Download From Human Prehistory to the Early Civilizations

Document related concepts

Proto-globalization wikipedia , lookup

Transcript
AP World History
Mr. Walker
Class Study Guide
walkerapworld.blogspot.com
walkerapworld.pbworks.com
Packet Number
Table of Contents
Quarter 1
Table of Contents ---------------------------------------------------------------------------------------------------------------Textbook Reading Schedule---------------------------------------------------------------------------------------------------Disclosure Statement -----------------------------------------------------------------------------------------------------------Why is History worth Studying -----------------------------------------------------------------------------------------------1. From Human Prehistory to the Early Civilizations -------------------------------------------------------------------2. Classical Civilization: China ---------------------------------------------------------------------------------------------3. Classical Civilization: India ----------------------------------------------------------------------------------------------4. Classical Civilization in the Mediterranean: Greece and Rome -----------------------------------------------------5. The Classical Period: Directions, Diversities, and Declines by 500 C.E. ------------------------------------------6. The First Global Civilization: The Rise and Spread of Islam --------------------------------------------------------7. Abbasid Decline and the Spread of Islamic Civilization to South and Southeast Asia --------------------------8. African Civilizations and the Spread of Islam -------------------------------------------------------------------------9. Civilization in Eastern Europe: Byzantium and Orthodox Europe -------------------------------------------------10. A New Civilization Emerges in Western Europe ---------------------------------------------------------------------Quarter 2
11.
12.
13.
14.
15.
16.
17.
18.
19.
20.
21.
22.
The Americas on the Eve of Invasion -----------------------------------------------------------------------------------Reunification and Renaissance in Chinese Civilization: The Era of the Tang and Song Dynasties ------------The Spread of Chinese Civilization: Japan, Korea, and Vietnam ---------------------------------------------------The Last Great Nomadic Challenges: From Chinggis Khan to Timur ---------------------------------------------The West and the Changing Balance of World Power ---------------------------------------------------------------The World Economy ------------------------------------------------------------------------------------------------------The Transformation of the West, 1450 – 1750 ------------------------------------------------------------------------The Rise of Russia -------------------------------------------------------------------------------------------------------- -Early Latin America ------------------------------------------------------------------------------------------------------- Africa and the Africans in the Age of the Atlantic Slave Trade -----------------------------------------------------The Muslim Empires ------------------------------------------------------------------------------------------------------ Asian Transitions in an Age of Global Change ------------------------------------------------------------------------Quarter 3
23.
24.
25.
26.
27.
28.
29.
30.
31.
The Emergence of Industrial Society in the West, 1750 – 1914 ----------------------------------------------------Industrialization and Imperialism: The Making of the European Global order -----------------------------------The Consolidation of Latin America, 1830 – 1920 -------------------------------------------------------------------Civilization in Crisis: The Ottoman Empire, the Islamic Heartlands, and Qing China --------------------------Russia and Japan: Industrialization Outside the West ----------------------------------------------------------------Descent into the Abyss: World War I and the Crisis of the European Global Order -----------------------------The World Between the Wars: Revolutions, Depression, and Authoritarian Response -------------------------A Second Global Conflict and the End of the European World Order ---------------------------------------------Western Society and Eastern Europe in the Decades of the Cold War ---------------------------------------------Quarter 4
32.
33.
34.
35.
36.
Latin America: Revolution and Reaction into the 21st Century -----------------------------------------------------Africa, the Middle East, and Asia in the Era of Independence ------------------------------------------------------Rebirth and Revolution: Nation-building in East Asia and the Pacific Rim ---------------------------------------The End of the Cold War and the Shape of a New Era: World History 1990 – 2006 ----------------------------Globalization and Resistance --------------------------------------------------------------------------------------------Appendix
37.
38.
39.
40.
Writing for the AP Exam-------------------------------------------------------------------------------------------------Developing a Thesis and Planning an Essay---------------------------------------------------------------------------Analyzing a Map-----------------------------------------------------------------------------------------------------------World History Generic Document Based Question (DBQ) Scoring Rubric----------------------------------------
2
41.
42.
43.
44.
45.
46.
47.
48.
49.
50.
51.
52.
53.
54.
55.
World History Change and Continuity Over Time (CCOT) Essay Rubric----------------------------------------World History Compare and Contrast (CC) Essay Rubric-----------------------------------------------------------Leader Analysis Sheet----------------------------------------------------------------------------------------------------Peoples Analysis Sheet--------------------------------------------------------------------------------------------------- Conflict Analysis Sheet--------------------------------------------------------------------------------------------------Change Analysis Sheet---------------------------------------------------------------------------------------------------Societal Comparison Sheet----------------------------------------------------------------------------------------------Document Analysis Sheet------------------------------------------------------------------------------------------------The Dialectical Journal---------------------------------------------------------------------------------------------------Inner / Outer Circle-------------------------------------------------------------------------------------------------------Additional Questions Divided by Eras---------------------------------------------------------------------------------Timeline--------------------------------------------------------------------------------------------------------------------The Exam-------------------------------------------------------------------------------------------------------------------Periods, Key Concepts, and 5 Themes---------------------------------------------------------------------------------Vocabulary Card Format--------------------------------------------------------------------------------------------------
3
AP World History Textbook Reading Schedule ~ 2011-2012
The Global Experience 6th Edition
Please read each section BEFORE class. The pages indicate what material we will be
discussing in class that day. So, for instance, if we will discuss P. 8-18 ON Wednesday
September 1st - have it read BEFORE that class. We have 36 chapters to study in addition
to various outside readings I will assign so it is imperative that you keep up with this
textbook reading, on your own, so as not to fall behind. The most important assignment in
this class is the reading- you will get the most out of lectures, discussions and various
activities if you stay on top of the reading. To know and understand history you must
read it.
There will be other Primary and Secondary Source Material Assigned!!!
1st Quarter Monday, August 29, 2011 – Thursday, November 3, 2011
Part I Unit 1 Ch 1-5 Human Prehistory -the Classical Era (500/600 CE)
Ch 1- Human Prehistory to the Early Civilizations 2.5 Million – 1000 BCE
DATE
Mon
Wed
Fri
Wed
Aug 29
Aug 31
Sep 2
Sep 7
Reading to be discussed/class activity
Assignment/Assessment
First Day of Class
Introduction to Part I
Ch 1 #1 P. 8-18
Ch 1 #2 P. 18-31
None
None
Study Guide Due
Study Guide Due
Ch 2- Classical China 1000 BCE-500 CE
DATE
Fri
Tue
Sep 9
Sep 13
Reading to be discussed/class activity
Assignment/Assessment
Ch 2 #1 P. 32-44
Ch 2 #2 P. 44-54
Study Guide Due
Study Guide Due
Ch 3- Classical India 1000 BCE-500 CE
Thu Sep 15
Ch 3 #1 P. 56-64
Mon Sep 19
Ch 3 #2 P. 64-74
Study Guide Due
Study Guide Due
Ch 4- Classical Greece & Rome 100 BCE-500 CE
Wed Sep 21
Ch 4 #1 P. 76-87
Study Guide Due
Fri
Sep 23
Ch 4 #2 P. 87-96
Study Guide Due
Ch 5- Classical Period: Declines, Diversities & Declines by 500 CE
Tue Sep 27
Ch 5 #1 98-107
Study Guide Due
Thu Sep 29
Ch 5 #2 107-119
Study Guide Due
Mon Oct 3
Unit 1 Test
Readings/IDs Due
Part II Unit 2 Postclassical Era Ch 6-10: Rise of Islam, Postclassical Africa & Medieval
Europe 500-1450 CE
DATE
Reading to be discussed/class activity
Ch 6 The Rise and Spread of Islam
4
Assignment/Assessment
Wed Oct 5
Mon Oct 10
Ch 6 #1 P. 120-134
Ch 6 #2 P. 134-148
Study Guide Due
Study Guide Due
Ch 7 Abbasid Decline and the Spread of Islamic Culture to Asia
Wed Oct 12
Ch 7 #1 P. 150-160
Study Guide Due
Fri
Oct 14
Ch 7 #2 P. 160-170
Study Guide Due
Ch 8 Postclassical Africa
Tue Oct 18
Ch 8 P. 172-192
Study Guide Due
*** Fall Recess ***
Ch 9 Eastern Europe, Byzantium & Orthodox Europe
Mon Oct 24
Ch 9 P. 194-209
Study Guide Due
Ch 10 Medieval Europe
Wed Oct 26
Ch 10 #1 P. 210-221
Fri
Oct 28
Ch 10 #2 P. 221-232
Tue Nov 1
Unit 2 Test
Study Guide Due
Study Guide Due
Readings & IDs Due
No Quarter Final
End of 1st Quarter
2nd Quarter Thursday, November 3, 2011 – Thursday, January 19, 2012
Part II Unit 3 Postclassical Era: Ch 11-15: The Americas, East Asia, Mongols & “The Rise of
the West” in The Post-Classical Era: 500-1450 C.E.
DATE
READING TO BE DISCUSSED
ASSESSMENT/ASSIGNMENT
Ch 11 The Americas on the Eve of Invasion
Thu Nov 3
Ch 11 #1 P. 234-244
Tue Nov 8
Ch 11 #2 P. 244-255
Study Guide
Study Guide
Ch 12 Reunification & Renaissance in Chinese Civilization: Tang & Song
Thu Nov 10
Ch 12 #1 P. 256-267
Study Guide
Mon Nov 14
Ch 12 #2 P. 267-276
Study Guide
Ch 13 The Spread of Chinese Civilization: Japan, Korea & Vietnam
Wed Nov 16
Ch 13 #1 P. 278-290
Study Guide
Fri
Nov 18
Ch 13 #2 P. 290-300
Study Guide
Ch 14 Last Great Nomadic Challenges/Threat: Genghis Kahn-Tamerlane
Tue Nov 22
Ch 14 #1 P. 302-312
Study Guide
*** Thanksgiving Recess ***
Mon Nov 28
Ch 14 #2 P. 312-322
Study Guide
Ch 15 “The Rise of the West”
Wed Nov 30
Ch 15 P. 324-338
Fri
Dec 2
Unit 3 Test
Chapter Study Guide
Reading, IDs Due
5
Part III Unit 4 Early Modern Era: Ch 16-19: The World Economy, Transformation of the
West, The Rise of Russia, Early Latin America in The Early Modern Era: 1450-1750 C.E.
Ch 16 The World Economy
DATE
Tue
Thu
Dec 6
Dec 8
READING TO BE DISCUSSED
ASSESSMENT/ASSIGNMENT
Ch 16 #1 P. 340-353
Ch 16 #2 P. 353-366
Study Guide
Study Guide
Ch 17 The Transformation of the West 1450-1750
Mon Dec 12
Ch 17 #1 P. 368-376
Study Guide
Wed Dec 14
Ch 17 #2 P. 376-386
Study Guide
Ch 18 The Rise of Russia
Fri
Dec 16
Ch 18 P. 388-402
Chapter Study Guide
Ch 19 Early Latin America
Tue Dec 20
Ch 19 #1 P. 404-416
Thu Dec 22
Ch 19 #2 P. 416-430
Study Guide
Study Guide
*** Winter Recess ***
Unit 4 Take-Home Test. Test, Readings, IDs Due Mon January 3rd 2012
ALSO Ch 20 Study Guide P. 432-454
Chapter Study Guide
Part III Unit 5 Early Modern Era: Ch 20-22: Africa and Africans in the Age of the Atlantic
Slave Trade, The Muslim Empires, Asian Transitions in an Age of Global Change in The
Early Modern Era: 1450-1750 C.E.
Ch 20 Africa & Africans in the Age of the Atlantic Slave Trade
Tue Jan 3
Ch 20 P. 432-454
Chapter Study Guide
Ch 21 The Muslim Empires
Thu Jan 5
Ch 21 #1 P. 456-467
Mon Jan 9
Ch 21 #2 P. 468-480
Study Guide
Study Guide
Ch 22 Asian Transitions in an Age of Global Change
Wed Jan 11
Ch 22 #1 P. 482-491
Study Guide
Fri
Jan 13
Ch 22 #2 P. 491-502
Unit 5 Test, Reading, IDs Due
No Quarter Final
END OF 2nd Quarter
3rd Quarter Thursday, January 19, 2011 – Friday, March 23, 2012
Part IV Unit 6 Industrialization in the West: Ch 23-25: The Emergence of Industrial Society,
Industrialization & Imperialism- The European Global Order, & The Consolidation of Latin
America
6
Chapter 23: The Emergence of Industrial Society
DATE
READING TO BE DISCUSSED
ASSESSMENT/ASSIGNMENT
Wed Jan 18
Mon Jan 23
Wed Jan 25
Ch 23 #1 P. 506-519
Ch 23 #2 P. 519-527
Ch 23 #3 P. 527-536
Study Guide
Study Guide
Study Guide
Chapter 24: Industrialization & Imperialism-European Global Order
DATE
Fri
Tue
Jan 27
Jan 31
READING TO BE DISCUSSED
ASSESSMENT/ASSIGNMENT
Ch 24 #1 P. 538-549
Ch 24 #2 P. 549-561
Study Guide
Study Guide
Chapter 25: The Consolidation of Latin America
DATE
READING TO BE DISCUSSED
ASSESSMENT/ASSIGNMENT
Thu Feb 2
Mon Feb 6
Wed Feb 8
Ch 25 #1 P. 562-574
Ch 25 #2 P. 571-588
No Textbook Assignment
Study Guide
Study Guide
Unit Test
Part IV Unit 7 Industrialization in the East: Ch 26-27: Civilizations in Crisis- Ottoman Empire &
Middle East, Qing China
Chapter 26: Civilizations in Crisis- Ottoman Empire & Middle East
DATE
Fri
Tue
Feb 10
Feb 14
READING TO BE DISCUSSED
ASSESSMENT/ASSIGNMENT
Ch 26 #1 P. 590-601
Ch 26 #2 P. 601-612
Study Guide
Study Guide
Chapter 27: Qing China
DATE
READING TO BE DISCUSSED
ASSESSMENT/ASSIGNMENT
Thur Feb 16
Tue Feb 21
Thu Feb 23
Ch 27 #1 P. 614-625
Ch 27 #2 P. 625-635
No Textbook Assignment
Study Guide
Study Guide
Unit Test
Part V Unit 8 Contact and Conflict: Ch 28-31: World War I, Great Depression/Authoritarianism,
WWII, The Cold War
Chapter 28: WWI
DATE
READING TO BE DISCUSSED
ASSESSMENT/ASSIGNMENT
Mon Feb 27
Wed Feb 29
Ch 28 #1 P. 636-649
Ch 28 #2 P. 650-669
Study Guide
Study Guide
Chapter 29: Great Depression/Authoritarianism
DATE
READING TO BE DISCUSSED
ASSESSMENT/ASSIGNMENT
Mon Mar 5
Wed Mar 7
Ch 29 #1 P. 672-687
Ch 29 #2 P. 687-703
Study Guide
Study Guide
READING TO BE DISCUSSED
ASSESSMENT/ASSIGNMENT
Ch 30 #1 P. 706-717
Ch 30 #2 P. 718-729
Study Guide
Study Guide
Chapter 30: WWII
DATE
Fri
Tue
Mar 9
Mar 13
Chapter 31: The Cold War
DATE
READING TO BE DISCUSSED
ASSESSMENT/ASSIGNMENT
7
Thu Mar 15
Mon Mar 19
Wed Mar 21
Ch 31 #1 P. 732-745
Ch 31 #2 P. 745-760
Unit Test
Study Guide
Study Guide
No Textbook Assignment
Part V Unit 9 Latin America, Africa & Asia: Ch 32-36 Latin America- Revolution & Reaction, Africa
& the Middle East, Asia & the Pacific Rim, End of the Cold War 1990-NOW, Globalization &
Resistance
Chapter 32: Latin America- Revolution & Reaction
DATE
Fri
Mar 23
READING TO BE DISCUSSED
ASSESSMENT/ASSIGNMENT
Ch 32 #1 P. 762-772
Study Guide
Part V Unit 9 Continues in 4th Quarter
End of 3rd Quarter
4th Quarter Friday, March 23, 2011 – Friday, June 1 2012
Part V Unit 9 Latin America, Africa & Asia: Ch 32-36 Latin America- Revolution & Reaction, Africa
& the Middle East, Asia & the Pacific Rim, End of the Cold War 1990-NOW, Globalization &
Resistance
From 3rd Quarter
Chapter 32: Latin America- Revolution & Reaction
DATE
READING TO BE DISCUSSED
ASSESSMENT/ASSIGNMENT
Wed Mar 28
Fri
Mar 30
Ch 32 #1 P. 762-772
Ch 32 #2 P. 772-782
Study Guide
Study Guide
Chapter 33: Africa, the Middle East and Asia in the Era of Independence
DATE
READING TO BE DISCUSSED
Wed Apr 3
Ch 33 #1 P. 784-796
Study Guide
*** Spring Recess ***
Ch 33 #2 P. 784-808
Study Guide
Tue
Apr 10
ASSESSMENT/ASSIGNMENT
Chapter 34: Rebirth & Revolution: Nation-building in East Asia and the Pacific Rim
DATE
READING TO BE DISCUSSED
ASSESSMENT/ASSIGNMENT
Thu Apr 12
Mon Apr 16
Ch 34 #1 P. 810-823
Ch 34 #2 P. 823-836
Study Guide
Study Guide
Chapter 35: The End of the Cold War and the Shape of New Era: World History 1990-2006
DATE
READING TO BE DISCUSSED
ASSESSMENT/ASSIGNMENT
Wed Apr 18
Fri
Apr 20
Ch 35 #1 P. 838-847
Ch 35 #2 P. 847-857
Study Guide
Study Guide
Chapter 36: Globalization and Resistance
Tue Apr 24
Ch 36 P. 860-876
Thu Apr 26
Unit 9 Test
Review
DATE
Material TO BE DISCUSSED
Mon Apr 30
Wed May 2
Part I Unit 1
Part II Unit 2 & 3
Study Guide
No New Assignment
8
Fri
Mon
Thu
Mon
May
May
May
May
4
8
10
14
Part III Units 4 & 5
Part IV Units 6 & 7
Writing Strategies
Writing Strategies
AP TEST May _____, _____AM
(This is a B Day and you will be excused from class from 8 AM to Noon)
Post-Test
DATE
Wed
Fri
Tue
Thu
Tue
Thu
Fri
May 16
May 18
May 22
May 24
May 29
May 31
Jun 1
Country Reports
Research
Research
Research
Research
Presentation
Presentation
Graduation
End of 4th Quarter
9
AP World History Bingham High School 2011-2012
Instructor: Mr. Matthew J. Walker
Disclosure Statement
A. Class Description:
AP World History is a social studies course designed for all sophomores, juniors, and seniors. This will be a
difficult class in that it will require you to analyze, evaluate, and reflect on the history of the world. You will
be expected to do two hours of homework each night to stay up to date with the class. Students will be
expected to learn about the history and culture of many different societies from around the world. Key topics
studied in this class include:
First Quarter
Foundations: 8000 B.C.E – 600 C.E: Origins of Civilization, Egypt, Mesopotamia, Classical Greece & Rome, Classical
India & China
Second Quarter
The Postclassical Era: 600 C.E. – 1450: Fall of the Roman Empire Rise of Medieval Europe, Post-Classical India & China,
Feudal Japan, The Era of the Mongols, Rise of Islam & Christianity.
Third Quarter
The Interaction of World Cultures: 1450 – 1750: The Age of Discovery, Early Global Commerce & Industrialization
Western Global Hegemony: 1750 – 1914: The Age of Industrialization and Revolutions (So many revolutions!)
Fourth Quarter
The 20th Century in World History: 1914 – Present: World War I, The Global Depression, World War II, The Cold War,
The Rise of Globalism, The Post-Cold War Era (So far)
B. Behavior Strategies:
In order for students to learn, and assist in the learning of their peers, personal responsibility and social
responsibility must be practiced. Students exhibit these responsibilities through their adherence to
classroom, as well as school, rules and procedures. Students need to take responsibility for their education
and take an active part in the classroom, making their contributions to the class, and interaction with peers,
constructive and positive.
1. Attendance Policy.
Receiving academic credit for this class is dependent upon the conditions outlined in the Bingham High
School Attendance Policy. Absences in excess of 2 per class per quarter that are not made up, will result
in loss of course credit. You can’t expect to learn if you are not in class.
2. Tardy Policy.
A. Students must be in their seats when the bell rings in order to be on time.
B. Students who arrive to class 15 or more minutes after the bell will be considered absent, not tardy
and will be sent to the attendance office to be checked into school.
C. After the first tardy, considered a warning, students will receive a 2% reduction from their final
grade for each additional tardy. Students can lose up to 20% of their grade.
D. In order to make up any tardy received after the first warning; a student must complete an in-class
school study session of 20 minutes.
10
C. Materials:
1. Students will need a notebook with three rings for this class. The notebook will be used to keep notes and
prepare writing assignments. This needs be brought to class EVERY DAY.
2. Students will need to bring a pen and/or pencil, along with the textbook, EVERY DAY.
3. Students will need color pencils.
D. Media:
1. Students watch a variety of historical movies and documentaries. Most of the time they are only shown
five minute clips to illustrate a point of view or a key concept. Sometimes students will watch full
video’s usually when they have a substitute teacher. Students need permission for all PG-13 videos in
the Jordan School district. By signing this disclosure statement you are giving your child permission to
watch the following movies.
Clash of the Titans (2010)
Jason and the Argonauts
Alexander the Great
Spartacus
Cleopatra
El Cid
Monty Python and the Holy Grail
Seven Samurai
Kagamusha
Master and Commander
Zulu Dawn
Marie Antoinette
The Message
The Last Emperor
Memoirs of a Geisha
PG-13
PG
PG
PG-13
NR
NR
PG
NR
PG-13
PG-13
PG
PG-13
PG
PG-13
PG-13
Gallipoli
Red Baron
Lawrence of Arabia
Gandhi
U-571
Valkrie
A Bridge to Far
Life is Beautiful
Fat Man and Little Boy
Empire of the Sun
Hunt for Red October
Dr. Strange Love
13 Days
Invictus
Hotel Rwanda
PG
PG-13
PG
PG
PG-13
PG-13
PG
PG-13
PG-13
PG
PG
PG
PG-13
PG-13
PG-13
E. Class work and Grading:
1. Students will receive a percentage grade for the semester. The percentage grade will be based on all the
assignments, quizzes and tests from the semester.
2. Daily assignments and activities will account for approximately 50% of the semester grade with the
remaining approximate 50% earned from tests, quizzes, DBQ’s, CC’s, and CCOT’s.
Letter grades are determined by the following percentage scale:
94-100 = A
80-82 = B67-69 = D+
90-93 = A77-79 = C+
63-66 = D
87-89 = B+
73-76 = C
60-62 = D83-86 = B
70-72 = CBelow 60 = F
3. If a student is absent on the day of a test, he/she must take the make-up test within one week of the
original test date.
4. No retakes will be given for tests without special circumstances to be determined by the teacher.
5. Class work that has been missed due to an excused absence will be accepted for one week following the
student’s return without penalty. In the event of unexcused absences, no credit will be given for
assignments or tests.
6. No late work is accepted, without special circumstances to be determined by teacher.
7. Work assigned before midterm will not be accepted after the midterm..
11
F. Classroom Expectations:
1. Respect yourself, respect others, and respect the classroom.
2. Honesty in all work. Students are expected to submit all original work and those found to be using the
work of others will receive no credit for the assignment or class.
3. Students will, at all times, be in accordance with the Jordan School District Dress Code Policy.
4. Cell phones must NEVER be seen or heard in the classroom. Each and every time a cell phone is seen
or heard in the classroom, the student will be penalized 2% of their final grade. The cell phone will be
confiscated and given to a vice principle. I don’t need to see it or hear it to take it away.
5. Students will be given two hall passes per quarter.
6. Students could be video and audio recorded for educational and media purposes.
7. Students can be expected to earn extra credit only when the work is completed well above the teacher’s
expectations.
8. This disclosure is subject to change by teacher and the administration at any time.
AP World History ___________________________ Period: _____
We, the undersigned, have reviewed the AP World History disclosure statement and I am aware of the
requirements and responsibilities of this class. We also understand that it is our responsibility to obtain any
further clarification we may need or want.
Student Name (Print)
_______________________________ Date:_____________________
Student Email Address
_______________________________ Date: _____________________
Student Signature
_______________________________ Date:_____________________
Parent Name (Print)
_______________________________ Date: ____________________
Parent Email Address
_______________________________ Date: ____________________
Parent/Guardian Signature
_______________________________ Date:_____________________
Administrative Signature
_______________________________ Date:_____________________
Teacher Signature
Matthew J. Walker
Date:
May 15, 2011
---------------------------------------------------------------------------------------------------------------------------------------
12
Why is History worth Studying?
A few years ago a book about women in the past appeared with an eye catching title: Herstory. Suddenly, the real
meaning of a commonly used word became a lot clearer. History is indeed a story, not specifically about women or men,
but about all those who have left some imprint on the age in which they lived.
History can be defined most simply as the story of human actions in past times. Those actions tend to fall into
broad patterns, regardless of whether they occurred yesterday or 5,000 years ago. Physical needs, such as the need for
food, water, and breathable air, dictate some actions. Others stem from emotional and intellectual needs, such as religious
belief or the search for immortality. Human action also results from desires rather than absolute needs. Some desires are
so common that they recur in every generation; some examples might be literary ambition, or scientific curiosity, or the
quest for political power over others.
History is the record of how people tried to meet those needs or fulfill those desires, successfully in some cases
unsuccessfully in others. Many generations of our ancestors have found familiarity with that record to be useful in
guiding their own actions. The study of past human acts also encourages us to see our own present possibilities, both
individual and collective. Perhaps that is history’s greatest value and has been the source of its continuous fascination for
men and women who have sought the good life.
Many people are naturally attracted toward the study of history, but others find it difficult or (even worse)
“irrelevant.” Some students perhaps yourself! Dread history courses, saying that they can see no point in learning about
the past. My life, they say, is here and now; leave the past to the past. What can be said in response to justify the study of
history?
Insofar as people are ignorant of their past, they are also ignorant of much of their present, for the one grows
directly out of the other. If we ignore or forget the experience of those who have lived before us we are like an amnesia
victim, constantly puzzled by what should be familiar, surprised by what should be predictable. Not only do we not know
what we should know, but we cannot perceive our true possibilities, because we have nothing to measure them against.
The non-historical mind does not know what is missing and contrary to the old saying, that can definitely hurt you!
A word of caution here: this is not a question of “history repeats itself.” This often quoted cliché is clearly
nonsense if taken literally. History does not repeat itself exactly, and the difference in details is always important. But
history does exhibit general patterns, dictated by common human needs and desires. Some knowledge of and respect for
those patterns has been a vital part of the mental equipment of all human societies.
But there is another, more personal reason to learn about the past. Adult persons who know none of their history
are really in the position of a young child. They are objects, not subjects. Like the child, they are acted upon by forces,
limited by restrictions, or compelled by a logic that they not only can do little about, but may not even perceive. They are
manipulated by others’ ideas, wishes, and ambitions. They never attain control of their lives, or at least, not until the
young child grows up. The sad thing is that the unhistorical adult has grown up, physically, but less so mentally.
The historically unconscious are confined within a figurative wooden packing crate, into which they were put by
the accident of birth into a given society, at a given time, in a given place. The boards forming the box enclose these
people, blocking their view in all directions. One board of the box might be the religion or lack of it into which they were
born; another, the economic position of their family; another, their physical appearance, race, or ethnic group. Other
boards could be whether they were born in a city slum or a small village, or whether they had a chance at formal
education in school (about three fourths of the world’s children never go beyond the third year of school). These and
many other facts are the boards of the boxes into which we are all born.
If we are to fully realize our potential as human beings, some (at least some!) of the boards must be removed so
we can see out, gain other vistas and visions, and have a chance to measure and compare our experiences with others
outside. Here “outside” refers to the cross section of the collective experience of other human beings, either now in the
present, or what is more manageable for study, in the knowable past.
Thus, the real justification for studying history is that it lets us see out beyond our individual birth box, into the
rich variety of others’ lives and thoughts. History is a factual introduction into humans’ past achievements; its breadth
and complexity vary, depending on the type. But whatever the type of history we study, by letting us see and giving us
perspective that enables us to contrast and compare our lives with those of others, history liberates us form the invisible
boards that confine us all within our birth box.
13
Chapter 1, From Human Prehistory to the Early Civilizations
Summary:
The earliest known humans lived in east Africa about 2.5 million years ago. These humans lived by hunting and
gathering. Gradually, the most advanced human species, Homo sapiens sapiens, migrated from Africa to the Middle East,
then into Europe, Asia, Australia, and the Americas. They developed tools out of stones, sticks, and other natural objects.
Agriculture began form about 10,000 years ago onward. This in turn encouraged the development of civilization. Early
civilizations arose in five different sites, four along the fertile shores of great environments and the search for food
supplies. The development of agriculture offered different opportunities for humans, including altered family forms,
formal political structures and cities, and monumental buildings. But change took place during this time period slowly.
The impact of this change in human civilization can be seen with children who were more supported, nurtured and
disciplined because they were a vital part of the family labor force in agricultural societies.
Key Concepts:
Human Life Before Agriculture:




Humans learned simple tool use, tamed fire, and developed bigger brains and a more erect posture during the
Paleolithic (Old Stone) Age, which lasted from about 2.5 million years to about 12,000 B.C.E.
Over time, the hunting and gathering species Homo sapiens sapiens, which originated in Africa and from
which all modern humans are descended, came to dominate other human types.
Stone tool use gradually improved, and humans developed speech, rituals, and culture as they gradually spread
across the globe.
In the Mesolithic (Middle Stone) Age, from about 12,000-8,000 B.C.E, humans made more advanced tools,
fought in more wars, and increased their population considerably.
The Neolithic Revolution:




In the Neolithic (New Stone) Age, between roughly 8,000 and 3,500 B.C.E., some human societies experienced
one of the most dramatic developments in human history.
These groups mastered sedentary agriculture (this is often called the “Neolithic Revolution”) and domesticated
animals. These innovations produced the food surpluses and rising populations that made possible the
founding of cities and the increasing specialization of occupations within human societies.
At the same time, pastoral nomadism developed, but these nomads remained the periphery of civilizations and
sedentary agricultural zones.
Soon after the introduction of agriculture, societies in the Middle East began replacing stone tools with those
made of metal—first copper, then bronze. These tools improved agriculture, aided in warfare, and benefited
manufacturing artisans.
Civilization:



The emergence of civilization occurred in many agricultural societies. It often built on additional changes in
technology including the introduction of metal tools.
Most civilizations had common features including cities, writing, formal institutions (especially government
and religion), stratified classes, and trade. Catal Huyuk is an excellent example of an important town in an
early Neolithic civilization.
Early civilizations included those in Mesopotamia, Egypt, the Indus River Valley, and northern China.
The Heritage of the River Valley Civilizations:
14


River valley civilizations left a number of durable innovations, but most declined after about 1,200 B.C.E,
This declines was often due to nomadic migrations across Eurasia by pastoral nomadic chariot peoples from the
central Asian steppe.
A number of small population centers emerged in the Middle East. These civilizations introduced further
innovations including the religion of Judaism, the alphabet, iron tools, and extensive trade connections across
the Mediterranean basin.
The First Civilizations:



The river valley civilizations created a basic set of tools, intellectual concepts such as writing and mathematics,
and political forms that persisted across three continents.
The rise of civilizations reduced local autonomy, as kings and priests tried to spread trade contacts and cultural
forms and warred to gain new territory.
Despite wars and trade, civilizations had little contact with each other and thus developed separate cultural
patterns.
Key Terms:
Bronze Age
Pastoralism
Nomads
Hunting and Gathering
Specialization
Cuniform
Monotheism
Judaism
Valley Civilizations
Sumerians
Iron
Agrarian Revolution
Phoenicians
Chapter 1, Quiz Questions
1) Hunting and gathering societies
A) are not able to produce art.
B) are always warlike and require little land.
C) organize rather small groups into political units.
D) could not survive after Middle Eastern people developed agriculture.
E) generally produce a food surplus.
2) A characteristic of the human species before the advent of civilization was
A) the ability to spread to various geographic settings and climate zones.
B) the ability to organize large political units.
C) the inability to communicate about abstractions such as death.
D) that all tasks were shared equally by men and women.
E) land ownership was equal.
3) The development of agriculture caused important changes in all of the following EXCEPT
A) population size and life expectancy.
B) male-female relations.
C) the tendency to believe in many gods.
D) the stability of human settlements.
E) the development of complex social patterns.
4) Why did the original inhabitants of Australia not develop agriculture?
A) Australian soil was too barren to grow crops.
B) The Australian climate was too severe.
C) They were too isolated to learn of developments elsewhere until recently.
D) Australia never experienced an ice age.
E) They were prevented from doing so by the Neolithic revolution.
5) Once developed, metal tools were preferred over stone tools for all of the following reasons EXCEPT
15
A) they were easier for ordinary people to make at home.
B) they were sharper and more precise.
C) they permitted more diverse shapes.
D) they could be used to make accurate weapons.
E) they were more durable.
6) A society is almost certainly a civilization if
A) it practices sedentary agriculture.
B) it involves tool use.
C) it has religious rituals.
D) it has some political structure.
E) it gathers food to survive.
7) The development of writing
A) resulted from new technologies, notably the invention of paper.
B) helps explain why agriculture could develop.
C) helps explain why governments could become more formal and bureaucratic.
D) resulted from the needs of the various river valley civilizations to communicate with one another.
E) was unusual in an agricultural society.
8) Egypt differed from Mesopotamian civilization by stressing
A) well-organized, durable empires.
B) extensive trade.
C) firm religious beliefs.
D) greater social equality.
E) more modest building projects.
9) Among the early river civilizations
A) the Huang he culture in China was the most isolated.
B) sedentary agriculture first developed in Mesoamerica.
C) writing was only found in the Nile river valley.
D) west Africa developed the first empire.
E) the use of metal tools spread very slowly.
10) Jewish monotheism
A) was spread actively by Jewish missionaries throughout the Middle East.
B) proposed a less human-like and more abstract God.
C) included worship of various lesser gods.
D) emerged at the high point of Sumerian civilization.
E) influenced no other religions.
Essay Questions:
From Human Prehistory to the Early Civilizations
1. What advantages does an agriculturally based society have over a hunter gatherer based society?
2. Compared to non-civilized societies, what are the major drawbacks of civilization?
3. Why is the development of writing important in the history of the river valley civilizations?
4. Compare the main features of Egyptian and Mesopotamian civilizations. What did the two civilizations have in
common as early civilizations? What were their main differences in values and organization?
5. Why was Jewish monotheism a significant development in the religious history of early civilization?
16
Chapter 2, Classical Civilization: China
Summary:
The major development during the classical period was the formation of large regional civilizations in China, India, the
Mediterranean, and the Middle East. These areas had by far the largest concentration of population. Furthermore, the
influence these civilizations extended into surrounding regions outside their direct control. Much of the development of
each civilization was separate and the establishment of distinctive cultural and institutional patterns was a key legacy of
this period. One of the triggers for the clear transition into the classical period was the introduction of iron tools and
weapons. With this development each classical civilization developed its own social structure, religion, political system,
system of science, and style of art. While the introduction of iron in the classical period, the period itself did not witness
sweeping technological developments. Patriarchal culture prevailed with a new emphasis on the respect for the
achievement of old age.
Key Concepts:
Patterns in Classical China:





Three dynastic cycles cover the many centuries of classical China: the Zhou, the Qin, and the Han.
Political instability and frequent invasions caused the decline of the Zhou Dynasty and promoted debate over
China’s political and social ills.
In the last centuries of the later Zhou era, some of China’s greatest thinkers, including Confucius, tried different
ways to restore order and social harmony. Central to culture were the family, filial piety, harmony, reciprocal
social relationships, and deference to social superiors.
Shi Huangdi, the brutal founder of the Qin Dynasty, centralized power in China, and began construction of the
Great Wall.
Wu Ti, most famous of the Han rulers, supported Confucianism and promoted peace.
Political Institutions:


For most of recorded history, the Chinese people have been the most tightly governed people in any society in the
world.
Political institutions became one of classical China’s hallmarks. Among the most permanent aspects of Chinese
culture was the belief in the unity and the desirability of a central government in the hands of an emperor
assisted by an educated, professional bureaucracy.
Religion and Culture:






Chinese culture began coalescing during the last, calamitous centuries of Zhou rule. During this time, three
critical secular philosophies arose, each of which emphasized the role of education to achieve social ends.
Confucianism, an ethical system based on relationships and personal virtue, became the predominant philosophy.
Legalism countered Confucianism by favoring an authoritarian state and harsh rule.
Daoism taught harmony with nature and humble living. Laozi was Daoism’s most popular figure.
Art in classical China was mostly decorative, and appeared in many forms, including calligraphy, carved jade
and ivory, and silk screens.
Science and mathematics emphasized the practical over the theoretical, and the ancient Chinese were particularly
adept at astronomy.
Economy and Society:


China’s classical economy focused on agriculture. All Chinese philosophies extolled the virtues of the peasants
and their world.
Sharp class division existed between 1) the landowning aristocracy and educated bureaucrats – Mandarins, 2)
the laboring masses, peasants, and urban artisans, and 3) the “mean” people, or those without meaningful skills.
17



The state also fostered an extensive internal trade, even while maintaining some ambivalence about merchants
and commercial values.
Technological advances were plentiful, including ox – drawn plows, water – powered mills, and paper.
Socially China was hierarchical, deferential, and patriarchal, and tight family structure was valued.
How Chinese Civilization Fits Together:



China’s politics and culture meshed readily, especially around the emergence of a Confucian bureaucracy.
Economic innovation did not disrupt the emphasis on order and stability, and family structures were closely
linked to political and cultural goals.
Classical Chinese civilization evolved with very little outside contact. Though internal disagreement existed,
most Chinese saw the world as a large island of civilization (China) surrounded by barbarians with nothing to
offer save periodic invasions.
Key Terms:
Qin Shi Huangdi
Silk Road
Analects
Patriarchalism
Daoism
Confucianism
Legalism
Han Dynasty
Qin Dynasty
Zhou Dynasty
Chapter 2, Quiz Questions
1) One difference between classical civilizations and river valley civilizations was that in classical civilizations
A) most people farmed the land.
B) political organizations were more elaborate.
C) trade was introduced.
D) writing was developed.
E) beliefs were widely held.
2) One difference between classical China and the earlier Huanghe river valley civilization was that
A) human sacrifices were suppressed.
B) traditions emphasized the harmony of nature.
C) irrigation was widely practiced.
D) China became relatively isolated.
E) farming was important.
3) The "Son of Heaven" concept promoted all of the following EXCEPT
A) loyalty to the emperor.
B) centralization of power in the state.
C) priests' control of the state.
D) the remoteness of emperor from subjects.
E) an explanation of the decline of dynasties.
4) Chinese views of nature emphasized
A) harmony and balance.
B) a powerful, all-seeing God.
C) that nature was uncontrollable.
D) that the key to understanding lay through scientific experiments.
E) deities that punished sinful people.
5) Which of the following was a Confucian belief?
A) Change should be encouraged and modeled by the emperor.
B) A good society has a hierarchy both in family and state.
C) Merchants must be valued for their money-making skills.
D) Governments must not interfere with individual rights.
18
E) People of all social classes and abilities should be actively involved in government.
6) Ceremony and hierarchy became an important part of upper-class Chinese life because
A) the Chinese believed that women should regulate the household.
B) the Chinese believed that it would help unify society and prevent greed.
C) the Chinese believed that polite behavior was a way to please the gods.
D) the Chinese believed that courtesy would win salvation in heaven.
E) the Chinese religion contained many public celebrations.
7) Daoists would agree with Confucianists on all of the following EXCEPT
A) the importance of political activity.
B) scorn for greed.
C) basic harmony of nature.
D) importance of restraint in personal life.
E) the importance of tradition.
8) The Chinese government accepted Daoism for all of the following reasons EXCEPT
A) Daoists did not have great political ambitions.
B) Daoists believed that nobles were holier than peasants.
C) Daoists came to acknowledge the Son of Heaven.
D) Daoists provided spiritual insights for many in the upper classes.
E) Daoism embraced traditional Chinese beliefs.
9) Chinese art featured
A) frequent shifts in style.
B) careful craftsmanship and detail work.
C) monumental temples and stadiums.
D) emphasis on God and the holy family.
E) images of perfectly proportioned humans.
10) One of China's key economic strengths was
A) extensive overseas trade.
B) government encouragement of the merchant class.
C) high levels of technological innovation.
D) early introduction of steam-powered equipment.
E) cheap slave labor.
11) Women in Han Chinese society
A) could rise to the level of the emperor but only if they had no children.
B) sometimes become quite powerful in a household.
C) tended to be poor and were sometimes sold into slavery.
D) tended to marry much younger men.
E) would not have children until much older.
12) A famous example of "cultural diffusion" in early Chinese history was
A) the spread of paper-making technology from the Middle East.
B) the use of the iron plow from Rome.
C) the development of written Mandarin Chinese characters.
D) the introduction and spread of Buddhism from India.
E) the use of the chariot, which was brought in by the Mongols.
Essay Questions:
Classical Civilization: China
1. What kinds of political problems was the development of bureaucracy in Han China meant to solve?
19
2. How does one explain why the early Chinese people were so creative yet their society was relatively isolated?
3. How did Chinese culture, particularly Confucian philosophy, support the political structure of the empire?
4. Why was China able to accept two different basic belief systems, Confucianism and Daoism?
5. Aside from periods of outright political decline, what were the chief tensions in Chinese society and culture?
How were they handled?
20
Chapter 3, Classical Civilization: India
Summary:
The oldest and most impressive civilizations of ancient India we know little beyond the archeological evidence. The
exotic picture writing remains largely undecipherable. The Maurya and Gupta Empires were inheritors of the Aryan cast
system. They expanded the cast system and promoted Buddhism to give their empires a loose control of India.
Key Concepts:
The Framework for Indian History: Geography and a Formative Period:



Geography (including the mountainous northern region and agricultural regions along the Indus and Ganges
rivers) and climate were major influences on Indian civilization.
The Aryan culture, which dominated India after the fall of the Indus River Valley civilization, also played a
formative role. Among other things, the Aryans brought the rudiments of the caste system.
The Vedas, the Mahabharata, the Ramayana, and the Upanishads formed the basis of a great Aryan literary
tradition.
Patterns in Classical India:




Two major empires formed at the crucial periods in classical Indian history, the Mauryan and, later, the Gupta.
The Greek conquest of the Indus and the exchange of ideas with the Mediterranean basin and southwest Asia
influenced the rise of the Mauryan dynasty.
Chandragupta Maurya was the first Mauryan ruler, and Ashoka the greatest. Ashoka expanded the empire and
promoted Buddhism.
The Guptas arose after a period of nomadic invasions, and created along period of political stability.
Political Institutions:


Regionalism and political diversity dominated classical Indian political life, so central authority was relatively
weak.
The increasingly complex caste system promoted public order the way more conventional government structures
did in many other cultures.
Religion and Culture:

Hinduism and Buddhims were integral parts of classical Indian life. They had great influence on the arts and
sciences, and both tended to promote religious tolerance.
 Hinduism is a polytheistic faith that gradually became more complex. It stresses reincarnation, the shallowness
of worldly concerns, and dharma, the moral path.
 Buddhism founded by Siddhartha Gautama in the 6th century B.C.E., scorned caste and the material world in
favor of self control and the Eightfold Path to nirvana.
 By the last centuries B.C.E., the Indian civilization developed a written language, built cities, and produced art
and literature, and nurtured two of the great world religions. Artistic patterns linked to religion and a significant
scientific tradition developed.
Economy and Society:


Dominated by the caste system, India developed extensive internal commercial and international maritime trade.
However, India’s economy remained essentially agricultural.
Family life combined patriarchy with an emphasis on mutual emotional support.
Indian Influence:
21

Classical India had an enormous effect on other parts of the world. India emerged as the center of a Eurasian
trade system, a source of great wealth and a means of exporting Indian culture abroad.
China and India:


China and India offer important contrasts in political emphases, social systems, and cultures.
They also resembled each other in seeking to build stable structures over large areas and in using culture to
justify social inequality.
Key Terms:
Kushans
Maurya Dynasty
Ashoka
Hinduism
Buddhism
Caste System
Vedas
Aryans
Chandragupta
Chapter 3, Quiz Questions
1) Which of the following does NOT help explain why India was invaded more often than China?
A) Geographical position
B) Hostility to warfare
C) Regional diversities
D) Political tradition
E) Greater contact with other societies
2) The Aryan conquerors brought to India
A) its first civilization.
B) distinctive religious ideas.
C) admiration for India's earlier inhabitants.
D) new agricultural techniques.
E) political democracy.
3) India's political tradition
A) involved the renunciation of violence and warfare.
B) stressed the importance of regional and local units.
C) emphasized the emperor as Son of Heaven.
D) insisted on religious uniformity.
E) required frequent wars of expansion.
4) The Indian caste system served to an extent as a political institution by
A) enforcing rules about social behavior.
B) unifying the subcontinent under a single government.
C) creating widespread interest in constitutional issues.
D) promoting a belief in individual rights.
E) causing unrest and rebellion.
5) Confucian and Hindu values both
A) focused attention on the afterlife.
B) helped justify and preserve social inequality.
C) urged the importance of political activity.
D) tried to outlaw war.
E) resulted in the building of magnificent temples.
6) Hindu ethics involved
A) a detailed set of prohibitions on sexual activity.
B) emphasis on an individual carrying out the obligations of life.
22
Gupta
C) attack on all opposing religious faiths.
D) condemnations of money-making.
E) finding ultimate happiness.
7) Buddhism differed from Hinduism by not believing
A) in the caste system.
B) in holy leaders.
C) in nirvana.
D) in the importance of moral obligations.
E) in spreading the faith.
8) Which of the following does NOT describe important features of Indian art?
A) Lively color
B) Linked to religious beliefs
C) Opposed to the use of animal figures
D) Influenced by styles from other cultures
E) Showed human figures
9) Compared to China, Indian social and economic structure
A) showed greater interest in technological innovation.
B) made it easier for a peasant to rise to higher status.
C) relied on conquest of foreign territories.
D) gave a stronger role to merchants.
E) tended to discourage commerce.
10) Compared to China, India
A) lacked regional diversity and was more centralized.
B) had greater contact with other societies and civilizations.
C) had a more flexible social structure that gave a greater role for women.
D) was more secular in outlook with a focus on commercial values.
E) had much more recent origins due to the invasion of the Aryans.
11) In the classical period, both China and India
A) showed considerable tolerance for different religions.
B) attempted to conquer the most territory possible.
C) developed a lasting tradition of strong, centralized government.
D) welcomed influences from other cultures.
E) created cultural traditions which rapidly died off.
12) In contrast to China, the social values that developed in classical India
A) promoted considerable equality between men and women.
B) encouraged greater emotional spontaneity.
C) urged that children not be required to work.
D) discouraged scientific research.
E) led to great political expansion.
Essay Questions:
Classical Civilization: India
1. Why did the caste system develop and how was it perpetuated?
2. What were the chief differences between Buddhism and the developing Hindu tradition?
3. What differences between Indian and Chinese geography help explain differences in social and cultural patterns?
23
4. Which society, Gupta India or Han China, was more successful?
5. How did the Indian caste system differ from the organization of Chinese society?
6. Compare the political implications of Hinduism and Confucianism. How does each belief system help explain the
political history of its native society in the classical period?
7. Compare the social and family structures of India and China in the classical period. What are the main
differences? The main similarities?
24
Chapter 4, Classical Civilization in the Mediterranean: Greece and Rome
Summary:
The civilizations of Greece and Rome rivaled those of India and China in cultural richness and their effect on world
history. Their institutions and values reverberated in the later histories of the Middle East and Europe and Europe’s
colonies around the world. The study of classical Mediterranean civilization is complicated because it includes Greek and
then Roman political, social, and economic institutions, which were sometimes shared but often unique.
Key Concepts:
The Persian Tradition:

Founded by Cyrus the Great, the Persian Empire was tolerant of local customs, developed iron technology,
organized an effective government and military, developed a new religion (Zoroastrianism), and supported a
great artistic tradition.
Patterns of Greek and Roman History:




The rise of the dynamic city – states of classical Greece began around 800 B.C.E, reaching a high point in the 5th
century B.C.E., when Pericles governed Athens.
Following the Peloponnesian Wars between Athens and Sparta, decline set in, but a new pattern of expansion
occurred under Alexander the Great. Greek values spread widely in the ensuing Hellenistic period.
As Hellenism declined, Rome was emerging as an expanding republic, defeating Carthage in the Punic Wars
and later becoming the Roman Empire after the death of Julius Caesar. For roughly 200 years, the Empire
enjoyed great power and prosperity.
Despite the efforts of emperors like Diocletian and Constantine, the ensuing 250 years brought a slow but decisive
fall.
Greek and Roman Political Institutions:




Greece and Rome featured an important variety of political forms. Both tended to emphasize aristocratic rule,
but there were significant democratic elements as well.
In the Greek polis, those who were citizens participated actively in political life. In Athens, the system of direct
democracy allowed citizens to shape policy in general assemblies.
In the Roman republic, the Senate was the main legislative body, but under the autocratic empire, the Senate’s
influence waned.
Later, Rome added emphasis on law and created the institutions necessary to run a vast and decentralized empire.
Religion and Culture:



Greek and Roman culture did not directly generate a lasting major religion, though Christianity arose in the
classical Mediterranean context. Greco – Roman religion used epic poems and mythology to explore human
foibles and passions.
An emphasis on rationality, especially in philosophy, science, and strong artistic and architectural tradition,
permeated classical Mediterranean culture.
Socrates, Plato, and Aristotle are the most well – known Greek philosophers.
Economy and Society in the Mediterranean:


Greek and Roman societies mirrored many standard social features of an agricultural economy, including a large
peasantry and a land – owning aristocracy and dependence on trade and commerce.
Differing versions of the patriarchal family structure existed in both Greek and Roman culture.
25

Distinctive features included slavery and a slightly less oppressive attitude towards women than was true in
classical China.
Toward the Fall of Rome:


Rome began to decline after about 180 C.E. Symptoms were gradual, including loss of territory and economic
reversals.
Ultimately, invaders periodically raided Rome until the empire finally collapsed.
Key Terms:
Zoroastrianism
Athens
Carthage
Alexander the Great
Cyrus the Great
Senate
Greek City-States
Diocletian &
Constantine
Twelve Tables
Roman Republic
Chapter 4, Quiz Questions
1) If the Greek genius was politics, the Roman genius was
A) engineering.
B) poetry.
C) science.
D) agriculture.
E) music.
2) Compared to modern American ideas of democracy, Athenian democracy was distinctive in
A) urging that the state adopt policies to benefit ordinary citizens.
B) separating foreign residents from citizens.
C) naming experienced leaders as military generals.
D) urging that all citizens participate directly in lawmaking and policymaking.
E) electing representatives to govern the city-state.
3) Greek politics resembled Indian politics in
A) the wide interest in diverse political theory.
B) the role of slave labor in providing revenues.
C) the inclusion of women as political leaders.
D) the tendency of regional fragmentation.
E) the use of military rulers.
4) From a Confucian viewpoint, the Roman Empire might have been criticized for placing too much confidence in
A) divine status of the emperor.
B) public works functions.
C) laws rather than trained officials.
D) harsh punishments of criminals.
E) religious conviction.
5) Republican Romans and democratic Athenians would have agreed that all of the following were politically important
EXCEPT
A) proper worship of the gods.
B) a strong military.
C) division of powers within the state.
D) splendid public buildings.
E) an involved citizenry.
6) Roman emperors tried to prevent popular disorder by
A) dividing the great landed estates.
26
B) granting the vote to lower-class citizens.
C) abolishing slavery in Rome and the provinces.
D) organizing food supplies and distribution.
E) enlarging the empire through the conquest of Gaul.
7) The Socratic Method emphasized the importance of
A) faith in authority.
B) laboratory experiment.
C) questioning.
D) harmony.
E) respect for elders.
8) The Greek and Hellenistic approach in science
A) stressed the importance of practical applications.
B) used mathematics to try and explain nature's patterns.
C) heavily influenced China's scientific approach.
D) emphasized the mysterious forces of nature.
E) was purely theoretical.
9) Greek and Roman agriculture
A) exported grain to Asia.
B) concentrated on vegetables and dairy products.
C) employed only a minority of the population.
D) tended to develop large, commercial estates.
E) ignored grape and olive cultivation.
10) Both ancient Rome and Greece depended on slavery. One result of this dependence was
A) both groups lagged in technological advances compared to the Chinese or the Indians.
B) many wars between the Romans and nomadic groups broke out when the Romans attempted to capture slaves.
C) the development of the idea of democracy, which led to the eventual fall of Rome.
D) the rise of a new social class known as the "metics" in Roman society.
E) a vast improvement in agricultural techniques leading eventually to the split of the Roman Empire.
Essay Questions:
Classical Civilization in the Mediterranean: Greece and Rome
1. What were the main similarities and differences between Greek and Roman political structures?
2. What is the significance of the Hellenistic period in Middle Eastern and Mediterranean history?
3. Write an “intellectual biography” essay on one of the following: Socrates, Plato, Sophocles, Aristotle, Vergil,
Cicero.
4. Compare the main features of the social structure of the classical civilizations in the Mediterranean, China, and
India.
5. In which society-India, China, or the Mediterranean – would you have wanted to be a merchant in the classical
period? Explain your choice.
6. What were the main similarities and differences between the empires of Rome and Han China? Why did Rome’s
empire prove less durable?
7. Early in the classical period, both Greece and India exhibited religions with many gods and goddesses. Why and
how did India go on to form a major, durable religion around such phenomenon while Greece did not?
27
8. Compare the scientific achievements and approaches of classical India, China, and the Mediterranean.
9. Compare Greek, Roman, and Confucian ideals.
28
Chapter 5, The Classical Period: Directions, Diversities, and Declines by 500 C.E.
Summary:
The basic themes of the three great classical civilizations of China, India, and the Mediterranean involved expansion and
integration. Throughout the classical world, these themes faltered between 200 and 500 C.E., signaling the end of that
era. The response of major religions to political decline formed a leading direction in the next phase of world history.
Meanwhile, developments outside the classical orbit gained new prominence.
Key Concepts:
Expansion and Integration:



Common themes for the classical civilization involve territorial expansion and related efforts to integrate the
new territories.
China united through centralization, India united through religious values, and the Mediterranean world united
through cultural achievements.
Integration required territorial and social cohesion. Each civilization valued social distinctions.
Beyond the Classical Civilizations:



Outside the centers of civilization important developments occurred. Significant civilizations operated in the
Americas (the Olmecs, Maya, and Incas) and also in Africa (Kush, Axum, and Ethiopia).
Agriculture and other developments spread across northern Europe and northern Asia, where semi – civilized
peoples developed extensive contacts with older civilizations.
Nomadic societies played a vital role, particularly in central Asia, in linking and occasionally disrupting classical
civilizations. Important popular migrations across Eurasia led to the rise of new cultures.
Decline in China and India:






A combination of internal weakness and nomadic invasions led to important changes, first in China, and then in
India.
The central Asian nomadic Huns attacked all three classical civilizations.
About 100 C.E., the Han dynasty began a serious decline. Weakened central government, social unrest led by
overtaxed peasants, and epidemics were the most prominent sources of decline, combining to make the
government unable to stop invading nomads.
However, by 600, China revived, first with the brief Sui dynasty and later (and more gloriously with the Tang.
The decline in India was not a drastic as in China. By 600, Huns destroyed the Gupta Empire. For several
centuries, no native Indian led a large state there. Hinduism gained ground as Buddhism, unappealing to the
warrior caste, declined.
After 600, Islam entered India and Arab traders took control of Indian Ocean trade routes. What survived was
Hinduism (Islam never gained adherence from a majority of the population) and the caste system.
Decline and Fall in Rome:



Decline in Rome was particularly complex. Although its causes have been much debated, certain issues may
have contributed; population declined, leadership faltered, the economy flagged, tax collection became more
difficult, a series of plagues swept the empire, and a sense of despondency pervaded much of the citizenry.
When Germanic tribes invaded in the 400’s there was little power or will to resist.
Developments also varied between the eastern and western portions of the Empire, as the Mediterranean world
fell apart. The eastern, or Byzantine Empire, continued for another 1,000 years after the western empire
collapsed.
The New Religious Map:
29


The period of classical decline saw the rapid expansion of Buddhism and Christianity. This religious change had
wider cultural, social, and political implications.
Later, Islam appeared, and spread, following the previous spread of Hinduism across south and southeast Asia.
The World Around 500 C.E.:




Developments around 500 C.E. produced three major themes for world history in subsequent periods.
First, there was a collapse of classical civilizations. Societies across Eurasia faced the task of reviving or
reworking their key institutions and values after decline and invasion.
Second, new religions arose and older ones spread. These would form the basis of future civilizations.
Finally, new developments across the globe, whether through indigenous developments or contacts with older
centers led to the rise of the new civilization.
Key Term
Inca
Mahayana
Yellow Turbins
Animism
Tang Dynasty
Olmec
Axum
Shintoism
Jesus of Nazareth
Coptic Christianity
Chapter 5, Quiz Questions
1) The first kingdoms in eastern Africa below the Sahara showed the influence of
A) Egypt and Hellenism.
B) Rome and Phoenicia.
C) Indian merchants and missionaries.
D) the flight of Jews from Israel.
E) Persia.
2) The end of the Gupta Empire differed from the decline of Rome in that it did not involve
A) a change in political institutions.
B) outside invasions such as the Huns and others.
C) the introduction of a new religion for the majority.
D) the weakening of central government.
E) big cultural changes.
3) Which of the following best survived the Hun invasions in India?
A) Hindu beliefs
B) Political unity
C) Nationalist beliefs
D) Buddhist beliefs
E) Aryan traditions
4) Events in late Han China and the late Roman Empire show that the decline of a civilization, whether temporary or
permanent,
A) is not simply the result of attack by outside invaders.
B) follows inevitably from centralized, unrepresentative government.
C) results from undue dependence on slavery.
D) results from social rebellion in which the poor attack the rich and destroy their institutions.
E) results from a lack of religious conviction.
5) One important early symptom of Rome's decline was
A) the drop in population due to a series of plagues.
B) the use of slave labor.
C) the replacement of republic by empire.
30
D) the weakness of the eastern portion of the empire compared to the west.
E) acceptance of Christianity.
6) The eastern portion of the Roman Empire experienced less decline than the west for all of the following reasons
EXCEPT
A) the east had older traditions of civilization.
B) the east resisted the spread of Christianity.
C) the east faced less pressure from barbarian invasions.
D) the east had more active trade.
E) the east was more wealthy.
7) After 200 C. E., an increasing number of people in Asia, Europe, and North Africa began to adapt faiths characterized
by
A) polytheism.
B) monotheism.
C) animism.
D) secularism.
E) ancestor worship.
8) Despite major differences, Christianity, Hinduism, and Buddhism all show interest in
A) strong priesthood.
B) clearly organized church structures.
C) absolute hostility to the worship of religious images and spirits of nature.
D) life after death.
E) polytheism.
9) Everywhere it spread, Buddhism stressed
A) worship of Buddha as a god.
B) a strong church organization.
C) meditation and ethical behavior.
D) the impossibility of attaining nirvana except by multiple reincarnations.
E) the worthlessness of all competing religions.
10) Compared to Hinduism, Christians are more likely to
A) see humans as superior to the rest of nature.
B) believe that women are morally superior to men.
C) approve of sexual pleasure.
D) tolerate other beliefs.
E) be polytheistic.
11) Compared to Hinduism and Buddhism, all of the following constitute distinctive features of late-Roman Christianity
EXCEPT
A) intolerance for competing beliefs.
B) belief in a divine trinity.
C) non-believers cannot join the church.
D) a strong hierarchy of church officials.
E) a strong evangelizing impulse.
12) By 600, looking at the entire world, a good definition of "barbarian" would be
A) someone who fights better than a peasant-soldier.
B) someone who is not Christian.
C) someone who is not part of a civilization.
D) someone who is illiterate.
E) someone who is a knight.
31
Essay Questions:
The Classical Period: Directions, Diversities, and Declines by 500 C.E.
1. Explore the differences in the eastern and western portions of the Roman Empire. How did these differences
arise? In what ways can it be argued that the Roman Empire survived in the eastern Mediterranean even after it
collapsed in the west?
2. What were the main factors in Rome’s decline? Which do you judge most important? Why?
3. Why did the results of Han china’s decline differ from those of the Roman Empire’s decline?
4. Compare the major beliefs and religious organization of Christianity and Buddhism.
5. Compare the major beliefs and religious organization of Christianity and Hinduism.
6. Taking into account both Egypt and Kush, what were the main features of civilization in Africa prior to the first
century C.E.
.
32
Chapter 6, The First Global Civilization: The Rise and Spread of Islam
Summary:
In the 7th century C.E., the Arab followers of Muhammad surged from the Arabian Peninsula to create the first global
civilization. They quickly conquered an empire incorporating elements of the classical civilizations of Greece, Egypt, and
Persia. Islamic merchants, mystics, and warriors continued its expansion in Europe, Asia, and Africa. The process
provided links fro exchange among civilized centers and forged a truly global civilization. Although united in belief of
Muhammad’s message, the Islamic world was divided by cultural and political rivalries. The disputes did not undermine
the strength of Muslim civilization until the 14th century.
Key Concepts:
Desert and Town: The Pre – Islamic Arabian World:



Islam appeared first on the Arabian Peninsula, an area occupied by pastoral nomads and on the periphery of the
civilized zones.
Much of the peninsula is desert, which supported both goat and camel nomadism among peoples called Bedouin.
Sedentary agricultural communities were limited to the far south of the peninsula, and trading towns like Mecca
developed along the coasts.
The Bedouin tribal culture of clan loyalty and rivalry provided a critical backdrop for the emergence of Islam.
Women enjoyed somewhat greater freedom, art was largely nonexistent, and religion was a blend of animism and
polytheism.
The Life of Muhammad and the Genesis of Islam:


In the 7th century C.E., a new religion arose in the Arabian Peninsula. Built on the revelations received by the
prophet Muhammad, a trader from Mecca, the new faith won over many camel – herding tribes of the peninsula
within decades.
Islam united Arabs and provided an important ethical system. Though initially an Arab religion, Islam’s beliefs
and practices (including the five pillars) eventually made it one of the great world religions.
The Arab Empire of the Umayyads:






Although some Bedouin tribes renounced their allegiance to Islam following Muhammad’s death in 632, the
Prophet’s followers were able to conduct military campaigns restoring the unity of the Islamic community.
Abu Bakr assumed leadership of the umma (community of the faithful). Ali, Muhammad’s son-in-law, was
passed over, which would later cause an important rift in the Muslim community.
Once the rebellious tribesmen were brought back into the umma, Muslim armies began to launch attacks on
neighboring civilizations outside of Arabia.
Within a short period of time, Arab armies exploited weaknesses in their enemies’ forces and captured
Mesopotamia, northern Africa, and Persia. A new dynasty, the Umayyads, ruled this Arabic empire.
The question of succession soon led to the Sunni-Shi’a split.
Umayyad extravagance ultimately led to the empire’s overthrow.
From Arab to Islamic Empire: The Early Abassid Era:




The Abbasid rulers move the Empire’s capital to Baghdad, and lived a life of luxury that alienated many
followers.
The Abbasids fully integrated the mawali, or non-Arab Muslims, into the Islamic community.
Merchants and landlords grew in wealth and status. Cities grew, the dhow improved sailing, and slave labor
became increasingly important.
Arab learning flowered, as scholars sought to preserve the great works of Greek and Roman civilization.
33
Key Terms:
Mawali
Umayyad
Bedouin
Mecca
Ka’ba
Qur’an
Hajj
Abbasids
Sunni
Shi’a
Chapter 6, Quiz Questions
1) Which of the following statements concerning bedouin society is NOT accurate?
A) Bedouin herders lived in kin-related clan groups.
B) Bedouins lived in highly mobile tent encampments.
C) Clans were commonly congregated together in larger tribal groupings.
D) Arabian society fostered strong dependence on loyalty and cooperation with kin.
E) Bedouins were rarely found living in urban areas.
2) Which of the following statements concerning inter-clan relationships in bedouin society is most accurate?
A) Clans within the same tribe almost never engaged in warfare, but violence between different tribes was common.
B) Arabic society was too mobile to result in many contacts between clans, and therefore violence was minimal.
C) Inter-clan violence over control of water and pasturage was common.
D) Inter-clan violence was regulated by a universally recognized code of law imposed by the Quraysh in Mecca.
E) Violence in Bedouin society was generally limited to slave uprisings.
3) What was the result of inter-clan rivalries?
A) They kept population down in a region that could support few people.
B) They tended to weaken the bedouin in comparison to neighboring peoples and empires.
C) They allowed for intermarriage between clan groups, thus preventing social isolation.
D) They prevented mobility and migration that would have debilitated efforts at more complex social organization.
E) They strengthened be bedouin and enabled them to challenge their neighbors.
4) What was the major difference between Medina and Mecca?
A) Political dominance in Medina was contested between a number of Jewish and bedouin tribes.
B) Mecca was established in an oasis, and Medina was in a mountainous region.
C) Medina was engaged in long-distance caravan trade, while Mecca was not.
D) Medina was located on the western side of the Arabian Peninsula, while Mecca was located on the Persian Gulf.
E) Medina was controlled by Coptic Christians while Mecca was controlled by the Banu Hashim clan.
5) Which of the following statements most accurately describes the status of women in bedouin society prior to Islam?
A) Women were regarded as little more than property with neither rights nor status.
B) Descent in bedouin tribes was strictly patrilineal.
C) Women were the equals of males in the rugged society of the desert bedouin.
D) They enjoyed greater freedom and higher status then Byzantine and Sasanian women.
E) Women were permitted to take more than one husband (with approval of their mother).
6) What was the nature of the material culture of bedouin society?
A) The bedouins constructed numerous temple complexes featuring monumental architecture in the form of pyramids.
B) Although their nomadic lifestyle did not permit the development of monumental architectural forms, the bedouins were
skilled painters and sculptors.
C) Mecca was a major center for the development of art and architecture, but the desert bedouin produced little of cultural
value.
D) Except in the sedentary agricultural communities of the South, there was little art or architecture and the chief focus of
cultural creativity was oral poetry.
E) The bedouins preserved the learning of classical cultures through writings that included prose-like epics.
7) What was the nature of pre-Islamic bedouin religion?
A) Most of the bedouin were Christians.
34
B) Most of the bedouin were Jews.
C) It was a blend of animism and polytheism.
D) The bedouin were monotheists who worshipped Allah.
E) The bedouins had no religious beliefs.
8) Which of the following represents a mounting pressure for change in pre-Islamic society?
A) Invasion from sub-Saharan Africa agricultural cultures incompatible with nomadic life
B) Greater Byzantine and Sasanian control over Arabic tribes of the peninsula and Arabic migration to Mesopotamia
C) The increasing influence of Hindu animism from the expansion of the Gupta Empire
D) The increasing influence of polytheism throughout all cultures around the Mediterranean
E) The increasing scarcity of natural resources to support the life and culture of the bedouins
9) What was the initial response of the Umayyads to Muhammad's new faith?
A) They regarded him as a threat to their wealth and power as he questioned the traditional gods of the Ka'ba.
B) They sought to protect him from a plot on his life by the Banu Hashim.
C) The Umayyads immediately accepted Muhammad as their religious and political leader and the chief power in Mecca.
D) The Umayyads simply ignored Muhammad as an insignificant member of a weak clan.
E) They sought him as an ally against the Sassinians and the Byzantines.
10) What was the principle advantage of the Islamic concept of the umma?
A) It provided a clear principle of political succession that would provide the basis for an Islamic state.
B) It provided for an annual treaty that would restore the trade routes of Arabia.
C) It provided dietary restrictions that allowed for more equitable distribution of food in Arabia.
D) It transcended old tribal boundaries and made possible political unity among Arab clans.
E) It emphasized the value of individualism and fostered self-reliance among the bedouin tribes.
11) Which of the following statements concerning the ethical system of early Islam is NOT correct?
A) Islam stressed the dignity of all believers and their equality in the eyes of Allah.
B) Islam stressed the responsibility of the wealthy and strong to care for the poor and weak.
C) A tax for charity was obligatory in the new faith.
D) The teachings of the Prophet and the Quran were not formally incorporated into a body of law.
E) It recognized the truth of similar ethical ideas in Judaism and Christianity.
12) What was Muhammad's teaching with respect to the revelations of other monotheistic religions?
A) Muhammad accepted the earlier Christian revelations, but rejected completely any influence from Judaism.
B) Muhammad accepted the earlier Judaic revelations, but rejected completely any influence from Christianity.
C) Muhammad accepted the validity of earlier Christian and Judaic revelations and taught that his own revelations were a
final refinement and reformulation of earlier ones.
D) Muhammad stressed that only his own revelations had merit and that others were works of the devil.
E) Muhammad taught that monotheistic religion was compatible with polytheism.
13) Which of the following was NOT a reason for the early expansion of Islam beyond Arabia?
A) The desire for booty
B) The sense of common cause and united strength
C) The desire to convert new populations to Islam
D) A means to release the energies of the bedouin tribes against others than themselves
E) The weakness of their adversaries.
14) Why did the Arab warriors not want to convert large numbers of people to Islam?
A) Muhammad specifically stated that Islam could only be spread among the Arabs.
B) They would have had to share their booty and would have lost tax revenues.
C) They lacked the political organization to govern them and feared insurrection by non-Arabs.
D) Conversion would have slowed down the process of conquest.
E) They wanted to keep high religious offices among themselves.
35
15) What was the Umayyad attitude to other religions?
A) The Umayyads suppressed all religions within their territories other than Islam.
B) The Umayyads converted to Christianity, but continued to permit the open worship of Islam.
C) The Umayyads displayed tolerance towards the religions of dhimmi peoples.
D) Christianity and Judaism were suppressed as heresies, but other communities were permitted to retain their religions.
E) Zoroastrians and Hindus were never accepted.
41) What was the most significant of the transformations brought about by the Abbasids' rise to power?
A) The final defeat of the Byzantine Empire with the capture of Constantinople
B) The mawali were admitted as full members of the Islamic community
C) The destruction of absolutism within Islamic government
D) The destruction of Sunni influences within Islam
E) Victory in the Crusades and the conquest of Spain
16) What was the nature of the Abbasid government?
A) The Abbasids abandoned the formality and absolutism of the Umayyads and established an open and representative
government.
B) The Abbasids outdid the Umayyads in establishing an absolutist government symbolized by the growing powers of the
wazirs and the sinister presence of the executioner.
C) The Abbasid government represented a return to the principles of government in the first days of the Orthodox
caliphate.
D) The Abbasids continued the policies of the Umayyads virtually without change, including the maintenance of an
exclusively Arabic elite.
E) The Abbasid government was extremely efficient due to the influence of Byzantine advisors.
17) What was the nature of the economy of the Abbasid period?
A) It was a period of general prosperity typified by urban growth and the restoration of the Afro-Eurasian trade axis.
B) There was a general crisis in the agricultural economy resulting from the constant warfare of the period.
C) The economic downturn of the era was typified by the breakdown in the trade between the Middle East and China.
D) Although commerce was generally resuscitated during the Abbasid period, artisan production dropped off
significantly.
E) Trade was initiated with northern Europe, which eventually led to the rise of slave trade with Africa.
18) What was the status of artisans in Abbasid cities?
A) Handicraft industries were staffed by slave labor exclusively.
B) The number of artisans decreased along with the economic crisis of the Abbasid period.
C) Artisans were free men who owned their own tools and who formed guild-like organizations to negotiate wages.
D) Artisans were able to utilize their guild-like organizations to seize political control of most Abbasid towns.
E) Artisans found no markets for their goods and were finally incorporated into the ayan.
19) What was the nature of slavery within the Abbasid social system?
A) According to the Quran, slavery could not exist in Islam, and the Abbasid freed all former slaves.
B) Slavery was limited to the non-Arab converts to Islam, and died out during the period of the Abbasid Empire.
C) Because most unskilled labor was left to the unfree, slaves could be found in both the towns and countryside of the
Abbasid Empire.
D) Slavery was known in Abbasid cities, but was virtually unknown in the countryside where most labor was performed
by a free peasantry.
E) Most slaves worked under favorable conditions but were never allowed to convert to Islam.
20) What was the primary cultural contribution of the Muslims during the Abbasid period?
A) The Muslims were able to recover and preserve the works of the ancient philosophers as well as transmit ideas and
culture from one civilization to another.
B) The Muslims became extraordinarily adept at portraiture, focusing on depictions of Muhammad and the early caliphs.
C) Although the material culture of the Abbasid period remained poor, Muslims were able to make some advances in
music.
36
D) Islamic learning was necessarily unique, as they had no access to the ancient traditions of philosophy and science.
E) Islamic culture combined the achievements of earlier civilizations such as the Harappan and the Aryans.
Essay Questions:
The First Global Civilization: The Rise and Spread of Islam
1. Describe the social organization of the Arabs prior to the introduction of Islam.
2. Compare and contrast the status of women in Islam during the pre – Islamic period and the Abbasid Empire.
3. Why was Islam able to appeal to people of so many different cultures?
4. Why was the essential dispute within Islam over the succession to the Prophet Muhammad?
5. What was the difference between the Abbasid Empire and the Umayyad Empire?
37
Chapter 7, Abbasid Decline and the Spread of Islamic Civilization to South and Southeast Asia
Summary:
By the mid-9th century, the Abbasids were losing control over their vast Muslim Empire. Distance hampered efforts to
move armies and control local administrators. Most subjects retained local loyalties. Shi’s dissenters were particularly
troublesome, while slave and peasant risings sapped empire strength. Mongol invasions in the 13th century ended the very
weakened state. Despite the political decline, Islamic civilization reached new cultural heights, and Islam expanded
widely in the Afro-Asian world through conquest and peaceful conversion.
Key Concepts:
Islamic Heartlands in the Middle and Late Abbasid Eras:





The Abbasid leadership’s excess and sumptuous living (as exemplified by Harun al-Rashid) was apparent form
early in the caliphate. This led to the gradual disintegration of the vast empire between the 9th and 13th
centuries.
Civil wars drained the treasury, and revolts against the ensuing taxes spread among the peasants.
Slavery increased, and the position of women was further eroded.
Divisions within the empire (brought by groups like the Buyids and Seljuk Turks) opened the way for Christian
crusaders from Western Europe to invade, and for a short time, establish warrior kingdoms in the Muslim
heartlands.
Political decline and social turmoil were offset for many by the urban affluence, inventiveness, and artistic
creativity of the Abbasid Age.
An Age of Learning and Artistic Refinements:






As the Abbasid dynasty fell politically, trade links and intellectual creativity grew dramatically.
An expansion of the professional and artisan classes demonstrated increasing urban prosperity.
Persian gradually replaced Arabic as the court language, and many great works of literature were authored.
Scientists and doctors also made many important advances in chemistry, astronomy, and human biology.
Islam saw an increase in both mysticism (the Sufis) and orthodox religious scholars (the ulama).
The Abbasid caliphate eventually fell to the Mongols, then the forces of Tamerlane.
The Coming of Islam to South Asia:



From the 7th century onward, successive waves of Muslim invaders, traders, and migrants carried the Islamic
faith and elements of Islamic civilization to much of the vast south Asian subcontinent.
The first wave of influence occurred as a result of Muhammad ibn Qasim’s incursions. The second occurred
when Muhammad of Ghazni conquered much of the Indus River Valley and north central India.
Conversion to Islam was peaceful; in particular, many Indian Buddhists became Muslim. However, the onceegalitarian Islam generally succumbed to the caste system. Hindus took many measures to maintain
dominance, including emphasizing devotional cults, and as a result Islam did not spread in India as it had
elsewhere.
The Spread of Islam to Southeast Asia:


The spread of Islam to India set the stage for its further expansion. Arab traders and sailors regularly visited the
“middle ground” ports of Southeast Asia.
From the 13th century, traders and Sufi mystics spread Islam to Java and the islands of modern Indonesia. As was
the case in India, conversion was generally peaceful, and the new believers combined Islamic teachings and
rituals with elements of local religions that had spread to the area in preceding centuries.
Key Terms:
38
Ulama
Chinggis Khan
Huran al-Rashid
Mongals
Bhaktic Cult
Sati
Saladin
Sufis
Crusades
Sultans of Delhi
Chapter 7, Quiz Questions
1) Which of the following is NOT a reason for the decline of the Abbasid dynasty by the ninth century C.E..?
A) The collapse of the cities in the Islamic heartland
B) The retention of regional identities by the population
C) The difficulty of moving armies across the great distances of the empire
D) The difficulty of compelling local administrators to obey
E) Disputes over succession of the caliph
2) Which of the following groups did NOT revolt against Abbasid rule?
A) Slaves
B) Peasants
C) Sunnis
D) Shi'as
E) Buyids
3) What was al-Mahdi's attitude toward the Shi'as?
A) He viewed them as opponents of his dynasty and attempted to eliminate them.
B) He appealed to the moderate factions of the Shi'as to support the Abbasid dynasty.
C) He accepted the fundamental doctrines of the Shi'as and abdicated.
D) In order to placate the Shi'as, al-Mahdi lived a pious life of poverty and simplicity.
E) He instituted a program of conversion so they would become Sunni.
4) How did the administration of al-Rashid set a trend for subsequent Abbasid rulers?
A) He was at the outset of his reign heavily dependent on Persian advisors, a practice that became commonplace
thereafter.
B) He divided the empire into a series of states each of which elected representatives to a caliphal parliament at Baghdad.
C) He removed all regional governors and established strict absolutism from the court at Baghdad.
D) He stressed the importance of piety and simplicity, a position subsequently adopted by his successors.
E) He made the position of caliph all-powerful by building a new capital complex in Cairo.
5) What was the result of the civil wars following the death of al-Rashid?
A) The Abbasid dynasty came to an end with the defeat in the Battle of Zamzam.
B) Succession disputes led to the build up of personal armies, often of slave soldiers.
C) Private armies were abolished in an attempt to curtail future violence over the succession.
D) The practice of electing caliphs in Mecca was once again instituted.
E) The Shi'as were able to take over the throne with the election of Ali.
6) Which of the following statements concerning the mercenary armies of the later Abbasid era is NOT accurate?
A) Mercenary troops became a disruptive force in the life of Baghdad and other cities.
B) They were consistently a major player in the factional contests for control of the capital and the empire.
C) They often consisted largely of slave troops.
D) Despite their tendency toward random violence, they loyally defended the Abbasid caliphs.
E) They helped foster the decline of the empire.
7) What accounts for the disruption of the agricultural economy of the Abbasid Empire?
A) The decline of the cities led to a fall in the demand for food supplies and consequent drops in agricultural prices.
B) Progressive desiccation of the region led to a diminution of the land available for agriculture.
C) Spiraling taxation, the destruction of the irrigation works, and pillaging by mercenary armies led to destruction and
abandonment of many villages.
39
D) The government ordered regions of the empire populated by Shi'as abandoned.
E) The ayan class began to import large numbers of slaves to work the land.
8) What was the attitude of the Abbasids toward the institution of slavery?
A) The Abbasid dynasty forbade slavery in general.
B) The dynasty permitted slavery of Muslims, but forbade the enslavement of members of other religions.
C) The Abbasid elite demanded growing numbers of both male and female slaves for concubines and domestic service.
D) The dynasty forbade all slavery except for the mercenary armies loyal to the Abbasids.
E) The local customs determined the status of slaves as stated in the Qur'an.
Answer: C
9) Which of the following statements concerning women during the Abbasid era is most accurate?
A) No Islamic women engaged in labor.
B) Women often married at puberty, set at age nine.
C) Rich women had many career outlets in Islamic cities.
D) Abbasid women had vastly greater freedom than did women in the first century of Islam.
E) Women often practiced polygamy.
10) What was the impact of the Seljuk conquest of Baghdad on the Abbasid Empire?
A) The empire continued to crumble as a result of the military successes of Fatimid Egypt and the Byzantine Empire.
B) The imposition of a Christian government in the name of the Abbasid caliphs temporarily restored order.
C) The Seljuks abandoned the Middle East for further conquests in the Indian subcontinent.
D) It restored the ability of the empire to meet the challenges of Egypt and the Byzantine Empire.
E) It allowed the crusaders to conquer Egypt and Jerusalem with little difficulty.
11) What accounts for the success of the First Crusade?
A) The overwhelming military superiority of Western military technology
B) The contemporary emergence of the Christian Seljuk Turks in Baghdad
C) Muslim political fragmentation and the element of surprise
D) The support and cooperation of the Jewish community of the Holy Land
E) The Byzantine Empire provided naval support and the use of "Greek fire"
12) What was the impact of the Crusades on the Christian West?
A) Christians adopted military techniques, words, scientific learning, and Arabic numerals among other things.
B) Christians rejected most Muslim influence, although they did gain a taste for Muslim wines and liquors.
C) There was no Muslim influence on the Christian West.
D) The Crusades interrupted the trade of the Mediterranean and cut off the West from Islam until 1293.
E) The Crusades led to an extension of feudalism through the use of Islamic silver.
13) What was the impact of the Crusades on Islam?
A) The Muslims adopted military technology, words, and scientific knowledge among other things from the West.
B) Although they resisted most influence, the Muslims did acquire a taste for Western cuisine.
C) In the long run, there was little impact on Islamic culture and society.
D) The Crusades temporarily cut off all exchange between the West and Islam.
E) Islam fell into decline and was eclipsed by the Mongols.
14) What was the trend of urbanization during much of the Abbasid Empire?
A) Because the Abbasids abandoned Baghdad for other capitals, cities within the empire tended to wither and die.
B) Successive invasions led to a decline in urbanization with many city dwellers moving to the countryside.
C) Despite political disintegration and a declining agricultural sector, towns continued to grow rapidly.
D) Towns established in the early years of the dynasty were able to hold their own, but there was little growth.
E) The Abbasids forcibly relocated people to the desert in such areas as Yemen.
15) What was the level of trade in the Abbasid Empire?
A) Long-distance trade with Africa, the Mediterranean, India, and China continued to flourish despite periodic
40
interruption.
B) Trade with the East grew, but the Crusades eliminated the western trade routes.
C) Trade with Africa and the Mediterranean continued to expand, but the wars in India disrupted the eastern trade routes.
D) As a whole, long-distance trade along the traditional caravan routes virtually ceased during the Abbasid Empire.
E) The economy shifted to a focus on trading only with other Muslims.
16) Which of the following statements concerning the ulama is most accurate?
A) It stressed an increasingly restrictive conservatism within Islam, particularly with respect to scientific inquiry.
B) It stressed withdrawal from the rest of the community of believers and the creation of Islamic monasteries.
C) It brought mysticism into Islam and placed less emphasis on evangelism.
D) It embraced the full consequences of fusing Greek and Quranic traditions willingly.
E) It favored a type of Byzantine worship stressing the use of icons with the picture of Muhammad.
17) Which of the following statements concerning the Sufi movement within Islam is most accurate?
A) The Sufi movement stressed an increasingly restrictive conservatism within Islam.
B) The Sufi movement stressed withdrawal from life and into monastic communities.
C) The Sufi movement incorporated mysticism with a trend toward evangelism.
D) The Sufi questioned the Islamic interest in the Greek traditions in science.
E) Sufism was a rationalistic movement based on the teachings of Ibn Khaldun.
18) How did the political center of Islam change after the Mongol invasions?
A) Baghdad remained the capital of Islam, but under the control of successive Mongol dynasties.
B) The center of Islam passed with the withdrawal of the invaders into the steppes of central Asia.
C) Baghdad was supplanted by Cairo to the east and soon thereafter Istanbul to the north.
D) The political center of Islam was removed to sub-Saharan Africa.
E) Mongolian became the language of politics and commerce.
19) What was the difference between the Islamic invasions of India and previous incursions of the subcontinent?
A) With the Muslims, the peoples of India encountered for the first time a large-scale influx of invaders with a civilization
as sophisticated as their own.
B) With the Muslims, the peoples of India encountered for the first time an invasion from the west rather than the east.
C) The Muslims were rapidly able to unify all of India into a single empire.
D) The Muslims, unlike previous invaders, bypassed the Gangetic plain in preference for southern India.
E) Islam proved to be a temporary setback to India but was quickly replaced by Buddhism.
20) How did Islam and Hinduism differ?
A) Hinduism stressed the egalitarianism of all believers, while Islam was more rigid in terms of orthodox belief.
B) Islam stressed the egalitarianism of all believers, while Hinduism was more rigid in terms of orthodox belief.
C) Islam stressed the egalitarianism of all believers, while Hinduism embraced a caste-based social system.
D) Hinduism stressed the egalitarianism of all believers, while Islam embraced a caste-based social system.
E) Hinduism was monotheistic, while Islam was polytheistic but both had a supreme God.
21) How did the Muslim conquerors of Sind treat the Hindu and Buddhist residents of the region?
A) All non-Muslims were put to death but their children were allowed to convert.
B) Hindus and Buddhists were treated as dhimmis or "peoples of the book."
C) Most of the higher-caste Hindus were treated well, which led to mass conversions.
D) Forced conversions of the native population followed the Muslim military successes.
E) Hinduism was prohibited and Buddhism tolerated, but many Hindus were encouraged to move back to Islamic
heartlands such as Iran.
22) What was the most critical cultural advance as a result of the increased contact between Muslims and Indian
civilization?
A) Muslims adopted the Hindu pantheon of gods.
B) Muslim commerce was increasingly dominated by the merchant caste of India.
C) Muslims adopted the Indian system of mathematical notation.
41
D) Muslims adopted the highly stratified social system common in Indian civilization.
E) India adopted Islamic science and Sufi literary techniques.
23) Why were the Sufis effective missionaries within the Indian subcontinent?
A) They enjoyed the support of the Hindu princes because of their support for brahmin ritual.
B) In both style and message they shared much with Indian mystics and wandering ascetics.
C) They were supported by huge armies of Arabs who migrated to India in search of land.
D) They rejected low-caste Hindus in preference for converts among the brahmin elite.
E) Their message was totally new to India but appealing to brahmins.
24) How did Hinduism respond to the challenge of Islam?
A) Hindus abandoned their emphasis on many deities in favor of monotheism in the person of Shiva.
B) Hindus placed greater emphasis on the devotional or bhaktic cults of gods and goddesses such as Shiva and Vishnu.
C) Hindus converted to Islam in increasing numbers, until Muslims outnumbered Hindus in the subcontinent.
D) The brahmins accepted Islam as a variety of orthodox Hindu belief, while anticipating the incorporation of the Muslim
immigrants into the Indian caste system.
E) Hindus created a new warrior class and began a military campaign, which led to the eventual demise of Islam in India.
25) In general, how did Islam spread in southeast Asia?
A) Port cities were points of dissemination to other links in trading networks.
B) Most of southeast Asia was converted to Islam after the military victories of Qutb-ud-din Aibak.
C) Islam was carried to southeast Asia from China.
D) Trade to southeast Asia from Africa and Persia established Islamic centers on the mainland from which conversion
took place.
E) Isolated regimes were visited by dervishes and other Sufi sects.
26) What was the nature of Islamic religion that developed in Southeast Asia?
A) Because most of the missionaries were ulama from Arabia, the religion most closely resembled Islam as practiced in
the first generations after Muhammad.
B) Because Islam came to Southeast Asia from India and was spread by Sufi holy men, it developed a mystical nature that
incorporated much of indigenous religion.
C) Because Islam was carried to Southeast Asia from China, it bore many of the characteristics of Buddhism.
D) Because Islam was carried by conquering warriors from India, it rejected the native Buddhism and Hinduism in
preference for more conservative Islamic orthodoxy.
E) Because Islam adopted many Buddhist ideas, a large number of native groups in the area rejected it.
Essay Questions:
Abbasid Decline and the Spread of Islamic Civilization to South and Southeast Asia
1. Explain the reasons for the political disruption of the Abbasid Empire prior to the rise of the Buyids.
2. Describe the general economy of the later Abbasid Empire. Include a comparative evaluation of urban and rural
economies.
3. Discuss the religious trends of the Abbasid era and their impact on the expansion of Islam.
4. In what sense did the Muslim incursions into the Indian subcontinent result in Islamic influences on Hindus? To
what extent were Muslims affected by Indian culture?
5. By what means did Islam spread to Southeast Asia?
42
Chapter 8, African Civilizations and the Spread of Islam
Summary:
Africa below the Sahara for long periods had only limited contact with the civilizations of the Mediterranean and Asia.
Between 800 and 1500 C.E., the frequency and intensity of contacts increased. Social, religious, and technological
changes influenced African life. The spread of Islam in Africa was influenced both by indigenous and Islamic inspiration.
States like Mali and Songhay built on military power and dynastic alliances. City-states in western and eastern Africa
were tied to larger trading networks. African civilizations built less clearly on prior precedent than did other postclassical
societies. Older themes, such as Bantu migration, persisted. Parts of Africa south of the Sahara entered into the
expanding world network; many others remained in isolation.
Key Concepts:
African Societies Diversity and Similarities:





A wide variety of societies in Africa. This diversity meant political unity was difficult. Though universal states
and religions did not develop in Africa, universal religions from elsewhere did impact the region.
Many Africans lived in stateless societies, which were organized around kinship or other forms of obligation and
lacking concentration of political power.
Despite Africa’s remarkable diversity, some commonalities existed, including a common linguistic base (Bantu),
and a tendency toward animistic religions. Economic conditions varied by geographic region, and historians have
few reliable population numbers.
In the second half of the 7th century, Mohammad’s followers swept through North Africa and brought Islamic
influence. The Berbers, a people of the Sahara desert, joined the Almoravid and Almohadi Islamic reform
movements, Launching into jihad against Spain and the savanna kingdoms of Africa. Islam’s attractive promise
of egalitarianism was not always fulfilled in practice, however.
Early Christian kingdoms developed in northeastern Africa (in particular, Nubia and Ethiopia) and resisted
Muslim encroachment for many centuries. These regions practiced a unique Coptic brand of Christianity.
Kingdoms of the Grasslands:






Trans-Saharan Muslim traders brought Islam to the Sahel grasslands of Africa. Ghana, which converted to
Islam by the 10th century, was the first, great West African empire.
The effective control of subordinate societies and the legal or informal control of their sovereignty are the usual
definition of empires. The Sudanic states of Ghana, Mali, and Songhay fit that definition.
Mali, created by the Malinke people in the 13th century, was an agricultural, Islamicized state that also depended
on gold reserves. Trade was facilitated by Malinke merchants, or juula. Griots, or Malian oral historians,
celebrated Sundiata, the founder of Mali’s empire, as did the noted Arab traveler Ibn Batuta. Mansa Musa,
perhaps Mali’s greatest ruler, made a famous pilgrimage to Mecca in 1324.
Timbuktu became a great trading and learning center, though most people in Mali relied on subsistence farming
to survive.
Songhay, founded by Sunni Ali and expanded by Muhammad the Great, was third great Sudanic state.
Songhay fell at the end of the 16th century. The Hausa kingdoms of northern Nigeria also combined pagan and
Muslim traditions.
The development of unified states provided an overarching structure that allowed the various groups and
communities to coexist. Sharia was not always followed (for example, women tended to have greater freedom in
the Suanic states) as pagan traditions fused with Islam. Muslim influence also meant that more Africans became
slaves than ever before.
The Swahili Coast of East Africa:

A string of Islamicized trading ports along Africa’s Indian Ocean coast increased that region’s contact with the
Arabic, Indian, Persian, and Chinese worlds.
43


Zenji (the Arabic word for the Swahili Coast) came under Muslim influence and many of its port towns were
thriving by the 13th century.
By the time the Portuguese gained control of Mozambique in the early 16th century, Swahili culture (a hybrid of
Bantu and Arabic language and custom was entrenched.
Peoples of the Forest and Plains:





Across central Africa, many preliterate agrarian societies thrived. Over time several kingdoms developed.
In Nigeria, the Nok culture reflected early artistic achievement; later, Yoruba culture was highly urbanized and
politically organized. East of Yoruba, the Benin city-state was powerful enough to impress Portuguese visitors.
South of the Zambezi River, beyond the influence of Islam, many central African peoples had begun their own
process of state formation by about 1000, replacing the pattern of kinship-based societies with forms of political
authority based on kingship.
Along the Congo River, the state of Kongo was spread out in family-based villages and towns. There was a sharp
division of labor between women and men.
Another large Bantu confederation, with extensive trade connections, developed east of the Congo. Its
headquarters were at Great Zimbabwe, and impressive set of stone structures.
Key Terms:
Juula
Ibn Batuta
Muhammad the Great
Berbers
Bantu
Timbuktu
Great Zimbabwe
Mali
Sudanic States
Mansa Musa
Chapter 8, Quiz Questions
1) Between 800 and 1500 as the frequency and intensity of contact with the outside world increased, which of the
following had the most significant impact on sub-Saharan Africa?
A) The arrival of the Portuguese
B) The arrival of Christianity
C) The arrival of Islam
D) The arrival of Chinese merchants
E) The arrival of the Mamluks
2) What was one of the major differences between African civilizations and other post classical societies?
A) African civilizations built somewhat less clearly on prior precedent than did other post-classical societies.
B) African civilization was almost entirely dependent on cultural importations from Islam and the Arabic world.
C) Prior to 800, African civilizations had no prior contacts with civilizations outside of the African continent.
D) There were no civilizations in Africa until after the post classical period.
E) African civilizations were based on hierarchal and patrilineal models, not democratic and matrilineal.
3) Which of the following statements concerning political and religious universality in Africa is most accurate?
A) Although a universal empire did not develop in Africa, Islam provided a principle of universality in the continent.
B) During the post classical period, Africa was politically united under a single government but remained religiously
diverse.
C) Universal religions found no adherents in Africa, a fact that helps to account for the failure of a universal political
system to develop.
D) Neither universal states nor universal religion characterized Africa, but both Christianity and Islam did find adherents
in Africa.
E) There were no similarities in the various African religious beliefs, which led to an easy acceptance of universal
religions.
4) African societies organized around kinship or other forms of obligation and lacking the concentration of political power
and authority were referred to as
A) stateless.
44
B) pastoral groupings.
C) Bantu conical clans.
D) Islamic tribes.
E) proto-empires.
5) What was the function of secret societies in African culture?
A) They smuggled valuable gold across the Sahara and established vital trade routes with the Mediterranean.
B) Because secret societies were restricted to females, they permitted women to have an invisible, but powerful, role in
political affairs within African societies.
C) Because their membership cut across lineage divisions, they acted to maintain stability within the community and
diminish clan feuds.
D) They served as a disruptive and revolutionary force in African society forestalling the formation of larger states.
E) They brought the practice of slavery to the African continent, which led to the eventual adoption of Christianity by a
majority of African tribes.
6) Which of the following statements best describes the indigenous religion of much of sub-Saharan Africa?
A) Much of sub-Saharan Africa was Christian but later converted to Islam under military pressure.
B) Animistic religion, belief in the power of natural forces personified as deities, characterized much of Africa.
C) African religion prior to the arrival of the Muslims was typified by an independent form of monotheism characterized
by worship in monumental temple complexes.
D) Uniquely, African societies lacked any religious principles prior to the arrival of the Christians and Muslims.
E) Sub-Saharan groups were influenced by Hindu beliefs brought in through east African trading ports such as Kilwa.
7) Which of the following statements concerning the economies of Africa is NOT correct?
A) North Africa was fully involved in the Mediterranean and Arab economic world.
B) Settled agriculture and iron working had been established in many areas before the post classical period.
C) Trade was handled by professional merchants, often in kinship groupings.
D) Much of the region lacked a market economy and was based on self-sufficient agricultural units.
E) Stateless societies found it difficult to maintain long-distance trade because of external conflicts.
8) Which of the following was NOT a belief shared by practitioners of many indigenous African religions?
A) The idea of a creator deity
B) A moral code based on one supreme god
C) The veneration of ancestors
D) The view that the land had religious significance
E) Well-developed concepts of good and evil
10) What does the phrase "equality before God and inequality within the world" mean?
A) It refers to the indigenous African religions' emphasis on social equality.
B) It refers to the powers exercised by the African rulers.
C) It refers to the equality that all clan heads enjoyed within African society.
D) It refers to the disparity between law and practice in many African societies
E) It means that Africans favored a variety of socialism.
11) Which of the following statements concerning the Sudanic states of Mali and Songhay is most accurate?
A) Although powerful, the Sudanic states never reached the level of empires.
B) Sudanic states had territorial core areas in which the people were of the same ethnic background, but their power
extended over subordinate communities.
C) What provided the cultural unity necessary for the establishment of states in the Sudan was the conversion of many
people to Christianity.
D) The Sudanic states were distinguished from other African civilizations by the peculiar lack of family or clan lineages
as an organizing principle of society.
E) All the Sudanic populations converted to Islam upon their first contact with Islamic military forces.
12) Which of the following statements concerning the social organization of the Malinke people is most accurate?
45
A) Following their conversion to Islam, the Malinke removed the worst aspects of social stratification in order to achieve
social equality.
B) The Malinke recognized only two sorts of people, those who were free and slaves.
C) The Malinke regarded all members of the tribal group as children of the ruler and thus essentially equal.
D) Malinke society was divided into three groups, clans of freemen, people devoted to religion, and specialists and
tradesmen.
E) There was always a sharp division between urban, rural dwellers and outside groups such as Muslim traders.
13) The African traders associated with the Mali Empire were called
A) Soninke.
B) Zimbabwe.
C) Jenne.
D) Juula.
E) Griots.
14) What was the social and political function of the griots?
A) Griots were religious diviners whose function was to foretell the future and guide the decisions of kings.
B) Griots were the classes of people of the conquest states of the Mali kings who were consigned to labor within the
empire's mines.
C) Griots mastered the oral traditions of the Malinke and by knowing the past were considered excellent advisors of kings.
D) Griots were Malinke merchants who served as trade middlemen throughout Africa and most of the Islamic world.
E) Griots were Islamic religious leaders in African empires and as such, played a powerful role in government.
15) What was the nature of urbanization within the Mali Empire?
A) As a conquest empire, Mali possessed garrison cities for its soldiers, but failed to develop commercial centers.
B) Mali possessed "port cities" along the Niger River such as Jenne and Timbuktu, which flourished both commercially
and culturally.
C) The "cities" of Mali were essentially religious and palace complexes that lacked populations of specialists other than
men devoted to religious observances.
D) Mali failed to develop cities prior to the rise of a strong monarchy under Sundiata.
E) Few mosques were ever built in Malinke cities, which led to a gradual decline in urban areas.
16) Why was Islam so readily adopted by rulers within the Sudan?
A) They were all conquered by overwhelming Muslim armies and forcibly converted to Islam.
B) The Muslim concept of a ruler who united civil and religious authority reinforced traditional ideas of kingship.
C) The Muslim concept of religious equality allowed rulers to dispose of the traditional clans and lineages of Africa.
D) As a monotheistic religion, Islam was much like the traditional religions of Africa.
E) Their conversion had been prophesied for many years by the griots who were considered the most important advisors
for the monarchy.
17) What was the relationship between Islam and the indigenous religions of Africa?
A) Islamic teachers attempted to eradicate the animist indigenous religions of Africa.
B) Because both indigenous African religion and Islam were monotheistic, the two became inextricably intertwined.
C) Islam was able to accommodate pagan practices and beliefs in the early stages of conversion
D) Islam successfully overcame indigenous religious beliefs, and almost all Africans converted to Islam.
E) Islam was less flexible than Christianity in accepting the styles and forms of native worship.
18) How did contact with the Muslim world affect the African slave trade?
A) Because of the Muslim emphasis on equality of all believers, early Muslim rulers suppressed the slave trade.
B) Slavery was unknown in African society until the Muslims introduced it.
C) Slavery became a more widely diffused phenomenon and the slave trade developed rapidly.
D) Despite the Muslim acceptance of slavery and its widespread use in Islamic society outside of Africa, Muslims
generally refused to accept black slaves.
E) Muslims forced southern Africans to give up slavery.
46
19) How was the institution of slavery viewed in Muslim society?
A) In theory, slavery was seen as a stage in the process of conversion of pagans to Islam.
B) Slavery was believed to be a permanent condition that rendered the enslaved incapable of entering heaven.
C) Slavery was viewed as so demeaning that those who were enslaved were good for nothing beyond labor in the fields or
the mines.
D) Slavery was eradicated in Islamic society because of the emphasis on the equality of believers.
E) Slaves could never be used as eunuchs or concubines but instead, had to convert to Islam before they could be sold
again.
20) What was the common cultural trait of the urbanized trading ports of the east African coast?
A) Membership in the Soninke tribe
B) The artistic style of the Nok culture
C) Bantu-based and Arabic-influenced Swahili language
D) A single ruling family from the Malinke tribe
E) A common descent from refugees from Bagdad
21) How did the expansion of Islam aid in the creation of international trade on the east African coastline?
A) Islam expanded to India and southeast Asia providing a religious bond of trust between those regions and the
converted rulers of the cities of east Africa.
B) Because Islam regarded Christians as peoples of the book, Muslim merchants came to trade at the Christianized ports
of east Africa.
C) The connection with the Islamic states of north Africa permitted the urbanized ports of east Africa to trade widely with
northern Europe.
D) The direct trade routes between the African states of west Africa and the coast of east Africa stimulated commerce
between the cities of the east and the Atlantic Ocean.
E) Islamic merchants established banking houses on the coast, which in turn, led to an expansion of trade with the African
interior.
22) To what extent did Islam successfully penetrate the populations of east Africa?
A) As in Asia, the conversion of the coastal cities led to almost complete conversion of the populations lying inland.
B) Most clans and the most important lineage groups outside of the east African cities were converted to Islam in order to
participate in trade.
C) Islam penetrated very little into the interior among the hunters, pastoralists, and farmers, and even the areas near the
trading towns remained relatively unaffected.
D) Islam spread from the cities to the hinterland in such a way that eventually the countryside was more Islamic than were
the cities.
E) Few literate people converted to Islam but the vast majority of herders converted rapidly.
23) What was the impact of the Portuguese arrival on the trading patterns of the east African coast?
A) Despite great effort to shift the focus of trade into their hands, the Portuguese were never able to control trade on the
northern Swahili coast.
B) The Portuguese arrival disrupted normal trade lines so severely that African trade with India and southeast Asia ceased
to exist for centuries.
C) The Portuguese rapidly used military superiority to control all aspects of trade along the east African coast.
D) The Portuguese allied themselves with Christians from Ethiopia in a combined assault on the east African coast.
E) The Portuguese bypassed the Swahili coast on their way to more valuable trade in Java.
24) The "Demographic Transition" is often associated with what change in society?
A) The militarization of society
B) The adoption of private property
C) Industrialization
D) Conversion to Christianity
E) Agricultural revolution
25) Which of the following states represents the development of Bantu concepts of kingship and state-building?
47
A) Mali
B) Kingdom of Kongo
C) Songhay
D) Mombassa
E) Ghana
26) Which of the following statements concerning the impact of Islam on sub-Saharan Africa is most accurate?
A) Islam cut off north Africa from the regions of sub-Saharan Africa.
B) The spread of Islam into Africa brought large areas into the global community.
C) With the conversion of regions of the continent to Islam, Africa became the center of the Islamic world.
D) Despite widespread conversion of Africans to Islam, the continent remained outside the trading sphere of the Islamic
world.
E) African development can be understood solely in terms of Islamic influences.
Essay Questions:
African Civilizations and the Spread of Islam
1. What were the common elements of African society prior to the incursion of Islam?
2. By what means was conversion of sub – Saharan Africa carried out? What were the primary avenues of Islamic
entry into sub – Saharan Africa?
3. Islamic influence was strongest in the Sudanic states and the Swahili coast. Compare and contrast the political
forms of these regions. What does this suggest about the nature of Islamic influence in Africa?
4. Compare and contrast the political forms of the African states not affected by Islam (Yoruba, Benin, Kongo,
Zimbabwe) with those that were (the Sudanic states and the Swahili coast).
5. Given the geographical location of those African states most affected by Islam, what were the most important
points of contact between African and Islamic societies? What does this suggest about the nature of Islamic
Conversion?
48
Chapter 9, Civilization in Eastern Europe: Byzantium and Orthodox Europe
Summary:
In addition to the great civilizations of Asia and North Africa forming during the postclassical period, two related major
civilizations formed in Europe. The Byzantine Empire, in western Asia and Southeastern Europe, expanded into Eastern
Europe. The other was defined by the influence of Catholicism in western and central Europe. The Byzantine Empire,
with territory in the Balkans, the Middle East, and the eastern Mediterranean, maintained very high levels of political,
economic, and cultural life between 500 and 1450 C.E. The empire continued many Roman patterns and spread its
Orthodox Christianity civilization through most of Eastern Europe, Belarus, Ukraine, and Russia. Catholic Christianity,
without an imperial center, spread in Western Europe. Two separate civilizations emerged from the differing Christian
influences.
Key Concepts:
The Byzantine Empire:






Constantine built Constantinople as his capital in the 4th century, and the city became the capital of the Byzantine
Empire when the Roman Empire split. Complex administration around a remote emperor, who was surrounded
by elaborate ceremonies, increasingly defined the empire’s political style. Eventually Greek became the empire’s
official language.
Justinian’s positive contributions to the Byzantine Empire lay in rebuilding Constantinople, including the
remarkable Hagia Sophia, and systematizing the Roman legal code. His 6th century military gains (made with
the help of his general, Belisarius) were accomplished at great cost.
Justinian’s successors were able to hold off Arab invaders from the east (“Greek fire” was instrumental in this
process), but the empire’s size and strength was greatly reduced. The empire also successfully defeated
challenges from Bulgaria.
The Byzantine political system had remarkable similarities to the earlier patterns in China. The emperor was held
to be ordained by God, head of church as well as state. Women could (and did) serve as emperor. An
elaborate bureaucracy organized the empire militarily, socially and economically, while cultural life blended
Hellenism and Orthodox Christianity.
In 1054 longstanding disagreements came to a head, and the church split into two traditions: one Western (or
Roman Catholic), and one Eastern (or Orthodox).
The Byzantine Empire entered a long period of decline following the church schism. It was able to survive by
careful diplomacy until Turks conquered Constantinople in 1453.
The Spread of Civilization in Eastern Europe:






Christian missionaries like Cyril and Methodius helped bring Orthodoxy northward into Russia and the Balkans,
and created a new alphabet, Cyrillic.
Roman Catholicism also competed for converts in Eastern Europe. Jews, who valued education and literacy,
migrated into the region in large numbers, gaining strength in local commerce.
Kievan Rus’, which began along the trade route between Scandinavia and Constantinople, gained influence and
power after Vladimir converted to Orthodox Christianity around 1000. The state soon developed its own Russian
Orthodox Christianity around 1000. The state soon developed its own Russian Orthodox Church, and Yaroslav
issued a unifying code of laws.
Russian culture borrowed much from Byzantium, though the bureaucracy and education system were not as
developed. Boyars, the Russian nobility, were less powerful than their western counterparts.
Mongol invasions (the Tartars), aided by rival princes and the fall of the Byzantine Empire, ended this period of
Russian history.
This cut the region off from western contacts, stifling economic, political, and cultural sophistication.
Key Terms:
Byzantine Empire
Constantinople
Boyars
49
Hagia Sophia
Roman Legal Code
Justinian
Sassanian Empire
Cyril and Methodius
Russian Orthodoxy
Tartars
Chapter 9, Quiz Questions
1) Which of the following is most correctly seen as a direct continuation of the Roman Empire?
A) Frankish Empire
B) Ottoman Empire
C) Byzantine Empire
D) Abbasid Empire
E) Holy Roman Empire
2) The significance of the Byzantine Empire included all of the following EXCEPT
A) the empire's ability to survive for almost a thousand years.
B) the importance of the empire's capital at Constantinople as a major urban center.
C) the ability of the empire to spread its cultural and political influence to the Balkans and southern Russia.
D) the empire's conquest of the Ottoman Empire and its inclusion of all of the Middle East.
E) its development of Orthodox Christianity, which broke off from Rome in 1054.
3) What was the most important "stepchild" of the Byzantine civilization?
A) Italy
B) Bulgaria
C) Russia
D) Poland
E) Greece
4) Which of the following represents a difference between the spread of civilization in eastern and western Europe?
A) They produced different versions of Christianity, culturally as well as organizationally separate.
B) Only eastern Europe developed north-south commercial ties.
C) Centralized government and well-organized bureaucracy was more a feature of western Europe than eastern Europe.
D) Eastern Europe retained less fully the culture of the later Roman Empire than did the West.
E) Only the East faced the threat of an Islamic invasion.
5) Which of the following does NOT represent a similarity between the spread of civilization in eastern and western
Europe?
A) Civilization spread northward from a Mediterranean base.
B) Animism gave way to monotheism.
C) Northern kingdoms dominated both areas.
D) In both cases newly civilized areas looked back to the Greco-Roman past.
E) Christianity was spread in both cases.
6) What was the difference in the military organization of Byzantine and western Roman empires?
A) The western Roman Empire depended on citizen soldiers until the 5th century.
B) The Byzantine Empire recruited armies from the Middle East.
C) The Byzantine Empire recruited barbarians almost exclusively while the Roman Empire of the West depended on
Islamic mercenaries.
D) The Byzantine Empire depended on the strength of Constantinople's walls and did not recruit an army.
E) The Byzantine Empire gave higher status to the infantry than the West did.
7) The Byzantine Empire began
A) in the 9th century C.E., with the missionary work of Cyril and Methodius.
B) in the 4th century C.E., with the building of Constantinople.
C) in the 1st century C.E., during the reign of Augustus.
D) in the 5th century C.E., with the fall of Rome.
E) in the 11th century C.E., with the first Crusades.
50
8) Which of the following was a result of the conflict between the Byzantine Empire and the Arab Muslims?
A) The Arab threat to the Byzantine Empire was permanently removed.
B) The position of small farmers in the empire was weakened as a result of heavy taxation, resulting in greater aristocratic
estates.
C) The Byzantine Empire was able to recover the provinces of Syria and Egypt, thus regaining valuable agricultural land
and increased wealth.
D) The commercial significance of Constantinople was destroyed by the 8th century, forcing the Byzantine Empire to
depend increasingly on trade with the West.
E) The Byzantines made an alliance with Kievan Rus, which led to greater cooperation between the two empires.
9) Which of the following conclusions that might be drawn about the Byzantine Empire as the result of its conflicts with
its neighbors is most true?
A) Despite all of its difficulties, its wars with its neighbors demonstrate that the empire had real core strength.
B) The outcomes of the wars demonstrate that by the 10th century the empire was completely decadent and incapable of
defending itself.
C) The wars with Islamic powers demonstrate the inherent weakness of the Byzantine military.
D) The role of Theodora and other women in the wars demonstrate that the intrusion of women into Byzantine politics
was highly destructive.
E) The Byzantine Empire continued to lose tax revenues through the loss of territory in the Balkans.
10) In which of the following ways were the Byzantine bureaucracy and the Chinese bureaucracy similar?
A) There was an extensive state exam system in both.
B) Emperors played little role in either government.
C) There was no linkage of the bureaucracies to local administration.
D) Both bureaucracies were open to talented commoners, not just aristocrats.
E) They were driven by the authority of the religious authorities.
11) Which of the following statements concerning the Byzantine bureaucracy is NOT accurate?
A) Many of the officials closest to the emperor were eunuchs.
B) Aristocrats predominated, but there was some openness to talent.
C) An elaborate system of spies maintained loyalty to the central government.
D) Bureaucrats had limited education and ability.
E) Provincial governors kept tabs on the military.
12) Which of the following statements concerning urbanization within the Byzantine Empire is most correct?
A) Constantinople controlled the economy and grew to enormous size, but other cities were relatively small.
B) Constantinople began to decline in population in the later years of the Byzantine Empire and was surpassed by the
growth of other urban centers.
C) Like China, the Byzantine Empire was heavily urbanized with many cities numbering more than 100,000.
D) Most people in the Byzantine Empire lived in five great cities: Constantinople, Nicaea, Smyrna, Rome, and Athens.
E) Constantinople was never a center of trade, but dominated the empire culturally.
13) Which of the following statements concerning the merchant class of the Byzantine Empire is most accurate?
A) Because of its lack of export products, Byzantine commerce was controlled completely by foreign merchants.
B) Byzantine merchants, because of their wealth, rapidly became the most powerful force in the government of
Constantinople and the empire.
C) The Byzantine merchant class never recovered from the loss of territories to the Muslims and was not a significant
factor within the empire.
D) There was a large and wealthy merchant class in the Byzantine Empire, but it never gained significant political power
because of the elaborate network of government controls.
E) By comparison to western Europe, the merchant class was relatively small but was very influential in Byzantine
political circles.
14) Why did Vladimir I prefer Orthodox Christianity to Roman Catholicism?
51
A) He preferred to avoid the pitfalls of the veneration of icons.
B) He believed that Roman Catholicism implied papal interference, while Orthodoxy embraced the control of the church
by the state.
C) He was not familiar with Roman Catholicism, because the Western form of Christianity had not penetrated into eastern
Europe.
D) He did not believe in clerical celibacy, which was required of the Roman Catholic priesthood.
E) He rejected the Roman Catholics' emphasis on the sacraments and the use of the trinity.
15) What was the extent of western European influence in Kievan Russia?
A) Despite the adoption of Orthodox Christianity, most of the ecclesiastical influences were Roman Catholic.
B) There was direct Western influence in Russia in both music and art forms.
C) Like western Europe, Russia also moved away from the popular veneration of icons and representational images.
D) Russia's religious culture and social and economic patterns developed separately from western Europe's.
E) Russia borrowed many customs and artistic forms from both Rome and the Byzantines.
16) Which of the following statements concerning the Tatar invasion of Russia is most accurate?
A) Tatar control of Russia lasted for four decades.
B) The Tatars used Russia as a springboard for their successful invasion of western Europe.
C) Tatar supervision did not destroy Russian Christianity or a native Russian aristocracy.
D) The Tatars rapidly devised a closely supervised local administration for the Russian cities.
E) Russia became a Mongol possession permanently.
Essay Questions:
Civilization in Eastern Europe: Byzantium and Orthodox Europe
1. Compare and contrast the spread of European civilization in eastern and Western Europe.
2. What was the political organization of the Byzantine Empire?
3. What were the factors in the decline of the Byzantine Empire?
4. In what ways was the culture of Kievan Russia an extension of the Byzantine Empire?
52
Chapter 10, A New Civilization Emerges in Western Europe
Summary:
The Postclassical period in Western Europe, known as the Middle Ages, stretches between the fall of the Roman Empire
and the 15th century. Typical postclassical themes prevailed. Civilization spread gradually beyond the Mediterranean
zone. Christian missionaries converted Europeans from polytheistic faiths. Medieval Europe participated in the emerging
international community. New tools and crops expanded agricultural output; advanced technologies improved
manufacturing. Mathematics, science, and philosophy were stimulated by new concepts.
Key Concepts:
Stages of Postclassical Development:










The postclassical West suffered from several key problems, including the political fragmentation of Italy, the
Muslim conquest of Spain, Viking raids, and a decline in intellectual life.
Effective political organization was usually local, and manorialism created an oppressive system of political and
economic organization between landlords and peasants, many of whom were serfs. The moldboard and threefield system gradually improved agriculture.
The Christian church was the most organized institution in Western Europe. It had a relatively clear hierarchy
and established a chain of monasteries. Clovis’s conversion to Christianity helped him gain power over the
Franks.
Charlemagne established a substantial empire in France and Germany around the year 800, but his empire did
not survive his death as Europe split increasingly into regional monarchies.
Agricultural advances, population increases, and growing towns helped breathe economic and cultural vitality into
Europe after 900.
The key military and political system in the Middle Ages was feudalism, a system in which greater lords
provided protection and aid to lesser lords, called vassals; vassals, in turn, owed their lords military service, some
goods or payments, and advice. The introduction of feudal monarchy generally took time, though and advice.
The introduction of feudal monarchy generally took time, though William the Conqueror introduced feudal
monarchy to England following his invasion in 1066.
Feudal lords cut into aristocratic power. In 1215 noblemen forced King John to sign the Magna Carta, and
parliaments swerved as further checks on royal authority.
Western Europe’s expansionist impulse led to the conquest of Spain, the first European steps in the Americas,
and (spurred on by pope Urban II) the Crusades. Though the Crusades ultimately ended in defeat, they opened
Western Europe’s eyes to new possibilities, particularly for trade.
Several important reforms impacted the church, including those advocated by Franciscans, the Order of Saint
Clare, and pope Gregory VII. The investiture controversy helped give the church power over monarchies.
Several key creative tensions characterized the High Middle Ages.
Western Culture in the Post-Classical Era:



Theologians like Peter Abelard and Thomas Aquinas helped to assimilate Greek philosophical ideas into the
Catholic religious tradition. Bernard of Clairvaux opposed this process, but ultimately scholasticism prevailed.
Though some pagan traditions continued, Christian devotion among lay people increased.
Medieval architecture, literature, and art reflected religious themes. Gothic cathedrals rose up around western
Europe.
Changing Economic and Social Forms in the Post-Classical Centuries:


Economic activity and social structure developed innovative common features around Western Europe, and the
region became a growing commercial zone.
Peasants gained more freedom with agricultural advances.
53


Urban growth allowed more specialized manufacturing and commercial activities, which, in turn, promoted
still greater trade. The Hanseatic League is perhaps the best example of cities working together for mutual
economic benefit. Guilds grouped people in the same business or trade in a single city, stressing security and
mutual control.
Men placed new limits on the condition of women, and patriarchal structures seemed to be taking deeper root.
The Decline of the Medieval Synthesis:


The devastation and anti-feudal innovations of the 14th century’s Hundred Years War suggested that change was
at hand. Agricultural resources could no longer sustain increasing population, and devastating plagues like the
Black Death swept Europe. The growth of professional armies shook the authority of feudal lords, who turned
to a ceremonial style of life. A series of controversies over papal authority distanced the church from everyday
devotion. Intellectual and artistic life gradually moved out form under Church influence.
The legacy of the Medieval period includes academic institutions, political ideas, and a marked change in the
relationship between the West and the regions around it.
Key Terms:
Three Estates
Scholasticism
Magna Carta
Charlemagne
Three-Field System
Manorialism
Romanesque
Pope Urban II
Vikings
Ferdinand & Isabella
Chapter 10, Quiz Questions
1) Which of the following statements concerning the impact of Christianity on polytheistic religions in western Europe is
most accurate?
A) Christianity eradicated all traces of those earlier religions as the new religion became universal in western Europe.
B) The process of conversion produced a religious blend in which beliefs in magic and supernatural spirits coexisted with
Christianity.
C) Although Christianity made inroads, many areas of Europe retained polytheistic beliefs and rejected the new religion.
D) Small islands of polytheistic belief remained, but most Europeans converted from polytheistic faiths in the initial postclassical centuries.
E) Few polytheistic religions existed in Europe during the Middle Ages, but their influences grew.
2) Which of the following statements concerning the intellectual activity of the medieval West prior to the 8th century is
most accurate?
A) Classical rational traditions were actively united with Christian mysticism to carve out a new intellectual world.
B) With the few literate people concentrated in monasteries, little was achieved other than copying older manuscripts.
C) Universities rapidly created a new intellectual climate in which logic was applied to matters of Christian doctrine.
D) All literacy and contact with the ancient culture was lost in the centuries following the fall of Rome.
E) Western scholars achieved more during this period than their Islamic counterparts.
3) Which of the following statements about the manorial system is NOT true?
A) It was technologically sophisticated.
B) It had originated in the Roman Empire.
C) Its obligations bore heavily on serfs.
D) Agricultural productivity was low.
E) It was practically self-sufficient.
4) Which of the following statements concerning the three-field rotation system is most accurate?
A) Introduced in the 8th century, the three-field rotation left a third of the land unplanted to regain fertility,
B) The three-field system removed more land from production than before by reserving one-third for fallow.
C) The three-field system was rapidly replaced after the 8th century by the two-field system that offered greater flexibility
in terms of crop rotation.
D) The three-field system removed fallow fields and replaced them with nitrogen-bearing crops.
54
E) It represented a net increase in productivity where it was used because although crops were rotated, no fields were left
fallow.
5) What belief did the conversion of Germanic kings create among Western religious leaders, particularly the pope?
A) That the Church was subordinate to the secular monarchs
B) That the Church was superior to the secular rulers
C) That the Church should avoid conversion of northern Germanic kings
D) That such conversion represented a danger to the papal hierarchy
E) That the church should fear powerful kings
6) Which of the following was NOT a benefit of the monastic movement in western Europe?
A) They disciplined the intense spirituality of the medieval West in order to promote Christian unity.
B) Many monasteries helped improve the cultivation of the land.
C) By copying ancient texts, monks preserved classical culture for later intellectual inquiry.
D) Their political organization provided the foundation for the political order established in France, Germany, and
England.
E) Monks exemplified holy lives for church members and church leadership.
7) Which of the following statements concerning the Holy Roman Emperors after the 10th century is most accurate?
A) They built upon the Carolingian foundations to establish the most centralized government found in the medieval West.
B) Building on a feudal framework rather than the Carolingian Germanic foundations, the Holy Roman emperors created
a strongly centralized government.
C) Discarding much of the former Carolingian Empire, the Holy Roman emperors reestablished a centralized government
in northern Italy.
D) The rule of the Holy Roman emperors became increasingly hollow, because they did not build a solid monarchy from
regional foundations.
E) Dukedoms and city-states yielded political authority to the Holy Roman Empire throughout western Europe.
8) Which of the following was NOT a positive development that introduced new sources of strength by the 9th and 10th
centuries to western Europe?
A) New agricultural techniques
B) End of Viking raids
C) Development of imperial government
D) Greater regional political stability
E) Towns served as trade centers
9) What was the impact of the improved economy after the 10th century on the social system of western Europe?
A) The improvements in the agricultural system retarded the development of towns and restricted social mobility.
B) Harsh serfdom became the rule throughout western Europe.
C) The increased pace of economic life created a less rigid structure.
D) Despite the improved economy, the rigid social system associated with feudalism continued to dominate western
Europe.
E) The merchants lost considerable power to the monarchs.
10) Which of the following statements about feudalism is most accurate?
A) Although it inhibited the development of strong central states, some kings were able to use feudalism to build their
own power.
B) Although it provided initial political stability, feudalism was rapidly replaced by a western European imperial system.
C) Feudalism represented only a brief, and largely unsatisfactory, attempt to create political stability in western Europe.
D) Feudalism produced centralized monarchies by the 8th century.
E) Feudalism caused rapid economic gains in all parts of Europe including England.
11) Which of the following was NOT a power of the papacy immediately after 500?
A) The ability to send directives and receive information
B) Regulation of doctrine or dogma
55
C) Sponsorship of missionary activity
D) The appointment of all bishops
E) Excommunication
12) How did the introduction of feudal monarchy into England compare to the political experience of France?
A) English feudal monarchy developed more gradually and slowly in response to the improving economy.
B) English feudal monarchy was introduced abruptly following 1066, while French feudal monarchy developed more
slowly.
C) French feudal monarchy arose almost immediately in the 10th century as a result of the defeat of the Normans.
D) France failed to develop feudal monarchy until the 15th century unlike England.
E) England never developed a strong feudal monarchy similar to what happened in France.
13) Which of the following did NOT occur as a result of the Crusades?
A) The Kingdom of Jerusalem was established and controlled by the West for nearly a century in the Holy Land.
B) The Fourth Crusade resulted in the temporary conquest of Constantinople.
C) The Crusades helped to open the West to new cultural and economic influences from the Middle East.
D) The Crusades demonstrated a new Western superiority in the wider world.
E) As far as the Muslims were concerned, the Crusades had little impact on the Islamic world
14) In what way was the educational system of the medieval West different from that of China?
A) The West abandoned its classical heritage.
B) The universities were not tied into a single bureaucratic system.
C) In the West, there were no state bureaucracies to hire university graduates.
D) The West lacked a formal system of education.
E) University education was secular-based.
15) Which of the following developments was NOT a result of the improved economy of the High Middle Ages?
A) Urban growth allowed more specialized manufacturing and commercial activities, including banking.
B) Some peasants were able to throw off the most severe constraints of manorialism, becoming almost free farmers.
C) Rising trade permitted the redevelopment of commerce within the Mediterranean and beyond.
D) Conflicts between peasants and the landlords became rare, if they did not disappear altogether.
E) A money economy began to replace the traditional barter system used throughout western Europe.
16) All of the following were functions of the merchant and artisan guilds EXCEPT
A) limitation of membership.
B) regulation of apprenticeship.
C) ensuring a free-market economy.
D) guaranteeing good workmanship in their products.
E) giving its members an equal share in resources.
17) Which of the following was NOT true of the career of Jacques Coeur?
A) He used his wealth to arrange for his 16-year-old son to become an archbishop.
B) He died a rich and honored advisor to the king of France.
C) He was tortured, admitted to various crimes, and had his property confiscated.
D) He had the largest fleet ever owned by a French subject.
E) Coeur built an elaborate palace at Bourges.
18) Which of the following was a result of the Hundred Years War during the 14th and 15th centuries?
A) Kings reduced their reliance on feudal forces in favor of paid armies
B) An English victory, but only after an invasion of France by Richard the Lionhearted
C) Mounted knights continued their dominance over foot soldiers and archers
D) Major battles resulted in enormous loss of life over the course of the war
E) Physical destruction of both France and England, with loss of trade as well
19) Which of the following was NOT a threat to the sources of Western vitality at the end of the Middle Ages?
56
A) The Black Death
B) The increasing inability of agriculture to keep pace with population growth
C) The economic tail-spin and impending depression
D) New social disputes, involving both peasants and landlords and artisans and their employees.
E) Manufacturing and technology developed more quickly
20) Which of the following was NOT typical of the challenges to typical medieval institutions in the 15th century?
A) The landowning aristocracy lost its dominance as the chief military force
B) The balance between church and state began to favor the dominance of the state
C) The medieval intellectual and artistic synthesis was breaking down
D) A single imperial government replaced the smaller kingdoms of the Middle Ages
E) Parliaments limited the power of kings and provided major support to the merchant class
21) Which of the following statements concerning the medieval economy is most true?
A) Medieval economic thought and practice was of no consequence to later Western economic thinkers and actors.
B) Medieval economics was a combination of capitalistic and feudal practices
C) Medieval economics simply repeated the thought and practice of earlier economic thinkers.
D) Medieval economics, overall, tended to discourage merchant activity and technical innovation.
E) Medieval economic systems were based on global trade with China and India.
Essay Questions:
A New Civilization Emerges in Western Europe
1. Define monorialism and feudalism. How do they provide the building blocks for medieval political structure and
society?
2. What were the characteristics of feudal monarchy as demonstrated in France and England between 1000 and
1300?
3. How did the theological outlook of Western Europe change between 1000 and 1400?
4. What were the development that led to increases in monarchic power at the end of the Middle Ages? How was
royal authority limited?
57
Chapter 11, The Americas on the Eve of Invasion
Summary:
American societies during the postclassical era remained isolated form other civilizations. The societies continued to
show great diversity, but there were continuities. American civilizations were marked by elaborate cultural systems,
highly developed agriculture, and large urban and political units. Columbus’s mistaken designation of the inhabitants of
the America’s as Indians implies a nonexistent common identity. The great diversity of cultures requires concentration on
a few major civilizations, the great imperial states of Mesoamerica (central Mexico) and the Andes, plus a few other
independently developing peoples.
Key Concepts:
Postclassical Mesoamerica, 1000-1500 C.E.:






After the collapse of Teotihuacan, the Toltecs moved into the political power vacuum and established a culture
with a strong military ethic and a cult of human sacrifice and war.
Toltec influence spread over much of central Mexico. The legend of Topiltzin/Quetzalcoatl, which claimed that
a Toltec faction would one day return and claim the throne, was well known to the Aztecs (successors to the
Toltecs) and my have influenced their response when the Europeans arrived.
The Aztecs gained control of the important Lake Texcoco region in the post-Toltec era, and made their capital at
Tenochtitlan. The Aztecs had a reputation as tough warriors and fanatical followers of their gods.
By the time of Moctezuma II, the Aztec state was dominated by a king who represented civil power and served
as a representative of the gods on earth. The cult of human sacrifice and conquest was united with the political
power of the ruler and the nobility.
Aztec religion, which incorporated many traditional Mesoamerican elements, was a vast, uniting, and sometimes,
oppressive force in which little distinction was made between the world of the gods and the natural world. Major
deities included Tlaloc, god of rain, and Huitzilopochtli, the Aztec tribal god. Nezhualcoyotl, and Aztec king
and poet, promoted a kind of monotheism, but the idea did not last. Human sacrifice increased considerably.
To feed their people, the Aztecs used an ingenious and successful system of irrigated agriculture highlighted by
chinampas (beds of aquatic weeds, mud, and earth that had been placed in frames made of cane and rooted to the
lake floor). A special merchant class, the pochteca regulated markets, and the state oversaw a vast tribute
network.
Aztec Society in Transition:



As the empire grew, a new social hierarchy replaced the old calpulli (kinship based clan) system of social
organization.
The rights of Aztec women seem to have been fully recognized, but in political and social life their role, though
complementary to theat of men, remained subordinate. Lack of technology meant women were required to
spend significant time hand-grinding maize, a staple crop. The area controlled by the Aztecs may have included
20 million people.
Each city-state was ruled by a speaker chosen from the nobility. In many ways, the Aztec Empire was not unlike
the subject city-states over which it gained control. These city-states, in turn, were often left unchanged if they
recognized Aztec supremacy and met their obligations.
Twantinsuyu: World of the Incas:
 With a genius for state organization and bureaucratic control over peoples of different cultures and languages, the
Incas achieved a level of integration and domination previously unknown in the Americas.
 The coastal empire of Chimor preceded the Incas. With the help of their leader, Pachacuti, and his successors,
Twantinsuyu (the Incan Empire) spread from modern-day Columbia to northern Argentina.
 The Incas adopted the practice of royal split inheritance, which required new land and wealth. This may have
caused the empire’s growth. The Temple of the Sun at Cuzco was the center of Incan religious life.
 The Incas developed a state bureaucracy, headed by an inca and four regional governors (who, in turn, divided
their realms). They spread their language, Quechua, used colonists, and built extensive road networks (dotted
58


with tambos, or way stations) to encourage unity. The empire also demanded mita, mandatory labor on church
and state lands. Andean people practiced parallel descent. In addition to local ayllus (clans), a class of yanas
(people living outside their ayllu) provided important service. Though the empire was a masterpiece of statecraft,
a system of royal multiple marriages as a way of forging alliances created rival claimants for power and the
possibility of civil war on the eve of the Spanish invasion.
Incan cultural achievements included beautiful pottery, art, and metalworking, the quipu (a system of knotted
strings for recording numerical information) land and water management, extensive road system, statecraft, and
architecture.
The Incan and Aztec empires are best viewed as variations of similar patterns and processes, of which
sedentary agriculture is the most important. Basic similarities underlying the variations can also be seen in
systems of belief and cosmology and in social structure.
The Other Indians:




The diversity of ancient America forces us to reconsider ideas of human development based on Old World
examples.
Population figures are difficult to pin down, but in 1500 the Americas may have had roughly the same number of
inhabitants as Europe (between 57-72 million people).
Chieftanships based on sedentary agriculture could be found outside the major American empires. Cultural
diversity was particularly great in North America. Most American societies (outside the Incas and Aztecs) were
strongly kin-based, unlike in Europe and Asia.
The America contained a broad range of societies, from great civilizations with millions of people to small bands
of hunters. In many of these societies, religion played a dominant role in defining the relationship between
people and their environment and between the individual and society.
Key Terms:
Maize
Cuzco
Hernan Cortes
Chichen Itza
Inca Socialism
Temple of the Sun
Calpulli
Pochteca
Parallel Descent
Tlaloc
Chapter 11, Quiz Questions
Multiple-Choice Questions
1) Which of the following was NOT a characteristic of American civilizations during the post classical period?
A) Large cities based on elaborate political and economic organization
B) Elaborate cultural systems
C) Monotheistic practices
D) Highly developed agriculture
E) Diverse civilizations
2) What was the relationship between the Toltecs and their predecessors in central Mexico?
A) The former residents of central Mexico were wiped out during the Toltec invasions.
B) The entry of the Toltecs into central Mexico marks an abrupt break in the cultural development of the region.
C) The Toltecs adopted the animistic religion of their predecessors, but failed to develop cities or ceremonial centers.
D) The Toltecs adopted many cultural features from their predecessors to which they added a strong military ethic and
human sacrifice.
E) The Toltec rejected all economic and social practices but did use similar religious practices.
3) How did the Aztecs view the cultural achievements of the Toltecs?
A) As barbarians who lacked culture
B) As slaves, fit only for conquest
C) As the givers of civilization
D) As heretics, who practiced a forbidden religion
59
E) The Toltec culture was unknown to the Aztecs
4) Which of the following did NOT occur as a result of the Aztec rise to power?
A) The increasing dominance of the nobility
B) The emergence of a ruler with supreme powers
C) The abandonment of human sacrifice
D) The subsequent expansion of power and the boundaries of Aztec control
E) A highly urbanized society
5) What was the impact of expansion and conquest on the Aztec social system?
A) From a loose association of clans, Aztec society became a highly stratified society.
B) Conquest opened up Aztec society to incursions by the indigenous peoples who began to form a trained bureaucracy.
C) Aztec society was transformed in the sense that the Mexicans adopted the social patterns of the Maya.
D) Despite the stress of warfare and invasion, the Aztec society remained remarkably unchanged by the process.
E) Aztec social institutions became more inclusive leading to a more equalitarian society.
6) Which of the following statements concerning Aztec religion is most accurate?
A) Shortly after establishing their empire, the Aztecs abandoned all gods other than their patron, Huitzilopochtli.
B) Aztec deities were normally associated only with male forms.
C) Aztec deities were numerous and had different forms or manifestations somewhat like the avatars of the Hindu deities.
D) There was little or no animism in the religious world of the Aztecs.
E) Tonatiuh and Tezcatlipoca became the paramount gods along with Nezhualcoyotl.
7) Which of the following was NOT one of the major themes or cults of Aztec religion?
A) Gods of fertility
B) Creator deities
C) Gods of warfare and sacrifice
D) Deities devoted to contemplation and salvation
E) The agricultural cycle
8) What was the Aztec view of history?
A) They believed in a linear view of history dedicated to the premise of Aztec superiority for eternity.
B) Like other Mesoamerican peoples, the Aztecs believed in a cyclical pattern of repetitive destructions of the world.
C) Unlike other Mesoamerican peoples, the Aztecs rejected the cyclical view of history for a more modern historical view
based on the history of their empire.
D) Because they lacked a calendar system, the Aztecs had no formal historical viewpoint.
E) They believed in a linear history ending with their total destruction at the hands of the people from the East.
9) What was the nature of the Aztec economy?
A) The Aztecs failed to develop a merchant class, so all distribution of goods was carried out by the state.
B) The Aztecs developed a free-market economy in which all trade was in the hands of specialized merchants.
C) The Aztec state redistributed many goods received as tribute and there was a specialized merchant class.
D) There was little trade within Aztec society, as almost all communities were self-sufficient.
E) The Aztec traded the entire length of the inhabited Pacific coast as far south as the Inca empire.
10) Which of the following was NOT a function of the Aztec kinship groups?
A) Assignment of people to cult groups
B) Distributing land to household heads
C) Organizing labor units for state service
D) Organizing military units for warfare
E) Maintaining temples and schools
11) While the position of Aztec women in many ways paralleled that of women in other civilizations at a similar stage of
development, what was the significant difference between the lives of women in Mesoamerica and in the Mediterranean
world?
60
A) Women in Mesoamerica participated fully in the military.
B) There was no polygamy practiced in Mesoamerica.
C) Aztec women were unable to inherit or to pass property on to heirs.
D) Aztec women had to spend many more hours grinding grain for food.
E) Peasant women were more highly educated in Mesoamerica.
12) What was the nature of the Aztec administration of subject territories?
A) The Aztecs placed members of the Aztec nobility as rulers over subject peoples.
B) All territories conquered by the Aztecs became part of a singular administration run by a trained bureaucracy located in
Tenochtitlan, much like the Byzantine Empire.
C) Conquered territories were often left relatively unchanged under their old rulers as long as they recognized Aztec
supremacy and paid tribute.
D) The Aztecs established a military administration with subject territories controlled by regional generals.
E) Aztecs rarely collected tribute from subject territories but did take hostages for human sacrifice.
13) What was the primary difference in the political situation between the Mesoamerican and Andean zones following the
breakup of the classical states?
A) In Mesoamerica there was no real political decline as new peoples simply took over the institutions of their
predecessors.
B) In the Andean zones a number of relatively large states continued to be important, rather than the breakdown of power
that was typical of Mesoamerica.
C) The Andean political experience lacked the militaristic overtones that accompanied the breakdown of power in
Mesoamerica.
D) The transition of power that took place in the Andean zone was accomplished by invasion from outside the region.
E) The Andean region was more compact and therefore quickly reunited under the leadership of the Inca.
14) What do the authors suggest was the principal reason for Inca conquest and expansion?
A) Their need for human sacrificial victims
B) The practice of split inheritance
C) The absence of irrigation systems within the Inca empire
D) The existence of long-distance trade merchants within the imperial administration
E) The cult of the sun
15) What was the Inca practice of split inheritance?
A) On the death of the previous ruler, the throne passed to two descendants from the ruler's family.
B) On the death of the previous ruler, the family's wealth was equally divided among all male heirs.
C) On the death of the previous ruler, the inheritance passed through the family of the senior wife to her oldest brother.
D) All political power and titles went to the ruler's successor, but his wealth was kept in the hands of the male descendants
to support the cult of the dead Inca's mummy.
E) All power and wealth was inherited by the eldest son but religious leadership fell to the youngest daughter.
16) Which of the following was utilized in the Inca empire, but NOT by the Aztecs?
A) A semi-divine emperor
B) Extensive use of colonization
C) Use of local rulers in exchange for recognition of sovereignty
D) Identification of the nobility with the administrative and military functions of the state
E) Elaborate road systems
17) Which of the following existed in the Aztec empire, but NOT in the Inca empire?
A) A tribute system
B) Redistribution of tribute by the state
C) Extraction of labor for use on temple projects
D) An extensive merchant class
E) Recognition of local groups and leaders
61
18) The modern image of the Inca empire as a carefully organized system in which every community collectively
contributed to the whole and the state regulated the distribution of resources on the basis of need is referred to as
A) Inca despotism.
B) Indian utilitarianism.
C) Inca socialism.
D) utopian positivism.
E) Mesoamerican capitalism.
19) Which of the following views of the Aztec practice of human sacrifice can be associated with Marvin Harris's
interpretation of the "cannibal kingdom"?
A) The incidence of human sacrifice was grossly exaggerated by the Spanish for political purposes.
B) It was an essentially religious act central to their belief that humans must sacrifice that which was most precious to
them in order to receive the sun, rain, and other blessings.
C) It was the intentional manipulation of a traditional practice to terrorize their neighbors and to keep the lower classes
subordinate.
D) It was a response to the lack of available protein in the form of large mammals.
E) It was immoral and unnecessary and was a waste of economic resources.
20) Which of the following represents a cultural difference between the Incas and Aztecs?
A) Monumental architecture
B) Practice of human sacrifice
C) Lack of a writing system
D) Lack of a priest class
E) Relative isolation
21) In terms of the integration of a centralized empire, how did the Incas and Aztecs compare?
A) The Aztecs and Incas both made little attempt to integrate conquered territories and permitted self-rule in return for
loyalty and tribute.
B) The Aztecs were more advanced than the Incas in consolidating their government into an integrated unit.
C) The Incas attempted to create an over-arching political state and made conscious attempts to integrate their empire as a
unit, while the Aztecs did less in this regard.
D) Both the Incas and the Aztecs created fully integrated empires complete with central bureaucracies and military
administrative units that controlled all conquered regions.
E) To create their empires both allowed little variation from region to region and city to city.
22) Which of the following represents a significant difference between the Andean and Mesoamerican civilizations?
A) Climate and geography
B) Kin-based social groups
C) Sedentary agricultural systems
D) The existence of a nobility
E) Tribute systems
23) Which of the following statements about the population of the Americas is most true?
A) The population of the Americas is easy to calculate.
B) North America was more densely populated than Mesoamerica or the Andes.
C) The population estimates of the Americas continue to be revised upward.
D) The early 20th-century estimate of 8.4 million still seems the most accurate.
E) Most estimates are approximately the same.
24) By 1500, agriculture was
A) largely diffused throughout the Americas, although not always in sedentary agricultural communities.
B) virtually unknown outside of the Andean and Mesoamerican civilization zones.
C) practiced in sedentary communities throughout the Americas.
D) practiced in sedentary communities and by nomadic pastoralists throughout the Americas.
E) practiced sparingly inside Mesoamerica but was used extensively in regions such as the Mississippi.
62
25) Which of the following does NOT represent a characteristic of most Indian societies other than the Andean and
Mesoamerican civilizations?
A) Strongly kin-based societies
B) Wealth as the basis of social ranking
C) Communal action and ownership of resources
D) Important social and political roles for women
E) Highly urbanized populations
Essay Questions:
The Americas on the Eve of Invasion
1. In what ways did the Aztecs continue the culture of the classical Mesoamerican civilization and the Toltecs?
2. Describe Aztec social organization.
3. Compare and contrast the Inca and Aztec empires in terms of political administration.
4. How did the Indian cultures outside the Andean and Mesoamerican civilization zones contrast in political and
social organization with the Aztec and Inca empires?
5. How have historians explained the existence and prevalence of human sacrifice among the Aztecs?
63
Chapter 12, Reunification and Renaissance in Chinese Civilization: The Era of the Tang and Song Dynasties
Summary:
Basic themes of Chinese civilization underwent vital consolidation during the postclassical period. Less fundamental
innovation occurred than in the Americas and Europe. Important developments took place in technology. Political
turmoil followed the fall of the Han during the Period of the Six Dynasties (220-589 C.E.), and the empire’s bureaucratic
apparatus collapsed. The scholar-gentry class lost ground to landed families. Non-Chinese nomads ruled much of China,
and a foreign religion, Buddhism, replaced Confucianism as a primary force in cultural life. There was economic,
technological, intellectual, and urban decline. New dynasties, the Sui and Tang, from the end of the 6th century brought a
restoration of Chinese civilization. Political unity returned as nomads and nobility wee brought under state control and
the bureaucracy was rebuilt. Major changes occurred in economic and social life as the focus of a revived civilization
shifted from the north to the Yagzi valley and southern and eastern coastal areas. The Song dynasty continued the revival;
their era saw the restoration of the scholar-gentry and the Confucian order. It was a time of artistic, literary, and
technological flourishing. Male dominance reached new heights.
Key Concepts:
Rebuilding the Imperial Edifice in the Sui-Tang Era:






The long factional struggle that followed the fall of the Han dynasty ended when Wendi unified China under the
Sui dynasty in 589. Wendi used alliances, intrigue, and warfare to achieve his goals.
Yangdi, who murdered his father Wendi to gain the throne, at first strengthened the empire and made legal and
educational reforms, but after military defeats and expensive building projects that overwhelmed his subjects,
widespread revolts threatened the realm.
Following Yangdi’s death, Li Yuan (the Duke of Tang) seized power, expanded China’s boundaries dramatically,
and founded the Tang dynasty.
The Tang also greatly expanded the Confucian-based examination system (administered by the Ministry of
Rites) that provided qualified bureaucrats. Those who passed the highest level exams were called jinshi. Despite
the system, many officials gained their positions through family connections rather than merit.
Buddhism enjoyed a resurgence. Among the masses, the Salvationist, pure-land strain of Mahayana Buddhism
won widespread conversions because it seemed to provide a refuge from an age of war and turmoil. Members of
the elite classes on the other hand, were more attracted too the Chan variant of Buddhism, or Zen as it is known
in Japan and the West. Empress Wu was particularly supportive of Buddhism.
Buddhist successes aroused the envy of Confucian and Daoist rivals, and by the reign of Emperor Wuzong in the
mid-9th century, the religion was openly persecuted. Buddhism survived, but in a weakened condition.
Tang Decline and the Rise of the Song:





Deadly family infighting led to the long reign of Emperor Xuanzong, whose reign marks a high point in Tang
civilization. As his interest in governing waned, his affection for the arts and his famous lover Yang Guifei
increased. In 755, the first of several revolts signaled growing discontent with Xuanzong, and nomadic tribesmen
former allies were impinging on Tang territory with impunity.
In 960, the scholarly general Zhao Kuangyin defeated most of the rivals scounging for power after Xuanzong’s
death and founded the Song dynasty. However, he could not defeat the Manchurian Liao dynasty (founded by
Khitan peoples) in the North, a fact that would prove fatal in time.
The Song favored the scholar gentry at the expense of the military, which meant that the empire was never as
formidable as the Tang.
Accordingly, Confucian ideals were again emphasized. Zu Xi and the neo-Confucians, or revivers of ancient
Confucian teachings, believed that cultivating personal morality was the highest goal for humans, arguing that
virtue could be attained through book learning and personal observation as well as through contact with men of
wisdom and high morality.
Signs of the Song dynasty’s decline included border kingdoms like the Tangut people’s kingdom of Xi Xia,
disdain for military expenditures among the scholar gentry, and Wang Anshi’s failed attempts to secure longterm reform.
64

In 1115, a new nomadic contender, the Jurchens, overthrew the Liao dynasty of he Khitans and established the
Jin kingdom north of the Song empire. What became known for the next century and a half as the Southern
Song dynasty was a weak state politically, but radiant culturally.
Tang and Song Prosperity: The Basis of a Golden Age:







Yandgi’s Grand Canal linked the original centers of Chinese civilization on the North china plain with the
Yagtze River basin more than 500 miles to the South. The canal made it possible to transport grain from the
fertile southern regions to the capital and to transfer food from the south to districts threatened by drought and
famine in the north.
Tang conquests led to increased trade and contact (primarily over the silk road and over the seas in Chinese
junks) with civilizations to the west. Urban centers grew, and “flying money.” For example, was a sign of the
increasing sophistication of Chinese trade.
The movement of the population southward to the fertile valleys fo the Yangtze and other river systems was part
of a larger process of agrarian expansion that Tang and Song leaders encouraged. Agricultural
improvements and leaders modestly successful attempts at land reform aided peasant quality of life.
Both within the family and in society at large, women remained clearly subordinate to men. But some
evidence suggests that, at least for women of the upper classes in urban areas, the opportunities for personal
expression increased in the Tang and early Song.
Neo-Confucian philosophers were leading advocates of male dominance. Men were allowed to have premarital
sex without scandal, to take concubines if they could afford them, and to remarry if one or more of their wives
died. No practice exemplifies the degree to which women in Chinese civilization were constricted and
subordinated as dramatically as footbinding.
The Tang and Song eras are remembered as a time of remarkable Chinese accomplishments in science,
technology, literature, and the fine arts.
As the Confucian scholar gentry supplanted the Buddhists as the major producers of art and literature, devotional
objects and religious homilies gave way to a growing fixation on everyday life and the delights of the natural
world. Li Bo’s poetry is a shining example of the everyday themes these intellectuals prized.
Key Terms:
Tang, Silk Road
Song Dynasty
Footbinding
Empress Wei
Zen Buddhism
Mahayana Buddhism
Tang Dynasty
Flying Money
Grand Canal
Li Bo
Chapter 12, Quiz Questions
1) Which of the following statements concerning urbanization in China during the Tang-Song era is most accurate?
A) Chinese urbanization was more pronounced in northern China, but lagged in southern China.
B) The imperial capitals were enormous, but there were few other Chinese cities of size.
C) The degree of urbanization reached during the Han dynasty was never achieved during the Tang-Song era, although
some recovery of cities accompanied the commercial revival.
D) Chinese urbanization mushroomed during the Tang-Song era with a higher proportion of the population living in cities.
E) Cities were depopulated as people moved back into the countryside due to the collapse of the Chinese economy.
2) What made possible the rapid revival of empire under the Tang?
A) The abandonment of Confucianism in favor of the more widely practiced Buddhism
B) The brevity of the period of political dislocation
C) The willingness of the Tang to abandon traditional approaches to government
D) The rebuilding of the imperial bureaucracy using Confucian ideology
E) Massive grain imports from southern China led to a population increase.
3) What made the reunification of China under the first Sui emperor possible?
A) The support of the nomadic warrior elite
B) The support of the ethnic Chinese aristocracy
C) The support of the Buddhist monasteries
65
D) The support of the Confucian scholar-gentry
E) The threat of barbarian invasion
4) Unlike his father, the second Sui emperor favored
A) the Confucian scholar-gentry class.
B) the great Chinese aristocratic families.
C) the Buddhists.
D) nomadic elements in northern China.
E) the Chinese peasants.
5) What led to the downfall of the Sui dynasty?
A) Nomadic invasions
B) Unsuccessful military campaigns
C) Widespread Buddhist rebellion
D) The dissatisfaction of the Confucian scholar-gentry class
E) Famines, which led to a depletion of economic resources
6) Which of the following statements concerning the extent of the Tang empire is most accurate?
A) The Tang extended the empire in all directions except westward, where the Turks remained entirely independent of the
Chinese emperor.
B) The Tang empire was unable to recover the territorial extent of the Han, but did recover northern areas from the
nomads.
C) The Tang empire incorporated India and southeast Asia as well as the areas north of the Yellow River plain.
D) The Tang built an empire that was far larger than the Han, an empire whose boundaries in many directions extended
beyond the borders of modern China.
E) The Tang empire stopped expanding and was divided into three parts when a civil war broke out.
7) What was the attitude of the Tang emperors toward the Confucian scholar-gentry?
A) The Tang continued to support and patronize the growth of Buddhism in China at the expense of the Confucian
scholar-gentry.
B) The Tang supported the rebirth of the Confucian scholar-gentry, often at the expense of the aristocracy.
C) The Tang feared the development of the scholar-gentry and continued to support the nomadic aristocracy of China.
D) Confucianism continued to wane during the Tang dynasty and was only resuscitated under the Song.
E) The scholar-gentry class began to evolve into a new merchant and trade class due to the intervention of the emperors.
8) Which of the following statements concerning entry into the Chinese bureaucracy is most accurate?
A) Birth and family connections continued to be important in securing high office.
B) Under the Tang family connections ceased to be of significance, as all candidates received office based on their score
in the examination system.
C) Although the examination system continued to be monitored, almost all official received positions as a result of family
connections.
D) The examination system was eliminated during the Tang dynasty, and only members of the imperial family served in
the bureaucracy.
E) The higher levels of the Chinese bureaucracy were dominated by many bright commoners.
9) Which of the following descriptions of Pure Land Buddhism is NOT accurate?
A) It was founded by Chinese monks.
B) It appealed to upper class Chinese.
C) It appealed to the Chinese commoners.
D) It was a salvationist variant of Buddhism.
E) It offered a refuge from war and turmoil.
10) What proved to be the most damaging attack on Buddhism's popularity with the people during the early Tang
dynasty?
A) The Buddhists' insistence on rebellion against the emperor
66
B) The Confucians' successful campaign to convince the emperor that the Buddhist monastic establishment represented an
economic threat
C) The aristocracy's concern that the growing Buddhist monastic establishment was monopolizing land that otherwise
would belong to them
D) The entry of nomadic invaders who were Islamic during the ninth century
E) The challenge from Daoism and other polytheistic religions, which were appealing to Chinese merchants
11) What was the result of the imperial attempt by the Tang to suppress Buddhism within the Chinese empire?
A) Despite official attempts to suppress Buddhism, monastic orders continued to exert political influence and control
landed wealth as they did in the first decades of Tang rule.
B) Buddhism survived the brief imperial persecution and was restored as the primary religion of the state during the Song
dynasty.
C) Buddhism survived the repression, but in a reduced state without the political influence of the early Tang years.
D) Buddhism was eradicated in China by the end of the Tang era and replaced by the traditional Daoist and Confucian
beliefs.
E) Buddhism became a sect of minor importance but with many followers among the scholar-gentry.
12) What was the impact on Confucianism of the Tang repression of the Buddhists?
A) Confucianism emerged as the central ideology of Chinese civilization until the 20th century.
B) Confucianism emerged in a strengthened condition, but still remained behind Buddhism in the sense of providing a
basis for the intellectual rationale of Chinese civilization.
C) Because Confucian scholar-gentry were associated with the persecution, Confucianism failed to generate much popular
support outside the imperial government.
D) Confucianism, like Buddhism, declined in popularity, and its place was taken by Daoism.
E) Confucianism and Daoism blended with Buddhism to form a new philosophy.
13) Which of the following was NOT a problem during the last years of the Tang dynasty?
A) The revolt of An Lushan
B) The entry of nomads into large areas of the northern China plain
C) The Huang-chao rebellion
D) A Buddhist rebellion in southern China
E) Independent rulers collected their own taxes
14) How did the Song empire compare to the Tang?
A) The Song empire was greater in territorial extent than the Tang empire.
B) The Song empire and the Tang empire were virtually identical in territorial extent.
C) The Song empire was smaller in territorial extent than the Tang empire.
D) Although approximately the same size, the Song empire extended farther north than the Tang.
E) The Song was larger than the Tang and Han.
15) Which of the following statements most accurately describes the position of the scholar-gentry under the Song
dynasty?
A) The Confucian scholar-gentry continued to exist, but the examination system weakened noticeably causing the
imperial bureaucracy to depend more and more on the aristocracy.
B) The domination of the scholar-gentry over its aristocratic and Buddhist rivals was fully secured in the Song era.
C) The Song emperors came to rely increasingly on the military as defense of the borders placed less emphasis on the role
of intellectual ideology and more on warfare.
D) The Song continued to rely on the Confucian scholar-gentry, but more rigorous examination systems resulted in fewer
men entering the civil service as a result of education.
E) The scholar-gentry attempted to revive warlordism, which led to the fall of the Song.
16) Which of the following was NOT an eventual impact of the development of neo-Confucian schools of thought?
A) Hostility to foreign philosophical systems like Buddhism
B) The stifling of innovation and critical thinking in favor of traditionalism
C) Destruction of the mandarins
67
D) Reinforcement of class, age, and gender distinctions
E) Historical experiences were held to be the most practical guide to life.
17) Which of the following was NOT one of the reforms introduced by Wang Anshi in the face of conservative
opposition?
A) The introduction of government-assisted irrigation projects to encourage agricultural expansion
B) Taxation of the landlord and scholarly classes
C) The establishment of well-trained mercenary forces
D) The end of tribute payments to nomadic dynasties on the northern border
E) Reorganization of university education.
18) What caused the flight of the Song dynasty from its capital in northern China?
A) The invasions of the Jurchens who had formed the Qin kingdom
B) The Huang-chao rebellion
C) A rebellion led by a nomadic general, An Lushan
D) Extensive flooding in the Yellow River basin
E) Attempted takeover by the Khitans
19) Why was the construction of the Grand Canal necessary?
A) Major river systems in China ran from north to south, and the canal was necessary to connect the coastal regions with
the western frontier.
B) Chinese population was increasingly concentrated along the northern plains of the Yangtze River.
C) The Yangtze River valley was becoming the major food-producing region of China by the late Tang era.
D) The canal connected the Tang capitals Changan and Loyang with the newly acquired regions in the North.
E) Peasants were moving from the countryside of southern China to the urban areas of the North.
20) Which of the following was NOT a result of the agricultural policies of the Sui and Tang emperors?
A) Numbers of free peasantry increased.
B) The scholar-gentry were removed as landlords.
C) Fortunes of the old aristocratic families declined.
D) Lands were distributed more equitably to the free peasant households of the empire.
E) The yields of peasant farmers greatly increased.
21) What was a primary difference between marriages, families, and households of the upper and lower classes in TangSong China?
A) In the upper classes, males frequently married females of a younger generation.
B) Extended family households were more common in upper-class households than in lower-class ones.
C) Members of the elite often married before the age of puberty.
D) Households of the lower classes tended not to be patriarchal.
E) Upper-class men rarely married foot-bound women.
22) Which of the following statements concerning the status of women in the Tang-Song era is most accurate?
A) Tang-Song law recognized no women's rights.
B) The condition of women in general improved during the Tang-Song era.
C) The assertion of male dominance within the family was especially pronounced in the thinking of the neo-Confucian
philosophers.
D) The spread of Buddhism provided women with greater opportunities and career alternatives within the monastic
movement.
E) Women had more rights than ever before due to the influence of the Legalists.
23) Which of the following statements about Chinese landscape painting is most accurate?
A) Landscape painting was virtually unknown until after the Mongol invasions.
B) As in the West, landscape painting was the special skill of the military elite.
C) Members of the ruling political elite in China produced many of the paintings in the Song era.
D) Song subsidies to foreign painters resulted in the production of magnificent landscapes.
68
E) Paintings never conveyed moral messages or used symbolism.
24) In what way did foot-binding serve to diminish the independence of Chinese women by the end of the Song era?
A) Because foot-binding could only be afforded by the elite, poorer women were assigned to a lower social status.
B) Foot-binding sufficiently crippled women to effectively confine their mobility to their household.
C) As foot-binding was required in order to practice certain professions, Chinese women found that occupational
alternatives were diminished.
D) Foot-binding, although considered socially attractive, was condemned by neo-Confucians.
E) Upper-class women considered it high status and hoarded money to pay for it.
25) Which of the following intellectual schools was responsible for the production of most literary and artistic works
during the Tang-Song era?
A) Confucian
B) Daoist
C) Pure Land Buddhist
D) Chan Buddhist
E) Legalist
Essay Questions:
Reunification and Renaissance in Chinese Civilization
1. Contrast the Era of Division with the Sui – Tang era.
2. Explain the decline of Buddhism in the later Tang and Song dynasties.
3. Compare and contrast the empire under the Tang and the Song dynasties.
4. What were the elements of Tang – Song economic prosperity?
5. In what ways did the Chinese empire during the Tang - Song era depart from previous developments in Chinese
civilization?
69
Chapter 13, The Spread of Chinese Civilization: Japan, Korea, and Vietnam
Summary:
The people on China’s borders naturally emulated their great neighbor. Japan borrowed heavily from China during the 5 th
and 6th centuries when it began forming its own civilization. To the north and west of China, nomadic people and Tibet
were also influenced. Vietnam and Korea were part of the Chinese sphere by the last centuries B.C.E. The agrarian
societies of Japan, Korea, and Vietnam blended Chinese influences with their indigenous cultures to produce distinctive
patterns of civilized development. In all three regions, Buddhism was a key force in transmitting Chinese civilization.
Key Concepts:
Japan: The Imperial Age:





The Chinese influence on Japan came to a peak in the Taika, Nara, Heian periods, (645-857). The Taika reforms
restructured the government following the Chinese model. Confucianism permeated Japanese culture from top to
bottom.
The Taika reforms were not completed because of resistance form the nobles and Buddhist monks. Moving the
capital to Heian (Kyoto), the emperor Kammu hoped to avoid monastic opposition. Failing in this, he restored to
the aristocracy all of their rights.
Heian society was extremely mannered and sophisticated, developing a poetic tradition in a Chinese script
tailored to the Japanese language. The classic Tale of Genji symbolizes the aesthetic of the period, in particular
the important, albeit limited, role of women at the Heian court.
The Fujiwara family was one of the most powerful, but typical in their cooperation with Buddhist monasteries to
reduce the power of the emperors.
A new force came to challenge the court aristocracy: the bushi, or warrior leaders. Some were of noble origin,
some not, but they had in common increasing power in their small domains, and the loyalty of samurai troops.
Unchecked use of force led to the preeminence of a warrior class and a warrior culture. The code the samurai
followed included the practice of seppuku, or ritual suicide following defeat. Growth of Samurai power
accompanied the reduction of peasant status.
The Era of Warrior Dominance:




Chinese influence, and direct contact with China, waned in the 9th century. From the 11th century, court families,
in conjunction with bushi allies, split the court with open rivalry. Eventually, open war broke out between the
Taira and Minamoto families in the 1180’s.
The Gempei Wars ended with the ascendancy of the Minamoto at their new capital at Kamakura.
The bakufu government of the first Minamoto ruler, Yoritomo, was supported by shoguns, military leaders. The
following centuries saw a complex system with titular emperors and Minamoto shoguns, real power being
wielded by the Hojo family. The latter were supplanted by the Ashikaga Shogunate. Royal authority was a mere
shadow, but the shoguns also lost power in the late 15th century, replaced by 300 daimyo kingdoms.
Court manners became irrelevant as making war took center stage. The plight of the peasants became desperate,
leading to unsuccessful revolts. At the same time, the dynamism of some daimyos led to economic growth and
the emergence of a merchant class. Among the merchant and artisans, women had a more prominent role, while
women of elite families saw their lives constrained.
Korea: Between China and Japan:


Korea, although strongly linked to Chinese cultural and political developments, had distinct origins, and long
followed its own path of development. The peninsula’s first kingdom, Choson, was conquered by China in 109
B.C.E., and subsequently Chinese settlers arrived. Korea broke form Chinese dominance, forming three
kingdoms: Koguryo, Silla, and Paekche. As in Japan, Sinification adoption of Chinese culture was largely
mediated by Buddhism. The Koguryo ruler applied a Chinese style law code.
Internal conflict int eh Three Kingdoms Era left Korea vulnerable to Chinese attack. The Tang allied with the
Silla to destroy Paekche and koguryo, leaving the Silla a subject kingdom.
70




Sinification peaked under the Silla and Koryo rulers. Tribute and acknowledgement of Chinese authority created
peaceful relations that stimulated Korean borrowing from Chinese culture.
Under the Silla, their capital at Kumsong copied the Tang capital. Both the royal family and the Korean elite
supported Buddhism. While Korean borrowing from China was heavy, in the areas of pottery and printing, they
exceeded their teachers.
Sinification was limited to Korean elite, while indigenous artisanry was allowed to decline. All of Korean society
was arranged to serve the needs of the aristocracy.
Periodic popular revolts were successful only in weakening the Silla and Koryo monarchies. The Mongol
invasion in 1231 began a period of strife, ending with the founding of the Yi dynasty in 1392.
Between China and Southeast Asia: The Making of Vietnam






The early history of the Viet people is little known. Early Chinese raids into Vietnam in the 220’s B.C.E
increased trade. Intermarriage with Mon-Khmer and Tai language groups furthered the development of a distinct
Vietnamese ethnicity. Many early traditions separated them from the Chinese, such as the nuclear family pattern
and a greater role for women.
The Han became dissatisfied with merely exacting tribute from the Viet rulers and began direct rule in 111 B.C.E
Sinification increased, and was used by the Viet rulers to consolidate their power over both their own peoples and
those to the west and south.
In spite of Chinese expectations, the Viets never became assimilated to Chinese culture. Indeed, a culture of antiChinese resistance developed. The rising of the Trung sisters in 39 C.E. underlined the continuing prominent
role of Vietnamese women.
Continuing Chinese influence in Vietnam depended on overcoming physical barriers, and on the competence of
Chinese rulers. Following the fall of the Tang, the Vietnamese freed themselves completely by 939. Yet Chinese
influence continued, particularly in the administration. An important exception was the scholar-gentry who never
gained an important role in the Vietnamese regime.
The lands of the Chams and Khmers attracted the Vietnamese. From the 11th to the 18th centuries, the latter
steadily expanded their territory at the expense of the Chams. Subsequently, they attacked the Khmers in the
Mekong delta.
The new southern territories were controlled only with difficulty by Hanoi. The Trinh family, ruling the north,
was challenged by the southern Nguyen family. The conflict left the Vietnamese oblivious to an outside threat:
the French and the Catholic Church.
Key Terms:
Taika Reforms
Fujiwara Family
Seppuku
Bakufu
Tale of Genji
Middle Kingdom
Shintoism
Shogans
Trung Sisters
Yi Dynasty
Chapter 13, Quiz Questions
1) What was the central purpose of the reforms of 646 in Japan?
A) To remake the Japanese monarch into an absolutist Chinese-style emperor
B) To destroy the Confucian scholar-gentry in favor of a military aristocracy
C) To increase the power of the Buddhist monastic structure
D) The destruction of the traditional peasant-conscript army
E) To decentralize the imperial government
2) What was the immediate impact of the imperial move to Heian?
A) Buddhism ceased to play a major role in Japanese society.
B) The scholar-gentry was able to assert itself through the state's acceptance of a formal examination system modeled on
the Chinese bureaucracy.
C) The aristocracy was restored to counterbalance the power of the Buddhist monasteries and took over most of the
positions in the central government.
D) Shintoism was formally suppressed following the marriage of the Empress Koken to a former monk.
71
E) The aristocrats gave up positions in the central government under pressure from the Buddhists.
3) What was the military organization of the Heian government in Japan?
A) The emperor formalized the creation of a peasant-conscript army on the model of the Chinese.
B) The emperor accepted the introduction of Chinese troops as the basis for the Japanese army.
C) Formal military organization was abandoned by the emperor as a way to control the bushi.
D) Local members of the aristocracy were ordered to organize militia forces.
E) Buddhist monasteries depended on the bushi for protection.
4) What appears to have been the primary concerns at the imperial Japanese court at Heian?
A) Military exercise
B) Science and technology
C) Social interaction and status
D) Commerce and trade
E) Interpreting laws
5) Which of the following statements concerning the rise of a provincial military elite during the 10th century in Japan is
NOT correct?
A) Provincial elite families often arose from local landowners, estate managers, or local state officials.
B) The provincial elite came to control land and labor locally and to deny these resources to the imperial court.
C) The rise of the provincial elite corresponded to the recovery of the imperial government and its overthrow of the
aristocracy of the court.
D) Within their little kingdoms, warrior leaders administered law, supervised public works projects, and collected
revenue.
E) A warrior code developed that stressed family honor and death rather than retreat or defeat.
6) Which of the following statements concerning the relationship of the imperial court to the provincial military elite is
most accurate?
A) The provincial military elite was kept in check by the continued development of the imperial peasant-conscript army.
B) The provincial military elite was rapidly subjected to the aristocratic armies of the imperial court.
C) In the absence of an imperial military force, law and order broke down, leading both the emperor and high officials to
hire provincial lords and their military retainers.
D) The emperor created an alliance with the Chinese that permitted him to crush the regional military lords.
E) The imperial court worked with the military elite to fight the growing civil unrest that was due to an unhappy
peasantry.
7) Which of the following statements concerning the nature of warfare among the bushi is most accurate?
A) The bushi depended on infantry tactics, equipping the samurai initially with long spears.
B) The introduction of gunpowder in the 11th century allowed the bushi to rely on cannon and rockets as their primary
means of assault.
C) Battles depended on the Japanese phalanx of mounted samurai and massed assaults predicated on the willingness of
the retainers to sacrifice themselves for their leaders.
D) Battles hinged on man-to-man duels of great champions typical of the heroic stage of warfare.
E) The bushi made possible the building of conscript armies using new techniques of fighting obtained from the European
kingdoms.
8) What was the impact of the rise of the samurai on the peasantry in Japan?
A) Japanese peasants were reduced to the status of serfs bound to the land they worked.
B) The samurai were critical to the development of a free peasantry on which the warriors depended for supplies of food
and arms.
C) The creation of the samurai created a period of great social mobility in Japan during which people rapidly moved out
of the peasantry and into the class of warriors.
D) Although separated from the warriors by rigid class barriers, the peasantry achieved greater levels of personal freedom
and economic prosperity.
E) Peasants who were willing to be trained could become samurai.
72
9) By the 11th and 12th centuries, what was the status of the Japanese court aristocracy?
A) They continued to dominate the imperial government and to depend on their alliance with the Buddhist monasteries.
B) Aristocratic families at the court depended on alliances with the provincial warrior elite in order to exercise any power.
C) The resuscitation of the emperor led to the destruction of the court aristocracy and the suppression of the regional
warrior elite.
D) The court aristocracy succeeded by the 11th century in setting aside the emperor in favor of regionalized government.
E) The aristocracy increased in power due to the influence of the Taira and Minamoto families.
10) The victory of the Minamoto marks the beginning of what period in Japanese history?
A) The Taika period
B) The feudal age
C) The Onin wars
D) The Tokugawa Shogunate
E) The Fujiwara empire
11) Which of the following statements most accurately describes the nature of the Japanese government following the
death of Yoritomo?
A) Real power rested in the Hojo family, who manipulated the Minamoto shoguns, who in turn claimed to rule in the
name of the emperor at Kyoto.
B) There was a two-tiered system of government with power resting with the restored emperor at Heian who commanded
a body of aristocratic warriors associated with the bakufu at Kyoto.
C) Power was rapidly dissipated after the emperor was officially deposed without a successor.
D) The successors of Yoritomo seized the imperial throne in their own name and set off a civil war between the old
imperial family at Nara and the new one in Kamakura.
E) Japanese unity was achieved and feudal rule was ended under the leadership of the shoguns.
12) What was the relationship between the Ashikaga Shogunate and the emperor?
A) The Ashikaga shoguns unseated the emperor at Heian and took the imperial title for themselves.
B) The Ashikaga shoguns swore fealty to the emperor at Kyoto and restored imperial authority.
C) The Ashikaga shoguns fought the emperor at Kyoto who refused to recognize their authority.
D) The Ashikaga shoguns brought the imperial family to Kamakura in order to more closely control their activities.
E) The emperor and the Ashikaga shoguns formed an alliance and worked together to defeat their enemies.
13) What was the political result of the wars that destroyed the Ashikaga Shogunate?
A) Restored authority for the emperor
B) Restored authority for the court aristocracy
C) Division of Japan into 300 little states under the daimyos
D) The rise of four large aristocratic states under powerful bushi
E) The China model seemed more relevant to the Japanese.
14) How did the principles of warfare change under the daimyos?
A) Heroic combat between champions remained the rule, but the weapon of choice changed from the bow to the curved
sword.
B) Peasant forces were reduced in significance, as they were replaced by professional soldiers.
C) Warfare based on spying, timely assaults, wise command, and organization of massive armies replaced heroic combat.
D) The rise of gunpowder and cannon made the fortresses and castles of the warrior elite obsolete.
E) They introduced new military techniques and strategies learned from contacts with Islamic traders.
15) Which of the following statements concerning the society and economy of the warlord or shogun era in Japanese
history is most accurate?
A) Due to the incessant warfare, the Japanese economy was reduced to barbarism.
B) The peasantry were further reduced in status as the agricultural economy experienced rapid reductions in the acreage
under production.
C) Despite political chaos, improvements in agricultural techniques and incentives led to occupation of previously
73
uncultivated areas.
D) The frequent combat of the warlord era suppressed the development of artisan or merchant classes in Japan.
E) The farm economy continued to improve due to the introduction of the three field system from Europe.
16) Which of the following statements concerning the status of women during the era of the daimyos is most accurate?
A) Women's status among the elite certainly improved during the era of the warlords, as they gained rights to inheritance.
B) Women often participated in martial training during the era of the warlords, thus improving their social status.
C) Women were excluded from inheritance and treated as defenseless appendages of their warrior fathers or husbands.
D) While the status of women among the artisan and merchant classes declined, the status of women among the warrior
elite probably improved marginally.
E) The rise of the handicraft industry caused the status of women to decline throughout the period.
17) During the period of the warring daimyos, what was the factor that led to a revival of Chinese influence on the cultural
level?
A) Confucianism
B) The strength of the merchant class
C) Chinese military technology
D) Zen Buddhism
E) Legalism
18) Which of the following statements concerning the Korean bureaucracy under the influence of China is most accurate?
A) With the establishment of the Confucian examination system, advancement within the civil service was determined
almost exclusively by test scores.
B) Although Korea emulated the Chinese bureaucratic structure, the satellite never created a Confucian examination
system as a means of qualification.
C) Korea established a Confucian examination system on the Chinese model, but admission to the bureaucracy was
determined almost exclusively by birth.
D) Korean absolutism depended more on a warrior elite than administrative sophistication, so a formal bureaucracy was
never established.
E) Korean nobility did not welcome Chinese bureaucratic institutions and tried to introduce Korean models.
19) Which of the following statements concerning the nature of the Korean society is most accurate?
A) Like the Japanese during the same period, Korean society is typified by the rise of strong merchant and artisan classes.
B) Like the Chinese, a wealthy merchant class arose in Korea, but according to Confucian dictates it never achieved high
social status.
C) Because of the monopolization of all things Chinese by the tiny Korean elite, neither a merchant nor an artisan class
developed in Korea.
D) Korean society followed the Chinese model of a strong scholar-gentry, a weak aristocracy, and a highly regarded
though politically impotent peasantry.
E) Korean universities fought the trend toward Sinification by using Korean examination systems.
20) Which of the following was NOT a difference between the Viet and Chinese cultures prior to their conquest by the
Han emperors?
A) Language
B) Village autonomy among the Viets
C) Higher social status for Viet women
D) Viet tendency to extended families and clan groups
E) Political institutions
21) What was the political result of the Vietnamese drive to conquer regions south of the Red River basin?
A) The creation of a highly centralized kingdom with its capital at Hanoi
B) The defeat of the Vietnamese and the fragmentation of the kingdom into 300 small kingdoms ruled by a warrior elite
C) The reconquest of the Red River valley by the Chinese during the Southern Song era
D) The division of the Vietnamese into two kingdoms with capitals at Hue and Hanoi
E) Vietnamese domination of the South Pacific and the Philippines
74
22) Which of the following cultural traits was NOT introduced into Vietnam from China following the Han conquest of
111 B.C.E.?
A) Chinese examination system and bureaucracy
B) Chinese agricultural cropping techniques and irrigation technology
C) Chinese reliance on the nuclear family
D) Chinese military organization
E) Chinese-style schools
23) Which of the following was a critical factor in the failure of the Chinese to conquer or assimilate the Vietnamese?
A) The Vietnamese resistance to Chinese crossed both class and gender barriers
B) The failure of the Chinese to introduce Buddhism
C) The continued cultural impact of Indian culture in Southeast Asia, particularly among the Vietnamese
D) The widespread destruction of the Vietnamese elite
E) The proximity of Vietnam to Chinese centers of power
24) Following the assertion of Vietnamese independence from China in the tenth century, what aspects of Chinese culture
were retained by the Vietnamese rulers?
A) Chinese family organization
B) The Chinese administrative system based on the Confucian examination system
C) A highly centralized administrative system manned by a powerful scholar-gentry
D) The successful suppression of the peasantry
E) Chinese popular culture, such as cockfights
Essay Questions:
The Spread of Chinese Civilization: Japan, Korea, and Vietnam
1. How did the Japanese importation of Chinese culture differ from that of Vietnam and Korea?
2. Discuss the government of Japan between the Gempei wars and the Tokugawa Shogunate.
3. How did the social development of post classical Japan differ from the development of Korea during the period of
Chinese influence?
4. Why was China unable to assimilate the Vietnamese despite direct rule for almost a millennium?
5. How did the extension of Chinese culture to its satellite civilizations differ from other global civilizations such as
Islam?
75
Chapter 14, The Last Great Nomadic Challenges: From Chinggis Khan to Timur
Summary:
The nomads of central Asia returned to center stage in world history during the 13th century. The Mongols ended or
interrupted the great postclassical empires while extending the world network. Led by Chinggis Khan and his successors,
they brought central Asia, China, Persia, Tibet, Iraq, Asia Minor, and Southern Russia under their control and dominated
most of Asia for one and a half centuries. The Mongols were the most formidable nomadic challenge to the sedentary
civilizations since the first century C.E. The Mongols are often portrayed as barbarians and destructive conquerors, but
generally in their vast possessions people lived in peace, enjoyed religious tolerance, and had a unified law code.
Peaceful contacts over long distances opened. Mongol territory was a bridge between the civilizations of the East as
products and ideas moved among civilized and nomadic peoples.
Key Concepts:
The Mongol Empire of Chinggis Khan:







Mongols were typical nomads: living off of their herds and trade, organized around the tribe, forming short-lived
confederations, electing leaders, and valuing warrior virtues.
Kabul Khan, in the early 1100s defeated a Chinese Qin army. His grandson, Temujin emerged from Kabul
Khan’s fragmented dominions. A kurlitai or meeting of the Mongol leaders convened in 1206. Temujin was
chosen khagan supreme ruler as Chinggis Khan, in 1206.
The Mongol army relied on mounted archers. Chinggis Khan brought unity and organization, creating tumens of
10,000 warriors. Scouting parties and messengers allowed the khan to hold together large areas, as did swift
punishment for disloyalty. Information gathering supplied Chinggis Khan with maps that facilitated his
conquests.
Chinggis Khan’s ambition led him to attack the northern Chinese Xi Xia kingdom, then the Jin Empire of the
Jurchens.
The Mongolian Kara Khitai Empire was next conquered by Chinggis Khan’s forces. Subsequently, the Mongols
defeated Muhammad Shah’s Khwarazm Empire. By the time of Chinggis Khan’s death in 1227, the Mongol
Empire extended from Persia to the North China Sea.
Chinggis Khan, although capable of great brutality, patronized artists and intellectuals in the realms he conquered.
At his capital at Karakorum, he gathered the greatest thinkers from China and from Muslim lands. The Mongol
imperium meant lasting peace for much of Asia. Merchants in particular profited from the calm.
Following the death of Chinggis Khan, his empire was divided by his sons and his grandson Batu. The kuriltai
then chose Ogedai as the next great khan. Ogedai extended the empire to the east and north.
The Mongol Drive to the West:




The Khanate of the Golden Horde was one of the four divisions of Chinggis Khan’s empire. The goal of the
Golden Horde was the conquest of Europe Division in Russia made it bulnerable to Mongol aggression. By 1240,
only Novgorod had avoided conquest.
Mongol rule was demanding, but also extended religious and cultural toleration. Moscow profited by Mongol
rule to rebuild and to strengthen its hegemony. Mongol rule of Russia had a negative impact, but only a minor
cultural legacy. Its greatest impact was in changing the direction of Russian history, leading its rulers to
consolidate their power, and temporarily cutting Russia off from western Europe.
Early news of the Mongols led Europeans to equate Chinggis Khan with the mythical Christian king, Prester John.
Even the news of the defeat of Russia failed to alarm the western Europeans. King Bela of Hungary
contemptuously rejected Mongol demands, only to be defeated in 1240. The Mongols then raided further north in
eastern Europe before withdrawing.
Hulegu, on of Chinggis Khan’s successors, captured Baghdad in 1258. The impact on the Islamic heartland was
enormous. Berke Khan threatened Hulegu’s domains from the north. Finally, the Mongols defeat at the hands of
the Mamluks under Baibars stopped Hulegu’s push to the west.
The Mongol Interlude in Chinese History:
76






The Mongols under Kubilai Khan continued their assault on China, having already conquered the Xi Xia and Jin
empires. Kubilai took the title of great khan, and the dynasty he founded was known as the Yuan. Under his rule,
Mongol and Chinese cultures were kept separate, and intermarriage was forbidden. The Mongol elite ruled the
ethnic Chinese. However, Kubilai Khan’s capital at Tatu followed Chinese precedents, as did court ritual.
Mongol women kept the freer roles to which they were accustomed. Kubilai’s wife Chabi played an influential
part in his government.
Kubilai and Chabi patronized and intellectuals, especially Persians and Turks. Travelers from many areas arrived
at their court, including Marco Polo.
Kubilai was more effective in his efforts to keep Mongols and ethnic Chinese separate than he was in encouraging
his people to adapt to Chinese ways. Chinese resentment of the invaders was exacerbated by Mongol support for
artisans and merchants, upsetting the traditional order. The Yuan dynasty saw a revival of urban life and high
culture. Kubilai Khan had plans, never fully realized, to lighten the tax burden on peasants and establish a system
of village schools.
The Yuan dynasty was short-lived, and lost much of its vigor at the death of Kubilai Khan. His successors’
abuses heightened hostility towards the Mongols. Crime became widespread, and secret sects the White Lotus
Society is and example found large followings. Order was restored under the leadership of Ju Yuanzhang, a
commoner, who founded the Ming dynasty.
The brief rule of Timur-i Lang again destabilized central Asia. From Samarkand, the Turkish leader conquered
Persia, much of the Middle East, India, and southern Russia. Although Timur was himself cultured, his legacy
was one of brutal destruction.
Key Terms:
Timur-I Lang
White Lotus Society
Marco Polo
Ming Dynasty
Kubilia Khan
Chinggis Khan
Mamluks
Golden Horde
Samarkund
Karakorum
Chapter 14, Quiz Question
1) In most ways, the Mongols epitomized what type of society and culture?
A) Nomadic
B) Sedentary agricultural
C) Hunting-and-gathering
D) Urban
E) Modern
2) What was the religious policy of the Mongol empire under Chinggis Khan?
A) He was converted to Islam late in his life.
B) He practiced no religious beliefs himself, but tolerated Islam only.
C) All religions were tolerated in his empire.
D) Buddhism became the state religion of the Mongol empire.
E) After the Russian campaign the Mongols became Orthodox Christians.
3) Which of the following was NOT one of the positive aspects of Chinggis Khan's imperial rule?
A) He promulgated a legal code to end divisions and quarrels among the Mongol clans.
B) He brought peace to much of Asia.
C) He promoted the growth of trade and commerce.
D) The movement of merchants and commercial goods facilitated the spread of disease.
E) He promised religious toleration for many different religious groups.
4) What accounts for the growing political dominance of Moscow under Mongol rule?
A) Moscow was the only Russian city that was not destroyed during the invasion.
B) As the city located farthest south in Russia, Moscow was better able to benefit from renewed trade under the Mongols.
C) Moscow became the capital of the Golden Horde.
77
D) The princes of Moscow collected tribute for the Mongol khans and became the seat of the Metropolitan of the
Orthodox church.
E) Kiev had been destroyed by the Golden Horde.
5) What was the social impact of the Mongol conquest on Russia?
A) The Russian nobility was exterminated giving rise to a society largely composed of free peasants.
B) Due to the crushing burden of tribute paid to Mongols and princes, the Russian peasantry was reduced to serfdom.
C) The cessation of trade destroyed the commercial and artisan classes of Russia.
D) Russian women were elevated to new levels of social prominence.
E) The feudal system ended and was replaced by a centralized monarchy based in Kiev.
6) What was the most significant impact of the period of the Mongol rule on Russia?
A) The period of Mongol rule reinforced the isolation of Russia from western Europe and the developments of the
Renaissance and Reformation.
B) The Mongols aided the Russians in gaining political dominance over the peoples of the Asiatic steppes.
C) The period of Mongol rule introduced many Islamic people into the region of Russia.
D) The Mongol domination resulted in the destruction of Eastern Orthodoxy and the rise of Nestorian Christianity.
E) Mongol rule introduced Russia to advanced Asian technology imported from China.
7) Why did the Mongols not pursue the conquest of western Europe?
A) They were defeated at the battle of Bratislava by King Bela of Hungary and allied knights of eastern Europe.
B) Attacks by Timur-i Lang on the southern limits of the territories belonging to the Golden Horde interrupted the Mongol
assault.
C) The death of Khagan Ogedei in Karakorum precipitated a struggle for the succession involving the khan of the Golden
Horde.
D) The Mongols had no interest in western Europe.
E) They lacked the necessary cavalry skills.
8) In addition to the destruction of the Abbasid political capital at Baghdad and the weakening of the Muslim military
strength, what significant impact did the Mongol conquest have on the Islamic heartland?
A) The destruction of cities from central Asia to the shores of the Mediterranean devastated the focal points of Islamic
civilization.
B) The successful assault on the east African city-states weakened the international trading system of Islam.
C) Shi'ism was eliminated as a major factor within Islam.
D) Much of the population of the Islamic heartland was converted to the animism common among the Mongolian nomads.
E) Most areas were permanently changed to a Mongolian culture.
9) Why did the khan of the Ilkhan horde decide to give up his plans to continue his conquest of the Islamic heartland in
1260?
A) The forces of the Christian crusader states were likely to bring the powerful armies of western Europe into the struggle
against the Mongols.
B) The khan died leaving a minor as the ruler of the Ilkhan horde.
C) The successor of Batu as ruler of the Golden Horde, Berke, had converted to Islam and was a potential ally of the
Mameluks.
D) The Ilkhan horde dissipated into rival clan groups following their defeat at the hands of the Mameluks.
E) The Islamic forces were already preparing a massive counterattack.
10) Which of the following was a major change in the administration of China under the Mongols?
A) The central bureaucracy was dismissed and the Mongol dynasty ruled with a military elite.
B) The Mongols discontinued the use of the examination system to keep the scholar-gentry from gaining too much power.
C) The Mongols divided all of China into four great khanates under separate and independent rulers.
D) Confucianism was suppressed and Daoism became the state religion of China.
E) All of the main divisions of the empire were removed and replaced by local Mongol elites.
11) What was the status of Mongolian women during the Yuan dynasty of China?
78
A) Mongolian women lost status as they fell under the social apparatus of the Confucian ideology.
B) Mongolian women suffered social and political isolation as Mongol men adopted the preference for women who had
undergone foot-binding.
C) Mongol women were increasingly prevented from participating in hunting and martial activities, although they retained
some influence in the household.
D) Mongol women remained relatively independent, refused to adopt the practice of foot-binding, and retained their rights
in property.
E) Mongol women wholeheartedly adopted Chinese culture including the practice of foot-binding.
12) What was the Yuan policy with respect to religion during the Mongol occupation of China?
A) Like the Chinese dynasties, over time the Yuan adopted Confucianism as the primary ideology of the state.
B) Because it was closer to the animism practiced by the Mongols on the steppes, the Yuan adopted Daoism as the state
religion.
C) Despite their desire to remain separate, the Yuan emperors were converted to Buddhism under the influence of Chabi,
Kubilai Khan's wife.
D) The Yuan, like their ancestors, insisted on religious toleration.
E) Muslims and Buddhists were excluded from public life but were allowed to practice their religion.
13) What two social groups that suffered low status under the Confucian social system benefited most from the Yuan
administration?
A) Peasants and aristocracy
B) Scholar-gentry and aristocracy
C) Merchants and artisans
D) Military elite and scholar-gentry
E) Criminals and slaves
14) Which of these comparisons between the Turkic expansion and the Mongol one is most accurate?
A) The Turkic expansion, although of briefer duration, actually covered greater territory.
B) The Turkic expansion, like the Mongol conquests, provided a period of peace and commercial expansion.
C) The Turkic expansion covered less territory, failed to increase trade, and provided no internal peace.
D) The Turkic expansion, unlike the Mongol conquests, resulted in the creation of a lasting kingdom based on the capital
at Samarkand.
E) The Turkic expansion was less brutal and destructive but did lead to internal peace.
15) What was one of the most important but indirect and unintended impacts of the Mongol drive to the west?
A) The demographic blow to Eurasian populations caused by the Black Death
B) The closing of the Silk Road and other major trade routes led to isolation of the East
C) The tendency of the nomadic peoples to give up their lifestyle in preference for sedentary agriculture
D) The progressive desiccation of the central Asian steppes
E) The expansion of overseas trade to the Americas
Essay Questions:
The Last Great Nomadic Challenges: From Chinggis Khan to Timur
1. Define the territorial extent of the Mongol empire at its largest. How did this affect inter-cultural exchange?
2. In what way was the political impact of the Mongol conquests of Russia and the Islamic heartland similar?
3. In what ways did the Mongol dynasty of China (the Yuan) attempt to alter the traditional Chinese social structure?
4. What can be said to have been the positive aspects of the Mongol Conquest?
79
Chapter 15, The West and the Changing Balance of World Power
Summary:
By 1400, there was a shifting balance between world civilizations. The international role of the Islamic world, with the
fall of the Abbasids and other Mongol disruptions, was in decline. The Ming dynasty of China attempted, for a time, to
expand into the vacuum. The most dynamic contender was Western Europe. The West was not a major power, but
important changes were occurring within its civilization. Italy, Spain, and Portugal took new leadership roles. The
civilizations outside the international network, the Americas and Polynesia, also experienced important changes.
Key Concepts:
The Decline of the Old Order:




The Byzantine Empire and the Islamic Caliphate continued to dominate the Middle East into the 13th century.
However, by the mid – 15th century, the Ottoman Turks had taken Constantinople.
Cultural change came to the Middle East with political change. The popularity of the Sufi accompanied a general
shift toward mysticism and away from the sciences. A widespread decline in agriculture meant the reduction of
many peasants to serfdom.
Fragmentation of the Islamic world continued under the Ottoman Empire. The Mongols had taken advantage of
the fragmentation, but their decline again left a power vacuum
The new Ming Dynasty emerged in 1368, pushing out the Mongols. Ming emperors began a series ./of trading
voyages to India in 1405, led by admiral Zenghe. The expeditions were stopped in 1433, and this line of
development was not pursued. Instead, the emperors turned to strengthening their position in China, pursuing
traditional policies.
The Rise of the West:






The 15th century was a period of profound change in the West. The aristocracy was losing its place as the
defenders and leaders, turning to jousting and court ritual. Famine and the Black Death had deeply changed
European culture and society. One third of the population had died in 30 years.
The medieval monarchies retained their vigor. The European economy revived, after a period of decline, along
with increasing urbanization.
The expansion of the Mongol Empire had brought the west into more contact with the east. A variety of
innovations made their way to Europe: the compass, paper, gunpowder. The great demand for eastern luxury
goods led to a gold drain to the east. This demand, added to the threat of the Ottoman Empire, impelled
Europeans to seek new routes to the east.
The Italian Renaissance, a cultural and political movement that looked to the antique past, began to take shape in
the 14th century. The individual was central to the Renaissance.
Florence was preeminent in the Renaissance, extolled by men such as the poet Francesco Petrarch. The painter
Giotto began to move painting away from medieval canons, aiming at more realism. Italian trade continued to
flourish providing the funding for these cultural developments.
The Iberian Peninsula was another area of dynamism in the 15th century. The Reconquista, the conquest of the
peninsula under Christian monarchs, was completed by the end of the century the united monarchy of Castile and
Aragon.
Western Expansion: The Experimental Phase:


One of the earliest Atlantic voyages was undertaken by the Vivaldi brothers. The Vivaldis never returned, but
subsequent ventures took Europeans to the Canary Islands, the Madeiras, the Azores, and down the western coast
of Africa. The compass and the astrolabe made venturing into open seas possible.
Prince Henry of Portugal Prince Henry the Navigator was particularly important in supporting the sciences
necessary for trans-Atlantic voyages. He also began the process of colonization, starting with the Azores. A
pattern was established: cash crops grown on large estates, and the use of slaves to work the plantations.
80
Outside the World Network:





Outside the Asia-Africa-Europeans sphere, the Americas and Polynesia developed in relative isolation. Changes
in the two areas were making some societies vulnerable to attack.
The Aztec and Inca empires were fragmented, their central governments controlling their vast territories with
difficulty.
In Polynesia, the period 700 to 1400 saw expansion and migration to the Society Islands. During the same period,
Hawaii was part of this greater Polynesian world, but it was cut off from about 1400. Hawaii was divided into
small kingdoms, and organized hierarchically.
Perhaps as early as the 8th century, Polynesians began to settle New Zealand. As in Hawaii the Maori became
isolated after 1400, and were particularly vulnerable to western colonizers.
Patterns perceived around the world can mask independent developments. While some elements, such as
technology, were hard links between world regions, other developments were indigenous. Moreover, although
increasing ties between regions had an important role, native cultural traditions overwhelmingly survived.
Key Terms:
Iberian Peninsula
Maori
Giotto
Italian Renaissance
Ottoman Turks
Inquisition
Prince Henry
Admiral Zenghe
Black Death
Ethnocentrism
Chapter 15, Quiz Question
1) Which of the following was NOT a symptom of decline in the Arabic caliphate by 1400?
A) The narrowing of intellectual life symbolized by the triumph of religion over literature, philosophy, and science
B) Landlords seizing power over peasants
C) The decline of the Sufis
D) Decline of tax revenues for the state
E) Landlords ceasing to experiment with new agricultural techniques.
2) Which of the following statements concerning Arabic trade after 1100 is most accurate?
A) Arabic control of the seas was strengthened following 1100.
B) Although Arabic trade was reduced, Muslims remained active in world markets.
C) The total collapse of the Islamic world in the 12th century can best be compared to the fall of the Roman Empire.
D) The Arab trading complex was reduced after 1100 to the Middle East.
E) Their economic decline could be compared to that of Rome.
3) Which of the following statements concerning the political fragmentation of the Arabic world in the 1400s is most
accurate?
A) After the fall of the Abbasid caliphate, the emerging Ottoman Empire soon mastered most of the lands of the old
caliphate plus the Byzantine corner.
B) The political fragmentation caused by the fall of Baghdad lasted for several centuries under the decentralized
administration of the Seljuk Turks.
C) The Mongol conquests eliminated any form of centralized government in the Middle East until the 17th century.
D) Following the fall of the Abbasid caliphate, the Middle East became part of the colonial empire of the emerging feudal
states of western Europe.
E) The political system was chaotic for 300 years until the rise of a new political order under the Ottoman Turks.
4) Which of the following statements concerning the Ottoman Empire is most accurate?
A) The rise of the Ottoman Empire restored the full international vigor that the Islamic caliphate had possessed.
B) Turkish rulers promoted trade more actively than did their Arab predecessors.
C) The expansionist power of the Ottoman Empire was very real, but it was not the sole hub of an international network.
D) The Ottoman Empire had no expansionist interests or capabilities.
E) The Ottomans competed with western Europe for Atlantic trade.
81
5) What area represented a new conquest for the Ottoman Empire in the late 1400s?
A) Asia Minor
B) North Africa
C) Southeastern Europe
D) Mesopotamia
E) Russia
6) Which of the following statements concerning the Ottoman Empire is most accurate?
A) Turkish rulers did not promote maritime trade as vigorously as had the Arabs.
B) Scientific and philosophical investigations reached the level of innovation that they had enjoyed under the Abbasids.
C) The Turks refused to patronize the traditional Persian artists and craftsmen who had dominated the later Abbasid court.
D) The Ottomans were more interested in cultural patronage than in military organization.
E) The Ottomans never mastered the full territorial extent of the old caliphate.
7) Which of the following was NOT a reason used by the Ming dynasty to halt the trading expeditions?
A) The opposition of the scholar-gentry and bureaucracy
B) The technological inferiority of Chinese ships and navigation
C) The growing military expenses of the campaigns against the Mongols
D) The traditional preference of the Chinese for Asian products
E) The expense of building the new capital in Beijing
8) Which of the following statements concerning the cessation of state-sponsored trade by the Ming dynasty is most
accurate?
A) The cessation of trade severely damaged the internal economy of China and produced the inevitable peasant
revolutions that overthrew the Ming dynasty.
B) The end of international trade signaled a general decentralization of government in Ming China.
C) Because of the Chinese dependence on imports from abroad, the decision to end the state-sponsored expeditions was
particularly critical in initiating cultural decline.
D) In Chinese terms, it was the brief emphasis on trading and commerce that was unusual, not its cessation.
E) China had long emphasized internal development at the expense of trade.
9) Which of the following was NOT a drawback to the West's emergence as a global power?
A) Western nations lacked the political coherence and organizing ability of imperial China.
B) The West did not begin to establish key maritime and commercial links until after 1450.
C) The Catholic church, long one of the organizing institutions of Western civilization, was under attack.
D) The lives and economic activities of ordinary Europeans, the artisans and peasants, were in serious disarray.
E) Population loss caused further economic disarray and lack of strong leadership.
10) Which of the following was NOT a contributing factor to the economic crises of the 14th century?
A) Withdrawal from the global trading network
B) Bubonic plague
C) Lack of technological advance in agriculture
D) Recurrent famine
E) Labor shortages
11) Which of the following was NOT a source of Western dynamism in the 14th and 15th centuries?
A) The strengthening of feudal monarchy
B) The growth of cities and urban economies
C) Advances in metallurgy
D) Two centuries of peace among the major European nations
E) A cultural reawakening
12) In comparison to medieval culture, Renaissance culture was
A) more concerned with Aristotelian philosophy.
B) more concerned with things of the earthly world.
82
C) disinterested in classical models.
D) based less on urban vitality and expanding commerce.
E) more other-worldly and religious.
13) Which of the following was NOT one of the reasons that Italy emerged as the center of the early Renaissance?
A) Italy was spared the Black Plague due to its geographic location.
B) Italy retained more contact with Roman traditions than did the rest of Europe.
C) Italy led the West by the 14th century in banking and trade.
D) Italy had closer contacts with foreign scholars, particularly those in late Byzantium.
E) Italy was more urbanized than most of Europe.
14) What Italian city-state was best placed to engage in the new, Western-oriented commercial ventures of the 15th
century?
A) Rome
B) Florence
C) Genoa
D) Pisa
E) Padua
15) What was unique about the development of states in the Iberian peninsula?
A) These governments were based on city-states rather than nation-states.
B) Based on Castile and Aragon, the Iberian states were unique in their adoption of Islam.
C) Spain and Portugal developed effective new governments with a special sense of religious mission and religious
support.
D) The states of Spain and Portugal were able to develop without emphasis on the military.
E) They had never participated in the feudal practices of the Middle Ages, which made them more open to change.
16) The key theme of Polynesian culture from the 7th century to 1400 was
A) the adoption of Japanese civilization in the island societies.
B) the development of a uniform written script.
C) contraction as a result of the world-wide epidemic of the 14th century.
D) spurts of migration and conquest that spread beyond the initial base in the Society Islands.
E) large-scale expeditions of discovery that were aimed at establishing colonies in South America.
17) Which of the following statements is most accurate?
A) Without European intervention, there is no reason to believe that the Inca and Aztec empires could not have survived
for several more centuries.
B) Without European interference, the likelihood is that the Inca Empire would have overwhelmed the Aztecs and
established a unified government in the Americas.
C) Because of internal weaknesses, both the Inca and the Aztec empires were receding and might not have survived, even
if the Europeans had not arrived.
D) Prior to the arrival of the Europeans, both the Inca and the Aztec empires had been replaced by other, indigenous
governments.
E) Both the Inca and the Aztecs stopped exploiting subject peoples after 1500 due to the intervention of the Black Plague.
18) Which of the following was NOT a result of the European contact with sub-Saharan Africa after 1500?
A) Trade patterns in west Africa shifted from the Mediterranean to the Atlantic
B) Trade shifted in west Africa from Muslim to European hands
C) Seizure of slaves for European use affected many regions deeply
D) Regional kingdoms lost all influence in west Africa and were replaced by European governments
E) European weapons played an increasing role in the tribal conflicts between north and south.
19) The practice of judging other peoples by the standards and practices of one's own culture of ethnic group is
A) ethnocentrism.
B) genocide.
83
C) anthropomorphism.
D) aversionism.
E) localism.
Essay Questions:
The West and the Changing Balance of World Power
1. Account for the decline of the Middle East in the 1400’s.
2. Evaluate the position of China in the international balance of power in 1450.
3. What were the sources of dynamism that fueled the Western advances relative to other civilizations?
4. Describe early Western exploration and colonization patterns.
5. In what sense were the civilizations outside the world network of cultural exchange (the Americas, Africa, and
Polynesia) vulnerable to interaction with other civilizations?
84
Chapter 16, The World Economy
Summary:
The rise of the West from the 15th and 18th centuries involved distant explorations and conquests resulting in a heightening
and redefining of relationships among world societies. During the classical era, larger regional economies and culture
zones had developed, as in the Chinese Middle Kingdom and the Mediterranean basin, but international exchanges were
not of fundamental importance to the societies involved. During the postclassical period, contacts increased and were
more significant. Missionary religions – Buddhism and Islam – and trade influenced important changes. The new world
relationships after 1450 spelled a new period of world history. The Americas and other world areas were joined to the
world network, while older regions had increased contacts. Trade became so significant that new relationships emerged
among societies and prompted reconsideration of existing political and cultural traditions.
Key Concepts:
The West’s First Outreach: Maritime Power:




By 1500, Europeans had become more aware of the wider world. Ignorance, however, hampered their
explorations. Early voyages on small ships brought little returns.
Technological change rapidly impacted both the scope and nature of European voyages. Mastery of the compass,
added to the use of the cannon, changed everything. Europeans were now able to go farther and protect
themselves on the seas.
The Portuguese led the way, moving down the coast of Africa, and eventually around the Cape of Good Hope.
Vasco da Gama’s arrival in India brought Europe into direct contact with the east. Spanish ships reached the
Americas in 1492, and Ferdinand Magellan sailed around Cape Horn into the Pacific in 1519.
Northern European nations joined the competition late, but soon took the lead. The French claimed Canada, in
1534, and the British followed. The Dutch and the British focused on commerce, forming East India Companies
with monopolies on trade.
Toward a World Economy:






As the voyagers crossed, they brought plants, animals, and microbes in both directions: the Columbian
Exchange. Diets on both sides of the Atlantic were affected, and populations were decimated by imported
disease. The ecological impact was immense.
Europeans began to dominate trade, although still limited to the coastlines in most areas. Trading outposts dotted
the shores of Africa and the Indian subcontinent. In more resistant areas, merchants gathered in areas set aside for
them in towns.
Competition between European nations was spurred by mercantilism, which dictated that colonizers had the
monopoly on exports to their colonies, which, in turn, furnished raw goods at low prices. The practice resulted in
large areas of the world dependant on European goods.
In this first phase of colonization, the impact of Europeans was growing but still limited. At the same time, new
systems of forced labor took root.
Was there a world economy at this time? Chinese adherence to tradition kept western influence to a minimum.
Japan and Korea also limited contact, and even withdrew from trade. In India and the Middle East, the level of
trade was so low that no impact was felt.
The volume of trade increased in the 17th century. Britain turned India into a market for its cotton textiles.
Western European demands for grain let to increased eastern European production, and the intensification of
serfdom.
Colonial Expansion:

Colonization of the American mainland began in Panama. Francisco Pizarro was one of many adventurers who
gained their own empires. The Inca and Aztec were conquered. Missionaries accompanied these enterprises.
85





North American colonies took a different path, often as havens for religious refugees. France and Britain vied for
dominance, yet the commercial attractions were not great. The lower density of natives in North America made
European inroads easier.
Europeans maintained their tradition of nuclear families in their North American colonies, and, in general, more
closely resembled the home countries than colonies in Latin America.
For the most part, colonization in Africa in this first phase was restricted to coastal trading posts. The Portuguese
search for slaves in Angola and the Dutch Cape Colony were important exceptions. As Britain and France
struggled for control of India, their impact intensified, as they made Indian leaders part of their rivalry. In the late
1700’s, Britain took a more active role in administering the colony.
The Impact of European colonization was initially greater at home than abroad. The Seven Years’ War was the
culmination of British-French overseas competition, and the growing European reliance on imports changed tastes
at home.
Globally, Europeans intensified their use of forced labor. At the same time, some areas benefited by the
movement of goods made possible by world trade.
Key Terms:
Mestizos
Mercantilism
John Locke
Mughal Empire
Columbian Exchange
East Indian Company
Treaty of Paris
Calcutta
Ferdinand Magellan
World Econom
Chapter 16, Quiz Questions
1) How was the global trade network of the 15th century different from that of previous eras?
A) There was no significant trade between civilizations prior to the 15th century.
B) Trade prior to the 15th century was limited to the Eastern Hemisphere as a result of the military dominance of Eastern
nations.
C) In previous eras, most attention was given to the development of regional economies and cultural zones, rather than a
global network.
D) Trade in previous eras was almost entirely in the hands of the West as a result of overwhelming advantages in
technology
E) Trade was controlled by the Chinese due to their naval presence in the Indian Ocean.
2) Why did the initiative in early conquest and exploration pass to northern European nations in the later 16th century?
A) Spain and Portugal were defeated in a critical war with the Ottoman Empire.
B) The Dutch and the British improved the design of oceanic vessels, producing faster ships than their Catholic rivals.
C) Famine and disease disastrously reduced the population of the Iberian peninsula after 1588.
D) The Spanish defeat of the English Armada cut England off from further advances in Europe and forced English
attention to foreign conquest.
E) Conquered nations rose up against Spain and Portugal, requiring large forces to suppress them.
3) What was the purpose of the early English voyages to North America?
A) To establish colonies
B) To create a fortified port and trading region
C) To discover an Arctic route to China
D) To drive the Spanish from the Americas
E) To convert natives to Catholicism
4) In what way were the early Dutch and British exploration and trade projects different from those of the Iberian nations?
A) The expeditions of Spain and Portugal did not enjoy government support.
B) Dutch and British exploration owed much to private initiative of merchant groups and the formation of chartered
trading companies.
C) Dutch and British exploratory expeditions were independent of their respective governments.
D) The Dutch and British operated joint explorations in the names of both governments while Portugal and Spain
competed in the competition for conquest.
86
E) The Dutch and British projects were financed with banking capital from Italy and the Florentines.
5) What was the impact of the introduction of American crops into Europe?
A) The introduction of corn and the potato led to major population growth in Europe.
B) Fungi introduced to Europe along with American crops led to a severe decline in agricultural productivity.
C) Although American crops were introduced around the world by European traders, they were not adopted in Europe
itself.
D) Plantation agriculture fueled by slave labor became the norm in European agricultural systems.
E) Most Europeans rejected the new foods as uncivilized and began to import rice from China.
6) Which of the following areas of trade was NOT dominated by the West after the establishment of a global trading
network in the 17th century?
A) The Atlantic
B) The Pacific
C) The Indian Ocean
D) The Mediterranean
E) The Caribbean
7) Which of the following statements best accounts for the Spanish failure to hold a position of dominance in world trade?
A) The Spanish withdrew voluntarily from the race for world trade dominance and established a policy of international
isolation.
B) The Catholic church that dominated Spanish society argued against the establishment of a commercial mentality in
Spain.
C) Spain's interests were increasingly directed toward the destruction of the Ottoman Empire.
D) Spain's internal economy and banking system were not sufficient to accommodate the bullion from the new world and
lacked significant manufacturing capability.
E) Spain exported more finished goods than it imported due to the high quality of its artisans and craftsmen.
8) What economic policy encouraged the development of colonies, particularly by northern Europe countries?
A) Mercantilism
B) Free trade
C) Socialism
D) Communism
E) Boycotts
9) The dominant commercial nations of northern Europe made up the ________ zone of the world economy.
A) dependent
B) core
C) negative trade
D) free trade
E) peripheral
10) Which of the following regions were dependents to the core zone of the global trade network?
A) Holland
B) France
C) England
D) Sub-Saharan Africa
E) Spain
11) Dependence in the world economy and the consequent need to produce unprocessed goods cheaply led to the
development of ________ labor systems.
A) dependant
B) independent
C) coercive
D) socialist
87
E) free
12) Which of the following statements most accurately describes the impact of the development of core-dependent
economic zones on state formation?
A) Forced labor and European influence tended to generate weak governments in dependent regions, while increased trade
revenues tended to generate increasing government strength in core states.
B) While the profits of global trade tended to strengthen the governments of core regions, the creation of colonies
extended powerful governments to dependent zones as well.
C) The efforts of international trade tended to enrich private commercial interests but weaken core governments who were
unable to tap the wealth. Dependent zone governments, based on company organization, tended to be strong.
D) No state government benefited from the creation of core-dependent zones. Neither core regions nor colonies were able
to develop strong, centralized governments.
E) Dependent regions and their governments grew stronger in response to the European threat.
13) Which of the following statements concerning the relationship between Asian civilizations and the world commercial
network of the 16th and 17th centuries is NOT accurate?
A) Asian civilizations had ample political strength and economic sophistication to avoid dependent status.
B) East Asia constituted the civilization that remained most fully and consciously external to the world economy.
C) China depended on extensive government regulation to keep European activities in check.
D) China was able, thanks to the existence of its coastal navy, to prevent the establishment of European ports.
E) Most of the silver mined and exported by the Spanish ended up in China in order for the Spanish to purchase Chinese
manufactured goods.
14) What was the nature of the trade between eastern and western Europe in the 17th century?
A) Western Europe imported grain in increasing amounts from eastern Europe in return for art objects and manufactured
goods.
B) Eastern and western Europe remained economically isolated, as Russia, Poland, and Prussia extended their trade
relationships with the Ottoman Empire.
C) The inability of eastern Europe to produce anything of value to the West frustrated attempts to establish trading
connections between the two halves of Europe.
D) Eastern Europe emerged as a powerful challenger to Western domination of the global commercial network.
E) Eastern Europe provided a trade route for Asian goods because of the role Moscow played as a trade nexus.
15) In which of the following regions was European settlement a significant factor in the establishment of colonies?
A) West Indies
B) Indonesia
C) Dutch South Africa
D) China
E) Arabia
16) Which of the following represents an impact on western Europe from the development of colonies?
A) Colonial development resulted in a greater sense of cooperation among European nations.
B) The development of colonies hastened the growth of centralized governments and destroyed the growth of the
merchant class in western Europe.
C) The use of colonially produced sugar spread widely in Europe.
D) The decline of the Catholic church and its power resulted from many Europeans moving to the colonies.
E) Colonialism had limited impact on Europe due to the policy of natives selling free-access goods.
17) In which of the following colonies did a deeper appreciation of Western institutions and values take place?
A) Spanish Latin America
B) French and British colonies of North America
C) Brazil
D) India
E) Macao
88
18) Why did the southern colonies of the Atlantic seaboard win importance before those farther north?
A) Gold was discovered within the southern colonies of the Atlantic seaboard.
B) The harsh climate of the northern colonies left those regions virtually unsettled.
C) Only the southern colonies were able to eliminate the native Indian population.
D) Cultivation of cash crops produced by coercive labor emerged there.
E) The need to defend the southern colonies against Spanish settlers.
19) Which of the following statements concerning the interaction of North American colonists with the Indians is most
accurate?
A) Colonists interacted with Indians, learned from them, and misused them, but did not forge a new cultural grouping as
occurred in much of Latin America.
B) Constant warfare between the numerous sedentary agricultural tribes and the European colonists resulted in limited
immigration from Europe.
C) Rapid intermarriage between Europeans and the Native Americans resulted in the creation of a new class of people, the
mestizos, who continued to play a significant role in North American colonial development.
D) The occurrence of disease that rapidly decimated the Indian populations of Latin America did not take place in North
America, thus Indian populations remained large and intermingled with the European immigrants.
E) Indian populations grew as a result of new foods and goods introduced by European settlers.
Essay Questions:
The World Economy
1. Describe the early European exploration of the world. Characterize the nature of early settlement.
2. What was the nature of the “Colombian Exchange”?
3. The European expansion of the 16th and 17th centuries created an imbalance in world trade. Discuss the
emergence of dominant core areas and peripheral dependent zones.
4. Prior to 1600, what areas of the world remained outside the global trading network: what areas were then
incorporated into the network after 1600?
5. Of all the colonial systems, the Atlantic colonies of North America most fully incorporated Western civilization.
In what ways were the Atlantic colonies similar to the West? In what ways did they differ?
89
Chapter 17, The Transformation of the West, 1450-1750
Summary:
This chapter is about big changes in Western Europe during the early modern period. The core areas of Western
civilization changed dramatically between 1450 and 1750. While remaining an agricultural society, the West became
unusually commercially active and developed a strong manufacturing sector. Governments increased their powers. In
intellectual life, science became the centerpiece for the first time in the history of any society. Ideas of the family and
personality also altered. The changes resulted from overseas expansion and growing commercial dominance. The
internal changes, such as the Renaissance and Enlightenment, were marked by considerable internal conflict, with focal
points centered on the state, culture, and commerce, with support from technology.
Key Concepts:
The First Big Changes: Culture and Commerce, 1450-1650:








Francesco Petrarch was one of the first Europeans to typify the new spirit of the Renaissance.
The Italian Renaissance, after an abortive start in the 1300s, took shape in the 1400s. Humanism, the intellectual
basis of the movement, looked to ancient Greece and Rome for inspiration. Humanity, particularly in a secular
setting was the focus of artistic and intellectual endeavor.
The Northern Renaissance, starting some time after the Italian, but also spurred by merchant wealth, took its own
direction. It focused on the perfection of man in a Christian context with extraordinary writers such as
Shakespeare and Rabelais. Monarchy experienced a period of cultural and political dynamism.
Communication was deeply changed by the adoption of the printing press in the 1400s. In a different sphere, the
enduring European-style family because prevalent, based on nuclear family.
The Protestant Reformation, begun by Martin Luther, reshaped the religious, then the political, map of Europe.
Reformers promoted public education, particularly in Calvinist areas. The role fo the Catholic Church was
curtailed. However, responding to the Reformation, the Catholic Church finally acquiesced to the attempts of its
members to reform itself.
Religious strife both led to and inflamed growing nationalist conflicts. The Thirty Years War between Spain and
the Holy Roman Empire was fought over religious and nationalist issues. In England, the Civil War was fought
over religion, the extent of parliamentary control, and royal claims to sovereignty. While the religious conflicts
that afflicted Europe were resolved, with some improvement in women’s roles.
An influx of gold and silver form Latin America, and markets that grew as colonization advanced, transformed
the European economy. Commercial agriculture became the rule in Europe. Material wealth for the average
European greatly increased.
Others were hurt by economic change, notably the emerging proletariat. Popular uprisings were common in the
1600s, often leading to calls for a greater political voice. At the same time, persecution of witches burst out,
fueled by fear and hatred on the part of the dispossessed poor.
Science and Politics: The Next Phase of Change:




The impact of the Scientific Revolution spread beyond its immediate sphere, deeply transforming all areas of
thought in Europe.
Copernicus discovered, or possibly passed on, the heliocentric theory, revolutionizing the western view of the
cosmos. Regardless of how he developed his theory, it had an immense influence on scientific studies.
The work of Copernicus was ushered by Kepler’s planetary observations, and Galileo’s research using the
telescope. Broad, fundamental work done by Descartes and Newton established the scientific approach to
knowledge. Scientific studies became popular, and influenced the philosophy known as Deism. According to
Locke, everything could be known through the sense with the use of reason.
Led by France, monarchy entered a new phase: absolutism. Larger bureaucracies and elaborate court ritual raised
monarchs to new heights. Central European monarchs followed the French model, their power strengthened by
the defeat of the Turks. At the same time, ideas of the sovereignty of the people developed. In Britain and the
Netherlands, representative law-making bodies limited the power of monarchs.
90

All of the European countries had commonalities as nation-states: peoples bounded by common culture, within
clearly defined boundaries, constantly at odds with other nations-states.
The West by 1750:




Political forms crystallized around 1750, with few important developments. Only Prussia under Frederick the
Great showed any dynamism, improving agriculture and expanding religious toleration.
The Enlightenment, the spread of the Scientific Revolution to all areas of intellectual endeavor, took hold in the
18th century. With great faith in the goodness and capacity of humanity, and the power of the human intellect,
Enlightenment thinkers set out to fix the world. The movement took place in salons, in coffee houses, in lecture
halls, and was promoted by the growth of scholarly journals.
Consumer demand for imported products influenced colonization. Agriculture developed with improved drainage
and the importation of New World crops. Manufacturing, fueled by capitalism and employing large numbers of
workers, increasingly drove the economy.
Developments in commerce, the economy, and political forms were largely independent, but together they
transformed the West.
Key Terms:
Humanism
Scientific Revolution
Enlightenment
Capitalism
Johannes Gutenburg
Martin Luther
Niccolo Machiavelli
Isaac Newton
Proletariat
Jesuits
Chapter 17, Quiz Questions
1) In characterizing the period from 1450 to 1750 in the West, which of the following statements is NOT accurate?
A) What was once an agricultural society had become a predominantly manufacturing economy.
B) Government powers had expanded as nation-states began to evolve.
C) Science came to form the centerpiece of Western intellectual life.
D) The popular outlook, including ideas about personality and family as well as concepts of nature, had shifted.
E) The idea of Christian unity started to decline, which led to religious wars.
2) Which of the following statements about the Renaissance is NOT accurate?
A) The Renaissance challenged medieval intellectual values and styles.
B) The Renaissance was largely an artistic movement that began in Italy.
C) The Renaissance failed to develop any new ideas concerning political organization.
D) The Renaissance was built on a more commercialized economy.
E) The Renaissance had a southern and northern phase.
3) Which of the following statements concerning Italian humanism is most accurate?
A) Humanists focused on humankind as the center of intellectual and artistic endeavor.
B) Humanists attacked Christianity as rife with superstition and witchcraft.
C) Humanists carved out new literary styles without reference to classical or medieval models.
D) Humanists de-emphasized the corporate and communal aspects of human society.
E) Humanists rejected scientific explanations in favor of spiritual ones.
4) Which of the following accounts in part for the decline of the Italian Renaissance circa 1500?
A) Routes through Russia to the East undercut the Italian monopoly of trade.
B) The creation of a single nation-state in northern Italy sapped the vitality of artistic patronage.
C) French and Spanish monarchs invaded the peninsula, cutting down on political independence.
D) Much of Italy was conquered by the Ottoman Turks after the battle of Lepanto.
E) Atlantic trade routes increased the importance of Italian cities and their role in the slave trade.
5) What was one of the primary differences between the Northern and Italian Renaissances?
A) The Northern Renaissance occurred a century earlier than the Italian Renaissance.
91
B) Northern humanists focused more on religion than their Italian counterparts.
C) There were no major literary figures in the Northern Renaissance.
D) The Northern Renaissance did not make use of the classical languages typical of the Italian Renaissance.
E) Northern kings did not become patrons of the arts.
6) What was the European-style family pattern that emerged in the 15th century?
A) Extended families, early marriage ages
B) Nuclear families, early marriage ages
C) Extended families, late marriage ages
D) Nuclear families, late marriage ages
E) Extended families, marriage optional
7) Which of the following statements most accurately describes the reason Luther picked up widespread support among
the German elite?
A) Luther proposed moving the papacy from Rome to Germany.
B) Luther's support for a more centralized German government under the control of the Holy Roman emperor struck a
responsive chord in German nationalism.
C) German princes who turned Protestant could increase their independence from the emperor, seize church lands, and
control the church in their territories.
D) Luther proposed that indulgences should be collected by the Holy Roman emperor instead of the pope.
E) Luther was seen as anti-Muslim and many German princes saw a way to expand their power into eastern Europe.
8) Which of the following reasons suggests why common people supported the Lutheran Reformation?
A) Luther advocated the overthrow of the authority of the German princes.
B) Lutheranism sanctioned money-making and other earthly pursuits more wholeheartedly than did traditional
Catholicism.
C) Luther's reforms meant that indulgences and other ecclesiastical means of salvation would become less expensive and
more readily available to the poor.
D) Luther advocated redistribution of land and property throughout Germany.
E) Luther supported the Peasant Revolt that broke out throughout Germany.
9) What was the political impact of Calvinism?
A) Due to the location of the center of Calvinism in Switzerland, most governments that accepted the new religion were
city-states.
B) Because of the insistence of Calvinism of the acceptance of a single ecclesiastical authority, Calvinism spread rapidly
among the absolute monarchies.
C) Calvinism was regarded as so potentially revolutionary that it failed to find a foothold outside of Germany.
D) Calvinists sought the participation of all believers in church administration, which encouraged the idea of a wider
access to government.
E) Calvinist doctrine held that kings should appoint bishops and local priests.
10) Which of the following was NOT a result of the Thirty Years War?
A) It reduced German prosperity and power for a full century.
B) The treaty that ended the war established Spain as the principal power of western Europe.
C) The treaty that ended the war granted political independence to the Protestant Netherlands.
D) The population of the German territories was reduced by almost 60 percent.
E) The war established the principle of territorial toleration.
11) The religious wars that followed the Protestant Reformation led generally to
A) the restoration of Catholic unity.
B) the establishment of Protestant dominance.
C) a limited acceptance of the idea of religious pluralism.
D) the end of the involvement of the state in religion.
E) a monolithic Protestantism in Europe.
92
12) Which of the following statements most accurately describes a change in popular mentality as a result of the
Protestant Reformation?
A) Protestants were more likely to credit miracles or divine interruptions in nature's course.
B) Protestant churches, as physical structures, were more closely connected to market activities in the cities, encouraging
the idea that religion and daily life were related.
C) Protestants and Catholics considered the family in more positive terms, not simply as an institution necessary because
of human lust.
D) Religious change tended to discourage the growth of literacy in the era following the Protestant Reformation.
E) Protestants tended to believe what church authorities told them whereas Catholics began to question church doctrines.
13) Which of the following was a Western response to the commercial revolution of the 16th century that was proletarian
in outlook?
A) A more caring attitude toward the problems of the poor
B) A more elaborate family life including greater material wealth
C) A wave of popular protests resulting in uprisings caused by social tensions
D) Greater belief in personal achievement and the demystification of nature
E) Greater admiration for the merchant class and the Catholic church
14) Which of the following statements most accurately summarizes the view of Deists?
A) While there may be a divinity, its role is only to set natural laws in motion.
B) God can be found in all elements of creation, whether plant, animal, or mineral.
C) The authority of the church is paramount, and all political power is derived from divine sanction.
D) The institutional church has failed Western society, and it is necessary for the reestablishment of the church through
new institutions founded by the state.
E) God establishes governments on Earth, and kings are his instruments.
15) How did the Western view of science compare with that of other civilizations?
A) The West was the only civilization to develop scientific and technological expertise.
B) In China, science was based on practical, empirical advances.
C) The West was not alone in developing crucial scientific data, but its thinkers were the only ones to see science in
broader philosophical terms as central to intellectual life.
D) Islam remained vastly ahead of the West in terms of scientific knowledge, despite the clear advances made during the
Scientific Revolution of the 16th and 17th centuries.
E) In the West, science was seen as separate from religion leading to a more religious outlook in the long run.
16) Which of the following descriptions most accurately defines mercantilism?
A) Mercantilism was a scientific theory that attempted to limit the growing dominance of merchants within society.
B) Mercantilism held that government should promote the internal economy in order to improve tax revenues and to limit
imports from other nations.
C) Mercantilism was an economic philosophy that argued that natural laws would determine the flow of goods according
to supply and demand.
D) Mercantilism argued that the money supply should be freed from the supply of bullion and based instead on the flow of
goods within regional markets.
E) Mercantilism is the idea that all trade should be international and free of any restraints.
17) Adam Smith's economic theory advocated
A) government intervention in order to control the flow of bullion through extensive tariff systems.
B) the use of a controlled money supply as a means of limiting inflation.
C) that governments avoid regulation in favor of the operation of individual initiative and market forces.
D) the institution of state-controlled guilds to fix standards of production and wages.
E) that governments should allocate labor and capital whenever they engaged in warfare.
18) Which of the following was NOT a basic principle of the Enlightenment?
A) Society's goals should center on improvements in material and social life.
B) Religions that relied on faith or refused to tolerate diversity were wrong.
93
C) If people were not controlled, general social decline was inevitable.
D) Human beings are naturally good and can be educated to do better.
E) Humans behave according to natural laws based on reason.
19) Which of the following changes associated with treatment of children was associated with the Enlightenment?
A) Parents became more interested in freer movement and greater interaction for young children.
B) Physical discipline of children to encourage their development became more common.
C) Swaddling of infants continued as a means of protecting infants from injury.
D) Childhood was no longer perceived as a stage for learning and growth.
E) Childhood became a period of time where the labor of the children was freely available to the family.
20) Which of the following statements most accurately describes the nature of manufacturing in the later 18th century?
A) The factory system was well established since the commercial revolution of the 16th century and continued to develop
during this period.
B) By the 18th century the economic growth typical of the 16th century had halted, and manufacturing suffered from the
withdrawal of capital.
C) The 18th century witnessed a rapid spread of household production of textiles and metal products, mostly by rural
workers who alternated manufacturing with some agriculture.
D) The lack of new technology caused a bottleneck in the manufacturing processes and led to stagnation in European
productivity.
E) Large scale mass production was begun in places like France and Germany.
21) Which of the following statements concerning mid-18th-century Western society is most accurate?
A) Agricultural changes, commercialism, and manufacturing had combined to produce a rapidly growing population in
the West.
B) Radical changes in the nature of government resulted in the creation of essentially new political forms.
C) As a result of the Enlightenment, established churches no longer were forces to be reckoned with in Western society.
D) The spread of domestic manufacturing destroyed the traditional habits and family patterns of earlier Europe.
E) Most people believed that population growth was self-regulating due to the occurrence of famines.
22) By 1750, the strands of commercial, cultural, and political changes in Europe had combined to create
A) an unstable political environment that would eventually regress backwards.
B) proof of the innate superiority of Western civilization.
C) rapid adaptation not found in other civilizations.
D) an unusual version of an agricultural civilization.
E) a truly socialistic society based on Marxian principles.
Essay Questions:
The Transformation of the West, 1450-1750
1. In what ways was Western Europe in 1750 different from the medieval West?
2. How would you contrast the Renaissance and the Scientific Revolution?
3. In what ways did the commercial revolution of the 16th century change the social structure of the West?
4. How was the absolute monarchy of the 17th century different from the political forms of the Middle Ages?
5. How did the Enlightenment affect changes in popular outlook in European society?
94
Chapter 18, The Rise of Russia
Summary:
The rise of the Russian Empire, unlike the rise of Western colonial empires, although altering power balances through
Eurasia, involved only limited commercial exchange. After freeing themselves form Mongol domination by 1480, the
Russians pushed eastward. Some extension of territory also occurred in Eastern Europe. Regional states, many differing
from Russia, were present, with Lithuania and Poland rivaling Russia into the 17th century. Russia, with its Byzantineinfluenced culture, had been unimportant in world affairs before the 15th century. Russia then entered into new contacts
with the West without losing its distinct identity. Between 1450 and 1750, many lasting characteristics of the eastern
European world were formed.
Key Concepts:
Russia’s Expansionist Politics under the Tsars:




Mongol dominance of Russia lasted until the mid-15th century. Under Ivan III, Russia was liberated, and began
its rise to power.
Russian culture was not deeply changed by Mongol rule, but it had stagnated. The tsars began the process of
reviving and recreating Russian culture, largely by tying its past to that of the Byzantine Empire.
For the tsars, Russian independence meant territorial expansion, pushing back the Mongols. Cossacks, peasantadventurers, were used to settle the newly-taken lands. The lands, in turn, could be used to buy the loyalty of the
boyars or nobility. Russian expansion put an end to the periodic emergence of nomadic people from central Asia.
The tsars began a deliberate policy of contact with the West, both commercially and culturally, establishing the
enduring practice of following the Western lead. The Time of Troubles, a period of rebellion and invasion, was
ended by the new Romanov dynasty.
Russia’s First Westernization, 1690-1790:




Peter I, the Great, looked with confidence to the west for guidance in restructuring his empire.
Ruling as autocrat, Peter the Great used bureaucrats to avoid reliance on the nobility. He also instituted the longlived tradition of a secret police force.
Peter regularized the organization of Russia’s government, including provincial and urban governments. These
changes, along with increased manufacturing output, added to the power of the central government, including
provincial and urban governments. These changes, along with increased manufacturing output, added to the
power of the central government. The tsar also undertook cultural changes, attempting to bring Russian dress and
manners in line with other European countries. While his reforms brought change, they were also partial and
were deeply resented by many.
Catherine the Great, taking control of the Russian government from her husband, Peter III, continued many of
Peter I’s policies. Her reforms, too, were selective, restricting the freedom of peasants, and intellectuals wanting
further reforms. Her dynamic leadership added to the territory of the Russian state, and improved the position of
Russia in European politics. The reigns of Peter I and Catherine completely changed the political and cultural
place of Russia in European developments.
Themes in Early Modern Russian History:



Russian nobles had a more important role than their western counterparts, who, by this period, were largely
ornamental.
The position of serfs declined in the 1600s and 1700s, satisfying the nobility, who, in turn, effectively managed
this agrarian population for the tsars. In fact, Russian serfdom closely resembled slavery. In other areas of
Eastern Europe, peasants were similarly exploited. While peasant society was largely self-governed, the status of
the peasants declined throughout the 18th century.
Russian society tended towards polarization, with only a small merchant class. Agricultural practices saw little
improvement.
95


Russian intellectuals, influenced by western ideas, called for reform. Peasant revolts were brutally represses,
including the Pugachev rebellion, put down by Catherine the Great.
Eastern Europe in this period tended to fall into the western European sphere in this period. Copernicus, a Pole,
was an example of an intellectual with close ties to western European developments. Some smaller countries
were engulfed by more powerful neighbors. Poland is a notable example.
Key Terms:
Old Believers
Alexis de Tocqueville
Catherine the Great
Cossacks
Boyars
Ivan III
Peter I
Romanov Dynasty
Serfdom
Obruk
Chapter 18, Quiz Questions
1) What was the state of the Russian economy immediately after the expulsion of the Mongols in the 15th century?
A) Russia was already a dependent region within the global economy dominated by the West.
B) Fueled by the establishment of the Tatar trade routes with the East, Russia had developed a significant export trade and
merchant class.
C) Russia had become a more purely agricultural economy, dependent on peasant labor.
D) Russia's economic ties were almost exclusively with the Ottoman Empire and hence with Africa.
E) Russia was following the West into an economy dominated by merchants and capitalists.
2) Ivan IV, called Ivan the Terrible,
A) wished to confirm tsarist autocracy by attacking the authority of boyars.
B) abandoned the principles of territorial expansion in favor of centralizing power at home.
C) allied himself with the Russian aristocracy in a policy of political decentralization.
D) was responsible for the incorporation of Poland into the Russian empire.
E) destroyed the hold of the Vikings on Russia and eastern Europe.
3) Why did the Russian expansion policy focus particularly on central Asia?
A) The Russians wished to seize control of the trade routes with China.
B) Most of the Russian population remained ethnically Mongol with clear cultural ties to central Asia.
C) There were natural barriers to westward expansion.
D) The Russians were motivated by a desire to push the former Mongol overlords farther back to prevent renewed
invasion.
E) Russians wanted to control the lucrative fur trade in Siberia.
4) What was the impact of early Russian expansion on central Asia?
A) There was no impact at first but as time went by, there was a gradual assimilation of the Chinese speaking groups
B) Independent central Asia, the source of nomadic cultures and invasion forces, was eliminated.
C) Chinese trade was refocused through Russia and eastern Europe.
D) Russian society became more culturally and ethnically homogeneous.
E) Central Asia revived economically as a result of the expansion.
5) Which of the following was NOT a form of contact with the West during the reigns of the Ivans?
A) Diplomatic missions were sent to leading Western states.
B) Western merchants established outposts in Moscow and other centers.
C) Italian artists and craftsmen were imported for building projects.
D) Military alliances were signed with Spain and Portugal.
E) Territorial expansion along the Polish and Lithuanian border
6) Which of the following was NOT an accomplishment of Tsar Michael?
A) The reestablishment of internal order
B) Driving out foreign invaders
C) Updating practices in the Orthodox Church
96
D) Inclusion of part of the Ukraine, including Kiev, in the Russian empire
E) A successful war against Poland
7) Politically, what aspects of Western culture did Peter the Great emulate in Russia?
A) Parliamentary government
B) Aristocratic control of the bureaucracy
C) Royal control over the bureaucracy and reorganized military
D) Republicanism
E) Separation of church and state
8) In what way did Peter the Great shift the foreign policy interests of Russia?
A) Peter the Great initiated the Russian invasion of China.
B) Peter abandoned the Russian policies of expansion in favor of consolidation at the center.
C) Peter allied himself with the traditional Russian enemy, the Ottoman Empire.
D) Peter's wars with the Ottoman Empire and Sweden indicated a westward shift in Russian expansion.
E) Peter began Russian expansion into North America and Japan.
9) Which of the following was NOT one of Peter the Great's political reforms?
A) Improved military weaponry
B) Elimination of noble councils
C) Reduction of taxes on the peasantry
D) Systemization of law codes
E) Specialized bureaucracy
10) What was the purpose of Peter the Great's policy of economic development?
A) To create a free working class devoted to industry rather than agriculture
B) To create a wealthy merchant class capable of financing further expansion
C) To raise the standard of living of all Russians
D) To avoid the need of importing military armaments
E) To create a peasant class that could support the nobility
11) What was the limitation of Peter the Great's policies of cultural Westernization?
A) He made no attempt to introduce Western education, particularly in technological subjects.
B) Westernization was limited to the elite.
C) Peter made no attempt to enforce cultural reforms.
D) Despite tsarist proclamations, Westernization failed to have any impact on Russian society.
E) He did not have firsthand knowledge of Western institutions.
12) What was Catherine the Great's attitude toward the program of Westernization?
A) Catherine flirted vigorously with the ideas of the French Enlightenment, but failed to take steps to abolish serfdom.
B) Catherine rejected the concepts of Westernization in favor of a distinctive Russian culture.
C) Catherine earned the title of Enlightened monarch by fully embracing the ideas of the French Enlightenment, including
the abolition of the serfs.
D) Catherine was eager to continue the policy of Westernization, but was unable to attract Western philosophers to
backward Russia.
E) She was more interested in the process of Asianization and reforming the aristocracy.
13) What was the relationship between Catherine the Great's government and local administration?
A) The government established by Catherine the Great exercised exclusive control over both central and local
government.
B) The tsarist government combined a real monopoly of formal politics by a central administration, but had little power
over the noble-owned estates.
C) The powers of the government were exercised almost entirely by the nobility and little formal authority was exercised
by the central administration.
D) Under Catherine the Great, there was little central authority, but village governments under the jurisdiction of peasant
97
headmen continued to function.
E) The local bureaucracy held most of the judicial power whereas the tsarist government controlled tax revenues.
14) Which of the following statements concerning the political voice of the Russian nobility is most accurate?
A) The politics of the Russian nobility were expressed through service in the tsarist state and the preeminent power they
wielded over the peasant serfs.
B) The Russian nobility exercised enormous influence through the powerful assemblies that continued to legislate through
the 18th century.
C) The exclusion of the Russian nobility under Catherine the Great from any role in the central government or the military
relegated them to strictly local authority.
D) Under Catherine the Great the Russian nobility was virtually exterminated.
E) The nobility virtually ran the Russian empire with very little interference from the central government.
15) The development of coercive labor systems in eastern Europe was indicative of
A) Russia's economic dominance over Asian trade.
B) eastern Europe's growing economic subordination to the West.
C) the creation of a core economic zone in Russia.
D) the development of industrial strength in the region.
E) poor race relations in the East.
16) What was one of the primary differences between the social organization of the West and Russia in the 17th and 18th
centuries?
A) Russia's merchant class was more fully developed than that of the West.
B) The West had no formal aristocracy by the 18th century, but in Russia the nobility retained their political and social
function.
C) Russia saw a progressive intensification of serfdom while the West was relaxing this institution in favor of other labor
systems.
D) The agricultural labor of the West was subject to a more restrictive form of serfdom than that of Russia.
E) There was much more forced labor in Western Europe than in Russia due to the needs of the growing industrial base.
17) What was one of the primary limitations on the expansion of the agricultural economy?
A) The lack of cheap labor
B) The lack of markets for Russian surpluses
C) Lack of technological improvement
D) The lack of estate agricultural systems in Russia
E) The lack of available and arable land
18) Which of the following is NOT indicative of Russia's growing economic dependence on the West during the 18th
century?
A) Exportation of raw materials-furs, grain, and timber
B) Importation of luxuries and some manufactured goods
C) A coercive labor system
D) Self-sufficiency in metals and weapons
E) The need to imitate the West
19) A substantial merchant class in Russia during the 18th century
A) was restricted to the cities of Moscow, Novgorod, and St. Petersburg.
B) sprang from peasant origins.
C) emerged among the lesser nobility within the Russian cities.
D) failed to develop.
E) gained political power.
20) Compared to Western governments, how great a role did the Russian government play in economic development?
A) Because of the absence of a merchant class, the Russian government played a greater role than was common in
Western states.
98
B) Like the West, the government's role in the economy was largely restricted to establishing tariffs and protective
measures for domestic industries.
C) Because of the dependent nature of the Russian economy, the government played a relatively less significant role in
economic development.
D) The Russian government had no role in economic development, a task it abandoned to foreign investors.
E) Unlike the West, all production and distribution of goods was controlled by the government.
21) How did the Polish government differ from the Russian model after 1600?
A) Poland was more urbanized.
B) The central government was powerless.
C) There was an absence of a merchant class.
D) Poland lacked a landed aristocracy.
E) It was a monarchy.
Essay Questions:
The Rise of Russia
1. What was the nature of early Russian expansion under the Ivan’s?
2. In what way did Peter the Great reform the economy and government of Russia through Westernization?
3. In what ways were the policies of Westernization undertaken by Peter the Great and Catherine the Great more
appearance than substance?
4. What were the primary differences by the 18th century between Russia and the West?
99
Chapter 19, Early Latin America
Summary:
The new Latin American empires of Spain and Portugal maintained special contacts with the West. Western forms were
imposed on indigenous cultures as the militarily superior European invaders conquered their lands. Latin America
became part of the world economy as a dependent region. The Iberians mixed with native populations and created new
political and social forms. The resulting mixture of European, African, and Indian cultures created a distinctive
civilization. Indian civilization, although battered and transformed, survived and influenced later societies. Europeans
sought economic gain and social mobility; they used coerced laborers or slaves to create plantations and mine deposits of
precious metals or diamonds.
Key Concepts:
Spaniards and Portuguese: From Re-conquest to Conquest







The Christian re-conquest of the Iberian Peninsula shaped the monarchies of Spain and Portugal. Spain, formed
by the unification of Aragon and Castile, emerged as a militantly Christian state. Persecution of the peninsula’s
Jews followed.
Iberian traditions were transferred to conquered lands, including slaveholding and the patriarchal family. Portugal
and Spain created centralized colonial governments, dependant on professional bureaucrats and the clergy.
The first phase of conquest – from 1492 to 1570 saw the establishment of the administrative framework.
Encomiendas, grants of Indian workers, were given to settlers. From 1570 to 1700, the framework was fleshed
out as institutions took shape.
The Spanish experience in establishing control of the Caribbean islands was formative. Hispaniola (1493),
Puerto Rico (1508), and Cuba (1511) were settled by encomenderos. The Spanish developed a pattern of
colonial urban design, and political institutions such as governors. By the 1520s, a shift to ranching and sugar
plantations had taken place, with devastating results for the native populations.
The conquest of the Americas was undertaken by individuals under royal authority. Hernan Cortes succeeded in
taking Tenochtitilan and killing Moctezuma II. Spanish New Spain took the place of the Aztec confederacy. To
the south, Francisco Pizarro conquered the Inca, whose capital at Cuzco fell in 1533. Pedro de Valdivia founded
the city of Santiago, Chile in 1541. Francisco Vaszuez de Coronado led an expedition into the southwestern
United States.
The men who undertook these expeditions, from a variety of backgrounds, hoped to become wealthy. Their
superior military technology, disease, and existing division within the native empires led to their success.
The morality of destroying Indian societies was questioned by many. Yet it was justified by most on the basis of
natural inferiority. Bartolome de las Casas was one of the most outspoken critics of abuses. Yet by the time he
gained a hearing, the conquests were essentially accomplished.
The Destruction and Transformation of American Indian Societies:


Population decline occurred in all of the American peoples. The small numbers of natives remaining were then
more vulnerable to further disruption.
While enslavement of Indians was generally stopped by about 1550, forced labor was common. For political
reasons, the practice of granting encomiendas was stopped by the 1620s. The practice of mita, or forced labor, in
Peru, used Indians for mining and other state projects. In spite of widespread economic changes, indigenous
culture showed great continuity.
Colonial Economies and Governments:


In terms of revenue, mining was the most important colonial enterprise. The looted gold from the first decades of
conquest was followed across the Atlantic by newly-mined gold and silver.
Mexico and Peru held the greatest stores of silver. At Potosi in modern Bolivia, and Zacatecas, in Mexico,
mining communities developed. Modernization of silver extraction, using mercury from Huancavelica, greatly
100




increased silver exports. The mining industry led to the growth of other domestic industries, such as cloth
manufacture.
Indian traditional agriculture continued, along with the Spanish haciendas, rural estates. On the latter, crops such
as sugar were grown for export.
Although industries such as woolen cloth production made the colonies more self-sufficient, mining dominated
the economy. Spaniards had the monopoly on trade with Spanish America. All trade passed through Seville, with
its consulado, or merchant guild. Spain kept other nations out of its commercial sphere with galleons and the use
of fortified ports such as Havana. In spite of the great wealth flowing into Spain, much of it flowed out to the rest
of Europe. In any case, the revenue from taxation was greater.
By the Treaty of Tordesillas in 1494, Spain and Portugal divided their conquests. Spain ruled its empire with the
use of letrados, lawyers. The mass of colonial law was recodified in 1681 as the Recopilacion. The Council of
the Indies governed the colonies from Spain through two viceroys. Their domains were further divided into 10
judiciaries each: the audiencias. The Catholic Church was a major component of rule. The religious orders were
responsible for conversion and education, and sometimes for the protection of natives. The role of the orders was
later assumed by the secular church hierarchy, controlled by the Spanish crown.
European culture and thought was imported and disseminated largely through the church. Even offices of the
Inquisition were established.
Brazil: The First Plantation Colony:



The Portuguese first reached South America in 1500, at Brazil, but the territory was ignored for decades.
Portuguese nobles were granted captaincies, nearly autonomous domains. They were followed by Jesuits, and a
series of coastal settlements grew up.
Sugar can was by far the greatest commodity, demanding large amounts of capital and labor. A model plantation
colony, Brazilian society started with white planters at the top, and ended with slaves. A governor-general
represented Portuguese rule. Unlike Spanish America, Brazil was only part of the vast Portuguese rule. Unlike
Spanish America, Brazil was only part of the vast Portuguese empire. Its dependence on Portugal was great, and
the colony was slow to develop its own cultural life.
European political developments were echoed in the colonies. Competition in Europe was also played out in the
colonies. However, Portugal’s hold on Brazil was extended when Paulistas explored the interior. Gold strikes in
the region of Minas Gerais led to a gold rush. Massive importation of slaves led to their making up half the
population. By the mid-18th century, Brazil was the world’s greatest gold producer. Mining led to further
development of the interior, and destruction of more native populations. Rio de Janeiro became the colony’s
capital.
Multiracial Societies:


Three groups came together to form the complex Latin American society.
Miscegenation, mixing of races, produced mestizos, who were neither native nor Spanish. The sociedad de
castas – society of castes – used race, wealth, occupation, and place of birth to determine a person’s place in
society. Attempts to sort out the new categories had practical implications, as legal restrictions were applied to
different groups. Although at the top of the hierarchy, whites were also subdivided in to Spanish-born
peninsulares and those born in the colonies: creoles. Considerations such as age, class, and gender carried over
from Iberian traditions.
The 18th Century Reforms:



The intellectual climate of the Enlightenment had its impact on the colonies. Clubs of amigos del pais met to
discuss reform.
Spain’s hold on its colonies was threatened by piracy and the competition of other European nations. Spanish
oversight of its colonial systems flagged, leading to corruption and loss of revenue. The War of the Spanish
Succession, resolved by the Treaty of Utrecht in 1713, opened the Spanish trading empire to France and England.
The Bourbon reforms of Charles III and his successors revived Spanish control of the colonies. The reforms did
not recast the colonial structure, but sought to make it more effective. Jose de Galvez investigated the
administration of Mexico before taking over as minister of the Indies. He created a system of intendants that
101


made the administration more effective. The loss of Florida and Havana led Spain to strengthen the colonial
militia. Grants of monopolies increased the government’s control over key commodities. Trade was opened to
other nations, leading to a boom in some areas, such as the Buenos Aires ranching industry. However, some areas
experienced economic decline. In the long run, the reforms led to widespread dissatisfaction as groups, such as
creoles, lost their place in the government.
Under the Marquis of Pombal, similar reforms were carried out in Brazil. He followed the English model of
mercantilism, to make good the revenue loss due to declining gold production. He granted monopolies to develop
agriculture, and the region of the Amazon in particular. To people the area, he encouraged mixed marriages. His
reforms were only partially successful.
Population growth in Latin America went along with exploitation of new areas and a higher volume of exports to
Europe. Latin American society, disrupted by the reforms of the Bourbons and Pombal, was in ferment. The
Comunero Revolt of 1781 nearly ended colonial rule in New Granada. The native rising of Tupac Amaru in
Peru took three years to suppress. Brazil, on the other hand, experienced no major revolts. Deep divisions in
colonial society made unified protest unlikely.
Key Terms:
Sociedad de Castas
Peninsulares
Tupac Amaru
Charles III
Viceroyalties
Habsburg Monarchy
Creoles
Hernan Cortez
Encomiendas
New Spain
Chapter 19, Quiz Questions
1) Which of the following was NOT characteristic of Iberian society?
A) Heavy urbanization
B) Absence of slaveholding traditions
C) Emphasis on nobility
D) Emphasis on patriarchal ideals
E) Patriarchal families
2) How was the commercial experience of the Portuguese extended to the Americas?
A) The Portuguese were familiar with the routes to the Far East that resulted in the circumnavigation of the globe.
B) The Portuguese were responsible for the use of galleys in the Atlantic passage.
C) The Portuguese experience in Africa and their involvement in slave trading were extended to the Americas.
D) The Portuguese introduction of tobacco estate agriculture into the Caribbean was a significant factor in American
colonization.
E) The Portuguese began commercial trade after the Spanish discovered gold in the Americas.
3) How did Caribbean cities differ from those of Europe?
A) American cities were laid out in a grid plan.
B) American cities lacked churches.
C) There was an absence of commerce in American cities.
D) There were no Caribbean cities
E) They were built near water.
4) Which of the following statements concerning the men who conquered much of Latin America for Spain is NOT
accurate?
A) Few of the conquerors were professional soldiers.
B) Leadership was based on reputation and past achievement.
C) An agreement was drawn up between the leader and the Spanish crown that granted authority for the expedition in
return for a promise to pay one-fifth of all treasure to the crown.
D) The conquerors, many of humble origins, came to see themselves as a new nobility entitled to dominion over a new
peasantry, the Indians.
E) Most conquerors were cruel and ambitious and many were of humble origins.
102
5) All of the following were advantages the Spanish enjoyed over the Indians EXCEPT
A) epidemic disease that weakened the Indians and reduced their numbers.
B) the use of firearms and superior steel weapons.
C) internal divisions and internal rivalries among the Indians.
D) the failure of nomadic tribes to mount significant resistance to conquest.
E) the use of horses by the Spanish since the natives did not have any similar animal
6) Why were the encomiendas discontinued by the 1540s and all but gone by the 1620s?
A) The Spanish crown was unwilling to see the growth of a new nobility and the decline of the Indian population made
them less attractive.
B) The Indians refused to continue to serve under the imperial conditions established in the 1500s and demanded a new
arrangement with the Spanish crown.
C) Despite the continued economic prosperity of the encomienda system, the Spanish crown discontinued them in order to
establish a free labor system in the Americas.
D) The viceroys of the American colonies ordered their abolition in favor of enslavement of the Indian population.
E) As new land became scarce, existing encomiendas were divided among owners instead of new ones being created.
7) Which of the following statements most accurately describes the nature of the economy in Spanish America?
A) The majority of people in Latin America were rapidly organized into a light industrial economy intended to produce
goods for American society.
B) Although the majority of people were engaged in agriculture, the whole Spanish commercial system was organized
around the mining economy.
C) Latin America received almost its entire food supply from Europe, because the Spanish colonies were entirely geared
to the production of sugar on estate agricultural systems.
D) The agricultural economy of Latin America absorbed virtually its entire population because of the absence of large
domesticated animals prior to 1800.
E) Most people made their living in manufacturing, with a small segment working in the agricultural sector.
8) Which of the following statements concerning the agricultural system of Spanish America is NOT accurate?
A) Colonists faced with declining Indian populations found landownership more attractive.
B) In places where large sedentary populations existed, Indian communal agriculture of traditional crops continued.
C) Plantation crops like sugar and later cacao were exported to Europe in sufficient quantities to exceed the value of
bullion exports.
D) Spanish America remained predominantly an agrarian economy.
E) South America was not known as a producer of wheat or rye.
9) Which of the following statements concerning the Spanish commercial system is most accurate?
A) The merchant guild in Seville had virtual monopoly rights over goods shipped to America and handled much of the
silver received in return until the 18th century.
B) All trade from Spain after the mid-16th century was funneled through the city of Madrid.
C) Nearly all trade with the Spanish colonies was carried in ships built in the New World and captained by colonists.
D) The intent of the consulado was to keep prices in the Spanish colonies low.
E) The Council of the Indies regulated all trade and established a sub-council in Spanish America.
10) In what way did the importation of American bullion negatively affect the Spanish economy?
A) The apparent dependence of Spain on its bullion supply caused European bankers to avoid loaning money to the
Spanish government.
B) American bullion made up nearly 90 percent of Spain's state revenue, so that any disruption in the arrival of silver from
America plunged the government into crisis.
C) The supply of bullion to Spain was highly irregular, so that the government could not accurately gauge its income or
anticipate its expenditures.
D) The arrival of American treasure contributed to a sharp rise in prices and a general inflation.
E) The supply of bullion kept the Spanish from purchasing manufactured goods from England.
103
11) The Catholic church introduced all of the following to American life EXCEPT
A) universities.
B) the construction of baroque churches.
C) the sense of independence from the state.
D) establishing missions in outlaying areas.
E) monasteries.
12) What was the primary difference between the Spanish and Portuguese empires?
A) Portuguese colonies did not have the heavy influence of the Catholic church found in Spanish colonies.
B) Portuguese colonies lacked the bureaucratic structure that characterized the Spanish colonies after the middle of the
16th century.
C) Unlike the Spanish empire that was almost exclusively American, the Portuguese empire included colonies and
outposts in Asia and Africa as well as Brazil.
D) The Portuguese colony of Brazil was more intellectually independent of the mother country than were the Spanish
colonies in Latin America.
E) The Portuguese treated their slaves better than did the Spanish due to the influence of the Catholic church.
13) What conditions undercut the position of the Brazilian sugar plantation economy?
A) A demographic disaster among the Indians of Brazil resulted in a shortage of labor for the sugar plantations shortly
after 1700.
B) Competition from English, French, and Dutch plantation colonies in the Caribbean led to rising prices for slaves and
falling prices for sugar.
C) The European market was flooded with sugar supplied from Asian colonies.
D) A series of unusually wet winters flooded the traditional sugar regions and caused Brazilian planters to seek new land
for the production of sugar.
E) The growth of manufacturing made plantations less important to the economy.
14) What was the negative impact of the discovery of gold on Portugal?
A) Portugal failed to develop internal industries because the supply of gold allowed the Portuguese to purchase
manufactured goods from other European countries.
B) Portugal was forced to hand Brazil over to the more powerful Dutch navy.
C) Portugal's agricultural economy was devastated by the flow of capital from domestic produce to Brazilian imports.
D) Portugal became increasingly dependent on France due to the wars of the 18th century.
E) Portugal's trade with China began to decline rapidly which caused a negative trade balance.
15) Which of the following statements concerning the 18th-century Spanish reforms in America is most accurate?
A) The English system of justices of the peace was introduced to replace the audiencias, causing the decline in influence
of the educated lawyers in America.
B) The Spanish colonies were largely demilitarized, as Spain became increasingly dependent on the navy of France to
protect its interests in America.
C) The French Intendancy system was introduced but the traditional patterns of influence and power among the Creole
bureaucrats was disrupted.
D) The Spanish reforms did little to alter the patterns of local administration and the fundamental structure of power and
authority in the Latin American colonies.
E) Governments were formed based on natural rights based on the writings of John Locke.
16) Which of the following was NOT an impact of the Bourbon reforms on colonial economy?
A) The establishment of state monopolies on items like tobacco and gunpowder
B) The extension of the plantation agricultural system to Cuba
C) The growth of Buenos Aires as a regional trade center
D) The frontiers of Spanish America became fixed
E) More government control over the economy
17) What was the impact of the 18th-century reforms on slavery in Brazil?
A) Slavery was abolished.
104
B) The slave trade with Africa was abolished.
C) Slave imports were restricted to encourage the elimination of the plantation economy.
D) Brazil remained as profoundly based on slavery in the late 18th century as it had ever been.
E) It led to an overall decline in economic exports to the mother country.
18) Which of the following statements most accurately describes the state of the Portuguese and Spanish American
colonies by the middle of the 18th century?
A) The population of the American colonies never recovered from the initial loss of Indian population.
B) While the population of the American colonies was growing, largely due to the importation of African slaves, the
economy was largely stagnant.
C) The American colonies of Spain and Portugal were experiencing considerable growth in population and productive
capacity.
D) Following the disruption of the plantation economies, the Portuguese and Spanish American colonies experienced
population loss and economic depression.
E) England was threatening to take over the Caribbean colonies and Spain fell into economic dependency status.
19) What accounts for the general failure of 18th century colonial revolutions against Spanish and Portuguese rule?
A) The various racial and social groups, fearful of unsettling the social hierarchy, failed to work together to unseat the
colonial governments.
B) The Spanish reforms were generally so successful that there were few dissatisfied elements in the colonies.
C) Rebel armies enjoyed no success against the military forces of the colonial government.
D) Indians were so little incorporated into American society that they were not interested in changing the political
organization of their masters.
E) They were challenging popularly elected governments led by the Creoles.
Essay Questions:
Early Latin America
1. In what sense can it be said that the nature of the Latin American colonies was simply a reflection of Iberian
society?
2. Compare and contrast the colonial economies of Spanish America and Brazil.
3. How was the society of the Latin American colonies organized? What determined the social hierarchy?
4. What was the intent of the Spanish reforms of the 18th century? To what extent were they successful?
105
Chapter 20, Africa and the Africans in the Age of the Atlantic Slave Trade
Summary:
The new Latin American empires of Spain and Portugal maintained special contacts with the West. Western forms were
imposed on indigenous cultures as the militarily superior European invaders conquered their lands. Latin America
became part of the world economy as a dependent region. The Iberians mixed with native populations and created new
political and social forms. The resulting mixture of European, African, and Indian cultures created a distinctive
civilization. Indian civilization, although battered and transformed, survived and influenced later societies. Europeans
sought economic gain and social mobility; they used coerced laborers or slaves to create plantations and mine deposits of
precious metals or diamonds.
Key Concepts:
The Atlantic Slave Trade:




The Portuguese led the way in exploring the African coast, establishing factories to facilitate trade, El Mina, in
West Africa, being the most important. More interested in trade than conquest, Portuguese relations with African
rulers was generally peaceable. They did seek conversion; their greatest success was Nzinga Mvemba, ruler in
the Kongo. On both sides, attitudes to the foreign culture were mixed. As the Portuguese explored down the
coast they founded Luanda, expanded later to the colony of Angola. Other nations followed the Portuguese,
bringing competition. The trade in slaves developed slowly, as only one of many commodities. Slavery in
Europe, except for Iberia, had disappeared. It took development of sugar plantations to raise a need for slave
labor.
How many slaves were exported? The numbers are problematic, but as much as 12 million were taken across the
Atlantic in four centuries. High mortality on board and on the plantations coupled with low birth rates, kept
demand high. Brazil received about 42% of the slaves. The trade was initially in Africans form Senegambia,
then form modern Zaire and Angola, and finally from Dahomey and Benin.
The trans-Saharan slave trade was mostly in women, for concubines in Islamic lands, while the trans-Atlantic
trade took men for agricultural labor.
For much of the history of the slave trade, Portugal controlled traffic. From 1630, competition increased, and the
Dutch seized El Mina in 1637. The British Royal Africa Company was followed by similar French enterprises.
Following the Portuguese example, other nations established small outposts on the coast. Purchases of slaves
were made through local rulers, although sporadic raids also occurred. A system based on a healthy male slave
the Indies piece set prices on the commodity. Was the trade profitable? In itself it was probably not as profitable
as it was crucial to the triangular trade that developed.
African Societies, Slavery, and the Slave Trade:



African traditions of slavery were deeply engrained in economic systems, and in the social hierarchy. The
condition of slaves varied greatly. The practices of using slaves as concubines was part of the widespread
practice of polygamy. In Islam, slavery was accepted, but not enslavement of Muslims.
Europeans tapped into the established slave trade, but also intensified the trade European penetration into the
continent brought other changes. Endemic warfare typified much of Sub-Saharan Africa, with the exception of
Songhay. One of the results was a constant supply of slaves. As Europeans settled along the coast, they provoked
a shift in the power relations of states in the interior. Ghana and Songhay were able to make use of their position
as intermediaries in commerce.
Asante and Dahomey serve as examples of the impact of the slave trade. Asante, slave trade. Osei Tutu unified
the Akan clans, taking the title asantahene, or supreme ruler. For nearly two centuries, ca. 1650-1820, Asante
ruled along the Gold Coast. Agaja, king of Dahomey, made use of European firearms to establish an empire.
Controlling the slave trade in its region, Dahomey remained an independent, unified state longer than most of its
neighbors. Cultural development continued on the continent. Political experimentation included increasingly
powerful monarchs and monarchs limited by governing councils. The arts continued to thrive, often led by artisan
guilds. Demand for African crafts to suit European tastes increased Africa’s contact with other world areas.
106

The Swahili east coast still formed part of the Indian Ocean trading area, bringing ivory, gold, and slaves from the
interior of the continent. Clove plantations using African slaves developed on Zanzibar and other islands. The
interior of peoples led to a network of dynasties in east central Africa. The kingdom of Bunyoro was one of the
most prominent in the 1500s and 1600s. Islamization, following the breakup of Songhay, took on a more militant
phase. Usuman Dan Fodio, a Muslim scholar of the Fulani peoples of the Sudan, was inspired by Sufism.
Under his leadership, the Fulani took over several Hausa states, creating the Sokoto kingdom. The expansion of
the Fulani had its impact on the West African interior. Slavery in the Sudan expanded, as a result of the wars and
of European pressures.
White Settlers and Africans in Southern Africa:


Southern Africa was initially little influenced by the slave trade. Bantu migrations into the area changed its
economy, earlier dominated by Khoikhoi and San hunters and sheepherders. The Bantu peoples in Southern
Africa were organized into small chiefdoms. Their expansion further south brought them into contact with Dutch
settlers, moving inland form the Cape Colony in search of land to farm. Under British control from 1815, the
colony’s expansion led to warfare with the Bantu. The Boer Great Trek coincided with upheavals among the
Bantu peoples.
A new ruler led the Nguni peoples from 1818: Shaka, chief of the Zulu. His ruthless leadership created a
powerful Zulu state that survived his death. Shaka’s work was part of the mfecane. Swazi and Lesotho emerged
at the same time, resisting Zulu expansion. All of Southern Africa was involved in the turmoil of the mfecane.
The African Diaspora:






The slave trade not only brought slaves forcibly into an alien culture, it also brought foreign products into Africa.
The Middle Passage was always traumatic for slaves, and often lethal.
Africans in the Americas were typically employed in agricultural labor, but other occupations existed.
A hierarchy developed distinguishing saltwater slaves newly arrived from their creole descendants. The latter
could gain more skilled work in better conditions, and stood a better chance of being manumitted. Slave
communities sometimes divided along lines similar to those in Africa. Their numbers grew until they were as
much as 80% of colonial populations. The North American slave population had a higher birthrate and less need
of newly enslaved Africans, and thus was more cut off from Africa than slaves in other areas.
Africans in slavery maintained as much of their culture as was possible, depending in part on whether they found
themselves with other slaves from their native region. Africans were converted to Christianity, but their religious
traditions obeah survived. In Brazil and Haiti, African religion survived intact in candomble and vodun
respectively. Resistance to slavery was omnipresent, but rarely successful. Palmares, a community of fugitive
slaves in Brazil was an exception, as were the Maroons of Jamaica. The Maroons of Suriname also established
their own enclave.
Abolition resulted from changes outside of Africa. While self-interest on the part of European countries was
possible reason, the main impulse seems to have come from European intellectuals. Influenced by such men as
William Wilberforce, the British stopped the slave trade in 1807. Slavery was finally abolished in the Americas
when Brazil stopped the practice in 1888.
Key Terms:
Asante
Dahomey
Great Trek
Middle Passage
Saltwater Slaves
Vodun
William Wilberforce
Cape Colony
Triangular Trade
Zulu Wars
Chapter 20, Quiz Questions
1) Which of the following was NOT considered a factor in the development of large kingdoms in Africa?
A) A population expansion that followed the diffusion of iron tools and improved agriculture
B) The collapse of the Christian kingdom in Ethiopia in the face of Muslim advance
C) European demand for slaves
D) The use of firearms
107
E) Improved agriculture
2) Which of the following statements concerning the early Portuguese trade forts is most accurate?
A) The Portuguese trade forts permitted the political control of much of the African interior.
B) Where Portuguese trade forts were established, large European colonies rapidly developed.
C) Most of the forts were established with the agreement or license of local rulers.
D) The Portuguese trade forts were the nodal points for colonial administration on the model of the American colonies.
E) They were intended as the first step toward inland colonization and domination.
3) Which of the following statements concerning the Portuguese presence in east Africa is most accurate?
A) After the 1570s, the Portuguese gained complete control over the east African trade in the Indian Ocean and the Red
Sea.
B) Unlike the Portuguese settlements on the coast of western Africa, Mozambique became the basis for a large European
colony.
C) Portuguese bases in east Africa gave them access to the gold trade of east Africa, but they were unable to completely
disrupt the Muslim trade with the region.
D) The Portuguese presence in east Africa precipitated the population movement among the Bantu that became the
Mfecane.
E) The Portuguese presence was preceded by French and the English colonization movements.
4) Following the model established by the Portuguese, which of the following was NOT one of the principal patterns of
European contact with Africa?
A) A system of fortified trading stations
B) The combination of force and diplomacy including alliances with local leaders
C) The predominance of commercial relations
D) Widespread European colonization
E) Fair trade with native peoples for free-access goods
5) Which of the following statements about slavery in Europe before 1450 was most accurate?
A) Europe had never had a tradition of slavery going back to the Roman Empire.
B) Slavery had been extensive in the ancient world, but had died out during the Middle Ages in all regions in the West.
C) Slavery had died out in the Middle Ages in most of Europe except along the military frontier between Christians and
Muslims in the Mediterranean.
D) Slavery was common to Europe in all eras prior to 1450, although it tended to become more dominant during the
Middle Ages.
E) During the Middle Ages, most European states took conquered people as slaves but only if they were Muslims.
6) What development led to an intensification of the Portuguese involvement in the African slave trade?
A) The extension of the agricultural system in Europe
B) The Black Death, which decreased the available supply of labor in Europe
C) The development of sugar plantations on the Atlantic island of Madeira
D) The growth of the Ottoman Empire
E) Spanish reconquista and the establishment of New Granada
7) Which of the following statements concerning the volume of the slave trade to the American colonies is most accurate?
A) The volume of the slave trade dwindled rapidly after the 17th century because the plantation economies of the
Americas collapsed.
B) The high volume of the slave trade was a necessity because slave mortality was high and fertility was low leading to a
loss of slave population.
C) The shift from plantation economies to mining economies in which slaves were not utilized rapidly diminished the
American demand for slaves.
D) Demand for slaves continued to remain high due to the competing demand for slaves in Europe.
E) The volume of the slave trade decreased as mortality rates increased and the American economy shifted to
manufacturing.
108
8) Which of the following statements concerning the shape of the commerce in African slaves is most accurate?
A) The emergence of the Atlantic slave trade caused the immediate end of the older trans-Saharan slave trade in the hands
of the Muslims.
B) The Atlantic trade drew its slaves almost exclusively from southern Africa until the 19th century.
C) The African states of the interior actively resisted the slave trade.
D) The Atlantic slave trade drew slaves from across the African continent and its concentration shifted from Senegambia
to central and western Africa over time.
E) Fewer slaves crossed the Atlantic than the Sahara due to the military needs of the Arabic kingdoms.
9) In what way did the trans-Saharan slave trade differ from that of the Atlantic slave trade?
A) The Atlantic slave trade was carried out almost exclusively by Muslims.
B) The trans-Saharan slave trade was carried out in much greater volume than the Atlantic slave trade.
C) The trans-Saharan slave trade concentrated on women, but the Atlantic slave trade concentrated on young men.
D) The African preference for retaining young male slaves to extend kinship lines implied that primarily women were
available to the Atlantic trade, while men converted to Islam were more likely trade objects for the trans-Saharan trade.
E) The trans-Saharan slave trade was much more arduous and cruel than the trans-Atlantic trade.
10) How did the profitability of the slave trade compare to that of other contemporary business ventures?
A) The slave trade was less profitable on the whole than other business ventures because of the high costs and risks
involved.
B) The slave trade became increasingly dangerous and risky, so that by the 18th century its returns were minimal and
most participants were bankrupted.
C) Profits from the slave trade in the 18th century were so lucrative that capital derived from the trade in human beings
was used to capitalize the European Industrial Revolution.
D) The slave trade was little more profitable than most business activities of the age and was not a major source for the
Industrial Revolution of Europe.
E) The slave trade was less profitable in the Sahara trade but extremely profitable for the Atlantic slave traders.
11) Which of the following statements concerning slavery on the continent of Africa before the arrival of the Europeans is
most accurate?
A) Most African societies were egalitarian—that is, most people enjoyed the same social status—and slavery was
generally unknown.
B) Slavery in Africa was restricted to those areas where Islamic influence had introduced the concept in the period after
the 7th century.
C) While common elsewhere in Africa, slavery was not common in the forest states of west Africa prior to the coming of
the Europeans.
D) In many African societies, the control of slaves was one of the few ways in which individuals or lineages could
increase their wealth and status.
E) Slavery had virtually been abolished in Africa before the arrival of the Europeans due to tribal and clan conflicts.
12) What was the political impact of the presence of Europeans on the African coast?
A) States were more likely to form in the savanna regions of Africa.
B) Strong centralized states began to form on the coastline in close proximity to the European trade forts.
C) West and central African kingdoms just inland from the forts began to redirect their trade and expand their influence.
D) State formation in Africa took place on the Indian Ocean coast away from the trade routes established by the
Europeans.
E) Europeans immediately divided up the entire continent and established colonies.
13) In what way did the European slave trade enable centralizing states to expand more rapidly?
A) Slaves were traded for firearms that allowed expanding states to overpower their neighbors, resulting in more slaves.
B) The Europeans rapidly created military alliances and added their armies to those of their slave-trading allies.
C) The European slave trade weakened the states of central and western Africa, allowing the centralizing states of eastern
Africa to expand without competition.
D) The slave trade was restricted to the coasts, leaving the political units of interior Africa free of European interference.
E) The slave trade declined in the cross-Sahara trade routes leading to the introduction of stronger Muslim kingdoms.
109
14) On the east coast of Africa, the Swahili trading cities
A) were decimated following European naval attacks.
B) fell entirely within the orbit of the Portuguese global trade network.
C) continued their commerce in the Indian Ocean with both the Portuguese and the Ottoman Turks.
D) were unique in Africa because of their refusal to participate in the slave trade.
E) abandoned trade with Islam in favor of trade with the Europeans.
15) One of the unique features of the east African coast that differed from west Africa was
A) the existence of the slave trade.
B) the establishment of plantations using African slave labor.
C) the existence of large European colonies.
D) the widespread appearance of epidemic disease.
E) connection to global trade.
16) What was the impact of the Mfecane?
A) It destroyed the Bantu peoples of eastern Africa.
B) As a result, the Portuguese were able to absorb most of the east African Swahili trading cities.
C) The Boers were forced out of southern Africa.
D) The resulting political disruption sent African groups fleeing before the Zulus into both Portuguese coastal regions and
the Boer farms of southern Africa.
E) Greater cooperation between natives and settlers
17) Why were Africans sought for plantation labor in the Americas?
A) There was no other labor supply available in the Americas.
B) West Africans were already familiar with metallurgy, herding, and intensive agriculture whereas Indians were not.
C) Sugar was a crop native to Africa and exported to the Americas from there.
D) Africans rapidly expanded their population in the Latin American colonies.
E) Native Americans refused to perform the labor and were few in numbers.
18) How were the British colonies of the southern Atlantic coast of North America different from the Latin American
colonies?
A) There was no slavery there.
B) Although urban slavery was common, there was no plantation agricultural system on the North American mainland.
C) Manumission of slaves tended to be more common in the British colonies.
D) The British colonies depended less on imported Africans because of the positive rate of growth among the slaves.
E) The British colonies were primarily industrial.
19) Which of the following statements concerning slave families is most accurate?
A) Despite enormous difficulties, slaves continued to live in family units.
B) The onerous conditions of slavery and the shortage of women resulted in the creation of artificial clans and extended
lineages.
C) Lacking women, men substituted slave "fraternities" for the more traditional organization.
D) The conditions of slavery destroyed family concepts among Africans in the New World.
E) Family relations were stronger in the slave families than in the settler families.
20) Which of the following statements concerning resistance to slavery is most accurate?
A) Recalcitrance, running away, and direct rebellion were present wherever slaves were employed.
B) Although there were one or two slave rebellions in isolated areas, in general there was no organized resistance to
slavery.
C) Resistance to slavery was experienced on the mainland, but the plantation economies of the Caribbean suffered no
rebellions.
D) The only consistent location for slave resistance and rebellion was the British colonies of the southern Atlantic coast.
E) Few slaves ever rebelled against their captivity or their owners.
110
21) Which of the following statements concerning the ending of the slave trade is most accurate?
A) In general the Enlightenment with its advocacy of free trade accepted the practice even if it did not approve of the
slave trade.
B) Leadership in the drive to abolish the slave trade was assumed by France.
C) While it is true that legitimate products began to replace slaves in the European commerce with Africa, it is difficult to
demonstrate a link between economic self-interest and the abolition of the slave trade.
D) The end of the Atlantic slave trade in the 19th century led to the end of slavery in Africa itself.
E) Rousseau and Smith favored the continuation of slavery but only under certain conditions.
Essay Questions:
Africa and the Africans in the Age of the Atlantic Slave Trade
1. What model did the Portuguese establish for the exploitation of Africa?
2. Describe the structure of the Atlantic slave trade. Consider the volume of slaves by century the places to which
slaves were shipped, and the origins of slaves within Africa.
3. Why did Europeans participate in the Atlantic slave trade?
4. To what extent was slavery a feature of African society prior to the coming of the Europeans? How was it
intertwined with the development of more centralized states?
5. Evaluate the causes for the end of the Atlantic slave trade?
111
Chapter 21, The Muslim Empires
Summary:
Much of Africa followed its own lines of development between the beginning of the 15th and 19th centuries. The rise of
the West and the Western-dominated economy, however, was a powerful force in influencing the course of African
history. The Atlantic slave trade predominated in economic affairs after the middle of the 17th century. The forced
removal of Africans had a major effect in some African regions and was a primary factor contributing to the nature of
New World populations. African culture became one of the important strands in the development of American
civilizations. Despite the rise of the West and the slave trade, nearly all of Africa remained politically independent and
culturally autonomous. Among the important trends, Islam consolidated its position in sub-Saharan and east Africa, while
in many parts of Africa, independent states formed and expanded.
Key Concepts:
The Ottomans: From Frontier Warriors to Empire Builders:






In the 13th century, the Mongols made it possible for the Ottoman Turks to move from a role as servants of the
Muslim world to become its masters. The Ottomans quickly moved across the Middle East and into Europe,
although the conquest of Constantinople by Mehmed II came only in 1453. By 1566, they ruled all of the former
eastern Roman Empire. Their land empire was matched by mastery of the Mediterranean Sea.
The Ottoman state granted great independence to the military aristocracy to which it owed its success. These
nobles, granted conquered lands, eventually came to threaten the sultans’ power. The Janissaries, infantry made
up largely of conquered peoples, formed the new military core of the empire.
Early sultans ruled directly, as political and military leaders. Later, the sultans ruled through their viziers, and
through manipulation of the powerful groups within the empire. As the empire grew and the sultans became
surrounded by ritual and luxury, the power of the viziers grew.
Mehmed II rebuilt and improved Constantinople. Suleyman the Magnicent’s Slueymaniye mosque was built at
the apex of Ottoman culture in the 16th century. The city was restored to its position at the point of commercial
exchange between east and west. Merchants and artisans were again central to the city’s culture. The Turkish
language became the official language of court and literature.
The long success of the Ottoman Empire has been shadowed by the disruption caused by its decline. Like other
empires, as conquests ended, some of the Ottoman dynamism was lost. Oversight of the vast empire was
hampered by poor communication, and widespread corruption among officials resulted. As concerns about
succession led to the sequestering of royal offspring, succeeding emperors were increasingly ineffectual. The
power of the sultans was usurped by other in the empire.
Weakness within the empire coincided with external pressure. The battle of Lepanto in 1571 ended Ottoman
naval dominance. As the Portuguese rounded Africa, they were able to bypass Ottoman control of the spice trade.
Silver form Latin America led to crippling inflation in the Ottoman Empire. A brief period of able rule in the
1600’s strengthened, but did not completely restore, the integrity of the empire.
The Shi’a Challenge of the Safavids:



The Safavids, like the Ottomans, came to power on the Eastern fringes of the Muslim world, as champions of
Islam. However, they embraced Shi’ism, and struggles with the Ottomans were intensified by religious conflict.
Established by Sail al-Din of a family of Sufi mystics, they converted the Turks near Ardabil. Their Shi’a
followers, called the Red Heads, grew in numbers. The victories of the Safavid leader Isma’il led him to be
named shah in 1501. The Safavid expansiono led to war with the Ottomans. The great Safavid defeat at
Chaldiran in 1514 did not end their power, but did stop the spread of their empire and Shi’ism.
Shah Tahmasp I, a Turkic successor to Isma’il restored the stability of the empire. Shah Abbas I, the great
brought the empire to its apogee. The shahs managed to turn the Turkik leaders that challenged their power into a
warrior aristocracy. Shah Abbas built up slave regiments, as had the Ottomans.
Although the Safavid rulers were of Turkic background, they adopted Persian as the court language. Their
worldly power was buttressed by claims to be imams, or successors of Ali. They also used mullahs to add
religious support for their rule. Shi’ism came to be an integral part of Safavid distinctiveness.
112



Abbas I was a major patron of craft and trade revival, as well as the arts. At Isfahan, his capital, the court
dominated city life. Magnificent mosques and royal tombs decorated the city.
The Safavid and Ottoman empires shared many cultural traits. In both, as the nobility grew in power, their
exploitation of the peasants increased. Shahs and sultans were important patrons of the arts and crafts. Women
were limited in both their public roles and even in creative pursuits allowed to them in other cultures. Women of
Turkic and Mongol backgrounds lost ground as their cultures were changed by contact with Arabic and Persian
traditions. At court in both empires, women could wield great power, though indirectly. It appears that women
could retain some control over inherited property.
In spite of Abba’s achievements, his empire was short-lived. Weak successors were easily manipulated, although
such shahs as Abbas II were more able rulers. In 1722, Nadir Khan Afshar usurped the throne, inaugurating a
period of unending conflict.
The Mughals and the Apex of Muslim Civilization in India:







Babar, founder of the Indian Mughal dynasty, showed the same leadership ability and cultivation of the arts as
Shah Abbas I and sultan Mehmed II. His main goal of reclaiming his kingdom in central Asia was never
achieved, but he managed to win much of northern India. His son Humayan was an able successor. However,
the dynasty reached its high point under Humayan’s son Akbar.
Although a minor at his succession, Akbar was able to hold on to his throne. His ambitious program, aimed at
unifying his empire, included social reforms, the creation of a new faith, and erasing divisions between Mughals
and Hindus. His proclamation of a new religion in 1582, Din-i-Ilahi was intended to marry Islam and Hiduism.
Like the Safavids and Ottomans, Akbar granted lands to his nobles, yet he left many Hindu rulers in place.
Akbar’s social reforms included improving the plight of the urban poor, and changing marriage customs to protect
women. He outlawed sati, and tried to ease the seclusion of women.
Akbar’s ambitious plans were only partially successful. Jahangir and Shah Jahan followed him in succession, but
did little to build on his achievements. Under the Mughals, India fell behind in the sciences, although exports of
the textile industry remained important.
Although Jahangir and Shah Jahan were much more devoted to pleasure than Akbar, their patronage of the arts
was substantial. Many of India’s finest monuments date from their reign. Mughal architecture blended Persian
and Indian traditions.
Nur Jahan, the wife of Jahangir, took the power her husband neglected. Her influence brought able men to court,
and was used to help charities. Mumtaz Mahal, consort of Shah Jahan, had a smaller role, but her tomb the TajMahal is the grandest of India’s monuments. For other Indian women, reforms did little to improve their status.
Aurangzeb, son of Shah Jahan, was an able ruler but devoted his energies to expansion and cleansing Islam of
Hindu impurities. He was very successful in the first ambition, but uprisings occurred in the north while he was
on campaign in the south. His attack on the position of Hindus was even more disruptive, and overturned Akbar’s
attempt to bring peace. Although the Mughal empire was large at his death, it was weakened by rivalries.
Marrata risings and the emergence of new sects added to the strain. Attacks on the Sikhs turned the Din-i-Ilahi
sect from original goal of blending Hindu and Muslim traditions, to a rigidly Hindu, anti-Muslim religion.
Key Terms:
Mughal Dynasty
Taj Mahal
Imams
Sail al-Din
Gun Powder Empires
Sikes
Janissaries
Red Heads
Hagia Sophia
Golden Horn
Chapter 21, Quiz Questions
1) Which of the following statements concerning Ottoman naval power is most accurate?
A) The Ottoman Empire never developed a navy, thus were unable to control the Mediterranean or challenge the growing
naval supremacy of the West.
B) Powerful Ottoman galley fleets captured major island bases in the eastern Mediterranean, but were unable to prevent
Western advances in the Indian Ocean.
C) The Ottoman Empire was sufficiently powerful at sea that until the 18th century they were able to monopolize the
Arabian Sea.
113
D) The Ottomans retained their naval dominance in the Indian Ocean, but almost immediately lost all of their
Mediterranean possessions.
E) The Ottoman Empire used both Western and Chinese naval technologies and established naval bases in Italy.
2) What permitted the Janissaries to gain a position of prominence in the Ottoman Empire?
A) Their control of artillery and firearms gave them prominence over the aristocratic Turkish cavalry.
B) Their control of the bureaucracy made them indispensable to the operation of the empire.
C) As members of the royal family, they had access to the sultans.
D) They rapidly gained control of the mosques of the Ottoman Empire and were able to define religious orthodoxy.
E) Islamic law defined their authority as written in the Quran and the Sunna.
3) What did the Ottomans do to Constantinople following its fall in 1453?
A) The Ottomans destroyed the city and moved their capital to Sophia.
B) The original city remained, but in a much reduced condition that the Ottomans did little to restore.
C) Soon after its conquest, the Ottoman sultan undertook the restoration and beautification of Constantinople.
D) The Ottomans rapidly abandoned Constantinople to the leaders of the Orthodox church who were responsible for its
restoration and the construction of significant churches.
E) They used it only as a port city but did rebuild the walls and turned the churches into mosques.
4) In what way were the artisans of Constantinople similar to their counterparts in the West?
A) They had begun to form a proletariat.
B) Like their counterparts in medieval European towns, the artisans were organized into guilds.
C) In the capital city of the Ottoman Empire, artisans were free of governmental supervision.
D) Like the earlier West, craft production was limited and there were few independent artisans.
E) They produced goods that were much in demand in Asia and other eastern markets.
5) Which of the following was NOT a cause for the decline of the Ottoman Empire?
A) Local officials began to retain increasing amounts of revenue for their own purposes.
B) Oppressive demands of local officials caused the peasantry to abandon their holdings and flee.
C) The ability of individual sultans to rule declined.
D) The addition of European military technology such as light artillery made the Janissaries so powerful that they could
challenge the authority of the sultan.
E) Western Europe grew more powerful and challenged their existence.
6) Which of the following represents a difference between the declines of the Abbasids and the Ottomans?
A) Only the Ottoman Empire suffered from the intervention of mercenary soldiers in politics.
B) Only the Abbasids declined as a result of revolts by peasants and townsmen oppressed by the landed classes.
C) Only the Ottomans failed to develop a principle of succession leading to civil strife.
D) The Ottomans were at a much greater disadvantage as a result of the more potent threat from the West.
E) The Abbasid decline was much more gradual than that of the Ottoman Empire.
7) Why was the battle of Chaldiran in 1514 so important?
A) The battle established the military supremacy of the Safavids over the Ottomans and marked the end to eastern
expansion of the Ottoman Empire.
B) The Safavids were dealt a devastating defeat that checked the westward advance of Shi'ism and decimated the ranks of
the Turkic warriors who had built the Safavid empire.
C) The combined armies of the Safavids and Ottomans defeated the Mughal armies and ended the policy of expansion
undertaken by the Mughal emperors of India.
D) The defeat of the Safavids by a Western army reduced the Islamic empire to economic dependency on the West and
military inferiority to the other Muslim empires.
E) It signaled the beginning of Islamic regional dominance, which would lead to the beginning of secular type
governments in the Middle East.
8) What was the status of the Turkish chiefs under the Safavid shahs?
A) Unlike the Ottomans, the Turkish cavalrymen were driven from the Safavid empire.
114
B) The former Turkish warriors were replaced entirely by the indigenous Persian nobility who formed a mercenary
military force.
C) Like the Ottomans, the Turkish chiefs were gradually transformed into a warrior nobility with assigned villages and
peasant labor.
D) The former Turkish nomads were reduced to slavery, but still composed the foundation of the Safavid military.
E) Turkish chiefs were nominally independent of any centralized control or rule.
9) How did the Savafid economy compare to that of the Ottomans?
A) Only the Ottoman sultans actively sought to encourage handicraft production and trade in their empire.
B) The Ottoman empire benefited in the short run from non-Muslim traders (Christians and Jews) who had extensive
contacts with overseas empires that the Safavid empire lacked.
C) The Safavid empire made a more concerted and successful attempt to enlarge their market economy, particularly by
attracting merchants from the West.
D) The Safavid economy, because of the geographical location of the empire, was oriented exclusively toward the East
and had no contact with Western merchants.
E) The Safavids traded mainly in the Indian Ocean basin, and the Ottomans traded in the western Mediterranean.
10) What led to the rapid demise of the Safavid empire?
A) Like the Ottoman Empire, the lack of a principle of succession led Abbas the Great to eliminate all capable rivals,
leaving no capable ruler following his death.
B) The Safavid defeat at the battle of Panipat at the hands of a Russian army stripped the empire of its military forces just
as pressure from outside enemies increased.
C) The collapse of the Safavid economy in the 18th century diminished the revenues of the empire to the point that the
central government could no longer function.
D) The successful conquest of the Ottoman Empire overextended the Safavid resources, so that the central government
became increasingly inefficient.
E) The threat of Mongol conquest destabilized the government, which led to a revolt of the Janissaries.
11) Which of the following descriptions of the accomplishments of Babur is NOT accurate?
A) He was a fine military strategist and fierce fighter who went into battle alongside his troops.
B) He wrote one of the great histories of India and was a fine musician.
C) He reformed the ineffective Lodi bureaucracy to create a streamlined administration.
D) He was a fine musician and designed gardens for his new capital at Delhi.
E) He defeated a much larger force in 1526 at the Battle of Panipat, which led to the establishment of his dynasty.
12) Which of the following statements most accurately characterizes the reign of Humayan?
A) Humayan was able to build on the reforms of his father and enjoyed one of the most stable and prosperous reigns of
the Mughal dynasty.
B) Humayan completed the task of crushing opposition to the Mughal dynasty during the first 20 years of his reign, but
settled into dissolute behavior later in his life.
C) Despite the seeming stability of Babur's reign, his son Humayan was exiled by 1540 and forced to fight to restore the
Mughal rule.
D) Humayan was assassinated shortly after his accession and the Mughal dynasty was briefly replaced by a Hindu family.
E) Humayan fell ill and during his long illness the army mutinied leading to a new dynastic ruler.
13) What was the critical feature of Akbar's new religion, the Din-i-Ilahi?
A) It attempted to blend elements of the many faiths with which he was familiar as a means of reconciling Hindus and
Muslims.
B) It sought to purify the Islamic faith and remove Hindu influences by adopting many of the Sufi teachings already
extant in India.
C) Like the Safavids to whom early members of the Mughal dynasty fled in exile, Akbar adopted militant Shi'ism.
D) Although it retained the formal appearance of Islam, the popular aspects of the new religion were almost entirely
Buddhist.
E) It was very similar to Zoroastrianism but included elements of Christianity and Judaism.
115
14) What was the outcome of the Din-i-Ilahi?
A) Accepted by Hindus who benefited from the lenient policies of Akbar, the new religion was rejected by the Muslims.
B) Accepted by Muslims who saw it as a means of cementing the authority of the Mughals over the traditional Hindu
rulers, the new religion was rejected by Hindus.
C) The new religion was widely accepted during Akbar's lifetime and became the state religion of the Mughal empire.
D) By Akbar's death in 1605, the Din-i-Ilahi was rejected by both Muslims and Hindus.
E) It maintained a large number of believers until the introduction of Christianity in the 18th century by the British.
15) What was the state of the Mughal dynasty at the outset of the reign of Aurangzeb?
A) Although the cultural level of the empire had declined, the administrative reforms of Shah Jahan renewed the spirit of
the empire.
B) The status of women in the Mughal empire was higher than at the beginning of the dynasty.
C) The Mughal bureaucracy remained efficient and effective, but the military had lapsed into technological conservatism.
D) The Mughal bureaucracy was bloated and corrupt, the army was backward in weaponry and tactics, and the peasants
and artisans had seen living standards fall.
E) Affairs of state had taken precedence over military issues due to the demands of the bureaucracy.
16) What was the impact of Aurangzeb's religious policies?
A) Aurangzeb's religious policies gravely weakened the internal alliances between Hindus and Muslims and disrupted the
social peace that Akbar had established.
B) Aurangzeb's religious policies succeeded in removing Buddhism from India.
C) The emperor succeeded in reestablishing the political and social dominance of the Hindu majority in India.
D) Aurangzeb's introduction of Christianity to India began the process by which millions of Indians were converted to the
new religion.
E) More freedom for all religious groups with the exception of Christians
17) What was the state of the Mughal empire following Aurangzeb's death in 1707?
A) The Mughal empire had shrunken so much during Aurangzeb's reign that the dynasty controlled only Bengal.
B) The empire included more territory than ever before and there was greater religious homogeneity than earlier in the
reign.
C) The empire was far larger than earlier, but control and state revenues passed increasingly to regional lords who gave
little more than tribute payments to the emperors.
D) The empire collapsed in the face of a Safavid invasion from the Indus River valley.
E) The empire managed to arrange treaties with western powers, which in the long run provided enough resources for the
empire to grow.
Essay Questions:
The Muslim Empires
1. Discuss the social and political organization of the Ottoman Empire.
2. Compare and contrast the causes for decline and collapse in each of the Islamic early modern empires.
3. Compare and contrast the social and political organization of the Safavids to that of the Ottomans.
4. How did the religious complexion of Mughal India differ form that of the other Islamic empires? What impact
did religious diversity have on the Mughals?
116
Chapter 22, Asian Transitions in an Age of Global Change
Summary:
East and Southeast Asian early modern trends were highly diverse. Most Asian peoples, except in the islands of Southeast
Asia, were only marginally affected by the European arrival. India, China, and Japan were not fundamentally reshaped by
the West. The peoples o feast Asia developed new political and social strengths while following a policy of isolation in
response to global trends. Vasco da Gama’s voyage to India had opened the way to the East for Europeans, but it soon
became clear that Europeans had little to offer Asians in exchange for their desired to be conquered by Europeans, but the
latter’s sea power allowed control of spice exports and regulation of some parts of the Asian trading network. The
Europeans participated in the existing economic and political system, rather then attempting to capture it.
Key Concepts:
The Asian Trading World and the Coming of the Europeans:





The Asian trading network linked the Pacific and Indian oceans in three commercial zones. The Arab zone,
including the Red Sea and Persian Gulf, furnished glass and textiles from the Middle East. From the Indian zone
came cotton textiles from the Middle East. From the Indian zone came cotton textiles, and from China paper,
porcelain, and silks. Valuable spices still dominated the trade, coming from Ceylon and Indonesia. Coastal
routes were preferred by all. At the time the Portuguese arrived in the region, no central power controlled trade,
and military power was rarely used. The Portuguese changed the rules.
Lacking goods desired in the East, the Portuguese resorted to force to obtain the spices they came for. From
1502, when da Gama first entered Asian waters, the Portuguese used their advantage to capture ports. To fortify
their growing network, they took Goa and Ormuz. The Portuguese aimed to establish a system in which they
would control all traffic in the Indian Ocean.
The Portuguese were never able to extend the monopoly they desired, even by using the most brutal measures.
The Dutch and English arrived in the 1600’s with the Dutch taking an early lead. They built the port of Batavia
on Java, well positioned for the spice trade. The Dutch trading empire followed the same lines as the Portuguese.
Yet the Dutch eventually turned to peaceful cooperation, concentrating on transporting goods.
In general, the Europeans remained on the coastlines, with a few exceptions. For example, the Dutch controlled
the north of Java, installing coffee plantations. The Spanish conquest of the Philippines in the 1500s, starting with
Luzon and the nearby islands, failed to take Mindanao the northern islands. Tribute systems were established,
leaving local rulers in place.
Converting Asians to Christianity was made difficult by the secure position of Islam in many areas. Francis
Xavier, a Jesuit missionary, made headway in converting low-caste Hindus. Robert di Nobili learned Sanskrit, to
help convert high-caste Indians, but had little success. Spanish work in the Philippines led to more converts.
Christianity there, however, was a Filipino blend of traditional and Christian beliefs. Great continuity of customs
and religious practices in the Philippines mitigated the impact of Christianity.
Ming China: A Global Mission Refused:





The Ming dynasty was founded by Zhu Yuanzhang, of peasant stock. Influenced by time spent in a Buddhist
monastery, he led a rebel group and defeated the Mongol ruler. He claimed the title of Hongwu emperor in 1368,
and began a purge of Mongol influences.
The Ming emperors reestablished and extended the examination system.
Hongwu cleaned up corruption at court. In addition, he tried to rid the palace of intrigues of royal wives and
eunuchs.
His programs included public works, improving agricultural irrigation and encouraging farming on abandoned
lands. The increasing power of the land-lords greatly lessened the impact of these improvements. This gentry
class created its own culture to justify its increasing power. Under the Ming emperors, the low place of women
and the young was intensified.
Agricultural expansion and imports from the Spanish and Portuguese stimulated an economic boom. Maize,
sweet potatoes, and peanuts were imported and became staples. The high demand for Chinese goods meant an
influx of American silver. Foreign traders were allowed only on Macao and Canton, and Chinese merchants
117



fared well. In imitation of the gentry, merchants invested in land. The fine arts flourished as well, mainly along
traditional lines. The novel achieved its classic form.
Under Emperor Yunglo, admiral Zenghe led seven expeditions to the west. The size and scope of these
undertakings demonstrated the Chinese capacity to undertake global expansion.
Yet the Chinese retreated instead, closing themselves off more firmly than ever. Missionaries to china such as
Matteo Ricci and Alan Schall, chose to convert the country from the top down, but they met with little success.
Toward the end of the 16th century, the Ming court was weakened by corruption, and the rulers distanced
themselves from the people. Public works lapsed, and disaffected farmers turned to banditry. Rebellion brought
the Ming dynasty down in 1644, when the last emperor hanged himself to avoid capture.
Fending Off the West: Japan’s Reunification and the First Challenge:



A series of military rulers brought an end to daimyo warfare in Japan. The first, Nobunaga, used European
firearms to depose the Ashikaga shogun in 1513. After his death, Toyotomi Hideyoshi pursued his predecessor’s
killers. Ambitious as well as able, Hideyoshi undertook the conquest of Korea, but failed. His successor,
Tokugawa Ieyasu, was acknowledged shogun by the Emperor. Ruling from Edo, the Tokugawa shoguns ended
daimyo warfare.
European contacts with Japan were increasing in the period of unification, and indeed influenced events. In their
own way, firearms and commerce each helped the Tokugawa rulers. Nobunaga patronized Christian missionaries,
hoping to lessen the power of militant Buddhist orders. However, under Hideyoshi, Christians came to be seen as
a threat.
Hideyoshi expelled the Christian missionaries and then persecuted their converts. Ieyasu went further, attempting
to rid the islands of all Europeans. By the mid-17th century, European contact was limited to Dutch trade on
Deshima island. In this climate, the School of National Learning focused on the uniqueness of Japanese history
and culture.
Key Terms:
Malacca
Dutch Trading Empire
Caravels
Forbidden City
Manchu
Hongwu
Tokugawa Ieyasu
Edo
Nobunaga
Deshima
Chapter 22, Quiz Questions
1) All of the following were sources of disappointment to the Europeans who arrived in Asia in the 16th and 17th
centuries EXCEPT
A) Asians were uninterested in European trade goods.
B) Asians were uninterested in converting to Christianity.
C) Asian civilization seemed materially impoverished.
D) the Europeans were too few in numbers to exert force on Asian kingdoms.
E) Asian culture was thriving and diverse.
2) Despite their armaments, what factor convinced the Europeans that they could make little headway against the
kingdoms of Asia?
A) Inferiority of European ships
B) Distance from European ports
C) European lack of bullion
D) Large populations of Asian kingdoms
E) Armed resistance to European trade
3) What was the Portuguese lesson learned at Calicut?
A) Indian markets had little of interest to Western consumers.
B) Indians refused to trade with Europeans.
C) Western products brought for trade were of little or no value.
D) Western bullion was of no use in the East.
118
E) European goods were much sought after in India.
4) What was the significance of the mainland kingdoms and island states of southeast Asia that surrounded the three great
manufacturing zones of the Asian sea trading network?
A) These regions were virtually the sole consumers of goods made in the manufacturing regions.
B) These regions fed raw materials—precious metals and forest products—into the trading network.
C) These regions provided the military defense for the manufacturing regions.
D) These regions had no relationship to the three main manufacturing zones of the Asian sea trading network.
E) These regions provided the medium of exchange in the form of gold and silver.
5) What was the nature of the sea routes in the Asian trading network?
A) Well-established routes directly crossing the major oceans were maintained from ancient times.
B) Most of the navigation consisted of sailing along the coastlines and avoiding open seas.
C) Only the Chinese and Arabs practiced navigation in the Asian trading network.
D) The only sea-going routes crossed the Indian Ocean from the Swahili ports of east Africa to India.
E) They were restricted to the South China Sea and the Sea of Japan.
6) Which of the following statements most accurately describes the nature of the Asian sea trading network?
A) The entire network was dominated by Arab merchants who worked in common cause to establish a monopoly of trade.
B) The Chinese, as a result of their naval superiority, were able to secure military dominance of the system.
C) There was no central control, and force was usually absent from commercial exchanges.
D) The system functioned only so long as it was administered from the Ottoman Empire with the tacit support of the Ming
dynasty.
E) The Chinese empire kept the peace through the use of heavily armed junks.
7) Why were the Portuguese unwilling to exchange bullion for products within the Asian commercial system?
A) The doctrine of mercantilism equated possession of bullion with power and argued against negative trade balances.
B) They had none.
C) There was little merchandise that the Portuguese considered of value in the Asian trade network.
D) All of Portuguese bullion was exchanged for slaves in the African commercial system.
E) They were sending bullion to England to pay for weapons.
8) Why did the Portuguese believe they could successfully enter the Asian sea trading by force?
A) The Asian empires lacked navies.
B) There was little military force in any of the Muslim or Asian empires.
C) The Portuguese realized that their armies were more numerous than those of the Asian opponents.
D) Portuguese ships were more maneuverable and better armed than those of their Asian opponents.
E) The European trade zones were easily dominated by force.
9) How successful was the Portuguese monopoly on Asian products?
A) For some decades they were able to maintain a complete monopoly over Asian products shipped to Europe.
B) Though they managed to monopolize some spices grown in limited locales, the Portuguese lacked the manpower and
ships to sustain a monopoly.
C) The Portuguese were unable to achieve control over any Asian products due to competition from the Chinese
commercial navy.
D) The Portuguese monopoly was rigidly enforced over Asian products for almost two centuries.
E) Other European trade powers were frozen out of the market due to the success of the Portuguese establishing fortified
trade ports.
10) How did the Dutch commercial strategy within the Asian trade network differ from that of the Portuguese?
A) The Dutch lacked a substantial navy, and could not use warships to maintain their commercial advantage.
B) The Dutch did not make use of fortified towns and factories.
C) The Dutch were more systematic in their monopoly control of a limited number of specific spices.
D) The Dutch were more humane in their treatment of island peoples who cultivated the spices.
119
E) The Dutch were more interested in establishing permanent settlements such as Batavia.
11) In what way did the Dutch and English participation within the Asian sea trading network change by the middle
decades of the 17th century?
A) For both the Dutch and the English, peaceful commerce came to be more profitable than forcible control and
monopolies were aimed at European rather than Asian rivals.
B) Both northern European nations abandoned the commerce in spices in favor of cotton and silk textiles.
C) Unlike the Portuguese and Spanish, the northern European nations undertook wholesale conversion to Protestantism of
the inhabitants of the Spice Islands.
D) As allies, the Dutch and English were able to establish a naval supremacy in Asia sufficient to monopolize all trade
within the Asian sea trading network.
E) Military expansion enabled greater commercial control due to the series of religious wars that had broken out in
Europe.
12) In what sense was the Spanish conversion of the Filipinos similar to their experience in the Americas?
A) All Christian tenets were taught in the language of the indigenous peoples.
B) Filipino conversion to Christianity was predicated on political equality with the Spanish conquerors.
C) Like the Amerindians, the Filipinos' brand of Christianity represented a creative blend of earlier beliefs and practices
with Christianity.
D) Few Filipinos were converted to Christianity.
E) Filipinos were easily converted to Protestant faiths.
13) Which of the following was NOT a European contribution to the Asian sea trading network?
A) The addition of new routes, including the link to Europe around the Cape of Good Hope
B) The introduction of sea warfare into the Asian trade network
C) The establishment of new trading centers such as Goa, Calicut, and Batavia
D) The establishment of an exchange of new crops and diseases similar to the "Columbian Exchange" with the Americas
E) A global flow of silver starting in the Americas and ending in China
14) Which of the following reforms was NOT introduced by the first Ming emperor?
A) The position of the scholar-gentry within the bureaucracy was restored.
B) State subsidies for imperial academies and regional colleges were reinstituted.
C) Family influence in the selection of men to the Chinese bureaucracy was eliminated.
D) The civil service examination system was reinstated.
E) The position of chief minister was abolished.
15) Which of the following was a reform instituted by the first Ming emperor to reduce court factionalism and the power
of the scholar-gentry?
A) A chief minister was appointed from the royal family to oversee all work of the imperial bureaucracy.
B) Corrupt or incompetent members of the bureaucracy were punished by being beaten on the bare buttocks.
C) Imperial wives could only come from specified noble families of good repute.
D) Eunuchs were expelled from the royal household.
E) Exams were more complex and were based on the Legalist school.
16) Which of the following statements concerning Ming reforms in favor of the peasantry is most accurate?
A) The early Ming emperors were completely uninterested in the plight of the peasantry.
B) The first Ming emperor attempted to increase the forced labor demands on the peasantry in order to restore the Chinese
economy following the expulsion of the Mongols.
C) Despite some attempts to improve economic conditions for the peasantry, the growing power of the rural landlords led
to increased tenancy and landless laborers.
D) The Ming reforms resulted in a reduction in the authority of the local landlords and the establishment of small farming
operations throughout China.
E) Peasants were made exempt from all taxation due to Hongwu being a peasant himself.
17) Which of the following statements concerning Ming social organization is most accurate?
120
A) The adoption of more Buddhist beliefs began to break down the strict patterns of deference that had been customary in
Han and Song China.
B) Occupational alternatives for women of all social levels dramatically expanded during the Ming era.
C) Among the groups granted almost total freedom from the bonds of social status were the students seeking entry into the
scholar-gentry.
D) Under the continued influence of neo-Confucian ideology, Ming society remained rigidly stratified with emphasis on
deference of youth to elders and women to men.
E) Social roles were more flexible than before due to the influence of Jesuit missionaries such as Matteo Ricci.
18) Which of the following reasons is at least in part responsible for the peopling of the Yangtzi region in the southern
part of China during the Ming era?
A) The introduction of crops from the Americas that could be cultivated on inferior soils and did not require irrigation
B) The enforced migration of remaining Mongols within the Chinese population to the region of the Yangtzi
C) The abandonment of rice and millet cultivation in the region in favor of wheat introduced by Portuguese merchants
D) The opening of the region to settlement from Siam and Vietnam
E) The overpopulation of the northern provinces of China led to the Ming ordering a mass migration
19) Which of the following statements concerning the Ming economy is most accurate?
A) Merchants failed to realize profits from the Ming commercial boom.
B) Almost all commercial profits were reinvested in trade.
C) The Chinese government did not tax trade, thus did not profit from commercial growth.
D) Much merchant wealth was invested in land as a means of social advancement.
E) European markets became increasingly important to the Ming emperors.
20) Why did the Chinese abandon the commercial voyages of the Zheng He expeditions?
A) Many of the ships were lost as a result of poor ship design and inadequate sailing technology.
B) The size of the fleets was so limited that they could not compete with the greater capacity of the European voyages.
C) There was little of value for the Chinese to import in trade, and the voyages were expensive to carry out.
D) The trade with foreign regions produced a negative balance of trade for China that drained bullion from imperial
coffers.
E) Chinese commerce was not competitive enough due to competition from the Europeans.
21) Why did the earliest of the Japanese military centralizers accept Christian missionaries?
A) His wife was a Christian who was able to exert her influence throughout his household.
B) The Portuguese supplied a large army to rulers who offered to accept Christianity.
C) Christianity was seen as a counterforce to the Buddhist orders that opposed the imposition of central rule.
D) Prior to his first military victory, Hideyoshi saw a cross in the sky.
E) He was intimidated by the military might of the Portuguese.
22) Which of the following was NOT a policy imposed as a result of Japanese isolation in the 17th century?
A) Christianity was banned and Christians were persecuted.
B) Foreign traders were confined to the island of Deshima in Nagasaki Bay.
C) Neo-Confucian philosophy gave way to the influence of thinkers who championed the school of "National Learning."
D) The Japanese elite abandoned all contact with Western learning and technological advance.
E) Western books were banned.
Essay Questions:
Asian Transitions in an Age of Global Change
1. Define the characteristics of the Asian sea trading network. In what ways did the European incursion change the
Asian system?
2. What was the nature of the Ming restoration of traditional Chinese values? What innovations were made?
3. Compare and contrast Chinese and European means and motives for commercial expansion.
121
4. Following the establishment of the Tokugawa shogunate, what was the nature of Japanese isolation?
5. Evaluate the impact of the European entry into the Asian sea trading network.
122
Chapter 23, The Emergence of Industrial Society in the West, 1750-1914
Summary:
During the period of 1750 to 1914 new technologies and economies arose in parts of the world. The countries in these
parts of the world, generally in Europe, gained powerful advantages over the rest of the world. The triggers for this shift
in world history came form a series of inventions the originated in Great Britain and spread to Europe and the United
States. This industrialization led to new forms of work organization and the development of the factory system. It also
changed politics as a new middle class sought a political voice. Finally, industrialization provided a context for
imperialist tendencies of the West. Although these changes were revolutionary, its results were spread out over many
years with resistance on the regional and cultural level. The impact of the industrialization is most evident with the
transformation of leisure. New kinds of leisure were developed to decrease time form work. This trend also influenced
agricultural regions.
Key Concepts:
The Age of Revolution:







Eighteenth-century faith in progress was reflected in Condorcet’s Progress of the Human Mind, written in 1793
while the author was in hiding. The age of revolution was inspired by that spirit, but also shook its optimism.
Three influences came to overturn Europe by the mid-19th century. Intellectual excitement was pervasive. The
ideas of such philosophers as Jean-Jacques Rousseau were widely read. Secondly, more and more Europeans
were part of the commercial economy. Commercial wealth challenged traditional notions of the right to rule.
Thirdly, the population revolution resulting from better nutrition, and limited control of diseases, led to
population pressure. Proto-industrialization was spurred by the wealth of labor resulting from better nutrition,
and limited control of diseases, led to population revolution.
When the British Atlantic colonies rebelled it was not truly a revolution, but a war of independence. The socalled American Revolution established a new government in 1776, and achieved its aims in 1789.
The French Revolution soon followed, beginning a profound political restructuring. Ideology called for reforms,
and social change added a dimension to the struggles. However, the French government was to slow to adapt.
When Louis XVI finally called a parliament in the summer of 1789, reform was forced upon him. The
Declaration of the Rights of Man and the Citizen proclaimed the ideology of the reformers. The taking of the
Bastille, a symbol of royal power, on July 14th began widespread uprisings.
Calls for reform in 1789 turned to radicalism in 1792. The monarchy was abolished and the king beheaded by
guillotine. Maximilien Robespierre led the radical revolution. Leading the “Terror”, he also suppressed French
Catholicism, but was finally guillotined himself. Universal male suffrage, abolition of colonial slavery, and
universal conscription were among his policies. Nationalism added to calls for reform to crystallized adherence
to the movement. The arrival on the scene of Napoleon Bonaparte in 1799 transformed the revolution.
Monopolizing power, he nevertheless continued many of the liberal reforms. Legal reforms were promulgated
under the Code Napoleon. Ambitious wars led to the undoing of Napoleon in 1815. Yet, the revolutionary legacy
was great.
Following the defeat of Napoleon, the Congress of Vienna met in 1815 to settle the peace. The principle of the
balance of power was followed, and was successful for decades. The ascendancy of conservative goals was
signaled by the restoration of monarchies. Much of the Enlightenment ideology was pursued by Liberals, who
formed in opposition to the Conservatives. Radicals wished to push reforms further. Nationalists allied
sometimes with Radicals, sometimes with Liberals. The Greek Revolution in 1820 was one of many uprisings
inspired by the French Revolution and the ideas of nationalism. The British Reform Bill of 1832 achieved many
of the reformers’ goals without revolution.
Industrialization, added to the social upheavals, creating a working class with its own agenda. The Chartist
movement in Britain hoped to extend suffrage, enact educational reforms, and improve working conditions.
However, in the 1848-1849 revolutions, unrest peaked. Beginning with France, uprisings followed in Austria,
Germany, Hungary, and Italy. These short-lived revolutions did not bear immediate fruit. In France itself,
Napoleon III imposed authoritarian rule. The age of revolutions was over. Many of the goals of revolutionaries
had been met, but governments had also learned how to control challenges.
123
The Consolidation of the Industrial Order, 1850-1914:




The second half of the 19th century was marked by growth and a growing infrastructure linking European
countries. Urban life was improved by better sanitation and attention to the needs of the poor and sick. Crime
rates stabilized or fell.
Industrialization changed every aspect of life. Material well-being rose. Mortality rates among infants fell, and
more effective methods limited childhood diseases. The work of Louis Pasteur accounted in a large part for
massive improvement in health. At the same time, corporation’s transformed business, drawing on capital. Their
work forces became a major political and economic power, while the political power of peasants declined.
Politically, the agenda changed radically after the 1848 revolutions. The Argument over absolutism and
constitutions was resolved, and gave way to debates over workers’ rights and voting. Benjamin Disraeli, the
British conservative prime minister, expanded the vote for men. Count camaillo di Cavour also responded to
liberal demands. In Prussia, Otto von Bismarck extended the vote to all men, emancipated the Jews, and pushed
mass education. These new conservatives also called upon nationalism to consolidate their power. Natinal
platforms often took the form of imperialism, as in the case of British and U.S. expansion. In Italy and Germany
they were formative. Under Cavour’s leadership, Italy was unified in spite of papal opposition. Bismark
similarly used nationalism to unite Prussia. He maneuvered the country into war with Denmark and Austria,
culimianting in German unity in 1871. The American Civil War, form 1861-1865, reaffirmed the Union and
ended slavery. France returned to its republican system. Across Europe, parliamentary systems allowed
continuity in spite of changes in leardership, a process the Italians called transformismo.
Governments increased their domestic scope after 1870. Agencies regulating education, health, industry, and
welfare became common. Germany was a leader, implementing social insurance for illness and old age. The
social question came to dominate political debates, leading to the emergence of socialism. Earlier socialists had
general aims, often taking shape in utopian communities. Inspired by the ideas of Karl Marx, new generations
focused on his idea of the centrality of the class struggle, and the necessity of violent protest. Socialism became
an important force by the last decades of the 19th century. Revisionist socialism called for achieving Marx’s ends
without violence. Feminist movements added their agendas, demanding a political voice. Such leaders as
Emmeline Pankhurst used strikes and protests to gain suffrage.
Cultural Transformations:



While the growing working class copied middle-class tastes, they also demanded entertaining leisure pursuits. On
the supply side, meanwhile, producers had to stimulate demand to find a market for their goods. Mass leisure
culture, informed and shaped in part by high-circulation newspapers, called for entertainment. Team sports
developed along with nationalism to create new loyalties. In many ways mass culture replaced religion.
Unlike the sciences I the Enlightenment, in the late 19th century, the sciences moved away from other intellectual
pursuits, and professional scientists took the place of amateurs. In 1859, Charles Darwin published his
influential work on the theory of evolution. His ideas conflicted not only with religious notions, but with the
Enlightenment idea of an orderly natural world. Meanwhile, work in physics included discoveries in
electromagnetism, particle physics, and astronomy. Albert Einstein came up with the idea of realitivity, also a
challenge to accepted ideas of the physical world. The late 1800s were a period of great development in the social
sciences, including economics, sociology, and psychology. Sigmund Freud advanced his ideas on the human
subconscious.
In the arts, the prevalent movement in the early 1800s was romanticism. In reaction to the Enlightenment,
romanticism emphasized individual emotional responses in the plastic arts, music, and literature. The end of the
century saw a move to challenge every canon of representation, harmony, rhyme, and narrative.
Western Settler Societies:


Industrialization spurred the search for raw materials and markets. At the same time, it gave the West a military
advantage, in its use of guns and steamships. These factors drove the expansion of settler societies in the United
States, Canada, Australia, and South Africa.
The new United States came to be a world power in the 19th century. Massively enlarging its territory with the
Louisiana Purchase and settling the lands beyond the Mississippi, it was able to extend its influence partly due to
a huge influx of Europeans. The American Civil War, 1861-1865, violently resolved the economic and cultural
124

split between the north and south. The war pushed industrialization and the development of infrastructure. A
railway linking the east and west coasts was key to exploiting western lands. Outside the Western Hemisphere,
the influence of the United States was small.
The British dominions of Canada, Australia, and New Zealand had been relatively sparsely-populated by
indigenous peoples. In each area, governments followed western models. Canada was divided by French and
British settlers. From 1839, the British began the process of giving Canada autonomy, to avoid an independence
movement. As in the United States, railways facilitated settlement. Australia was originally colonized as a penal
colony, but by the middle of the century was an important wool producer. Gold strides spurred settlement.
Britain granted self-rule, and the Australian parliamentary government developed. New Zealand was different.
Its Maori people were politically organized, and resisted successfully for decades. Nevertheless, European
immigration began in 1840, and the Maoris were defeated by 1870, New Zealand, Canad, and Australia remained
closely linked to Britain, but developed along their own lines. With the United States and South America, they
received the bulk of European immigrants in the 19th century.
Diplomatic Tensions and World War I:



German unification undermined the balance of power in Europe. At the same time, European leaders were
looking for solutions through imperialism. Yet by 1900, there was little territory left to take.
The alliance system replaced the balance of power. The Triple Alliance; Germany, Austria-Hungary, and Italy.
The Triple Entente; Britain, Russia, and France. Militarization, spurred by nationalism and industrialists, made
the face off volatile. Russia and Austria-Hungary were the least stable elements, susceptible to internal
upheavals. As provinces of the Ottoman Empire sought independence, Balkan nationalism also threatened the
peace. Serbia emerged and then sought to expand its territory.
The causes of the First World War included militant nationalism. Industrialization was an indirect cause, as
leaders used international undertakings to appease social unrest. Militarization, an economic plus, also let to
conflict.
Key Terms:
Otto Von Bismarck
Napoleon Bonaparte
Industrial Revolution
Factory System
Age of Revolution
American Revolution
French Revolution
Imperialism
Nationalism
Count Camillo di
Cavour
Chapter 23, Quiz Questions
1) Which of the following statements concerning the political philosophy of the Enlightenment is most accurate?
A) Enlightenment philosophies were creatures of the monarchs who were their patrons and supported the powers of the
kings.
B) Although the Enlightenment philosophers were generally opposed to the authority of the church, they argued that only
monarchy could insure stability for the masses.
C) Enlightenment thinkers challenged regimes that did not grant full religious freedom or that insisted on aristocratic
privilege.
D) Enlightenment intellectuals were the firmest supporters of the church left in European culture.
E) Enlightenment philosophers withdrew from society and politics and adopted a policy of non-intervention in religious
matters.
2) Above all, population pressure in the 18th century
A) led to wholesale replacement of officeholders among the elite.
B) drove many people into the working-class proletariat.
C) led the business classes to reduce the number of children in their families.
D) caused the middle classes to reduce risk-taking.
E) kept many people trapped as agricultural serfs.
3) Proto-industrialization refers to
A) the strictly agricultural economy that preceded the Industrial Revolution.
125
B) the development of the rural factory system.
C) the employment of laborers who worked at home but in a capitalist system dependent on urban merchants.
D) the development of systems of transportation and communication necessary for full industrialization.
E) a society that has reached its full industrial potential.
4) Population upheaval and the spread of a property-less class working for money led to which of the following
developments?
A) An increase in the authority of the male heads of households
B) A general acceptance of authority, whether domestic or political
C) A decline in the percentage of illegitimate births
D) The adoption of more urban styles of dress
E) A sharp decrease in the crime rate
5) Which of the following was NOT a cause of the American Revolution?
A) Britain's attempts to impose new taxes and trade controls after 1763
B) A desire to overthrow the older colonial leadership
C) The British invasion of the Mississippi River Valley
D) Restriction on free movement into the frontier areas
E) British belief in mercantilism
6) Which of the following was NOT a cause of the French Revolution?
A) Enlightenment thinkers urging the need to limit the powers of the Catholic church and the aristocracy
B) The church seeking greater power over the royal government
C) The middle-class demand for greater political representation
D) The peasant desire for freedom from manorialism
E) Financial problems of the French government
7) Which of the following was a lasting reform passed during the initial, moderate phase of the French Revolution?
A) Universal male suffrage
B) The introduction of Protestantism
C) Peasants were freed from all traces of manorialism
D) Universal military conscription
E) End to mercantilism
8) Which of the following statements concerning the impact of the French Revolution on the rest of Europe is NOT
accurate?
A) The French Revolution spread key revolutionary legislation throughout much of Western Europe.
B) The revolution encouraged popular nationalism outside of France.
C) The French Revolution and its subsequent empire created a general consensus after the defeat of France for a more
liberal Europe.
D) The idea of equality under the law and attacks on privilege, whether aristocratic, guild, or ecclesiastical, spread
throughout Europe.
E) Conservatism remained the dominant political force until the coming of World War I.
9) After industrialization in 19th century Europe, a demographic transition occurred which led to a new system that
promoted stable population levels through
A) government-sponsored family planning.
B) legal sanctions against illegitimate births.
C) lower birth and death rates
D) increased mortality due to industrial accidents and environmental pollution.
E) increased mortality due to disease.
10) As industrialization spread, peasants in Europe improved their conditions by all of the following means EXCEPT
A) forming unions.
B) organizing cooperatives.
126
C) specializing in new cash crops
D) seeking education and new technical skills.
E) decreasing isolation.
11) Which of the following statements concerning conservative political strategies after 1850 in Europe is most accurate?
A) With the destruction of the liberal movement, conservatives felt free to restore the aristocratic bias of absolute
monarchy.
B) Conservatives began to ally more closely with socialists in order to offset the political advantages gained by liberals
during the revolutions of 1848.
C) Conservatives ceased to play any role in the political framework of Europe after 1850.
D) Conservatives strove to develop a new political consensus with liberals that would grant the appearance of
constitutional reform but retain aristocratic privilege.
E) Conservatives used military power to retain control over European society.
12) Which of the following did NOT represent an expansion of government functions in the West following 1870?
A) Civil service examinations
B) Extension of regulatory apparatus
C) The establishment of mandatory national unions for laborers
D) Wider welfare measures
E) Establishment of compulsory education to age 12
13) Which of the following statements could NOT be attributed to the political philosophy of Karl Marx?
A) Earlier socialist theories based on utopian schemes were silly and unrealistic.
B) In the aftermath of the victory of the proletariat, the state would emerge permanently as a powerful dictatorship.
C) History was shaped by the available means of production and who controlled them.
D) Revolution of the proletariat against the bourgeoisie was inevitable.
E) In the final historical phase, the state would wither away.
14) Which of the following statements most accurately describes the relationship between science and the arts in the later
19th century?
A) Science and art continued to follow the lines of classical and rational traditionalism.
B) Science and art of the 19th century were freed from the traditions of classical rationalism and embarked on a radical
shift that favored the emotional.
C) Science continued the Western trend of traditional rationalism, but art adopted the more emotional and impressionistic
theories of Romanticism.
D) There were few scientific advances after the early stages of industrialization and little if any innovation in the field of
art.
E) Government restrictions on academic and artistic freedom limited innovation in many European countries.
15) American exceptionalism suggests that the United States
A) developed on its own terms with only incidental contact with Europe.
B) depended exclusively for its cultural development on Europe.
C) had no cultural relationship of any kind with Europe.
D) achieved its cultural foundations from Native Americans.
E) does not have a culture that it can call its own.
Essay Questions:
The Emergence of Industrial Society in the West, 1750-1914
1. What were the permanent reforms of the American and French revolutions?
2. In what ways did the Napoleonic period, despite the creation of an authoritarian regime, result in the extension of
revolution to the rest of Europe?
127
3. What changes led to the Industrial Revolution in Britain? What social changes did the Industrial Revolution
cause?
4. After 1850 what political coalitions led to successful nationalist movements, particularly in central Europe?
5. How did the emergence of new central European nations after 1870 lead to increased diplomatic tensions?
128
Chapter 24, Industrialization and Imperialism:
The Making of the European Global Order
Summary:
Western European industrialization fundamentally altered the nature of European overseas expansion. In previous time,
Europeans sought desired material goods or moved against threats from external enemies. In the Americas, they seized
lands for plantation crops. Christian missionaries sought converts. Much of the secular and religious thrust was due to a
desire to strengthen Europe in the long contest with Islam. Industrialization brought new motives for expansion. Raw
materials were needed to fuel industrial growth, and markets were required for its manufacturing production. Christian
proselytizing continued, but private initiative replaced state direction. Another change was that the increased power of the
West made it fear European imperial rivalries more than indigenous opposition. Europeans then had gained the capacity
to push into and occupy territories once closed to them by disease or local resistance.
Key Concepts:
The Shift to Land Empires in Asia:






Up to the 19th century, European powers had patronized enterprises in Asia, as opposed to forming empires.
Profits were sought through cooperation with Asian countries. Communication with far-flung outposts was slow.
The only empires built were those made ad hoc by Europeans abroad.
Dutch Java is an example of on-the-spot empire building. Initially, the Dutch paid tribute to the sultans of
Mataram, in control of most of Java. However, the Dutch increasingly involved themselves in internal political
struggles. Locally recruited armies made the Dutch important players in local rivalries. At the same time, the
Dutch controlled more and more territory, until they controlled all of Java by 1760.
British rule in India developed along the same lines as Dutch rule in Java. Interference in Indian politics was
accompanied by the recruitment of Indian sepoy troops. Indian princes used British forces to best their rivals, with
the unintended result of increasing British power on the subcontinent. The British Raj grew in power as part of
British rivalry with the French. The battle of Plassey in 1757, gave Britain control over the south. Rober Clive,
a British military leader, used Indian spies and Hindu financial backing against the Muslim prince Sirnud-daula.
The battle was won more by espionage and corruption than by superior military power.
The involvement of the East India Company in India increased as Mughal power declined. British rule was
organized into the three presidencies of Madras, Bombay, and Calcutta. Divisions in India between princes and
between Muslims and Hindus made British expansion easier. India became the focus of the British Empire,
because of its position and its size.
Early Dutch and British rule in Java and India left native systems essentially intact. Colonists also adapted to
local customs in architecture, food, and dress. European men often lived with local women, and even marriages
became common. However, such attitudes changed later, in the 1800s.
The laissez-faire attitude of the Dutch and British in their early decades in Java and India was replaced by more
direct action. Reform in India came because of corruption among East India Company officials. The term nabob
was applied by the English to their countrymen who lived the high life in the colony. Lord Charles Cornwallis,
of Yorktown fame, led the reform of the British administration in India. Meanwhile, the British attitude towards
Indians changed. Some came to consider the Indians degenerate, and focused on social customs such as sati.
Ram Mohun Roy was on of many Western-educated Indians that supported the suppression of the latter practice.
In India as well as elsewhere, Britian began to methodically impose its standards, its values, and its infrastructure.
Industrial Rivalries and the Partition of the World, 1870-1914:


As Belgium, France, Germany, and the United States competed with Britain as industrial nations, they came to
view colonies as key to the competition. Repeated economic depressions brought social unrest, but no solutions.
Public opinion added its weight to the push for overseas territories.
Acquiring colonies was made easier for European powers by better communications – via railroads, new canals,
and telegraphs. Improvements in firearms made most battles extremely one-sided. Yet colonial expansion met
with determined resistance. The Zulu defeat of the British at Isandhlwana is an example, though the Zulu losses
were much greater than the British.
129
Patterns of Dominance: Continuity and Change:









European colonies can be divided into the “tropical dependencies” and settlement colonies. Among the latter, the
White Dominions included Canada, and Australia. Other areas, such as Kenya and Algeria combined
characteristics of both.
European rule in the tropical dependencies relied heavily on taking advantage of natural divisions within
countries. Privileged minorities were recruited as servants of colonial governments. European rulers and
administrators governed through local subordinates, generally from preexisting ruling groups. Higher education
was generally unavailable to Africans, polarizing societies there.
In the 19th centuries, the relationship between foreign rulers and the ruled changed in Africa and Asia. As
European immigration increased, isolation from local populations also increased. Miscegenation was condemned.
Ideas of white racial supremacy played a large role in these changes. In contrast with earlier colonists,
Europeans maintained their social order, their dress, and their manner of living.
Attempts to better extract resources form colonies led to imposition of European practices in Africa and Asia.
Forced, unremunerated labor was imposed, as was flogging, even to death, for failure to meet quotas. Export
crops replaced food crops in many areas. Most of the raw materials extracted went to European factories and
consumers.
The settler colonies in South Africa and the Pacific differed form India and the Belgian Congo, because the large
nubers of indigenous peoples were at least matched by large numbers of settlers. In the Americas, early conquest
decimated populations, and large numbers of settlers exported their culture virtually intact. The case of Australia
was similar. Colonies settled in the 19th century fared differently, because, in general, native populations were
more resilient to disease. Such was the case in the north of Africa, New Zealand, and many of the Pacific islands.
Colonization in southern Africa began with the Dutch Cape Colony. The Dutch Boers – farmer – moved into
areas with low population density, enslaving the Khoikhoi. British took the colony over in the 1790s, as a vital
link in their overseas empire. British rule brought a new, distinct group of settler, including some desirous of
ending slavery. The Boer Great Trek of the 1830s moved deep into the region of the Bantu Zulus and Xhosa,
with attendant violent conflict. At the same time, the Boers were often in conflict with the British. Two Boer
Republics were established, but discoveries of diamonds and gold meant new conflict between the two European
groups. The Boer War, launched by the Boers against the British, resulted in a costly British victory.
The South Pacific islanders had been isolated for centuries. As a result, they were particularly hard-hit by the
arrival of Europeans. The histories of Hawaii and New Zealand have much in common. Both had developed
sophisticated cultures and areas of high population density. Both were so adversely affected by the arrival of
Europeans that their cultures only survived with difficulty.
The Maori of New Zealand first suffered when Europeans arrived in the 790s, bringing smallpox and tuberculosis,
and involving the Maori in prostitution and alcoholism. Adjustment occurred, however, and the Maori survived.
The arrival of British farmers and ranchers in the 1850s brought a new cycle of suffering. The Maori were pushed
to the edge of extinction. Yet they survived, developed immunities to European diseases, and learned to fight
with and work with the settlers on their own terms.
Hawaii was annexed by the United States quite late, in 1898. Captain James Cook was the first European to reach
the islands, in 1777. King Kamehameha used British support to extend his rule. More cultural borrowing
followed, including conversion to Christianity. At the same time, western diseases decimated the population of
the islands. Outright seizure came after the power of the Hawaiian kings declined. U.S. rule displayed respect for
Hawaiian culture.
Key Terms:
Kamehameha
Boxer Rebellion
White Dominions
Queen Victoria
Cecil Rhodes
Boer War
Sepoys
British Raj
Mataram
Utilitarians
Chapter 24, Quiz Questions
1) Which of the following was NOT an aspect of imperialism in the period following European industrialization?
A) The search for markets for European manufactured products
130
B) The absence of Christian missions
C) The establishment of European colonies in the interior of Africa and Asia
D) The search for raw materials to feed the machines of Europe
E) The need to find profitable investments for excess capital
2) Which of the following statements concerning the management of colonial enterprises by the Dutch and British East
India companies in the 17th century is most accurate?
A) The directors of the companies were little interested in the acquisition of colonial territories.
B) The directors of the companies made precise plans for the expansion of company administrative control over the
governments of indigenous peoples.
C) The companies were granted monopolies by governments with the clear expectation that they would conquer new
territories for their respective nations.
D) The companies were mere figureheads for the active and direct intervention of European nations in the affairs of Asian
peoples.
E) The companies worked in partnership with indigenous industries to ship their products to China.
3) How were 18th-century land empires in Asia accumulated?
A) By direct government intervention
B) By the policy of the directors of the Dutch and British East India companies acting under the direction of their
governments
C) By the initiative of overseas agents of the Dutch and British East India companies acting in the absence of instructions
from the company directors
D) No 18th-century territorial acquisitions were made.
E) By insubordinate military leaders
4) Which of the following statements concerning the incursion of the Dutch East India Company into Java is most
accurate?
A) The Dutch were content in the 1620s to be the vassals of the Sultan of Mataram.
B) The Dutch won a series of military victories in the 1620s that established their military dominance in Java.
C) The Dutch relied on ships and military forces from Holland to establish their initial supremacy in Java.
D) The Dutch replaced the local rulers with a company directorate to govern the colony of Java in the 1620s.
E) Local peasants welcomed the Dutch as liberators, which led to the introduction and acceptance of Christianity.
5) In what way was the British East India Company's intrusion into India similar to the Dutch entry into Java?
A) The conversion of the Indian elite to Christianity
B) The use of mercenaries recruited from among indigenous peoples
C) The British removal of all local rulers in the 18th century
D) The direct intervention of the British government
E) Effective naval operations in the Bay of Bengal
6) In which of the following ways was the British experience in India different than that of the Dutch in Java?
A) The lack of involvement of the British East India Company
B) The absence of intervention in local squabbles and succession disputes in India
C) The failure to use indigenous peoples in recruited armies
D) The existence of a global imperial rivalry with the French
E) The support of the British government
7) Which of the following was NOT a handicap faced by the Indian princes in defending their kingdoms from the British?
A) The lack of a sense of national identity in India
B) The willingness of Indians to serve in the British armies
C) The continued warfare among the Indian princes
D) The success of the British in winning many converts to Christianity
E) Strength of the British military
8) All of the following were reasons why India became the pivot of the great British Empire EXCEPT
131
A) the size of the Indian land army.
B) the utility of Indian ports in maintaining British sea power.
C) the residence of more white settlers than any other British colony.
D) the existence of raw materials useful to the British industries.
E) British interest in global trade.
9) Which of the following statements concerning colonial society in India and Java prior to 1850 is most accurate?
A) The Dutch and British were content to leave the social systems of Java and India pretty much as they found them.
B) The massive conversion of the Javanese to Protestantism created a significant change in social mores, but the British
were unable to carry out a similar change in India.
C) The arrival of the British and the Dutch completely destroyed the original social hierarchies of India and Java.
D) The Dutch and British incursions resulted in the removal of the indigenous aristocracies and the substitution of direct
European control of the peasants.
E) In both cases a new indigenous ruling class soon emerged based on the teachings of the Utilitarians.
10) Which of the following statements concerning European interaction with indigenous peoples prior to 1850 is most
accurate?
A) Social taboos effectively prevented any social interaction between Europeans and indigenous peoples.
B) As most of the Europeans were male, social interaction was limited to sexual exploitation of females in brothels or as
slaves.
C) Mixed marriages between European males and indigenous females became widely accepted, particularly in Java.
D) Both European males and females intermarried with indigenous peoples on a common basis.
E) In all cases few interactions took place due to state mandates.
11) Which of the following was NOT a motive for expansion in the late 19th century?
A) The pressure of public opinion
B) The use of colonies as pressure valves to release the pressures of unemployed workers and surplus goods
C) The absence of influence of political leaders in the European countries
D) The need to ensure a supply of raw materials
E) The need to invest surplus capital
12) The jingoistic press and the extension of the vote to the lower middle and working classes
A) made public opinion a major factor in foreign policy.
B) left the planning of imperial expansion to the European aristocracy.
C) made imperial expansion impossible to achieve.
D) led to demands for massive programs of emigration from European countries.
E) tended to give the ruling classes a free hand in foreign policy.
13) Which of the following statements is most accurate?
A) Quarrels over the division of the colonial spoils were used to justify the arms buildup and general militarism.
B) European nations cooperated to defeat the outmanned armies of African nations.
C) European nations rapidly came to agreements over the territorial division of colonial holdings.
D) The League of Nations supervised the construction of European colonial empires.
E) Colonial disputes were usually referred to the World Court located at the Hague.
14) Which of the following statements is most accurate?
A) Faced with the advanced military technology of the Europeans, indigenous people ceased resisting the imperial
advance.
B) Despite advances in military technology, the Europeans remained unable to overcome the Asian advantages in
population.
C) African and Asian peoples often fiercely resisted colonial rule, although without realistic chances of permanent
success.
D) No African or Asian military forces won set piece battles.
E) Most Africans felt that European rule was a good thing.
132
15) Which of the following descriptions most accurately defines the term "tropical dependencies?"
A) Imperial possessions in which the numbers of European settlers and indigenous peoples were approximately equal
B) Colonies in which small numbers of Europeans ruled large numbers of non-Western peoples
C) Colonies with substantial majorities of white, European immigrants
D) Colonies that were largely unpopulated prior to the coming of the Europeans
E) Equatorial regions of South America under the control of Spain
16) Which of the following statements most accurately describes the European strategy with respect to the government of
tropical dependencies?
A) The Europeans exploited longstanding ethnic and cultural divisions between indigenous peoples.
B) European colonialism depended on wholesale conversions to Christianity.
C) In many cases, the Europeans utilized their military superiority to carry out genocide against African and Asian
peoples.
D) Colonial rulers tended to favor Muslims at the expense of other minorities in African and Asian colonies because of
the level of education of Muslim peoples.
E) They were de facto independent nations with governments operated by the local elites.
17) What was a critical factor in the growing tensions between the colonizers and the rising African and Asian middle
classes?
A) The growing numbers of Christians among the African and Asian peoples
B) The growing size of European communities in foreign nations and the increased number of women in European
settlements
C) The decline in European militarism
D) The decline in the size of European communities in colonial settlements
E) The growing military power of native peoples
18) Which of the following statements concerning the internal economies of the European colonies is most accurate?
A) The introduction of European technology such as railways and telegraphs was intended to improve the internal
economies of the colonies.
B) Slowly, the industrial system of the West, including factories and the production of manufactured goods, was
introduced into Africa and Asia.
C) By 1914, Asian and African colonies had won economic independence from the European colonizers.
D) Colonial economies were steadily reduced to dependence on the European dominated global market.
E) Much production and skilled labor moved to colonial areas from the industrial nations.
19) Which of the following was NOT a result of the first contact between the Maoris and Europeans during the 1790s?
A) The spread of alcoholism and prostitution
B) The introduction of European firearms to Maori warfare
C) The introduction of European diseases such as smallpox
D) Extensive intermarriage between the Maoris and the white settlers
E) Endangerment of the native ways of living
Essay Questions:
Industrialization and Imperialism: The Making of the European Global Order
1. In What ways was the British conquest of India similar to the Dutch colonization of Java? In what ways was it
different?
2. One of the first elements of European reform within the colonies was the introduction of educational systems. In
what sense did the introduction of Western education plant the seeds of decolonization?
3. In what ways did the process of industrialization and national centralization typical of 19th century Europe alter
the process of imperialism?
133
4. In what ways did the colonizers of the late 19th century control the indigenous people and increase economic
exploitation?
5. In what ways did the colonial experience of “contested settler colonies” differ from that of “tropical
dependencies”?
134
Chapter 25, The Consolidation of Latin America, 1830-1920
Summary:
Most Latin American nations gained independence from colonial control early in the 19th century. The political culture of
its leaders had been shaped by the Enlightenment, but they faced problems growing form their own history. Their
colonial heritage did not include participatory government; highly centralized states had created patterns of both
dependence and resentment. Class regional interests divided nations; wealth was unevenly distributed. The rise of
European industrial capitalism placed Latin American nation in a dependent economic position.
Key Concepts:
From Colonies to Nations:




Latin American creoles were increasingly critical of the policies of their home countries. Criticism was also
voiced by the mass of their fellow colonists.
The American and French revolutions inspired revolutionaries in Latin America. The rebellion of Haiti, led by
Toussaint L’Ouverture, impressed both those who desired change and those who feared violent upheaval. The
trigger for change was French intervention in the Iberian peninsula. Napoleon put in his own rulers, raising
questions of legitimacy.
Three centers of independence movements emerged. In Mexico, Father Miguel de Hidalgo raised support
among Indians and mestizos. Under Augustin de Iturbide, the creoles joined the uprising and in 1824 Mexico
gained its independence. Led by conservative creoles, a monarchy was established. In South America and the
Caribbean, colonies gained independence in reverse order of colonization: Argentina and Venezuela were first,
Cuba and Puerto Rico last. In the north, the new nation of Gran Columbia was formed, led by Simon Bolivar.
In Buenos Aires, Jose de San Martin led the movement or independence of the United Provinces of the Rio de
La Plata. San Martin then moved to Peru. All of Spanish South America was independent by 1825.
Brazil stood apart from the rest of the continent because of its size and economic clout. Its leaders feared that a
push for independence would be accompanied by a slave uprising. However, the Portuguese royal family arrived
in 1807, fleeing Napoleon’s invasion of the peninsula. Dom Joao VI ruled from Brazil until 1820. He then left
his son Pedro as regent and returned to Portugal. In 1822, Pedro claimed the title of constitutional emperor of an
independent Brazil.
New Nations Confront Old and New Problems:



The leaders of the newly independent Latin American nations had many ideals in common. Division appeared,
however, concerning the role of the church and slavery. The economies of some countries were deeply dependant
on slave labor. The place of Indian populations was another divisive issue. The society of Castas had not
disappeared.
Politically, Latin America can be divided into regions. Mexico chose monarchy, until the republic was
established in 1823. The rest of Central Ameirca formed a short-lived union. By 1838, however, the union
dissolved. The Caribbean islands slowly gained their independence. Gran Columbia fell apart in 1830, the year
of Bolivar’s death. Similarly, the United Provinces of Rio de La Plata, led by Argentina, fragmented. Peru and
Bolivia tried to unite, under Andres Santa Cruz, but then went their own ways. Chile maintained an independent
course. Huge distances and geographic barriers made unions difficult.
Long wars facilitated the rise of caudillos. Their bases of support varied from country to country. Rafael
Carrera, ruler of Guatemala, was a staunch advocate of the Indian majority. Everywhere, struggles emerged
between centralists and federalists, and between liberals and conservatives. In general, liberals and
conservatives rallied against and for the Catholic Church, respectively. Across Latin America, the place of the
church was a key issue, although the centrality of Christianity was not questioned. Liberal and conservative
parties emerged everywhere. Still, it was the individual leadership of such men as Juan Manuel de Rosas and
Antonio Lopez de Santa Anna that drew adherents. Latin America was thus a volatile mix for decades after
independence. Rapid changes brought down governments, although some areas such as Brazil were relatively
stable, politically.
135
Latin American Economics and World Markets, 1820-1870:






As Latin American colonies became independent, they entered international affairs. Spain attempted to turn back
the clock and seize control, while Britain and the United States supported independence. In the Monroe
Doctrine, the latter claimed that the Western Hemisphere was off limits to outside interference. Britain was
nevertheless successful in becoming the dominant economic power in the region. Dependence on foreign
producers weakened Latin American domestic industries.
The period 1820-1850 saw economic stagnation in Latin America. Cuba, still under Spain, was the exception.
From 1850, new exports – coffee, beef, hides, grains, and minerals – brought regrowth. At the same time,
transportation problems were eased by railroads and steamships. Patterns of change include wide-spread attempts
at liberal reforms, and a conservative response.
The end of the 19th century saw a surge of reform. Based largely on Auguste Comte’s ideology of positivism,
the new wave was made possible by economic growth. Leaders focused on joining the ranks of capitalist
countries. Post-1860 governments were generally led by men who believed in progress but distrusted their
citizens. Economic growth benefited landowners at the expense of peasants.
Mexico’s new republic was based on a constitution inspired by those of France, the United States and Spain.
However, the place of Mexico’s large Indian population was unresolved. Santa Anna, a typical caudillo,
depended on personal autocratic rue. Mexico was threatened by foreign intervention. North American settlers in
Texas attempted to gain autonomy. Suppressed by Santa Anna, the movement led to war with the United States.
The Mexican-American War ended with the Treaty of Guadalupe-Hidalgo in 1848, in which Mexico lost half
of its territory. Santa Anna now faced political challenges, especially by Benito Juarez, a Zapotec Indian. La
Reforma brought in liberal reforms and a new constitution. However, Juarez’s attempt a land distribution
backfired, and Indians were dispossessed of what little remained to them. A conservative backlash again
followed. Napoleon III was asked for support and Maximilian von Habsburg took the title emperor. The
emperor was assassinated, and Juarez took office again. He was the central figure in Mexican politics until his
death in 1872, when Porfirio Diaz succeeded him.
Argentina’s attempt to dominate the United Province of Rio de la Plata failed when the union dissolved. Liberal
reforms and conservative responses followed, as in Mexico. Juan Manuel de Rosas, a federalist, supported
ranchers and merchants. But his despotic leadership roused a coalition that forced him form power in 1852.
Incluenced by Juan Bautista Alberdi, a new compromise constitution was promulgated. The Argentine Republic
began a period of growth, while presidents pushed reforms similar to Mexico’s reforma. Economic prosperity
was built largely on ranching. Victory over Indians to the south solidified Argentine feelings of achievement.
While Brazil was stable for much of the 19th century, it had not resolved critical issues. Dom Pedro I abdicated in
favor of his son in 1831. The regency that followed was divided by revolts. Dom Pedro II began his personal
rule in 1840. Coffee plantations – fazendas – came to dominate export trade. Coffee growing depended on slave
labor. A growing infrastructure opened the interior of the country, while growing professional groups, and
working and middle classes changed the political makeup of the country. Large numbers of immigrants allowed
the abolition of slavery in 1888. This, along with other changes, reduced support for the monarchy. Pedro II was
deposed in 1889. All of these changes brought dislocation for some groups, provoking backlash uprisings.
Societies in Search of Themselves:


As Latin American countries freed themselves of Spain and Portugal, they were more open to other influences.
European culture was popular, influencing literature and the arts. Romanticism inspired the works of Antonio
Goncalves Dias and Martin Fierro. Latin in the century, realism was more influential, with novelists depicting the
bleaker side of life. The culture of the mass of Latin Americans showed great continuity.
Social changes were slow in coming. Women gained little ground, either at home or as professionals. An
important exception was in schools, where girls as well as boys received public education. The schools also
created a demand for teachers, which women often filled. The society of castas remained largely in place.
Indians were often at odds with central governments, in spite of some attempts to better their situation. Politics
and the economy were still controlled by a small group of white leaders.
The Great Boom, 1880-1920:
136



The decades between 1880 and 1920 saw enormous economic growth in Latin America. Based primarily on
exports, it was supported by a growing alliance between established wealth and new urban leaders. In each
country, a specialty crop in high demand brought in plentiful revenue, but made the national economies dependent
on international markets. Moreover, neighbors went to war to control key commodities. Rapid expansion in the
economy drew foreign investors, creating another area of dependency.
Mexico and Argentina illustrate responses to economic expansion. Porfirio Diaz was elected president of Mexico
in 1876. Centralization allowed the building of an industrial infrastructure. Positivists, cientificos, influenced
Diaz’s policies. Progress came at the expense of improvements in the rural sector. Diaz ruled for 35 years,
during which critics were methodically suppressed. Argentina took a different approach. Buenos Aires
dominated, with one fourth of the national population. European influence was extensive, producing a distinctive
culture. Socialist and anarchist groups emerged, inspiring strikes and walkouts. A Radical party came to power
in 1912, demonstrating the flexibility of Argentina’s political structure.
Following the end of the American Civil War in 1865, the United States was increasingly involved in Latin
America. U.S. investors were heavily involved in Cuba’s sugar exports. The Spanish American War was
fought over Spain’s remaining colonies of Puerto Rico and Cuba. The war ended with American occupation of
the two islands. The United States supported Panamanian independence, and then gained rights over the Panama
Canal. While the completion of the canal was a source for pride for the United States, Latin Americans viewed
the expanding power to the north with suspicion.
Key Terms:
Simon Bolivar
Monroe Doctrine
Manifest Destiny
Porfirio Diaz
Positivism
Mexican American War
Benito Juarez
Miguel de Hidalgo
Spanish American War
Panama Canal
Chapter 25, Quiz Questions
1) In what way was Latin America different from other regions that remained outside the direct control of Western
imperialism?
A) Latin America had no economic relationship with the West.
B) During the century of imperialism, Latin America cast off previous colonial controls through revolution.
C) Unlike other regions, Latin America had no previous political relationship to the West.
D) The ideals of the Enlightenment had little or no impact on Latin America.
E) It achieved rapid industrialization through the use of capital from the United States.
2) Which of the following was NOT one of the beliefs that Latin America shared with the West during the 19th century?
A) Belief in "progress"
B) Belief in reform
C) Belief in absolute monarchy
D) Belief in private property
E) Belief in democratic ideals
3) Which of the following events was rejected by Creole elites as a model of revolution because of excessive radicalism?
A) American Revolution
B) French Revolution
C) Independence movement in Portugal
D) Napoleonic Wars
E) English Civil War
4) What was the impact of the Haitian independence movement on Latin America?
A) It served as a model for national independence movements throughout South and Central America.
B) It demonstrated the power of Creole elites in leading revolutionary movements.
C) It led immediately to a general abolition of slavery throughout Latin America.
D) Creole elites viewed it with horror as an example of general social upheaval.
E) It led to freedom for slaves in Latin America and the United States.
137
5) In what way was the government of independent Brazil different from those of the other newly created Latin American
nations?
A) Brazil was a monarchy rather than a republic.
B) It was the only government that immediately abolished slavery.
C) It was ruled by the popular majority of former slaves and people of mixed race.
D) It was the only republican form of government established in Latin America.
E) It abolished all forms of racial discrimination.
6) Which of the following statements is most accurate?
A) While most leaders sought to maintain Catholicism as the official religion of the new states, some sought to end the
exclusion of other faiths.
B) Roman Catholicism had been the only state religion during the colonial period, and its status as the only permitted
religion remained unchallenged after independence.
C) The onset of independence was accompanied by a general trend away from Catholicism toward Protestant religions.
D) The defense of the Roman Catholic church became a rallying cry for Latin American Liberals.
E) As Spanish power declined, so did the power of the Roman Catholic church.
7) Which of the following statements concerning the political organization of Latin America to 1850 is most accurate?
A) Most of Latin America was divided up into consolidated units that mirrored the colonial vice royalties.
B) The excellent colonial road system enabled the creation of larger states after independence.
C) Permanent consolidation and union was more typical of Central America and southern South America than elsewhere.
D) Most attempts at consolidation and union had failed.
E) Traditional tribal boundaries formed the basis for new nation-states.
8) Which of the following characteristics is most accurately associated with the "centralist" theory of government in Latin
America?
A) Fiscal and commercial policies set by regional governments
B) Recognition of the political rights of Indians and mestizos
C) Democracy
D) Strong national governments with broad powers
E) Multi-party parliamentary governments
9) Which of the following political principles would NOT be associated with Latin American Liberals?
A) Rights of individuals
B) Secular society modeled on the United States
C) The retention of colonial governors and viceroys
D) Development of commerce
E) Literacy and education
10) In what way were both Liberal and Conservative parties in Latin America similar?
A) Their distrust for Roman Catholicism
B) Their acceptance of federalist political theory
C) Their endorsement of centralist political theory
D) The social origins of their leadership
E) Their beliefs about economic systems
11) Which of the following factors did NOT account for the economic stagnation in Latin America from 1820 to 1850?
A) Slow recovery of the mining sector after wars of independence
B) Lack of a transportation network and port facilities
C) Lack of capital for investment in industry
D) Absence of foreign markets for raw materials
E) A rural and industrial unskilled labor force
12) Which of the following descriptions of the politicians who led Liberal governments in post-1860 Latin America is
138
NOT accurate?
A) They represented a new generation of politicians who had matured after independence
B) They favored expansion of the franchise to Indians and mestizos who represented the "ancient" aspects of Latin
American civilization
C) Their inspiration was England, France, and the United States
D) They were firm believers in progress, education, and free competition within a secular society
E) They were intellectual heirs of the European Enlightenment.
13) Under Juan Manuel de Rosas, the United Provinces of the Rio de la Plata
A) adopted the federalist program of a weak central government and local autonomy.
B) overthrew the dominance of Buenos Aires province.
C) undertook a program of education and economic training among the Indians.
D) introduced a democratic regime that recognized political diversity without violence.
E) reintroduced Spanish culture and political institutions.
14) Following the 1830s, the literary and artistic style preferred in Latin America was
A) neo-classicism.
B) romanticism.
C) existentialism.
D) Dadaism.
E) impressionism.
15) Which of the following statements concerning the status of women in post-independence Latin America is most
accurate?
A) Following independence, women gained little ground and there was virtually no change in the attitudes toward
women's proper role in society.
B) Because of their participation in the independence movement, women were rewarded in post-revolutionary Latin
America with voting rights and access to political office.
C) Because women in general failed to support the revolutionary movements, they were not included in the massive
reforms that followed independence.
D) The status of women actually declined after independence, as they were cut off from areas such as public education
that had been available to them in colonial Latin America.
E) Women enthusiastically demonstrated for greater political and social rights.
16) Which of the following beliefs is NOT associated with the concept of "modernization" or "Westernization?"
A) Development was a matter of increasing per capita production in any society.
B) The more industrialized and urbanized any society became, the more social change and improvement were possible as
traditional patterns and attitudes were abandoned.
C) Change would take place through radical or revolutionary transitions rather than gradually.
D) As the process occurred, there would be a natural movement toward more democratic forms of government and
popular participation.
E) Education was the key factor in any modernization program.
17) The "dependency theory" of economic development is associated with what political group?
A) Liberals
B) Marxists
C) Conservatives
D) Monarchists
E) Positivists
18) Which of the following statements concerning the Díaz government in Mexico is most accurate?
A) Under Díaz, reforms were undertaken that finally began to resolve the inequity of land distribution in Mexico.
B) Labor unrest and political instability decreased significantly by the beginning of the 20th century.
C) Díaz's strongly centralized government actively discouraged foreign investment in Mexican mining and transportation.
D) Under the guise of modernization, the forms of Liberal government were maintained but were subverted in order to
139
keep Díaz in power.
E) All the people of Mexico benefited from the reforms of this era due to the growing economy.
Essay Questions:
The Consolidation of Latin America, 1830-1920
1. In terms of models of revolution available to Latin Americans in the early 19th century, which examples were
considered acceptable, which unacceptable, and why?
2. In what ways were the revolutionary movements in Mexico, northern South America, and southern South
America similar?
3. In what ways were the Latin American economies of the 19th century dependent on the industrialized nations of
Europe and North America?
4. What are the explanations offered for the relative “underdevelopment” of Latin America? Which explanation
seems to be the most sufficient explanation?
5. Discuss the relationship of the United States with Latin America from the mid-19th century to 1910.
140
Chapter 26, Civilizations in Crisis: The Ottoman Empire, the Islamic Heartlands, and Qing China
Summary:
The parts of Asia still independent from European dominance after 1750 suffered from political decline and form the
reactions to new challenges. They also faced the threat of Western imperialism and the West’s industrial lead. China,
under the Qing dynasty in the 117th century, enjoyed growth and prosperity and had the power to limit European
intervention. The Ottomans, on the contrary, were in full retreat. Russia and Austria seized territories, North African
provinces broke away, and local leaders throughout the empire became more independent. Economic and social
disruption accompanied the political malaise. Although the Ottoman rulers did not have a solution to their problems, they
regained some strength during the 19th century by following Western-style reforms. At the end of the century, the
foundations of Chinese civilization had been demolished by internal and external pressures.
Key Concepts:
From Empire to Nation: Ottoman Retreat and the Birth of Turkey:



Ottoman decline can be attributed to weak rulers in a system dependent on effective leadership. Urban artisans
suffered from competition from European goods, resulting in urban riots. With division at the top and the
empire’s commercial economy threatened, European neighbors could take advantage of Ottomans back in the
Balkans.
Yet the empire survived, in spite of military defeat and territorial loss. This was in part due to European efforts to
support the Ottomans against the Russians. Reforms within the empire only further divided the ruling elites.
Selim III attempted reforms, which were viewed as a threat to the Janissaries and other groups in power.
Mahmud II was more successful in pushing reform. Intentionally spurring the Janissaries to mutiny, Mahmud
then suppressed them. His reforms followed Western precedents. The Tanzimat reforms – from 1839 to 1876 –
included Western-style universities, legal reforms, and establishment of newspapers. Opening the economy to
foreigners adversely affected artisans. Pushing reforms against women’s seclusion, veiling, and polygamy had a
limited impact.
The reform movements brought Western-educated Turks to question the role of the sultanate. Abdul Hamid
attempted to establish autocratic rule, while still continueing reforms. The coup of 1908 brought the Young Turks
– members of the Ottoman Society for Unioin and Progress – to power. The constitution – set aside by Abdul
Hamid – was reestablished, with the sultan a figurehead. Internal struggles and Balkan conflicts nearly toppled
the movement. Arabs of the Fertile Crescent and Arabia were disappointed to find the Young Turks wished to
maintain the empire.
Western Intrusions and the Crises in the Arab Islamic Heartlands:



Arabs of the Ottoman Empire had some commonalities with the Turks, especially Islam, but were left undefended
from European attacks.
Napoleon’s invasion of Egypt in 1798 opened a new era in reforms between the Middle East and Europe. At the
time, Egypt had been led by Mamluk families, headed by Murad. Their defeat by Napoleon was a shock,
following as it did centuries of Mamluk military ascendancy. The conflict brought no lasing gains for France, but
it was a watershed. Muhammad Ali emerged to lead Egypt following Napoleon’s departure. He reformed the
army along Western lines. Egyptian peasants were forced to grow export crops. His successors were unable to
capitalize on his partial success. His descendants – the Khedives – ruled Egypt until 1952.
Muhammad Ali’s reforms made Egypt dependent on cotton exports and therefore at the mercy of European
markets. European lenders gained control of cotton prices and then shares in the Suez Canal. Courses proposed
among Egyptians to solve the problem of weak sultans and khedives included jihad and more borrowing from the
West. These two approaches were and are essentially at odds. Afghani and Muhammad Abduh favored the
latter course, partly because they valued the Islamic tradition of rational inquiry. The financial problems of the
khedives led to greater financial control of British and French bankers. Ahmad Orabi led a revolt in 1882, which
resulted in the British restoration of the khedives. British control of the puppet rulers and British financial control
began a new era.
141

Egypt had become involved in wars in the Sudan. Egyptian power, centered on to stop the slave trade.
Muhammad Ahmad emerged to lead these opponents as the Mahdi. He launched a jihad against Egypt and
Britain, motivated by a desire to purify Islam. His military skill led to control of modern Sudan. Following his
death, a successor, the Khalifa Abdallahi, built a state in the region. The British sent Lord Kitchener to Sudan in
1896. The Battle of Omdurman in 1898 ended the Mahdist state.
The Last Dynasty: The Rise and Fall of the Qing Empire in China:






Manchu , north of the Great Wall, were united by Nurhaci in the early 1600s. His banner armies were a powerful
force. For decades, the Manchu learned from the Chinese bureaucratic methods and employed scholar-officials.
Called in to help put down a rebellion, they instead took Beijing. Under the dynastic name Qing, they ruled
China. The Manchu elite ruled with few changes to court or bureaucratic procedure. They patronized traditional
Chinese arts and Confucianism. Kangxi was himself an important Confucian scholar.
Minimal changes occurred in Chinese society under the Manchu, except possibly a decline in the status of
women. Rural reforms attempted to bring more land into cultivation and restore the infrastructure of dikes, roads,
and irrigation. These improvements were partially successful, yet did little to mitigate the power of landlords.
Merchants did well under the Qing as exporters of tea and silk. These compradors linked China to the rest of the
world.
Qing decline went along familiar lines. The examination system ceased to fill its role in bringing forward able
administrators. Posts could be bought, and cheating was allowed. The abuses were troubling in a system based
on Confucian education, intended to engender concern for the people of China. Again, public works in rural areas
were abandoned. In the Shangdong peninsula, the Huanghe river was allowed to flood. Thousands died form
famine and disease. Banditry, on the rise, signaled a weakening dynasty. Many expected that a new dynasty
would now renew the historical cycle.
Yet the new “barbarians” threatening China could not be sinified and absorbed. In the 18th century, British
merchants had turned to opium for export to China. British depended on the trade, but the Chinese saw it as a
threat. As much as one percent of the Chinese were addicted, causing widespread social and administrative
problems. Efforts to stop the trade began in the 1820s. In the 1830s Lin Zexu was sent to end the opium trade.
To do so he confiscated opium, destroyed warehouses, and imposed a blockade. The resulting Opium War ended
with Chinese defeat. China was forced to open its ports to foreign trade. Hong Kong was developed as a British
outpost. British officials oversea Chinese trade, and the government was forced to accept foreign ambassadors.
Chinese defeat and growing foreign interference led to revolts. The Taiping Rebellion was led by Hong
Xiuquan against the Zing. Although successful militarily, the movement fell apart, especially under British
opposition. The Taiping Rebellion challenged not just the Qing government, but also the traditional order. The
scholar-gentry thus rallied to the regime. Men such as Zeng Guafan led the self-strengthening movement
against Western influence, while embracing Western technology. Manchu attempts at reform were blocked by
those resistant to change, such as the dowager empress Cixi. In 1901, the Boxer Rebellion tried to expel
foreigners. It resulted in greater European control.
Numerous secret societies formed to end Qing rule, without success. Yet they spawned a succeeding generation
of reformers, such as Sun Yat-sen. These revolutionaries targeted foreigners. In 1911, they forced the Manchu
from power. The revolution ended the Qing dynasty. In 1905, the civil service exams had been discontinued,
after 2,500 years.
Key Terms:
Qing Dynasty
Opium Wars
Puyi
Banner Armies
General Kitchner
Muhammad Ali (Egypt)
Khedives
Suez Canal
Tanzimat Reforms
Young Turks
Chapter 26, Quiz Questions
1) Which of the following statements concerning the Muslim economy at the beginning of the 18th century is most
accurate?
A) The Muslim economy remained dependent on Arab merchants who traded predominantly with Africa.
B) By holding the Europeans at bay, the Muslims were able to capitalize an indigenous industry based on the production
142
of cotton textiles.
C) The prosperity of the industrial base of the Muslim empires led to a close alliance between the artisans and the
government.
D) Merchants within the empire, especially those who were Jews or Christians, grew more dependent on commercial
dealings with European counterparts.
E) The discovery of new resources caused a resurgence of Ottoman economic power.
2) What was the result of the reforms of Sultan Selim III (1789-1807)?
A) Western-style education was introduced throughout the empire.
B) The Janissary corps was eliminated as a political and military force.
C) The sultan was toppled from the throne by a Janissary revolt.
D) Railways were constructed connecting the empire with Europe.
E) The empire was taken over by Muhammad Ali.
3) Which of the following was NOT part of the Tanzimat reforms in the Ottoman Empire?
A) Introduction of Western-style education in the universities
B) Addition of state-run postal and telegraph systems
C) Creation of a constitution based on European prototypes
D) Elimination of religious protection for minority religious groups
E) Extensive legal reforms
4) Which of the following statements concerning the improvement of women's status as a result of the Tanzimat reforms
is most accurate?
A) The inclusion of reforms for women was not even considered in Ottoman society during the period of the Tanzimat
reforms.
B) Despite widespread discussion of the practices of seclusion, polygamy, and veiling, few improvements in women's
social status were won in the 19th century.
C) While not all women benefited, elite women were freed from the restrictive aspects of Muslim society during the
period of the Tanzimat reforms.
D) Muslim restrictions against the social equality of women were swept away as part of the Tanzimat reforms.
E) Women in the Ottoman Empire became as free as those in western Europe and the United States.
5) Which of the following reforms resulted from the coup in the Ottoman Empire of 1908?
A) The sultanate was abolished.
B) The constitution of 1876 was restored.
C) Janissaries were removed as a political and military force.
D) Restrictions against women in Muslim society were removed.
E) Removal of the political influence of the officer corps
6) Which of the following statements concerning the relationships between the Young Turks and the Arabs of the
Ottoman Empire after the 1908 coup is most accurate?
A) The Young Turks harbored resentment against the Arabs of the empire for failing to support the 1908 coup.
B) Arab support of the 1908 coup waned when they discovered that the Young Turks had no intentions of abandoning the
concept of empire.
C) The 1908 coup resulted in the immediate independence of the Arab portions of the Ottoman Empire.
D) The close alliance between the Young Turks and the Arab leaders of the Ottoman Empire continued after the 1908
coup.
E) The Young Turks supported the idea of autonomy for the Arab portion of the empire.
7) Which of the following reforms was NOT introduced in Egypt after 1801?
A) Restoration of the Mamluk armies
B) Introduction of Western-style military conscription
C) Hiring of French military advisors
D) Importation of Western arms
E) Rebuilt the irrigation systems
143
8) What prevented Muhammad Ali from overthrowing the Ottoman Empire?
A) His failure to develop a modern army
B) Lack of a navy
C) His defeat by the Ottomans at Omdurman
D) Opposition of European powers
E) Completion of the Suez Canal
9) What was the result of the rebellion by Egyptian army officers in 1882?
A) The Khedival government was overthrown by an indigenous Egyptian government.
B) A new constitution was instituted, modeled on the Ottoman constitution of 1876.
C) The rebellion was crushed by the Turkish elements within the Egyptian army.
D) The Khedive called on the British to crush the rebellion, resulting in British overlordship of Egypt.
E) British influence in Egypt was ended and the Ottomans reasserted control.
10) Which of the following statements concerning the Manchu government is most accurate?
A) They destroyed the scholar-gentry in order to consolidate their grip on the government.
B) The civil service examination system was eliminated as a means of entering the government.
C) Though Manchus occupied a disproportionate number of the highest political positions, there were few limits on
Chinese promotions within the imperial bureaucracy.
D) Chinese officials were eliminated at the local administrative levels in order to prevent the extreme regionalization that
had led to the downfall of previous dynasties.
E) They welcomed modernization and Western influence after the Boxer rebellion in 1898.
11) In what area did the Manchus attempt to take strong measures of reform?
A) Elimination of the scholar-gentry
B) Removal of social restrictions on women
C) Overturning the Confucian social hierarchy of age and sex
D) Alleviating rural distress and unrest
E) Eliminating the influence of religion
12) What accounts for the general failure of Manchu attempts at reform?
A) Resistance on the part of the peasantry
B) Enormous population growth and the disappearance of open lands
C) Buddhist resistance
D) Loss of territory to nomads from the Asian steppes
E) Strong resistance from the scholar-gentry
13) All of the following signs of dynastic decline were apparent in the Qing regime by the beginning of the 19th century
EXCEPT
A) corruption of the examination system.
B) diversion of revenue from state projects to private fortunes.
C) failure of foreign commerce.
D) food shortages, mass migrations, and banditry.
E) explosive growth of the bureaucracy.
14) What was the impact of the British opium trade on China?
A) Its use was restricted to the peasantry of northern China, where production of food rapidly decreased.
B) The government was quickly able to halt the importation of opium, so that it did not have the disastrous impact on the
Chinese population that was expected.
C) Within years China's favorable balance of trade was reversed and silver began to flow out of the country.
D) Due to the addiction of the imperial court, the British were welcomed as a valuable trade partner of China.
E) The opium trade had little economic effect on China in the long run.
15) What was the outcome of the Opium War?
144
A) Despite technological advantages, the British forces were overwhelmed by the Chinese numerical superiority and were
unable to penetrate China's isolation.
B) The British soon swept the seas of opposition, but were prevented from entering China by opposition from other
European powers who feared Britain's overthrow of the Manchus.
C) The British victory was so overwhelming that the Manchu dynasty was overthrown by 1850 and replaced by a
republic.
D) British victory in the Opium War allowed European powers to force China to open trade and diplomatic exchanges.
E) The Opium War led directly to the Sino-Japanese War and China lost Korea to Japan.
16) What was the political and social position of the Manchu rulers at the end of the 19th century?
A) The Manchu rulers stubbornly resisted the far reaching reforms that were the only hope of saving the regime and
Chinese civilization.
B) The last decades of the dynasty were dominated by Cixi, a woman who proposed radical reforms of the social order.
C) The Chinese scholar-gentry and the provincial elite allied with the emperors to introduce significant reform of
landholding practices and regional administration.
D) The dynasty wholeheartedly embraced the ongoing Westernization of the Chinese government and economy.
E) The Manchu rulers adopted a wait-and-see attitude with regard to Western-style reforms.
Essay Questions:
Civilizations in Crisis: The Ottoman Empire, the Islamic Heartlands, and Qing China
1. How did the emergence of the industrialized West affect the pattern for the decline of civilizations?
2. How did the experience of the Qing dynasty in China and the Ottoman Empire differ between 1700 and 1900?
3. Compare and contrast the conditions leading to the overthrow of the Ottoman Empire in 1908 and the Qing
dynasty in 1912.
4. Compare and contrast the British intervention in Egypt with its intervention in China.
5. Contrast the reactions of the Islamic heartlands and China to the challenge of the West.
145
Chapter 27, Russia and Japan: Industrialization outside the West
Summary:
Russia and Japan defied the pattern of 19th-century European domination. By 1914, they launched significant
industrialization and accomplished other changes that preserved their independence. Both achieved economic autonomy
and were able to join in the imperialist scramble. There were differences between the two. Japan displayed more political
flexibility than did Russia. Change in Russia increased internal strains and led to revolution. Japan through its reforms,
pulled away form the rest of East Asia; Russia continued expanding its influence in Eastern Europe and central Asia.
Among the characteristics common to the two nations in their maintenance of independence was their prior experience of
cultural imitation: Japan from China, and Russia from Byzantium and the West. They were able to learn without
destroying their own cultures. Both also had improved their political effectiveness during the 17th and 18th centuries, a
situation allowing the state to sponsor change.
Key Concepts:
Russia’s Reforms and Industrial Advance:



In the wake of the French Revolution, Russia turned form following Western models. Alexander I supported the
Holy Alliance in their defense of the religious and political order. Yet Russian intellectuals maintained ties to the
West. Pushkin was one of many writers to embrace and enhance the Romantic style. The Decembrist Uprising
of 1825 pushed Nicholas I further to the right. Restrictions on political freedom followed. The revolutions of
1830 and 1848 skipped Russia. At the same time, Russia expanded its territory. A Polish national uprising in
1830-1831 was brutally suppressed. Pushing south, Russia took Ottoman lands and supported Greek
independence.
Russian industrialization did not keep pace with the West. Peasant labor service was increased to meet demands
for grain exports, and the grain trade did have a positive effect on industrialization. The Crimean War – from
1854 to 1856 – pitted Russia against the Ottoman Empire. France and Britain, fearful of expanding Russian
power, supported the Ottomans. Russian leaders saw the advantage industrialization had given Western powers,
and Alexander II pushed for reform. Serfdom was a key issue, and reforming the institution was clearly
necessary.
The emancipation of the serfs, in 1861was carefully planned to maintain tsarist control. The serfs received
lands, but had to pay redemption fees. Peasant revolts actually increased because of disappointment at the
limitations of the reforms. The tsar set up Zemstvos, which gave some political experience to more Russians.
The army was reformed and recruitment expanded. Literacy and demands for popular fiction increased.
Women’s roles broadened. Industrialization was par of these changes. The trans-Siberian railroad linked
western Russia to the Pacific, additionally stimulating the coal and iron industries. Industrialization picked p,
especially in Moscow, St. Petersburg, and Polish towns. Sergei Witte, the minister of finance from 1892 to 1903,
modernized the Russian economy. Foreign control increased, and Russia became a debtor nation. While the
volume of manufactures was large, Russia was still only partly industrialized.
Protest and Revolution in Russia:


Minority nationals raised concerns in Russia, but were secondary to the dislocations caused by industrialization.
Calls for reform developed along two lines. Liberal reforms were sought by businesspeople and professionals.
The intelligentsia and student groups called for more radical reform, but remained isolated. Lastly, anarchists
aimed to end all government. Failing to find popular support, they turned to violence. Alexander II responded by
withdrawing support, they turned to violence. Alexander II responded by withdrawing support for reform. He
was assassinated in 1881. Repressive measures followed, including anti-Semitic policies, and pogroms. Marxism
took hold by the 1890s. Vladimir Ilyich Ulyanov, Lenin, adapted Marx’s ideas to Russian conditions. His
version of Marxism was dopted by the Bolsheviks. Dissatisfaction grew among workers, who unionized and
organized strikes. These different currents of unrest made revolution in Russia likely.
Russia made gains against the Ottoman Empire in the late 19th century. Aiding the Serbian and Bulgarian
independence movements added to Russian pride. However, the Russo-Japanese War broke out in 1904, when
Russia threatened Japan’s regional control. The Russian defeat led to the Russian Revolution in 1905. The tsars
146

created the duma to satisfy liberals. The Stolypin reforms eased the peasants’ redemption payments. Kulaks,
peasant entrepreneurs, brought land to develop. The duma’s power was steadily weakened, and the Russian
government turned its attention to the Balkans.
Similar patterns existed in other eastern European nations. Some chose parliamentary governments, some
monarchies. Eastern Europe experienced a period of cultural flowering, with new pride in Slavic culture.
Japan: Transformation Without Revolution:






Japan’s shogunate ruled in the early 19th century, with few changes. Shrinking revenues weakened the power of
the shoguns after 1850. Developments in intellectual life included the terakoya, or public schools, leading to
literacy rates of 40% for men and 15% for women. Nationalist leanings led to the celebration of Shintoism and
Japanese culture. At the same time, Dutch studies continued in spite of bans on Western reading. Controlled by
monopolies, commerce boomed. Slowing economic growth after 1850 and riots in rural areas led to a climate
where change was welcome.
The arrival of Matthew Perry in 1853 threatened Japanese isolation. By 1856, two Japanese ports were open to
U.S. commerce. The emperor was pressured to open the country further. Samurai were especially keen, hoping
that the change would dislodge the shogun. The samurai began using America firearms in 1866 and defeated the
shogun’s troops. Reform came with the installation of a new emperor – Mutsuhito – called Meiji.
The Meiji government replaced the daimyo system with prefects. Samurai were sent to Europe and the United
States to learn, turning the group into a force for change. The reforms of 1873-1876 ended samurai privileges and
introduced conscription. Iwasaki Yataro is an example of samurai who changed his stripes. Founding Mitsubishi
in 1868, he built railroads and steamer lines. Political parties emerged. A new constitution in 1889 included a
diet, modeled on the German legislature. Japan was successful in borrowing from the West while maintaining
much of its traditional structure.
Reforms continued with an overhaul of the army and navy. Priority was given to industrialization. Internal tariffs
and guilds were ended to clear the way for a unified economy. The government was closely involved in the
process of industrialization. Western models were adapted to Japanese conditions. At the same time,
entrepreneurs from all levels of society played an important role in the changes. Industrial conglomerations,
zaibatsu, emerged. Industrialization was well advanced by 1900. Still dependent on imports, however, the
country lagged behind the West. Cottage industry and sweatshops were common.
Japanese society experienced change as a result of economic and industrial change. Population growth was an
important issue. Public education was offered to all, focusing on the sciences and technology. Rapid
Westernization in the 1870s was replaced by more attention to Japanese values and social structure. Western
lifestyles, clothes, and measures were adopted. Women’s roles saw little change Shintoism gained ground.
Foreign policy was adapted to Japan’s increasing involvement in the global economy. The Sino-Japanese War
gave Japan a quick victory over China, and hegemony in Korea. Forced by European powers to abandon territory
it had taken in mainland China, Japan turned its eyes to Russian lands. The Russo-Japanese War of 1904 was
another Japanese victory, and Korea was annexed in 1910.
Urbanization and industrialization resulted in strains in Japanese society. Politics reflected the tensions, with
assassinations and frequent dismissal of the Diet. Among intellectuals, questions were rife about Japanese
culture’s survival. Their government’s response was to promote nationalism. The country thus avoided the
revolutionary turmoil that affected China and Russia.
Key Terms:
Sino-Japanese War
Duma
Meiji Restoration
Russo-Japanese War
Zaibatsu
Decembrist Uprising
Emancipation of Serfs
Trans-Siberian Railroad
Bolsheviks
Chapter 27, Quiz Questions
1) Which of the following represents a significant difference between Russia and Japan?
A) Only Japan had a significant aristocracy prior to industrialization.
B) Only Russia participated in territorial expansion by 1914.
C) Russia engaged in selective borrowing from Western models by 1700.
147
Kulaks
D) Only Japan underwent significant political revolution prior to 1914.
E) Only Japan experienced Communist uprisings by 1905.
2) Which of the following reflects a significant similarity between Japan and Russia during the period of industrialization
prior to 1914?
A) Both experienced significant political revolutions.
B) Both Japan and Russia had prior experience of imitation, Japan from China and Russia from Byzantium and the West.
C) Both demonstrated remarkable political flexibility resulting in sweeping transformations of political structure.
D) Both engaged in territorial acquisitions in the Ottoman Empire.
E) Both had a large industrial labor force coupled with a small agricultural base.
3) Which of the following accounts for Russia's lack of significant revolution in 1830 and 1848?
A) Russia lacked a substantial history of autocracy.
B) Absence of a coercive labor system in Russia left scant cause for rebellion.
C) Political repression
D) Like England, Russia's history of participatory government and its national parliament forestalled revolution.
E) Major reforms, satisfying most Russians
4) Which of the following statements concerning Russian territorial expansion is most accurate?
A) Russia's loss of Poland in the revolt of 1830 stimulated other attempts at territorial expansion.
B) Russia actively opposed nationalist movements in the Balkans in keeping with its conservative tradition.
C) Western powers actively aided Russia's pursuit of territories in the Ottoman Empire.
D) No massive acquisitions marked the early 19th century, but Russia continued to be an aggressive competitor for
territorial expansion.
E) Russia expanded rapidly south of Alaska to Oregon and east to Utah.
5) Which of the following statements best describes the Russian economy at the beginning of the 19th century?
A) Russia had achieved economic autonomy in the 18th century, although most of eastern Europe remained largely
agricultural.
B) The Russian economy was geographically oriented to the Ottoman Empire, a feature of the Mongol domination of
Russia until the 15th century.
C) Russia's economic dynamism and innovation rivaled that of the West.
D) In return for low-cost grain exports, Russia and other east European areas imported Western luxury goods for
aristocrats to display as badges of respectability.
E) Russian heavy industry accounted for nearly all its economic growth.
6) What accounted for the West's victory over Russia in the Crimean War?
A) The war was fought far from Russia, necessitating lengthy lines of communication and supply.
B) Russia was forced to fight an offensive war against entrenched positions.
C) The war was fought almost entirely at sea where the Russians were unable to bring numerical superiority to bear.
D) The Western nations won because of industrial advantages.
E) Sardinian and Polish assistance to the Russians was ineffective in the long run.
7) Which of the following was NOT a consideration in the reform of serfdom?
A) An attempt to sweep away the tightly knit peasant communities on which serfdom depended
B) The development of a vigorous and mobile labor force
C) A desire to meet Western humanitarian standards
D) Periodic peasant uprisings focused on lack of freedom, undue obligations, and lack of land
E) The preservation of aristocratic power
8) Which of the following statements concerning the emancipation of the serfs in Russia is most accurate?
A) The emancipation of the serfs destroyed the Russian aristocracy.
B) Emancipation of the serfs loosened the grip of the tsarist state.
C) In addition to personal freedom, the serfs were granted parcels of land subject to redemption payments.
D) Following emancipation, peasants were free to move about Russia as they pleased leading to massive movements of
148
agricultural labor.
E) Few serfs were really emancipated due to the conditions imposed on their release.
9) What was the first step toward industrialization in Russia?
A) The construction of factories
B) The development of the mining sector
C) The creation of an extensive system of railways
D) The end of the grain trade with the West
E) Mechanization of agriculture
10) Which of the following statements concerning the capitalization of Russian industry is most accurate?
A) Capital for Russian investment was almost entirely derived from liquidation of agricultural estates in Russia.
B) By 1900 approximately half of Russian industry was foreign-owned by British, German, and French industrialists.
C) It was the contact with the Japanese that led to an influx of capital for Russian industrialization.
D) Russian industry was capitalized by a substantial middle class that had built up wealth in the grain trade with the West.
E) United States investors were the largest owners of machinery in Russia by 1900.
11) Which of the following was present during the Russian program of industrialization?
A) Attitudinal change among workers similar to the West
B) A large middle class
C) Rich natural resources
D) Small, but efficient, factories
E) A highly educated work force
12) What was the general goal of the Russian intelligentsia?
A) Political freedom, social reform, and retention of Russian culture
B) Political freedom, retention of the social hierarchy, and increased Westernization
C) Restriction of civil liberties, honor and deference to the emperor, retention of Russian culture
D) Radical Westernization as part of a program of increased industrialization
E) Globally competitive economy
13) Which of the following statements about Russian Marxism is most accurate?
A) Marxist insistence on careful revolutionary organization and a focus on the working class were rapidly assimilated by
anarchists and peasant groups.
B) Marxist doctrines were not imported from the West, but originated among the Russian intelligentsia.
C) Lenin introduced important innovations in Marxist theory, including the idea that a proletarian revolution could take
place without going through a middle-class phase.
D) Lenin was dedicated to the mass electioneering typical of Western socialist parties.
E) Marxist doctrines were most applicable to an agrarian economy, not an industrial society.
14) Which of the following Russian developments was NOT adopted in other east European states?
A) National parliaments
B) Emancipation of serfs
C) Economic autonomy from the West
D) Monarchic forms of government
E) Nationalism
15) Which of the following statements concerning the Tokugawa Shogunate in the 19th century is most accurate?
A) The Shogunate bureaucracy had been opened to talented commoners, a reform that improved the standing of the
government with the masses of the Japanese people.
B) By the 19th century, the Tokugawa were able to dispense with the feudal organization of earlier Japan.
C) Increasingly, the Shogunate depended on its long-standing alliances with Western powers to maintain its dominance.
D) The Shogunate continued to combine a central bureaucracy with semi-feudal alliances with regional daimyos and the
samurai.
E) The Shogunate managed its finances carefully, and never carried a deficit.
149
16) Which of the following was NOT an advantage of Japan over China in the competition to assume leadership and to
establish industrialization in Asia?
A) Japan's leadership was less secular and bureaucratic than that of China.
B) Japan already knew the benefits of imitation, which China had never acknowledged.
C) Japan had allowed a more autonomous merchant tradition.
D) Feudal traditions limited the heavy hand of government controls while stimulating a sense of competitiveness.
E) Western countries were attracted to China first, so Japan had early leeway.
17) Which of the following statements concerning Japanese political reforms in the period of the Meiji state is NOT
accurate?
A) Meiji leaders established a new conservative nobility, stocked with former nobles and Meiji leaders that operated a
British-style House of Peers.
B) Samurai, destroyed by the removal of government stipends, were banned from participation in the Meiji Diet.
C) The constitution issued in 1889 assured major prerogatives for the emperor along with limited powers for the lower
house of the Diet.
D) The bureaucracy was reorganized, insulated from political pressures, and opened to talent on the basis of civil service
examinations.
E) The Meiji came to power with very little violence due to the influence of the Western powers.
18) What was the primary difference between the reformed Japanese government and reformed Russian institutions by
1914?
A) Japan retained an emperor at the head of government.
B) Japan created a national parliament.
C) Japan's government was elected by a broad majority of the population.
D) Japan's government had incorporated business leaders into its governing structure.
E) Russian institutions were more secular than Japan's.
19) One of the major similarities between Japanese and Russian industrialization was the fact that
A) both lacked natural resources.
B) scarce capital and unfamiliarity of new technology compelled state direction.
C) neither was able to complete construction of a railway system.
D) neither had any experience of cultural exchange with the West.
E) a small group of independent entrepreneurs led to movement in each case.
20) Which of the following was NOT a sign of significant social stress in industrialized Japan?
A) Disputes between generations over Westernization
B) The increasing freedom and political influence of women
C) Growth of nationalism
D) Growth of urban slums
E) Racial unrest
Essay Questions:
Russia and Japan: Industrialization outside the West
1. Compare and contrast Japanese and Russian society prior to industrialization.
2. Compare and contrast the political reforms undertaken in Japan and Russia prior to and during industrialization.
3. Compare and contrast industrialization in Japan and Russia.
4. Why did revolution occur in Russia in consequence of industrialization, but not in Japan?
5. Why did Japan rather than China assume leadership in Asia in the face of the challenge of the West?
150
Chapter 28, Descent into the Abyss: World War I and the Crisis of the European Global Order
Summary:
The First World War “The Great War” was one of several key turning points of the 20th century. A combination of
imperialism, arms races, industrial might, and nationalism pushed the Great Powers of Europe into a regional conflict that
quickly exploded into a global war of unprecedented devastation. Among the many results were a loss of global power
for Europe, the rise of the United States and Japan, Bolshevism in Russia, increased nationalism among European
colonies around the world, and political and social power shifts in several nations.
Key Concepts:
The Coming of the Great War:


Germany, led by Kaiser Wilhelm II, was increasingly powerful and aggressive in the 1890s. Britain joined with
Russia and France, forming the Triple Entente, while Austria-Hungary, Italy, and Germany formed the Central
Powers. Italy’s membership was made problematic because of its conflicts with Austria-Hungary. Tensions in
Europe were exacerbated around the world. France and Germany faced off in North Africa, coming to the brink
of hostility more than once. The formation of the two alliances added to the war of rhetoric. An arms race
between Britain and Germany over naval power was matched by growing land forces. Mounting international
conflict was made worse by internal strife, largely resulting from industrialization.
European concerns focused on the Balkans, where a multiplicity of ethnicities struggled. It was the assassination
of the heir to Austria-Hungary, Archduke Ferdinand at Sarajevo, by a Serbian, that triggered the war. Russia
supported the Serbians, as fellow Slavs, transforming a regional crisis into a European war. Britain entered the
war, involving its vast empire and making the conflict a global war. Germany and France carefully planned the
kind of war they were sure would give them a quick victory.
A World at War:




Germany’s strategy of quickly moving through Belgium was stopped by British support of the latter. The war all
had expected to soon win turned into a long standoff on the Western Front. Digging trenches was the only
defense against the new artillery. Staggering casualties and the inability to gain any ground made the war a new
experience. Leaders on both sides failed to adapt to the conditions, sending one group of soldiers after another
“over the top” to die quickly form machine-gun fire.
In the east, Germany pushed Russia back, inflicting large casualties. Nicholas II personally led the fighting, but
with such poor results that it was one of the causes of the Revolution of 1917. Russia had some success against
Austria-Hungary, but gained little ground. Austria-Hungary and Italy turned against each other. British and
French aid helped stop the Austrian assault on Italy, but widespread desertion and the threat of invasion panicked
Italy.
While soldiers faced the inglorious reality of trench warfare, those at home continued to view the war with
undiminished zeal. States expanded to control transportation, direct the media, and impose rationing. Propaganda
was used to keep the home front loyal to the war. Although labor leaders were given a voice in industrial
management, workers’ protests were not silenced. Germany faced revolution in 1918-1919, as food shortages and
labor unrest created a precarious situation. Women took men’s places in factories, gaining better wages than ever.
Many of these gains were lost after the war, but women won the vote in Britain, Germany, and the United States.
Conflicts between European powers extended to their empires. Colonial subjects were called to serve the war.
Britain’s empire in particular expanded the scope of the war. Britain’s 1902 alliance with Japan drew the latter in.
Troops form Britain’s dominions were particularly important in the Middle East, for example in the fighting at
Gallipoli in 1915. British Indian and African troops, and French and German Africans fought in the war. The
Ottoman Empire supported Germany, following cooperation between Germany and the Young Turks. Blaming
the Armenian Christians for Turkish military disasters, the Turks launched the Armenian genocide in 1915. The
United States entered the war in 1917, heralding its real entry into world affairs. Americans were divided on the
question of joining the war, but U.S. businesses profited. German attacks on neutral shipping finally pushed the
United States into the war. By 1918, the large numbers of U.S. soldiers shipped to Europe had begun to the
impact the war.
151

On the Eastern Front, Russia’s withdrawal allowed the Germans to focus on the other front. With U.S. help, the
Germans were halted and then pushed back. The Austro-Hungarian fronts failed, and the Empire broke apart.
Germany agreed to an armistice on November 11, 1918. Having been informed only of victories, the Germans
were stunned, a feeling of betrayal that was later used by Adolph Hitler. With ten million dead and twenty
million wounded, the war far outstripped any that had preceded it. The influenza pandemic that followed claimed
millions more.
Failed Peace:

Peace negotiations were greatly influenced by pressures form each leader’s constituency. Georges Clemenceau of
France wanted the Germans to be punished, as did many British, while their prime minister, David Lloyd George,
balanced those demands with a desire for a more moderate peace. All of the Western powers including U.S.
president Woodrow Wilson, were agreed in applying the principle of self-determination only to European
peoples. Western overseas empires were not disturbed. The Peace of Paris laid down the terms of a peace that
the Germans subsequently fought to overturn. The Germans were intentionally humiliated both in negotiations
and in the terms of the peace. The Russians, Arabs, Chinese, and Vietnamese – in the person of Ho Chi Minh –
were also treated with disdain. The U.S. Congress refused to approve the League of Nations charter.
The Nationalist Assault on the European Colonial Order:






World War I saw the first outright conflict over colonial possessions. Although the colonial powers held onto
their colonies, the war was a period of growing industrial and commercial power of India, and gave the
subjugated peoples a lesson in the barbaric behavior of their masters. In addition, the European overseas military
presence was necessarily lessened. The potential danger this caused was held off by attractive promises, which
were not made good after the war. In short, the war shook imperial control, both by spreading doubts about
Western racial superiority and by weakening of the means of control.
India’s nationalist movement led the way in the colonies by virtue of the size of the colony and because of the
central role it had long held in the British Empire. The movement had all of the elements that were to appear in
later, similar developments: influential groups educated in the West, charismatic leaders that brought the
movement to the masses, and nonviolent means. India’s National Congress Party brought together disparate
groups, and was acknowledged by the British in 1885. Hoping to use the Congress Party to identify rebellious
elements, the British found instead that it became a powerful force for criticism of imperial rule. Many initially
loyal Indians became outraged at their treatment by racist British leaders.
Looking for a cause to mobilize more of their fellow Indians, nationalist leaders began to make use of the
negative economic impact of colonization. Indians paid for British armies, British civil servants, and public
works built using British materials, all of which helped the British economy. In the countryside, subsistence
agriculture and farming for Indian consumption had given way to crops for British consumption. The peasants
were beset by food shortages and epidemics, which were blamed on the British.
The Indian nationalist movement was split by the religious divisions between Hindus and Muslims. Leaders such
as B.G. Tilak supported the establishment of the Hindu religion as a state religion, largely ignoring the Muslim
population. Tilak gained a large following, but left out all but conservative Hindus. British rule was also
threatened by radical groups that sought change through terrorism. Yet more moderate leaders emerged, aided by
the British Morley-Minto reforms, leading to a more peaceful, inclusive independence movement.
The First World war saw the adherence of many Indians to the British cause. At the same time, economic
dislocations had an adverse effect. British failure, in 1818, to honor promises made to Indian leaders during the
war was ameliorated the next year. In 1919, the Montagu-Chelmsford reforms gave Indians some control of
legislation and administration, yet at the same time the Rowlatt Act attacked basic civil rights. In this climate,
Mohandas Gandhi emerged. His attraction lay in his successes in a similar situation in British South Africa, his
nonviolent protests – called satygraha or truth force – his legal background, and the charisma of a guru. He
appealed both to intellectuals and to the mass of Indians.
Nationalism in Egypt, unlike other colonized areas, predated conquest. Lord Cromers rule as high commissioner
included reforms that benefited the ruling elite and some urban areas. The ayan, rural landowners, took advantage
of the reforms to amass larger holding, while spending their time luxuriating in Cairo. Younger sons form the
small but growing middle class, the effendi or professional and business class, formed the independence
movement. Arabic newspapers voiced increasing criticism of the British rulers. In 1906, the Dinshawi incident,
152



resulting in the hanging of four Egyptian villagers, sparked Egyptian demonstrations. By 1918, the force of
nationalism led the British to grant a constitution and representation.
The Ottoman Empire was ended by division. Mustafa Kemal, called Ataturk, rallied the Turks against Greek
nationalism, establishing an independent Turkey rallied the Turks against Greek nationalism, establishing an
independent Turkey by 1923. His rule advanced westernization, but also followed the line of development begun
in the 19th century. France and Britain continued to occupy Arab portions formerly under the Ottomans. Hussein
led Arabian resistance to Britain, helped along by failed British promises for Arabic independence. British and
French mandates were threatened from the outset by the Arabs’ sense of betrayal. The Balfour Declaration,
promising land int eh Middle East to European Zionists, was made good. The Zionist movement, fueled by
pogroms in the late 19th century, was led by such leaders as Leon Pinsker and Theodor Herzl. The Society for
the Colonization of Israel began the process of forming a Jewish nation. The wrongful conviction of Alfred
Dryfus gave further momentum, as French Jews joined the movement. The World Zionist Organization
included Jews form across Europe. Herzl’s success in gaining Palestine for the Jews was a clear message to the
area’s Arabic peoples. British attempted to control both groups.
Egypt’s post-war situation differed from that of the Arab world, because it was already under British control and
did not experience the sense of betrayal over failed promises. However , Egypt was used as a staging ground for
the Entente forces, draining resources. Growing anger, increased when the Egyptian delegation to Versailles – the
wafd – was shunned, led to revolts. . The Wafd party was led by Sa’d Zaghlul. British inquiries into the
situation led a decision to a withdrawal from Egypt form 1922 to 1936. Increased political power was used by
many Egyptian leaders to consolidate their position and increase their wealth. Bankruptcy in the 1940s led to
Gamal Abdul Nasser’s coup of 1952. Massive economic inequities had fed unrest.
Africa differed from India in being colonized just decades before World War I broke out. Again, Westerneducated groups were influential. Again, broken promises had their effect. Again, increasing knowledge of
European weaknesses and repressive measures changed colonial attitudes. Although African resources were
instrumental in the war efforts, economic dislocation had an adverse impact in Africa. African Americans such as
Marcus Garvey and W.E.B DuBois were influential, creating pan-African organizations. Although these did
not lead directly to independence, they helped arouse anti-colonial feelings. The negritude literary movement
gained Africans more respect among the French. Leopld Sedar Senghor, Leon Damas, and Aime Cesaire used
their writings to celebrate their culture. In the post-war decade, many British colonials were given more political
freedom. Early groups usch as the National Congress of British West Africa were replaced by smaller groups,
each representing an individual colony.
Key Terms:
Triple Entente
Self-determination
Balfour Declaration
Mohandes Gandhi
Armenian Genocide
League of Nations
Zionists
Treaty of Versailles
Central Powers
Satygraha
Chapter 28, Quiz Questions
1) Germany's leaders counted on what to help them overwhelm the Belgians and French?
A) Their country's superb railway system
B) Small, but lightning-quick armies
C) Submarine warfare
D) Diplomacy
E) Use of air power
2) By 1915, conflict on the Western Front
A) had become a shifting game of rapid maneuver with few major battles.
B) had resulted in the surrender of France and the establishment of the Vichy government.
C) had resulted in victory for the British and French troops, who pushed the exhausted enemy to the borders of Germany.
D) had settled into a deadly stalemate in which hundreds of thousands of lives were expended for a few feet of trench.
E) had resulted in massive food shortages in both France and Great Britain.
3) Between 1914 and 1917, warfare on the Eastern Front
153
A) pitted the forces of Russia and Austria-Hungary against the invading Germans.
B) had reached a stalemate.
C) resulted in the Serbian knockout of the Austrian forces.
D) featured bloody trench warfare in which almost no land changed hands.
E) was as bloody as the Western Front.
4) Which of the following was NOT a feature of war on home fronts between 1914 and 1919?
A) Governments organized the major sectors of the economy to ration resources and production.
B) Executive branches of government increasingly took over from parliaments.
C) Governments controlled public opinion through manipulation of mass media such as newspapers.
D) Strict government regulation prevented material shortages and famine.
E) Union leadership served on industrial production boards designed to increase production.
5) Which of the following statements concerning the global aspects of World War I is most accurate?
A) The British dominions, Canada, Australia, and New Zealand remained aloof and virtually untouched by the war.
B) By 1914, the United States had not entered the scramble for colonial possessions.
C) American businessmen prior to 1917 profited by selling goods to both sides and by taking advantage of European
distractions to seize new world markets.
D) The United States aggressively entered the war in 1914 to demonstrate its new position as a world power.
E) Germany discouraged colonial nationalists in their struggle against British imperialism.
6) During World War I, the Ottoman Empire
A) vainly attempted to retain its neutrality in what the Turks perceived was a Christian conflict.
B) long attached to German military advisors, joined Germany in the war effort.
C) used the opportunity to reassert Turkish dominance over the Arab regions.
D) launched assaults indifferently against the colonial possessions of Britain, France, and Germany.
E) began a program of "genocide" in Greece, resulting in a massive migration of Greeks to Italy.
7) Which of the following was NOT included in the final set of treaties that ended World War I?
A) A League of Nations was formed, but the United States refused to join.
B) Russia was rewarded for its service to the Allies by the grant of substantial territories in Poland and the Baltic
republics.
C) Germany was forced to accept blame for the war and to pay huge reparations to the victorious Allies.
D) Austria-Hungary was divided up into a Germanic Austria as well as the independent states of Hungary,
Czechoslovakia, and Yugoslavia.
E) Poland was created from eastern German territory.
8) What made colonial regimes particularly vulnerable to challenges from within in the late 19th and early 20th centuries?
A) The growing industrialization of colonial societies
B) Colonial governments were built in collaboration with indigenous elite groups.
C) Their dependence on European military forces
D) The dependence of plantation economies on the West
E) Efforts of communist-based movements to appeal to the peasant classes
9) Which of the following statements concerning the position of the early Congress Party in India is most accurate?
A) It was committed to relieving the poverty of the Indian masses.
B) It demanded the immediate decolonization of India.
C) It was loyal to the British rulers and primarily concerned with interests of the Indian elite.
D) It was closely involved with the development of a strong labor movement within Indian industry.
E) Its leadership mostly came from the peasants of India.
10) Which of the following statements concerning the leadership of the decolonization movement in India just prior to
World War I is most correct?
A) Leadership was assumed by more radical members of the Congress Party such as Tilak just before 1914.
B) The Congress party lost its leadership role to the Socialist Party, which was more willing to court the masses of the
154
Indian peasantry.
C) Tilak's removal and the repression campaigns against terrorists, along with British reforms, strengthened the hands of
the Western-educated moderates in the Congress Party.
D) It is difficult, if not impossible, to identify leadership in the fragmented Congress Party of 1914.
E) The assassination of Gandhi led to a split between Muslim moderates and more extreme Congress Party leaders.
11) The radical wing of the Congress Party under B. G. Tilak proposed what formula as the basis for the party's political
program?
A) An appeal to conservative Hinduism, including boycotts of British goods, full independence, use of Hindu festivals as
opportunities for rallies, and opposition to women's education
B) An appeal to unity among the Christians, Buddhists, and Muslims of India
C) A reform program intended to remove cultural limitations on women
D) Emphasis on Muslim teachings, particularly the need to put off British cultural influence
E) An appeal to leading British industrialists and to Parliament itself to plead their case along with a propaganda
campaign using the British press and media to persuade the common English voter
12) Which of the following groups did NOT benefit from the British economic reforms in Egypt?
A) The ayan
B) The Egyptian bourgeoisie
C) The peasants
D) The Turco-Egyptian political elite
E) The khedives
13) Egypt is the one country in the Afro-Asian world in which
A) decolonization occurred in the 19th century.
B) decolonization failed to occur until the 1980s.
C) European countries failed to establish a colonial base until after World War I.
D) the emergence of nationalism preceded European conquest and domination.
E) both peasants and the educated elite form a secular nationalist party under Nasser.
14) What was the outcome of the Dinshawai incident in Egypt?
A) The British withdrew from Egypt prior to World War I.
B) The British were forced to suppress a revolt led by the Mahdi in the Sudan.
C) Mass support, including the emergence of ayan allegiance, for the national cause grew in Egypt.
D) Islamic mullahs began to preach against the nationalist movement.
E) The British established a mandate in Egypt, leading to the creation of the first democracy in the Middle East.
15) By 1913, the British
A) had granted full independence to Egypt.
B) had eliminated resistance to their regime in Egypt.
C) granted a constitution to Egyptian nationalists.
D) had withdrawn from Egypt.
E) had built a canal in Egypt between the Red Sea and the Persian Gulf.
16) Prior to World War I, what was the primary difference between Egyptian and the Indian nationalist movements?
A) In India a single political party dominated the nationalist movement, but in Egypt a variety of rival parties proliferated.
B) The Egyptian nationalist movement was dominated by peasantry, while in India Western-educated lawyers ran the
movement.
C) Whereas India had been dominated by the British since the 18th century, Europeans played no role in Egyptian
colonialism.
D) Egypt lacked an educated elite capable of assuming leadership of the nationalist movement.
E) The Egyptian nationalist movement had aligned itself with Lenin and the Bolsheviks while the Indian nationalist
movement had remained independent.
17) Which of the following statements concerning the early nationalist movements of Africa is most accurate?
155
A) French-speaking west Africans tended to concentrate their efforts at political representation within their colonies.
B) By the mid-1920s, racist views of African society were becoming more strident, and most Europeans refused to
countenance the concept of a progressive African culture.
C) Pan-Africanism, linking all Africans in a single national movement for independence, was the most successful
apparatus for achieving decolonization.
D) The early leadership of pan-African organizations was more often American and West Indian than African.
E) Most African nationalist movements were split between Muslim and Christian ideals.
Essay Questions:
Descent into the Abyss: World War I and the Crisis of the European Global Order.
1. Discuss the aftermath of World War I and its effect on European society.
2. How did World War I affect outside of Western Europe?
3. Discuss the changes in gender roles brought about by the war.
4. Discuss Ghandi’s approach to the removal of the British from India.
5. Discuss Zionism and how World War I helped make possible the creation of a Jewish homeland.
156
Chapter 29, The World Between the Wars: Revolutions, Depression, and Authoritarian Response
Summary:
The 1920s were profoundly shaped by World War I and by movements well underway before the war. Three major
patterns emerged: First, Western Europe recovered from the war only incompletely: second, the United States and Japan
rose as giants in industrial production; third, revolutions of lasting consequence shook Mexico, Russia, and China. Each
of these developments brought into doubt Western Europe’s assumptions about its place as the dominant global power.
Key Concepts:
The “Roaring Twenties”:





Europe faced massive economic problems after the First World War, yet an optimistic attitude prevailed. In the
arts, Pablo Picasso led the cubist movement, while writers and composers forged new styles. Albert Einstein’s
work challenged traditional physics. Mass consumption was a powerful force, changing as women become
important consumers. Yet signs of economic troubles worried some.
Canada, Australia, and New Zealand won independence and became equal members in the British
Commonwealth of Nations. In the United States, the pace of industrialization continued, with attendant changes.
Production was improved by the innovations of Henry Ford and others. The United States exported its own
culture for the fist time, in the form of jazz music and Hollywood films. The nation withdrew into isolation after
a period of involvement in world affairs. Japan continued to industrialize, relying on exports. Internal strains
increased in Japan between the military and the government.
In 1919, Benito Mussolini founded the fascio di combattimento, which gave fascism its name. Reliant on
aggressive nationalism, the movement called for corporate state. The roots of nationalism lay in the postEnlightenment disenchantment with liberalism. Postwar Italy was a land ripe for an ideology that rejected liberal
ideals in favor of action. In 1927, the king of Italy invited Mussolini to form a government. Mussolini suspended
elections in 1926.
New nations in Eastern Europe were born in a climate of intense nationalism. Rivalries weakened them form the
outset. The fall of agricultural prices in the 1920’s and the Great Depression led to social tensions that paved the
way for authoritarian governments.
Political developments in the 1920’s defy broad generalizations. The advance of democracy in some nations was
paralleled by challenges to democracy in others, or even in the same country.
Revolution: The First Waves:




In Latin America, industrialization brought social conflict. Some political change had taken place. Syndicalism
tapped labor unrest, while in Mexico, outright revolution occurred.
The Mexican Revolution was in part a response to the outbreak of World War I. During the Great War, Latin
American countries lost important markets became more economically independent. By the end of the war,
however, U.S. influence had replaced that of Britain. The dictatorship of Porfirio Diaz, in place since 1876, had
led the way in industrialization, but at the cost of silencing dissent. Even so, tensions persisted. The United
States owned up to twenty percent of Mexican territory. In 1910, Francisco Madero intended to run against
Diaz. When he was imprisoned and a rigged election put Diaz back in power, rebellion followed. The revolt was
led in the north by Pancho Villa, in the south by Emiliano Zapata. Diaz was replaced by Madero, and then
Zapata removed Madero. Victoriano Huerta began a dictatorship, but he too was forced out. Alberto Obregon
finally became president in 1921. The long war had led to 1.5 million deaths. The new Constitution of 1917
promised liberal reforms.
The revolution was largely fought over the issues of nationalism and indigenism. These also inspired such artists
as Diego Rivera and Jose Clemente Orozco. Writers and composers also took up these themes. At the same
time, the Cristeros fought against secularization. The war also brought renewed U.S. intervention. The Party of
the Institutionalized Revolution – the PRI – dominated Mexican politics in the 1920s and 1930s.
Food shortages resulting from World War I led to food riots and strikes in St. Petersburg in 1917. The workers’
soviet took the city, and the tsar then abdicated. Alexander Kerensky and other moderates sought liberal
reforms. However, as the war dragged on and the revolutionary leaders failed to implement real land reform,
157






unrest broke out. Lenin led the November Revolution of the Bolsheviks in 1917. Peace with Germany was soon
made irrelevant by Germany’s defeat. The Russian delegation was snubbed at Versailles. Lenin and his
followers lost to the Social Revolutionary Party in parliamentary elections. In response, Lenin put in its place a
Congress of Soviets, imposing Communist Party control. The United States, Britain, France, and Japan
intervened, with little impact. Economic and political chaos resulted from Lenin’s actions.
Leon Trotsky’s Red Army imposed order. Lenin’s New Economic Policy of 1921 helped to stabilize the
economy. By 1923, a new system was in place: the Union of Soviet Socialist Republics. The Supreme Soviet,
nominally a parliament, was made up of representatives chosen by the Communist Party.
The first years of communism in Russia saw a great deal of experimentation and debate. Lenin’s death in 1924
led to a struggle for power. Joseph Stalin emerged as victor. While Lenin had hoped the Russian example would
engender a global wave of communism, to be organized by the Comintern, Stalin emphasized nationalism. He
also pushed industrialization through a program of collectivization.
In China, the Qing dynasty fell when the last emperor abdicated in 1912. The conflict that followed led to the rise
of Mao Zedong. Military leaders such as Yuan Shikai were prominent. University students, intellectuals, and
secret societies presented their own solutions, but Japan’s intervention decided the issue.
Sun Yat-sen led a coalition of anti-Qing groups. He was elected president in 1911 by his Revolutionary Alliance,
but he ceded power to Yuan Shikai in 1912. It soon became clear the Yuan wanted to emperor. Japan entered the
European war as a British ally, quickly taking German territory in the region. Indecision vis-à-vis aggressive
Japanese demands led to Yuan’s fall in 1916. Japan gained control of northern China in the peace of Versailles.
Chinese Outrage at the concessions to Japan led to demonstrations and the May Fourth Movement. Calling for
democracy and repudiating traditional systems, the movement had a large following. Yet with warlords in power,
more was needed. The Bolshevik success in Russia prompted Chinese intellectuals to adapt Marxism to China.
Li Dazhao postulated that in China, peasants would take the place of urban workers in the revolution. Mao
Zedong was highly influenced by Li. A meeting of Marxist in Shanghai in 1921 formed the nucleus of the
Chinese Communist Party.
The Guomindang, or Nationalist party, led by Sun Yat-sen, prevailed in the south. They concentrated on
international and political issues, leaving aside critical domestic issues, including land reform. An alliance with
the Communists was declared in 1924. The Whampoa Military Academy, founded in 1924, was first headed by
Chiang Kia-shek.
The death of Sun Yat-sen in 1925 left an opening filled by Chiang Kaia-shek. His nationalists took Shanghai and
Beijing. He attacked the Communists, bringing Mao Zhedong forward in opposition. The latter led the Long
March in 1934 to create a new base in Shanxi. The Japanese invasions in the 1930s distracted Chiang form
opposing Mao.
The Global Great Depression:




The Great Depression was caused by structural weaknesses in industrial economies. A price collapse occurred as
a result of cheap agricultural imports and rising European production. Recovery in the 1920s was based partly on
U.S. loans. Production from Africa and Latin America also outstripped demand, causing hardship in those areas.
Responses were local: protectionism and other measures intended to protect national economies worsened the
situation.
In 1929, the New York stock market collapsed. Bank failures in Europe followed. Agricultural investment
slowed, production fell, and then unemployment followed, reaching new highs. Although similar to earlier
depressions, the Depression lowed, reaching new highs. Although similar to earlier depressions, the Depression
of 1929-1933 was more intense and of longer duration. Social disruptions included suicides and shantytowns.
Massive unemployment led to voluble criticism of governments. The Depression also provoked disenchantment
with the optimism of the postwar period. Depression also provoked disenchantment with the optimism of the
postwar period. Depression in the West spread to the rest of the world.
National responses to the Depression fed existing political and social problems. Parliamentary systems were
challenged everywhere, either becoming ineffective or being eliminated. In France, new political parties
emerged: socialist, communist, and the Popular Front. Deep divisions led to stagnation. In some countries, such
as Sweden, governments grew to resemble modern welfare states.
Elected president of the United States in 1933, Franklin Roosevelt presented the country with his New Deal. The
Social Security system offered protection in unemployment and old age, while the government took a larger role
158
in stimulating industry and regulating banking. While the New Deal did not end the Depression, it did promote
faith in the government, sidestepping the problems of paralysis and revolt that beset so many countries.
The Authoritarian Response:












In Germany, the Depression brought to power a fascist government. A result of the Great War, fascism offered a
different response than the discredited liberal program. The German National Socialist, or Nazi, Party made
fascism a major international force, stopping the spread of liberal democracies. Adolph Hitler promised a return
to traditional values, ridding Germany of Jewish influence, and solving Germany’s economic problems. Through
agreements with German leaders, Hitler then established a totalitarian state. He used the Gestapo to implement
control over every facet of life. Targeting Jews as the cause for most of Germany’s problems, after 1940 Hitler
aimed to eliminate all Jews form Germany in the Holocaust. Behind all of these goals lay intensive military
preparations.
Hitler’s success in Germany led to fascist movement in Hungary, Romania, Austria, and Spain. Mussolini was
emboldened, attacking Ethiopia in 1935. The League of Nations took no action, and the Italians took over the
country. In Spain, the advent of fascism led to the Spanish Civil War in 1936. Francisco Franco was backed by
the fascist Falange against forces supporting the Republic after three years of fighting, Franco won in 1939.
Liberalism in Latin America was foundering by the 1930s. Traditional social divisions were little changed.
Intellectuals, writers, and artists looked to Latin American solutions for Latin American problems. A reform
movement spread from Argentina to the rest of the continent. Socialist and communist movements arose.
The Great Depression had its impact on Latin America. Corporatism, echoing some of the ideals of fascism,
took hold. President Lazaro Cardenas of Mexico began thorough land reform, winning broad support.
In Brazil Getulio Vargas was elected president in 1929. His Estado Novo took Mussolini’s Italy as its model.
Joining the Western powers in World War II, Brazil benefited economically. Vargas’s suicide in 1954 ironically
ensured his policies would dominate subsequent regimes.
In Argentina, in 1929, an attempt to overthrow the Radical Party regime failed. Federations of workers emerged
as industrialization progressed. The military backed conservative governments in the 1930s, until in 1943 a
military government took power. Juan D. Peron was one of many military nationalist leaders. With the support
of his wife, Eva Duarte, he gained popular support, especially after failed U.S. attempts to discredit him. Peron
nationalized the railways, telephone systems, and the petroleum industry. In spite of broad support, his coalition
fell apart. He was forced into exile by the military, returning briefly in 1973. His death the next year opened the
door to military dictatorship.
The Depression had a deep impact on Japan, creating political schisms. In 1932, the military took control of the
government. War with China broke out in 1937 and led to Japanese control of Manchuria, Korea, and Taiwan.
Control turned to brutal oppression, particularly in Korea.
Political developments in Japan eased the effects of the Depression. Industrialization resumed in the 1930s, at an
accelerating pace. To boost loyalty, large companies awarded lifetime employment contracts to some.
The Soviet Union had been somewhat immune to the Depression. Stalin continued his program of
industrialization. Borrowing technology from the West, he nevertheless maintained government control of
production.
Collectivization – the establishment of state-run farms – began in 1928. It was a means of control as well as of
improving production. Although peasants in general welcomed reform, the kulaks did not. Failing to cooperate,
millions were killed or deported to Siberia. After intense disruption and famine in the move to collectivization,
the system did work. In the industrial sector, Stalin’s five-year-plans were very successful. Unlike the West,
industrialization in the Soviet Union concentrated on heavy industry. Strict distribution of resources was used to
produce remarkable results.
As in the West, industrialization led to overcrowded cities, but with the difference that welfare systems were in
place. In spite of strict control of all levels of production, worker’ issues gained more attention early on than they
had in the West.
Under Stalinism, the arts were carefully managed. Socialist realism celebrated the progress and camaraderie of
the socialist experiment. Stalin’s methods included use of the secret police, and purges of possible opponents.
The Politburo became just a rubber stamp for Stalin’s policies. Isolation gave way in the 1920s to some
international diplomacy. Hitler’s rise was a threat to Russia, especially given his disdain for the Slavic peoples.
An agreement with Hitler in 1939 gave the Soviet Union time to arm itself.
159
Key Terms:
Russian Civil War
Mexican Revolution
Spanish Civil War
Appeasement
Great Depression
The Roaring Twenties
Collectivization
Pancho Villa
Mao Zedong
Benito Mussolini
Chapter 29, Quiz Questions
1) The mid-1920s in western Europe could best be described as a period of
A) war and destruction.
B) stability and optimism.
C) depression and unemployment.
D) internal political unanimity.
E) international tensions building between East and West.
2) Which of the following statements concerning the economy of Europe during the "Roaring '20s" is NOT accurate?
A) The worst inflationary pressures were resolved, though at the cost of wiping out the value of savings for many
propertied groups.
B) Industrial production boomed.
C) Mass consumption standards, based on ready availability of new products, rose.
D) The United States lagged behind the European nations in both industrial production and mass consumption.
E) New artistic movements began in Europe and spread across the Western world.
3) Which of the following statements most accurately summarizes the state of Mexican political organization after the
revolution?
A) A multitude of political parties so confused the electoral scene that no meaningful political compromise could be
achieved.
B) Mexico was immediately dominated by labor parties who originated in the northern parts of Mexico.
C) The revolutionary leadership institutionalized the regime by creating a one-party system under PRI.
D) Presidents ruled much as the caudillos before them without limitation of powers or term of office.
E) Two major parties developed, creating a bi-party system between left and right.
4) What was the primary goal of Zapata's forces within the Mexican Revolution?
A) The presidency for Zapata
B) Industrialization
C) Extension of the plantation economy in Oaxaca
D) Sweeping land reform
E) Creating a Latin American regional federation
5) In the eight months following the beginning of the Russian Revolution, what term could best be used to describe the
revolutionary government?
A) Liberal
B) Anarchist
C) Communist
D) Conservative
E) Reactionary
6) Which of the following was NOT a cause of the failure of the initial revolutionary government?
A) It did not wish to establish parliamentary government.
B) It wanted to continue Russian participation in World War I.
C) The leaders hesitated to enact massive land reforms.
D) Liberalism was not deeply rooted in Russia due to the lack of a substantial middle class.
E) The alliance with Britain and France
7) The initial goal of the May Fourth movement was
160
A) Communist insurrection.
B) ridding China of all Western influences.
C) the creation of a liberal democracy along Western lines.
D) Chinese subjection to Japan.
E) ending the practice of Confucianism.
8) What made Chinese thinkers aware of a possible Marxist solution to China's ills?
A) The Russian revolution of 1905
B) The Russian revolution of 1917
C) Frederick Engels's visit to China in 1912
D) The Japanese example
E) The revolutions of 1848
9) How did early Chinese Marxist philosophy differ from Lenin's?
A) Chinese philosophers emphasized the role of the proletariat in the revolution.
B) Chinese thinkers stressed the gradualist approach to political change.
C) Sun Yat-sen taught that the revolution could only occur after the complete industrialization of China.
D) The study circle at the University of Beijing saw the peasants as the vanguard of revolution.
E) The dictatorship of the proletariat would last forever.
10) Early Marxist philosophers in China characterized all of China's society as
A) proletarian.
B) peasant.
C) bourgeois.
D) Westernized.
E) capitalist.
11) The Nationalist Party's greatest failure was its inability to
A) create a military wing of the party.
B) deal with the social and economic crisis of the Chinese peasantry.
C) industrialize China.
D) rid itself of its ties to the Qing dynasty.
E) forge an alliance with Japan.
12) What impact did the Nationalist smashing of the workers' movement have on the development of the Communist
Party in China?
A) It destroyed the early communist movement, leaving only small cells of intellectuals and students.
B) The slaughter of the workers in Shanghai had little impact, because the great concentration of workers was in Beijing.
C) It allowed Mao's program of revolutionary activity among the peasants to come to the forefront.
D) It forced the communists to become increasingly dependent on the Soviet Union.
E) Mao and his followers had to flee to the Soviet Union and regroup.
13) Which of the following statements is most correct?
A) The intensity of the Great Depression had similar precedence in the history of industrial society.
B) The intensity of the Great Depression had no similar precedence in the history of industrial society.
C) The Great Depression's duration was typical of most economic failures.
D) Most world governments saw the Depression coming, but were unable to affect it.
E) The effects of the Great Depression were hardest in the U.S.S.R.
14) Which of the following was NOT a factor leading to the development of fascism in Germany?
A) The impact of the depression
B) Treaty arrangements that forced Germany to accept the blame for World War I
C) The recent and shaky tradition of parliamentary government
D) The division of Germany into zones of occupation by the victorious Allies in World War I
E) The rise of leaders such as Hitler
161
15) Which of the following was NOT part of Adolf Hitler's political program?
A) The need for unity and the hopeless division of parliamentary politics
B) A return to more traditional ways
C) Alliances with socialist parties in Germany
D) A glorious foreign policy designed to undo the disgrace of Versailles
E) Creation of a secret police known as the Gestapo
16) In Germany under Hitler, government economic planning
A) did not exist.
B) was focused on a return to a pre-industrial system.
C) helped to restore production levels by emphasizing armament construction.
D) was modeled on the programs of the American New Deal.
E) was based on the economic theory of John Maynard Keynes.
17) The type of government that Hitler established in Germany can best be described as
A) anarchic.
B) totalitarian.
C) communist.
D) democratic.
E) liberal.
18) What caused the downfall of the first Peronist government in 1955?
A) The death of Peron
B) Peron's divorce of Evita
C) The coalition of labor, business, and the military fell apart.
D) A peasant revolution
E) A war between Argentina and Brazil, resulting in the defeat of Argentina
19) What type of government did Getelio Vargas institute in Brazil between 1929 and 1945?
A) Vargas restored the Brazilian monarchy.
B) Vargas introduced a centralized program based on ideas from Mussolini's Italy in 1937 before turning to the left by
1945.
C) Vargas introduced a socialist and totalitarian government in 1929 based on the model of Stalin's Russia.
D) Vargas was dedicated to the principles of liberal democracy and the retention of a vibrant and open political arena.
E) Vargas's program was a conservative reaction to socialist and communist movements.
20) Which of the following statements concerning the depression in Japan is most accurate?
A) Due to government controls, the depression never affected the Japanese.
B) The Japanese government failed to take any direct action to modify the impact of the depression.
C) After initial great misery, Japan suffered far less than many Western nations because of effective government action.
D) The total concentration of the government on military aggression led to an ineffective response to the depression.
E) Western nations such as Britain and the United States began a massive investment program in Japan.
21) Which of the following statements best describes the Japanese government during the 1920s and early 1930s?
A) Japan was ruled exclusively be a strong liberal party that dominated the lower house of parliament.
B) Japanese politics were fully democratic, leading to the growth of communism in Japan.
C) Japan's oligarchic political structure allowed elite groups to negotiate with each other for appropriate policy and
allowed military leaders to take a growing role.
D) Labor unions began to exert increasing control over economic policy in the aftermath of the government's failure to
take direct action during the depression.
E) The rise of the Communist Party in Japan led to a major reorganization of the government.
22) Which of the following was NOT typical of the Soviet economy?
A) Control of virtually all economic sectors
162
B) Emphasis on heavy industrial goods
C) Need to create a massive armaments industry
D) Ready availability of consumer goods
E) Emphasis on rigid planning
Essay Questions:
The World Between the Wars: Revolutions, Depression, and Authoritarian Response
1. Discuss the transition in revolutionary Russia from liberalism to communism.
2. What are the characteristics of 20th century revolutions?
3. In what ways did Stalinism alter the original concepts of Soviet economy and government?
4. In what ways did the cultural policies of the Stalin regime depart from traditionally Russian practices? In what
ways did it emphasize them?
163
Chapter 30, A Second Global Conflict and the End of the European World Order
Summary:
In contrast to the disorganized beginning of World War I, Word War II was provoked by deliberate aggressions of
Germany, Japan, and Italy. The failures of the Western policy of appeasement encouraged the Axis Powers’ militaristic
expansions. The most deadly conflict in history, World War II, resulted in the rise of the United States and the Soviet
Union to world preeminence and competition. Western European hegemony came to an end as independence movements
in Africa and Asia succeeded in the decades after the war.
Key Concepts:
Old and New Causes of a Second World War:

Chiang Kai-shek’s leadership of the Guomindang led the Nationalists to power in southern China, and they then
moved north. Japan was fearful of renewed Chinese control of Manchuria and invaded, eventually creating the
independent Manchukuo. In Germany, the Weimar Republic had been hard-hit by the Depression. Hitler
promised to end economic hardship and stop the advance of Communism. Both Germany and Italy under
Mussolini rearmed and took part in the Spanish Civil War. The conflict prepared Germany and the other nations
that ook part for World War II. Under Franco’s dictatorship, Spain withdrew from European affairs.
Unchecked Aggression and the Coming of War in Europe and the Pacific:

World War II began officially on September 1, 1939, but conflicts began much earlier in Asia. Europeans and
their leaders hoped to avoid a major war by pacifying Hitler. Some, including Winston Churchill, warned against
this policy. The Japanese, from their new base in Manchukuo, attacked China in 1937. After capturing Shanghai
and Canton, they also took Nanjing and slaughtered its citizens. The Guomindang moved into the interior. In
Europe, Hitler and Stalin signed a nonaggression pact in 1939, and then divided Poland. Hitler’s plans were now
clear, and Britain and France declared war.
The Conduct of a Second Global War:





Ally delays permitted Axis victories in the early phase of the war, but when Hitler turned to Russia, victory
eluded him.
The German strategy of blitzkrieg – lightning war – was highly successful. Poland was taken in 1939 and much
of France by 1940. France had been divided politically and had not prepared for war. Only the south was
semiautonomous under the Vichy regime. Germany failed in its massive assault on Winston Churchill’s Britain,
the Battle of Britain. Yet the Germans controlled much of Europe and the Mediterranean by the middle of 1941.
Erwin Rommel led German troops victoriously across north Africa, adding to the resources available to the
Germans. Hitler moved east and then on to Russia, but met Napoleon’s fate. Again, in 1942-1943, an assault on
Russia failed, destroying the German army. As the Germans retreated, the Russians retook areas of Eastern
Europe.
German attacks on the Jews and others deemed deleterious to the nation had begun in 1940. In 1942, Hitler
undertook the complete eradication of Jews and other undesirables. The Holocaust claimed as many as 12
million lives, at least half of which were Jews. The Allies failed to take action against the Holocaust.
The Battle of Britain absorbed most of the British war effort for almost two years. The United States joined the
war after the Japanese attack on Pearl Harbor. Britain and the United States joined forces against Rommel in
North Africa and then moved into Italy. Mussolini was captured and killed. Anglo-American forces then
attacked Germany in northern Europe, via Normandy. The Battle of the Bulge, 1944-1945, led the Allies into
Germany. Adolph Hitler committed suicide in 1945.
Following the attack on Pearl Harbor in 1941, the Japanese took British possessions in China, then Malaya,
Burma, the Dutch East Indies, and the Philippines. They were pushed out again by the British and fierce local
resistance, but U.S. forces played the largest part in the fighting. The Pacific theater centered on strategic islands.
In the Battle of the Coral Sea, the Japanese were halted and a month later defeated on Midway Island. Nearing
Japan, General Curtis LeMay ordered the bombing of the country in March, 1945. The United States then went
164
further that summer, dropping atomic bombs on Hiroshima and Nagasaki. The Japanese surrendered
unconditionally.
War’s End and the Emergence of the Superpower Standoff:


The peace treaties ending World War II lacked the scope of the Versailles Peace. The United Nations was
established, to be based in New York City. Control over world affairs was no longer to be monopolized by
Western powers. Although the primary mandate of the U.N. was to facilitate diplomacy, more specialized
branches were subsequently created.
The Cold War, which was to last four decades, resulted from a stalemate in the peace settlement. The Tehran
Conference, in 1944, allowed the Soviet Union to control portions of Eastern Europe, in the face of U.S.
objections. The Yalta conference the next year confirmed the U.N. and divided Europe into four occupation
zones. A meeting at Potsdam, the same year, allowed the Soviet Union to keep Poland. Austria was occupied by
the United States and the Soviet Union, and the two powers divided Korea. In the Middle East, Africa, India, and
Asia, much of the old colonial territory was reestablished. Two themes emerged. The first was decolonization;
the second was the Cold War.
Nationalism and Decolonization:





Japanese defeat of the Western powers in Asia added to a growing sense that victory over the colonial rulers was
possible. Total war had exhausted Europe, which was surpassed in global influence by the United States and the
Soviet Union. The Atlantic Charter of 1941, negotiated by Roosevelt and Churchill, included self-determination
for all.
A British representative, Sir Stafford Cripps, was sent India in 1942 to try to negotiate with the Indian National
Congress. The Quit India Movement began that year, making debate impossible. The British attempted
suppression. The Muslim League, led by Muhammad Ali Jinnah, was more willing to work with Britain. The
Labor government that came to power in Britain after 1945 decided to work with India to achieve independence.
Jinnah was persuasive in calling for a separate Muslim state. In 1947, the British handed control of the
subcontinent to the Congress Party in India and to Jinnah, first president of Pakistan. Sectarian violence followed
the partition. Gandhi was assassinated in 1948. Burma (Myanmar) and Ceylon (Sri Lanka) gained their
independence soon after. Other Asian empires also dissolved. The Philippines and Indonesia won their
independence.
During World War II, many African recruits fought for the Allies, but they gained nothing by their loyalty.
Industrialization to aid the war effort reversed European policies in Africa, and urbanization followed. Kwame
Nkrumah is an example of a leader that took the radical path to independence. Returning to the Gold Coast, he
formed the Convention People’s Party. Standing firm against British threats, he gained a large following and
was recognized as prime minister of Ghana in 1957. In other areas, independence came with few confrontations.
Leopold Sedar Senghor led Senegal peacefully to independence from France. Belgium retreated hastily from the
Congo. By the mid-1960s, decolonization was achieved in all but the settler states.
In the settler colonies, large numbers of Europeans blocked indigenous nationalist and independence movements.
European settlers opposed both the African majority and European administrators’ pushes for change. African
leaders, thus stymied, often turned to violence. In Kenya, Jomo Kenyatta and the Kenya African Union
supported radical action. The Land Freedom Army used terrorism and guerilla tactics, but imprisonment of
leaders blocked that strategy. Yet the British negotiated with nationalists, in spite of resistance from European
settlers. Kenya gained independence, with Kenyatta in charge. In Algeria, the independence movement gathered
around the National Liberation Front. As independence movement gathered around the National Liberation
Front. As in Kenya, although defeated, that Algerians gained freedom through negotiation. However, French
settlers formed the Secret Army Organization (OAS), which was responsible for ending France’s Fourth
Republic. A brief war ended with Algerian independence in 1962.
Angola, Mozambique, and Zimbabwe won independence by violent means. Only in South Africa did a white
majority retain control against a black majority. Afrikaners, distanced from their original home, felt themselves to
be native and, moreover, were buttressed by convictions of their racial superiority. The Afrikaner National
Party created apartheid, through a mass of legislation. Black Africans were denied equality with white
Afrikaners.
165

In the Middle East, many countries had feed themselves of European governance, if not influence. Palestine was
a point of contention. Muslim rebellions, in 1936-1939, convinced Britain to slow the movement of Jews into the
nascent Israel. A Zionist military force, the Haganah, was created. At the end of World War II, a stalemate
existed. In 1948, the U.N. approved the partition of Palestine. Israel defended itself effectively and gained some
territory.
Key Terms:
Final Solution
Vichy
Manchuria
Nazi-Socialism
Hiroshima & Nagasaki
United Nations
Yalta Conference
Muslim League
Potsdam
Cold War
Chapter 30, Quiz Questions
1) Hitler came to power in Germany
A) as a result of entirely legal and constitutional means.
B) with the support of socialists.
C) after a short, but violent, overthrow of the constitutional government.
D) after a lengthy civil war between forces of conservatives and communists.
E) with the support of the upper and lower classes but financed by the Soviets.
2) Japan's surrender in the Pacific was precipitated by
A) the use of atomic weapons on the cities of Nagasaki and Hiroshima by the U.S.
B) a massive land and sea assault on the Japanese home islands.
C) the loss of the Philippines to the U.S.
D) the British advance through Malaya into China.
E) the loss of China to combined British and American forces.
3) Which of the following was NOT a result of the peace treaties signed following World War II?
A) The United States occupied Japan.
B) Germany was divided into four zones of occupation.
C) The Soviet Union took much of eastern Poland, while the Poles were compensated by receiving part of eastern
Germany.
D) German industrial power was destroyed.
E) Austria was divided and occupied.
4) Which of the following statements concerning Zionism following World War II is most accurate?
A) Zionists turned to violent attempts to eject the British from Palestine in response to the British attempts to limit
immigration to the Middle East.
B) The Zionist movement turned to peaceful demonstrations and boycotts on the model of the Indian nationalist
movement and refused to participate in violence.
C) The Zionist movement, frustrated by the failure to achieve an independent nation, weakened after World War II.
D) The Zionist movement was eliminated after World War II by the combined action of the Palestinian Arabs and the
British.
E) The Zionists reached out to the Soviet Union and other communist nations for support.
5) Which of the following statements concerning the Algerian independence movement is most accurate?
A) Algeria won its independence from France in a peaceful movement led by white settlers in the colony.
B) Decolonization in Algeria was violent, as white settlers resisted independence through the OAS supported by powerful
elements within the French military.
C) Independence in Algeria was achieved as a result of the military victory of the FLN over the French army.
D) Unlike the rest of Africa, Algeria was never decolonized and remained a province of France.
E) Algeria became an example of a Cold War conflict where both the United States and the Soviet Union sent in troops
and aid.
166
6) What was the solution to the division in India between Muslims and Hindus in 1947?
A) The British established a single government with a Hindu majority, but with specific offices reserved for Muslims.
B) The government of India was divided between two houses of the Indian parliament, one for Muslims, one for Hindus.
C) The British simply withdrew from India without any political settlement of the problem of religious division.
D) The British decided to divide India into two nations, a Muslim Pakistan and a secular, but Hindu-dominated India.
E) A civil war broke out between Hindus and Muslims but was settled quickly with the aid of Gandhi.
7) How did the Indian Congress Party and nationalist leaders respond to British participation in World War II?
A) As in World War I, the Congress Party and nationalist leaders such as Gandhi led popular rallies in favor of the British
war effort.
B) Nationalist leaders of all parties opposed the war effort.
C) Congress opposed the war effort and its leaders were jailed, but the Muslim League rallied to the British cause.
D) The Muslim League and the Communists opposed the British war effort as a means of establishing independence.
E) At first, both parties opposed the war, but after the Battle of Britain both parties agreed to support the war.
Essay Questions:
A Second Global Conflict and the End of the European World Order
1. Define “total war.” How did the world wars of the 20th century demonstrate the application of “total war”?
2. In what ways did the settlement of World War II repudiate the Versailles treaties that ended World War I? In
what ways did the settlement affirm the concepts included in the Versailles treaties?
167
Chapter 31, Western Society and Eastern Europe in the Decades of the Cold War
Summary:
Both western and eastern Europe were devastated by World War II, yet the U.S.S.R. soon emerged as a superpower
rivaling the U.S. Eastern Europe was dominated by the Soviets for 45 years after the war, and Western Europe generally
followed the U.S. model. Only the West, however, showed strong economic recovery in the years following the war. A
consumer culture arose, women reached new heights of equality, and democracy was firmly established. In Eastern
Europe, advances in industrial capability were balanced by repression from the Communist system.
Key Concepts:
After Word War II: A New International Setting for the West:



Europe’s infrastructure, its economy, its people were devastated by World War II, to the point that survival itself
was in doubt for the first years following the wars.
The forces pushing toward decolonization became apparent soon after the war. Although violent, costly struggles
resulted in some areas, decolonization was generally smooth from the 1950s through the 1970s. Western powers
sometimes maintained positive relations with their former colonies. Yet the process also returned waves of
embittered colonists to their home countries.
The Cold War, between the United States and the Soviet Union, was one of the most important factors in the
postwar world. The Soviet Union created an eastern bloc, including Poland, Czechoslovakia, Bulgaria, Romania,
and Hungary. The United States, led by Harry Truman, was more antagonistic to the Soviet Union than were
European powers. Winston Churchill called the division between the two spheres the iron curtain. The U.S.
Marshall Plan, providing aid to Europe, was in part a means of resisting communism, In the immediate postwar
period, Germany was divided into East and West Germany. The North Atlantic Treaty Organization (NATO),
bringing together the North American and European powers, was matched by the Soviet-led Warsaw Pact
countries. The onset of the Cold War meant increasing U.S. intervention in Europe. However, the U.S. and the
Soviet Union were soon engaged in other world areas. U.S. militarization was one result of the Cold War, while
European powers devoted less of their budgets to arming.
The Resurgence of Western Europe:





Following the war, European leaders were greatly influenced by their wartime experiences. Military defeat
discredited fascism, and Europe moved to the left, supporting democracy and welfare institutions. Political
reconstruction in Germany was initially slow, and was then influenced by the cold War. Italy and Germany both
drew up new constitutions.
The welfare state grew out of the postwar need for reconstruction. In the United States, welfare programs began
with Depression-era New Deal. Typically, states passed unemployment insurance, public health measures, family
assistance, and housing aid. Governments relied on so-called technocrats, who were skilled in fields that Europe
required for rebuilding.
Student protests were common in Western countries in the 1960s. Material culture and social inequalities were
common targets. In the 1970s, the Green movement became a significant political force. Recession was also
widespread in the 1970s, reversing the trend toward larger governments.
During and after World War II, many Europeans desired greater harmony among their nations. By 1958, six
European powers had created the European Economic Community, later called the European Union. Initially
motivated by economic goals, as the union grew, it also added a parliament and judiciary. Europe gained a
mechanism for ensuring general peace.
Substantial economic growth in postwar Europe was helped by agricultural improvements and a shift to
production of consumer products. Steady growth occurred in the service industries. Immigration fed the need for
labor. Material wealth and spending on leisure and luxuries increased substantially.
Cold War Allies: The United States, Canada, Australia, and New Zealand:
168



While the North American allies Australia and New Zealand did not experience the burst of growth that occurred
in Europe after the war, growth did occur.
Canada followed its own path of development, but continued economic cooperation with the United States. Asian
immigrants changed Canada’s makeup, and French Canadians pushed for autonomy. Australia and New Zealand
shifted their alignment away form the British sphere to one dominated by the United States. Australia traded
increasingly with Japan.
In 1947, Harry Truman declared support for those resisting oppression. In part, this meant resistance to
communism. A number of U.S. agencies, including the Central Intelligence Agency, the Strategic Air command,
and the military, were important tools that supported the Truman doctrine. The United States invaded North
Korea in 1950, after the North Korean communist government had launched a surprise attack on South Korea.
Dwight Eisenhower’s presidency focused on containment of the Soviet Union. Under George Bush, and to a
lesser extent Bill Clinton, the United States continued to lead military actions overseas.
Culture and Society in the West:




Social conflicts in Europe were eased by greater prosperity, though class and race divisions did not disappear.
The greatest social changes in the postwar West involved women. Many women employed during the war
continued to work after the peace. By the 1920s, women comprised up to 44 percent of the workforce. Yet
women’s pay was often lower than men’s, and women were most frequently employed in the clerical positions.
Women won the vote, and increasingly attended universities. Advocates for women’s reproductive rights were
often successful. These changes are partially responsible for declining birthrates and for children starting school
earlier. Divorce became a common phenomenon. The new feminism, voiced by Simone de Beauvoir and Betty
Friedan, attempted to redefined women’s roles. The movement was successful in bringing forward new political
issues.
Despite profound changes in Western society, cultural development often followed well-established lines. The
United States was a growing power in intellectual life, as the country drew scholars from many areas. Important
European scientific research continued, with such work as Francis Crick’s study of DNA. In the arts, styles that
had been shocking and ultramodern in the 1920s became familiar in the 1950s. “Pop” art used new media to
bring art and popular culture together. Europeans generally took the lead in film. In the 1960s, Godard,
Antonioni, and Bergman further developed the art of filmmaking. It is difficult to generalize about the social
sciences, with the possible exception of a tendency to collect large databases of information for study.
European popular culture was heavily influenced by the United States. In particular, U.S. television series gained
large audiences in Europe. As in the United States, sexual behavior underwent a number of changes; for instance,
premarital sex became more common.
Eastern Europe After World War II: A Soviet Empire:



Soviet post-war policy included a wish to protect the country from invasion and a desire to maintain its position as
a world power. Pacific islands taken from Japan late in the war and influence in North Korea and Vietnam
increased the Soviet sphere.
While expanding its influence in many areas, the Soviet Union first extended its influence in Eastern Europe. The
many young nations of the area had struggled between the wars and then had fallen to the Nazis. The Soviets
took all but Albania, Greece, and Yugoslavia by 1948. In the region, the Soviet Union exported its
collectivization program, and industrialization, while silencing opposition. The Warsaw Pact formed a separate
economic sphere. Some social and economic problems were addressed, but the Berlin Wall was erected in 1961,
making the Soviet approach to choice clear. Easing of Stalinism in 1956 led to high expectations. Poland showed
some independence, with Soviet approval. A Hungarian uprising was viciously suppressed, but overall Soviet
control did loosen. A more liberal Czechoslovakian regime was condemned by the Soviet Union. The Polish
Solidarity movement was allowed to develop, under close scrutiny. While differences continued between
countries, by the 1980s Eastern Europe had been transformed by Soviet influence. Conversely, the need to keep
east European opposition under control kept the Soviet Union preoccupied.
Propaganda was used by the soviet leaders, vilifying the United States. Control of the media, of the media, of
travel, and of the borders allowed the government to maintain control over its own people. Stalin’s organization
of the state and society, dominated by the Communist Party, continued with few changes.
169
Soviet Culture: Promoting New Beliefs and Institutions:



The Soviet government was an innovative attempt to expand the state with popular support, while promoting a
new, common, culture. Its attack on the Orthodox Church began soon after the 1917 revolution, and mainly
consisted of hampering the church’s ability to influence the young. In the area of culture, as well, the government
set its own agenda, often in opposition to Western trends. Ballet and classical music were important exceptions.
Literature developed with relative freedom, often choosing themes that celebrated the Soviet experience. The
author of the Gulag Archipelago, Aleksander Solzhenitsyn, is an example of a writer that adhered to many
Russian values, while criticizing much of the Soviet government. The sciences and social sciences continued to
hold a preeminent position, though under government control.
Industrialization in the Soviet Union, along with most of Eastern Europe, was complete by 1960. Heavy industry
was still given priority over manufacturing consumer goods. The drive to increase production had a serious,
adverse impact on the environment. Leisure activities became important, and a division between workers and
managers followed industrialization in the Soviet Union, as it had in the West. Changes in family structure,
including a falling birthrate and increasing women’s employment, also resembled Western developments.
Stalin’s death in 1953 jeopardized the system he had created. However, the system was sufficiently entrenched to
survive. Nikita Khrushchev monopolized power in 1956, and condemned Stalin’s methods. More criticism of the
state was allowed. Khrushchev brought the Soviet Union close to war with the United States when he refused to
back down in Cuba. Khrushchev’s failed scheme to open Siberia to cultivation led to his fall from power. The
1960s and 1970s were relatively stable in the Soviet Union.
Key Terms:
Solidarity
European Union
New Feminism
Berlin Wall
Civil Rights Movement
Great Society
Marshall Plan
Iron Curtain
Nato
Warsaw Pact
Chapter 31, Quiz Questions
1) Which of the following statements concerning western European nations in NATO is most accurate?
A) Increasingly western European nations withdrew from NATO and established self-sufficient military defensive
systems.
B) Western European nations rapidly lost their fear of Soviet aggression leading to weakening of NATO.
C) Western European nations rapidly expanded their military expenditures to equal and exceed those of the U.S.
D) Europeans ultimately became rather comfortable with the concept of relying on U.S. protection.
E) West Germany took the lead in establishing a new anti-Soviet coalition and withdrawing from NATO.
2) Which of the following statements concerning the development of new governments in Europe after World War II is
most accurate?
A) Except for Germany and Italy, political stability was restored through the institution of more authoritarian
governments.
B) New constitutions established in western Europe uniformly established effective parliaments with universal (including
female) suffrage.
C) Constitutions formed after World War II were noteworthy primarily for the lack of durability.
D) As late as the 1980s several nations clung to semi-fascist, authoritarian regimes ruled by strongmen.
E) A strong pan-Europe movement developed which led to the creation of a super-European state.
3) The creation of the welfare state
A) was accomplished only in the United States.
B) was carried out as part of the program of conservatism in European governments following World War II.
C) was necessitated by the recurrence of the depression following World War II.
D) resulted from the leftward shift of the political spectrum in Europe following World War II.
E) was a worldwide movement found in the industrialized West as well as in colonial areas such as Africa.
170
4) Which of the following statements concerning the European welfare state is most accurate?
A) The imposition of the welfare state was accompanied by the elimination of the private sector in most European nations.
B) Middle-class people, in general, failed to realize any benefits from the welfare state.
C) Although some aspects of the welfare state redistributed income, it did not make a huge dent on western Europe's
unequal class system.
D) All of the tax schemes introduced by the welfare state were intended to redistribute income from the wealthy to the
poor.
E) The economic status of the lower class improved substantially from the end of World War II to 1960.
5) Which of the following was NOT an effect of the welfare state?
A) It cushioned citizens against big expenses and unusual hardships, rather than rearranging the social structure.
B) It protected the purchasing power of the very poor against catastrophe and contributed to improved health conditions.
C) It increased contacts between government and citizen and produced a host of new regulations.
D) Immediately upon its creation, it generated a storm of political protest from conservative political factions.
E) It proved to be very expensive for governments to manage and operate.
6) Which of the following paralleled the development of the welfare state?
A) Increased military spending
B) Increased government role in economic policy
C) Increased political conservatism
D) Decreased government spending overall
E) Decreased spending on technology and research
7) The European Economic Community is a good example of
A) Europe's continued national strife.
B) cooperation between European nations and a willingness to create a single European economy.
C) the need for Europe to develop a single foreign policy independent of the U.S.
D) the continued economic dependence of the European nations on the capital derived from the U.S.
E) the continued economic dependence of the European nations on the influence of Marxist ideas and the power of the
Soviet Union.
8) Which of the following statements most accurately describes the European economy of the post-1950s?
A) The European economy stagnated shortly after the withdrawal of the U.S. from European affairs.
B) Overall growth in gross national product surpassed the rates of any extended period since the Industrial Revolution
began.
C) With the exception of a major depression from 1958 to 1964, the European economy grew.
D) While southern European nations, long the least developed of the continent, demonstrated remarkable growth, northern
Europe slid into economic recession.
E) Eastern Europe became an economic powerhouse while northern Europe fell into economic depression.
9) Which of the following statements most accurately describes the situation of European peasantry in the social structure
of late 20th-century Europe?
A) The European peasantry disappeared following World War II.
B) The European peasantry decreased in size and became increasingly commercialized.
C) The European peasantry continued to remain exclusively rural and rejected 20th-century urban culture.
D) Alone among European social classes, the peasantry remained independent of the state bureaucracies.
E) The European peasantry began to join the military in large numbers as a way of improving their economic status.
10) Which of the following statements concerning Western culture during the later 20th century is most accurate?
A) In the fine arts, most artists attempted a return to the classical forms of the Renaissance and the ancient world.
B) More than any other scientific field, the social sciences abandoned the traditional emphasis on consistency in human
and social behavior.
C) In the field of physics, modern scientists were unable to go beyond the startling discoveries of the 19th century.
D) Western culture in the 20th century, both in art and in science, became largely relative rather than objective.
E) Romanticism was re-introduced with much success due to the increasing alienation among the intellectual class.
171
11) Which of the following statements concerning the Orthodox church under Stalin's regime is most accurate?
A) The regime appointed the Orthodox church the only approved religion within the state.
B) The Orthodox church became the primary instrument of the regime's policy of education.
C) The Orthodox church ceased to exist during the Stalinist regime.
D) Loyalties to orthodoxy persisted, but they were concentrated in a largely elderly minority.
E) The Orthodox church continued to endorse the political policies of the Soviet regime.
12) Which of the following was NOT a source of pressure on the Soviet family?
A) Religious constraints imposed by the Orthodox church
B) Crowded housing
C) Loss of ties to extended family life of the countryside
D) Mothers who worked long hours away from the home
E) Increased use of alcohol
13) In what way did the social organization of the industrialized Soviet Union come to resemble that of the West?
A) Emphasis on service industries and managerial hierarchies
B) The division of urban society between workers and a managerial middle class
C) The absence of a true proletariat
D) Lack of a peasantry
E) The use of the free market as an economic mechanism
14) Women in Russian industrialized society
A) rapidly reached the same status as males.
B) were less likely to be in the work force than women in the West.
C) dominated some professions, such as medicine.
D) were afforded the same type of domestic idealization typical of women in the West.
E) played a major role as political leaders.
15) Which of the following statements concerning the Soviet military following Stalin's death is most accurate?
A) Overall the Soviet Union played a cautious diplomatic game, almost never engaging in warfare but maintaining a high
level of preparedness.
B) The continued growth of the Soviet military led to an increasing willingness on the part of Russian rulers to engage in
direct military intervention around the globe.
C) Following Stalin's death, subsequent Soviet rulers enacted a policy of progressive demilitarization.
D) Following World War II, the Soviet military played no significant role in subsequent Russian history.
E) The Soviet military attempted to create a containment system when it came to countering American military moves.
Essay Questions:
Western Society and Eastern Europe in the Decades of the Cold War
1. Describe some of the societal and political changes of the 1970s.
2. In what ways did Stalinism alter the original concepts of Soviet economy and government?
3. In what ways did the cultural policies of the Stalin regime depart from traditional Russian practices? In what
ways did it emphasize them?
4. How did the failures of the Soviet economy lead, in part, to the changes in Soviet policy after 1985?
172
Chapter 32, Latin America: Revolution and Reaction into the 21st Century
Summary:
In Latin America, much of the 20th century witnesses a struggle between the forces of revolution and reaction. The focus
of this chapter and the next is on third world nations, which display great diversity and cultural emphasis. In the second
half of the 20th century, Latin America took an intermediate position between the nations of the North Atlantic and those
of Africa and Asia. Investments often came form the West, and Latin America was vulnerable to the world financial
system. Throughout the 20th century, it grappled with issues of social justice, cultural autonomy, and economic security.
Workers’ organizations emerged as a political force. Explosive urban growth and emigration were often key concerns.
Overall, the economy and politics were subject to broad shifts. Although much of Latin America was subject to the
rhetoric of social and political change, remarkable little change actually occurred. At the same time, significant
transformations took place in education, social services, women’s rights, and the role of industry.
Key Concepts:
Latin America After World War II:


Following World War II, authoritarian rulers held power in several Latin American countries, including Peron in
Argentina and Vargas in Brazil. Dissent was often countered by oppressive measures.
The PRI controlled Mexico until 2000. The Zapatistas emerged in 1994, calling to mind Emiliano Zapata’s
unfulfilled movement. Mexico joined the North American Free Trade Union (NAFTA), hoping to boost its
economy, leader of the National Action Party (PAN), ended decades of PRI domination.
Radical Options in the 1950’s:



Unsatisfied desire for reform built up in many countries, including Venezuela and Costa Rica, where elections
brought reformers to power. In 1952, a revolution broke out in Bolivia, conservative forces won the day.
Guatemala, like Bolivia, had an Indian majority and an extremely inequitable distribution of resources. Juan
Jose Arevalo was elected in 1944. His program included land reform, in the face of such large foreign compnies
as the United Fruit Company. Arevalo was replaced in 1952 by Jacobo Arbenz. The United States moved to
protect United Fruit form Arbenz’s more radical program and to stop perceived communism. The U.S. Central
Intelligence Agency organized an invasion force, bringing in a pro-U.S. regime. Guatemala’s problems
continued, and a guerilla movement emerged.
Unlike Guatemala and Bolivia, Cuba had a population mostly descended from European colonists and African
slaves. By the 1950s, Cuba was firmly in the U.S. sphere of influence. Fulgencio Batista ruled Cuba from 19341944. Undertaking reforms, his regime moved close to dictatorship. In 1953, Fidel Castro launched an
unsuccessful attack on the Cuban military. Fleeing to Mexico, Castro joined Ernesto “Che” Guevara, and the
two raised troops and invaded Cuba in 1956. By 1958, they were in control, and Castro’s movement had become
more radical. U.S. opposition pushed Castrol into the Soviet camp. The United States face-off with Cuba became
part of the Cold War. Castro’s reforms were sweeping and particularly successful in the area of social welfare.
The Cuban economy, on the other hand, required Soviet support. Reform in Cuba, and the island’s resistance to
U.S. pressure, made it an attractive model for other Latin American countries.
The Search for Reform and the Military Option:


In Latin America, revolution continued to be a likely option to resolve persistent problems. Another option, taken
by Mexico, was the stability of one-party rule. Christian Democrats, especially in Chile and Venezuela, offered
the support of the church in seeking social reform and protection of human rights. Liberation theology,
combining Catholic theology and socialism or Marxism, was another popular solution.
Military involvement in Latin American politics was a long-standing tradition. Often acting in reaction to the
threat of reform, including communist programs, Military groups took action in Brazil and Argentina. In Chile,
the socialist President Salvador Allende was removed by the military. Military regimes aimed to impose neutral
regimes that would stabilize their economies. Brutality was used when it was thought necessary. Economic
173


growth was achieved in some cases, but at the cost of more equitable resource distribution. Military regimes
varied, and in some cases sought popular support and social and land reforms.
By the middle of the 1980s, military regimes were giving way to civil governments. Elections were held in
Argentina in 1983. El Salvador and Guatemala were returned to civilian rule in 1992 and 1996. Inflation, debt,
and the drug traffic plagued Latin American economies. While democracy spread, some countries took more
radical paths. Hugo Chavez’s regime in Venezuela gained a following in other countries.
The United States cast a long shadow over Latin American developments throughout the 20th century. U.S.
influence included private investment, outright invasion, and sometimes both. More than 30 U.S. military actions
occurred in Latin America before 1933. Nicaragua’s Augusto Sandino led armed resistance against U.S. trained
forces, inspiring the Sandinista movement. U.S. intervention led to the establishment and then control of so called
banana republics. Widespread hostilitiy to U.S. interference was voiced by Pablo Neruda. Franklin D.
Roosevelt’s Good Neighbor Policy was a brief change to more equitable relations. The U.S. Alliance for
Progress provided over $10 billion to help economic development in Latin America. The 1970s and 1980s were
typified by U.S. involvement in Latin America on an ad hoc basis to protect U.S. interests. Financial support
from the United States to Latin America is largely for military spending.
Societies in Search of Change:




Important social changes occurred in Latin America in the 20th century, in spite of disappointments in attempts to
bring about larger social reforms.
Women’s roles changed slowly. Ecuador, Brazil, and Cuba granted women the right to vote by 1932. Feminist
and suffrage movements became more active. In some countries, women gained the vote, only to join parties that
denied them further rights. Entering the workforce in large numbers in the early decades of the 20th century,
women still lagged behind men in pay. Women in Peru, Ecuador, and Bolivia are influential in small-scale
commerce and have become a political force. Concerning the position of women, by the mid-1990s, Latin
America stood between industrialized and developing nations.
Population growth was high in Latin America, and accompanied by significant population movements. Mexican
migrant labor into the United States reached 750,000 per year by 1970, primarily coming form Mexico. The
figure is about 5 million for movement within the continent. Industrialization, political repression, and instability
have contributed to the phenomenon. Moreover, movement form rural to urban areas has created large urban
areas surrounded by shantytowns. Unlike rural workers moving into towns during European industrialization, the
new arrivals often failed to make it into the ranks of industrial workers a divided urban population has resulted.
Latin America remains overwhelmingly Catholic. Popular culture has maintained its energy. Striving for social
justice and welfare has invigorated artistic expression. Frustrated desire for change led some writers to pursue
“magical realism.” Writers such as Gabriel Garcia Marquez combined close observation of his own culture with a
fantastic setting.
Key Terms:
NAFTA
Juan Peron
Vicente Fox
Bay of Pigs
Third World
Fidel Castro
Liberation Theology
Banana Republics
United Fruit Company
Good Neighbor Policy
Chapter 32, Quiz Questions
1) Which of the following statements concerning the revolution in Guatemala in 1954 is most accurate?
A) The communist Arbenz government that ruled Guatemala was overthrown by liberal reformers under Arevalo.
B) Communist revolutionaries overthrew the conservative military government despite active U.S. support.
C) The U.S. Central Intelligence Agency aided conservative dissidents in overthrowing the nationalistic Arbenz
government.
D) The U.S.-supported regime that replaced the Arbenz government introduced significant land reform and limited
foreign ownership of Guatemalan industry.
E) The regime of Arevalo was overthrown by a U.S.-backed group of military dissidents.
2) Which of the following statements most accurately describes the outcome of the Cuban revolution?
174
A) Despite the successful overthrow of Batista, the revolutionary government failed to enact significant reforms in the
plantation economy.
B) The revolutionary government eventually announced its adoption of Marxist-Leninist leanings, broke off relations with
the U.S., and introduced sweeping socialist reforms.
C) The largely liberal government that resulted from the revolution returned to the constitution of 1940 and closer
relationships with the U.S.
D) After a brief sojourn in the U.S., Batista was able to return to power with the support of the U.S. military.
E) Castro and the United States agreed to a cease-fire and resumed normal relations by 1970.
3) The revolutionary government of Cuba traded economic dependency on the U.S. for
A) economic autonomy with a successful program of industrialization.
B) increasing political and economic ties with Japan.
C) increasing economic dependency on the Soviet Union.
D) a significant share of the world's petroleum market.
E) increased aid from other third world nations such as Nigeria.
4) Which of the following is NOT true in regards to officers of the Latin American military?
A) They began to see themselves as above the selfish interests of political parties.
B) They began to think of themselves as the true representatives of the nation.
C) They believed in the value of hierarchy and usually kept their place in society.
D) They believed they were the best equipped to solve their nation's problems.
E) They saw themselves as reformers of society.
5) The common thread running through all of the military regimes of Latin America was
A) they were all supported by the United States.
B) they were all supported by the working populations.
C) they were all reform-minded.
D) they were all nationalistic.
E) they were all supported by the USSR.
6) Which of the following statements concerning military governments in Latin America after 1960 is most accurate?
A) Military governments tended to favor labor and the working classes at the expense of the traditional oligarchy.
B) Political repression and torture were often used to silence critics.
C) None of the military governments was successful in introducing social or economic reforms.
D) Military governments were uniformly surrogates for conservatives in Latin American society.
E) They were all supported by the United States as part of a cold war strategy.
7) Which of the following statements most accurately reflects the progress of democratization in Latin America during the
1980s?
A) Despite return to democratic government in many Latin American countries, problems with populist movements,
threats from military leaders, inflation, and the drug trade weakened the new regimes.
B) Once democratic governments were restored in much of Latin America, the influence of the U.S. in the region began to
wane.
C) The return of democracy to Latin America was so universal that military governments ceased to exist.
D) Democratic governments in the 1980s ceased to be troubled by the existence of leftist, guerrilla movements.
E) The trend led to increased resistance from many communist nations around the world.
8) What led to the U.S. return to more aggressive policies regarding Latin America including direct military intervention
following World War II?
A) The desire to contain communism and the cold war
B) The discovery of uranium in Mexico
C) The increasing intervention of Japan into Latin American economies
D) The alliance of many Latin American countries with fascist governments during the war
E) The rise of many military-operated governments in the region.
175
9) Which of the following statements concerning women in Latin American politics and society is most accurate?
A) Nowhere in Latin America did women achieve the right to vote before 1955.
B) Women tended to join the national political parties, where traditional prejudices against women in public life limited
their ability to influence political programs.
C) Women continued to be excluded from the Latin American industrialized labor force, although they played a major
role in agricultural production.
D) By the mid-1980s, Latin American women continued to hold social and political status more similar to other areas of
the third world than western Europe and North America.
E) Women were unwilling to challenge the traditions that kept them out of the political arena.
10) Which of the following statements concerning Latin American population is most accurate?
A) Between 1950 and 1985, Latin American population remained stagnant due to poor health conditions and constant
internal warfare.
B) Despite improvements, Latin America's population continued to increase more slowly than that of North America.
C) Almost all population increase in Latin America can be attributed to immigration of European laborers.
D) Since 1950, Latin American population has more than doubled, while North American population has grown more
slowly.
E) The diffusion of the Latin American people into the United States has virtually ceased.
Essay Questions:
Latin America: Revolution and Reaction into the 21st Century
1. Define populism. How have populist politics had a role in Latin America?
2. Compare and contrast the radical revolutions of the 1950’s in Cuba and Guatemala.
3. How “revolutionary” has change been in the Latin American economy and society since 1910?
4. Discuss the role of the military in Latin American political development during the late 20th century.
176
Chapter 33, Africa, the Middle East, and Asia in the Era of Independence
Summary:
Deep divisions between ethnic and religious groups remained when European rulers disappeared form their former
colonies. Economic life was hampered by concessions made to the departing colonizers and by an international economy
that favored industrialized nations. They lacked technological and management expertise, and had to face steady
population growth and environmental degradation. Social unrest occurred due to corruption and breakdowns in traditional
culture. Failure to solve the problems produced dissent and disturbances that shook existing regimes. Opponents
included political and religious revivalist groups with widely different proposed solutions. Leaders adopted differing
strategies to remain in power, but many were replaced by military officers who assumed dictatorial authority. In Iran, an
anti-Western religious movement triumphed.
Key Concepts:
The Challenges of Independence:





Nationalism continued to be a force in newly independent nations, often used by leaders against departing
Europeans. Yet when the latter did leave, improvements were not as great as many expected. Distribution of
goods already in short supply often led to difficulties. Struggles for independence had often brought about unity,
which could disappear when the foreign regimes departed. When artificial boundaries established between rival
peoples disappeared, conflict often broke out. Bangladesh established its independence following years of
conflict arising from the partition of India. The work of just keeping countries together absorbed a great deal of
energy.
Rapidly growing populations were a problem in all of the developing countries. New crops, especially those from
the New World, led to population growth, as did better infrastructures under colonial rule. Moreover, since the
early 20th century, health care has added to population growth. The problem has been most obvious in Africa,
where population growth rates have been extremely high, in spite of the AIDS epidemic. Developing countries,
behind in industrialization, had trouble feeding or employing their growing populations. Cultural attitudes have
prevented birth control form becoming popular, in particular in areas where religion or society requires sons.
Infant mortality has also dropped.
The move from rural to urban areas that occurred in Europe in the 18th ad 19th centuries occurred also in
developing nations, with the difference that industrialization is generally absent, leaving newcomers unemployed
and destitute. Urban poor are often a volatile political force. Urban sprawl includes large unplanned
shantytowns. Overpopulation in rural areas has had a profound environmental impact.
Women’s suffrage was often incorporated into new constitutions, although women’s roles have often expanded
only slowly. The example of such powerful women as Indira Gandhi – daughter of Jawharlal Nehru – or Corazon
Aquino is misleading, because they came to prominence through their husbands or father. Benazir Bhutto, prime
minister of Pakistan, was preceded in the office by her father. Early marriages and large families leave the
majority of women in developing countries little time for other pursuits. Malnourishment among women is high
because of the tradition of giving the best food to their children and husbands. Rights granted by law are often
severely limited by religious revivalism in many countries.
Nationalist leaders hoped to industrialize, but were hampered by insufficient capital. Cash crops and mineral
resources are key in many nations. These primary products have been subject to price swings, leaving nations
vulnerable to market forces. Leaders in Africa and the Middle East have often blamed the neocolonial economy,
but other factors play a part. In many countries, a tiny minority absorbs a disproportionate amount of revenue.
The World Bank and the International Monetary Fund have aided industry in developing nations, but the aid often
required economic restructuring.
Paths to Economic Growth and Social Justice:


Leaders of new nations are still seeking solutions to the problems of development.
Authoritarian rule has been a common, but largely unsuccessful, response. The example of Kwame Nkrumah in
Ghana illustrates the point. Genuinely wishing for reform, he finally opted for Soviet support, alienating Western
powers. Crucial revenue form cocoa exports dropped when cocoa prices fell. To stay in power, Nkrumah
177





resorted to oppression and to the celebration of what he called a unique version of socialism. He accepted
comparisons with Confucius and Mohammad, increasing opposition. Suppressed dissenters rose up during his
brief absence in 1966 and deposed him. Military regimes have succeeded Nkrumah.
Military leaders have often used the force at their disposal to impose control after order has broken down.
Western governments tended to support these military leaders because they are generally anti-communist.
Military dictatorships have varied considerably, from the rule of Idi Amin in Uganda, to that of Gamal Abdul
Nasser in Egypt. The Egyptian Fee Officers Movement had its roots in the 1930s, beginning with a nationalist
agenda. It was allied for a time to the Muslim Brotherhood, founded in 1928 by Hasan al-Banna. The later
movement focused on social reform. The murder of al-Banna in 1949 failed to stop the movement. Egypt’s
defeat in the first Arab-Israeli war of 1948 and anger over British occupation of the Suez Canal led to a coup in
1952. The Free Officers took control, and Nasser emerged as leader. The state was used as a tool to bring about
land reforms and to establish an educational system. The regime controlled foreign investment and managed to
gain control of the Suez Canal zone. Land reform was flawed by corruption and by the maneuvers of large
landowners. The Aswan Dam project created significant, unforeseen problems. Anwar Sadat succeeded Nasser,
reversing many of the latter’s programs. Private initiative was favored over state-run projects. Sadat also
reversed the policy of hostility to Israel. His successor, Hosni Mubarak, has generally followed Sadat’s course.
The problems of population growth and massive poverty continue.
India, since it gained independence, has managed to avoid military rule. Rulers such as Nehru have pursued
social reform and economic development, while protecting civil rights in a democratic state. Nehru mixed public
with private investment. The Green Revolution has increased agricultural yields. High-tech industry has also
been an important part of the economy. Yet India’s overpopulation problem is immense, and in spite of a
growing middle class, a large part of the nation has not benefited form development.
Among postcolonial nations, Iran underwent a revolution under the Ayatollah Ruhollah Khomeini. Iran had
never been colonized, but had come under Western influence. Modernization under the Pahlavi shahs was briefly
stopped by a coup, but they were returned to power with the CIA’s support. The shah alienated both the
ayatollahs and the mullahs. Attempts at land reform angered the landowners. A drop in oil prices brought the
country to revolution in 1979. Khomeini’s promises of purification and a return to the golden age of the prophet
were aimed at removing the shah and the Pahlavi dynasty. Like the Mahdi, Khomeini claimed to be divinely led.
Iran was purged of Western influence, and moderate and leftist Iranians were condemned. Secularism was
eliminated from the law. The new government planned land reforms, but Saddam Hussein led Iraq’s seizure of
Iranian territory, leading the two nations to war. The Iran-Iraq war resulted in devastation for Iran, which finally
signed an armistice in 1988. Without regime change, the country did experience some easing of restrictions, and
more open elections took place in the 1990s.
Several African countries, including Angola and Mozambique, were still under colonial powers into the 1970s.
South Africa stood out, however, as by far the largest country still under white rule. The Afrikaners had imposed
white rule in their system of apartheid. Black and shites were kept strictly segregated. Overpopulated
homelands were reserved for “tribal” groups. The Afrikaners ruled a police state. The African National
Congress and other black organizations were declared illegal. Walter Sisulu and Nelson Mandela were two of
many African leaders imprisoned. Another, Steve Biko, part of the Black Consciousness movement, was killed
in custody. From the 1960s, African guerillas emerged, countered by suppression. International pressure coupled
with exhausting wars against Namibia and Angola led to a change in attitude in the South African government.
F.W. de Klerk and fellow moderate Afrikaners began to undo apartheid. All adult South Africans were allowed
to vote in the 1994 elections, which brought Nelson Mandela and the ANC to power. Ethnic hostility still plagues
the country, in spite of the peaceful ending of apartheid.
Some patterns emerge when examining the new nations. India was particularly successful in creating a
democratic state, partly because modern India continues pre-conquest traditions on the subcontinent. In the
Middle East, Islam continues to be a dominant factor. In Africa as in India, the impacts of colonization have
merged with older traditions. In the case of Africa, this often means a tendency toward “Big Man” rule.
Key Terms:
Nelson Mandela
Ayatollah Khomeini
Gamal Abdul Nasser
Anwar Sadat
Hosni Mubarak
Globalization
Neo-colonial Economy
Green Revolution
Apartheid
178
Muslim Brotherhood
Chapter 33, Quiz Questions
1) Why have ethnic rivalries and communal violence been endemic in decolonized African states?
A) The level of civilization in Africa was more primitive at the time of colonization.
B) Tribal life in Africa was traditionally more violent than other cultures.
C) The introduction of slavery by whites in the 19th century brutalized African culture.
D) Europeans colonized Africa and set boundaries without reference to ethnic groups or cultural homogeneity.
E) Africans received training and weapons from European colonists.
2) European colonizers contributed to African population growth by all of the following means EXCEPT
A) the introduction of new food sources from the New World.
B) by bringing an end to local wars.
C) by introducing railway lines that cut down on regional famines.
D) by encouraging immigration of large numbers of whites.
E) by outlawing all means of birth control.
3) Which of the following is NOT a factor in the high birth rate of third-world nations?
A) The ability to import food to feed the increased population
B) Resistance to birth control
C) Social status symbols attached to male virility and the ability of women to have male children
D) The need to extend family lineages in Africa
E) Lack of awareness and education regarding population growth issues
4) Which of the following statements concerning the urban poor in third-world cities is most accurate?
A) Despite their condition and large numbers, the urban poor of the third world have not had a political impact.
B) Development specialists have concluded that slums provide the only urban housing the poor are likely to find.
C) Cities in the third world, fueled by the existence of cheap labor supply, have become the most productive centers of the
economy.
D) Third-world cities generally display the markings of careful urban planning in their programs of expansion.
E) Within a few years most of the population will become middle class and will move to suburbs.
5) On what have third-world countries traditionally depended to finance industrialization?
A) Development of banking systems
B) Sale of resources left behind by colonizers
C) Profits of previous industrialization
D) Sale of cash crops and minerals
E) High taxes
6) "Neocolonialism" refers to
A) Europe's conquest of new colonies in Africa and Asia.
B) Japan's conquest of much of Asia during World War II.
C) the continued relegation of the third world to economic dependency after decolonization.
D) the creation of colonies by India and the more advanced nations of Africa in the last several decades.
E) new world powers are taking over areas once ruled by Europe.
7) Which of the following is NOT a drawback to accepting investment capital from first and second world nations?
A) Excessively high rates of interest
B) Required military alliances
C) Requirements for removal of state subsidies on food and other essential items
D) Commitments to buy products of investors
E) The adoption of Western economic models
8) Which of the following reasons was NOT a factor in explaining the frequency of military takeover in third-world
nations?
A) Regimentation rendered soldiers more resistant to division by religious and ethnic rivalries.
179
B) Isolation from Western and Soviet countries prompted a desire to demonstrate power to foreign governments.
C) The military possessed a monopoly of force essential in restoring order during political crisis.
D) Military personnel possessed some technical training that was often lacking among civilian nationalist leaders.
E) Most of the people in the upper classes favored strong military rule.
9) The military government of Egypt after 1952 attempted all of the following reforms EXCEPT
A) land redistribution schemes limiting the amount of land a single individual could own.
B) state-financed education through the college level.
C) state subsidies to lower the price of food staples.
D) an isolationist foreign policy to minimize military expenses.
E) government employment programs.
10) In which of the following ways was India similar to Egypt following decolonization?
A) Level of industrialization
B) Emphasis on socialism and state intervention
C) Military takeover of government
D) Size of the middle class
E) Minimal influence of religious issues
11) Perhaps more than any other third-world nation, India has been successful at
A) controlling population growth.
B) raising living standards.
C) equitable land redistribution.
D) preserving civil rights and democracy.
E) minimizing religious conflict.
12) In many respects, the Iranian revolution of 1979 is most like
A) the military coup in Egypt in 1952.
B) Gandhi's non-violent resistance to the British Raj.
C) Kwame Nkrumah's government.
D) the Mahdist revolution in the Sudan in the 1880s.
E) the Zionist movement in Israel.
13) Iran, unlike other areas of the third world,
A) had not been colonized by European powers, but had been reduced to an informal sphere of influence.
B) did not have problems with inequitable land distribution
C) possessed a substantial Western-educated middle class that supported the revolution
D) was heavily industrialized and not dependent on the export of cash crops or mineral wealth.
E) had few local traditions to overcome in achieving nationhood.
Essay Questions:
Africa, the Middle East, and Asia in the Era of Independence
1. What problems tend to be typical of all third world nations?
2. How have women fared in the newly independent nations of the third world?
3. Compare and contrast the postcolonial governments of India and Egypt.
4. In what sense was the Iranian revolution of 1979 a throwback to the fundamentalist revolts of the 19th century?
5. In what sense has the process of decolonization been a positive movement?
180
Chapter 34, Rebirth and Revolution: Nation-building in East Asia and the Pacific Rim
Summary:
The recent history of China, Japan, and Vietnam has significant differences from other Asian and African states. Japan
remained independent, industrialized, and became a great imperialist power. After World War II, Korea, Taiwan, and
other industrializing nations gave the Pacific Rim new importance. China and Vietnam suffered from Western and Asian
imperialists. With their traditional order in ruins, they had to face the usual problems of underdeveloped, colonial,
peoples. Full-scale revolutions occurred. By the beginning of the 21st century, the result of all the changes gave East Asia
a new importance in world affairs.
Key Concepts:
East Asia in the Postwar Settlements:




Asia was reorganized following World War II. Korea was occupied by the Soviet Union and the United States,
Taiwan went back to China, and elsewhere colonial rule was restored. Changes followed quickly, including
Indonesian, Malayan, and Philippine independence. China’s communist regime was transformed.
In spite of extensive destruction during the war, Japan was able to recover quickly. Occupied until 1952, the
government and infrastructure was deeply reorganized by the United States. Under a new constitution, the
Japanese undertook legal reforms, which nevertheless supported traditional values. The Liberal Democratic Party
monopolized the Japanese government from 1955 into the 1990s. Education was made available to more
Japanese. Following the end of occupation, traditional values such as repect for the elderly have been
emphasized.
Korea was divided in 1948 between the south, under U.S. domination, and the north under the Soviet Union.
North Korea was governed by Kim Il-Sung until 1994. South Korea was initially governed by Syngman Rhee. In
1950, the Korean War broke out between North and South Korea. The war ended with an armistice in 1954.
The country remained divided.
The Chinese Guomindang occupied Taiwan, while the communists controlled mainland China from 1946-1948.
Aid from the United States supported the Taiwanese into the 1960s. Hong Kong was returned to British control,
which lasted until 1997. Singapore was also under British rule until 1965. By about 1960, many of the smaller
east Asian nations had achieved stability.
Japan, Incorporated:


Japan’s Liberal Democratic Party ruled the country form 1955 to 1993. Many elements of the political system
date form the Meiji period. Economic development was based on cooperation between the public and private
sectors. Supporting birth control and abortion, state intervention has controlled the population growth. Japanese
cultural traditions have been carefully preserved and synthesized with Western borrowings. The writer Yukio
Mishima, for example, initially embraced controversial new themes, but later turned to more traditional values.
Japan’s economy grew remarkably, especially after the 1950s. The Japanese government played a large role in
economic development, partly through educational reform. Spending little on the military, the country could
afford to spend more elsewhere. Japanese labor policies included group exercise and lifetime employment
guarantees. A strong sense of group loyalty kept both labor and management productive. Compared to their
Western counterparts, Japanese women enjoyed more education, but spent more time on domestic duties. The
Japanese suffered less from feelings of isolation, but suffered more from stress than Westerners. Relief is sought
in drinking and in the company of geishas. Western influence shows in the popularity of baseball and golf.
Problems of population and political corruption have become important issues.
The Pacific Rim: New Japans?

After World War II, South Korea was ruled by Syngman Rhee until 1960. The military leader Park Chung-hee
succeeded Rhee, ruling until his assassination in 1979. The military government was removed in the late 1980s,
and opposition political movements and freedom of the press were soon established. Industrialization was heavily
supported form the mid-1950s. Industrial companies, for example Daewoo and Hyundai, now loom large,
181


creating housing and schools for their employees. Growing population pressures led to state-supported birth
control.
In Taiwan, developments in both agriculture and industry spurred economic growth. Private and public
investment improved education and led to economic and cultural change. Hostility between China and Taiwan
eased with the emergence of informal diplomatic ties. Chiang Ching-kuo replaced his father, Chiang Kai-shek,
in 1978. The rule of Singapore’s Prime Minister Lee Kuan Yew, ruling from 1965, resembled the Taiwanese
government. Oppression was silenced by the people’s Action Party. A well established shipping industry was
joined by manufacturing and banking. Hong Kong, also a banking center, connected China with the global
economy. The territory was returned to China in 1997.
Commonalties among the Pacific Rim states include cultural traits group cohesion preferred over individualism
and political direction, including significant government intervention. Malaysia, Indonesia, Thailand, and other
small nations in the region shared in the economic expansion. Growing concerns at the end of the 20th century
included rising unemployment and slowing growth. Predictions that restructuring would be necessary have been
contradicted by slow but definite recovery.
Mao’s China and Beyond:






Chiang Kai-shek’s struggle against communism was halted by the Japanese invasion of the 1930s, which led
Chiang to join the communists. His conventional forces were not successful against the Japanese, leading to U.S.
military support. By 1945, the communists were in the ascendancy. In the civil war form 1945 to 1949, the
communists defeated the nationalists. Chiang retreated to Taiwan, and Mao declared the new People’s Republic
of China. Mao had won support in China by defeating the Japanese, but also by land reform and by giving the
peasants a central role in the movement. Strong military leadership, exemplified by Lin Biao, gave the
communists the upper hand.
The communist struggle for power provided the new government with a ready made infrastructure, including the
party cadres and the People’s Liberation Army. These forces were used to block secession, and then to act
aggressively in Korea and Vietnam against United States involvement. Cooperation between China and the
Soviet Union diminished after the death of Stalin.
Completing the work of the revolution in the countryside was priority for the new government. The landowning
class and large landowners were purged, with as many as 3 million executed. Industrialization was another key
goal. Technocrats rose to power. From the mid-1950s, Mao undertook a new program, the Mass Line approach.
Beginning in 1955, rural collectivization reversed the distribution of land that had been achieved three years
earlier. In 1957, a call for comment on communist rule brought a vocal, critical response, which was harshly
silenced.
A new program launched in 1958, the Great Leap Forward, attempted to bring about industrialization at the
local, rural level, supervised by peasant communes. The result was resistance and corruption, leading to massive
famine. International proposals to implement family planning were dismissed. While the Chinese birth rate was
not extremely high, the country already had a large population. From the 1960s, Families were restricted to one
or two children. By 1960, the Great Leap was abonadoned, and Mao was replaced by pragmatists including
Zhou Enlai, Liu Shaoqui, and Deng Xiaoping.
Mao’s wife, Jiang Qing, helped her husband in reforming the place of women in China. Madam Chiang Kaishek had stressed traditional women’s roles, with the result of greater communist support for expanded women’s
rights. Women rose in the military, and in many other sectors, and gained legal parity with men. However, that
has not always translated to equal opportunity, and higher positions are held by men.
Mao remained head of the Communist party after his fall from power. His opposition to the new administration
culminated in the Cultural Revolution. His Red Guard forces attacked Mao’s rivals, who were killed, executed,
or exiled. The movement achieved the overthrow of the government that Mao had hoped, but then continued, out
of control. Mao ended the campaign in 1968, but political fighting continued. The Gang of Four, led by Jiang
Qing, plotted to overthrow the pragmatists, but was not successful. After Mao’s death, Deng Xiaoping led the
pragmatist majority. In spite of important failures, the Chinese have successfully redistributed wealth and
improved conditions for most of their people.
Colonialism and Revolution in Vietnam:
182





Vietnam attracted the attention of the French from the 1600s. In the 1770s, the Tayson Rebellion overthrew the
Nguyen dynasty, and the Trinh dynasty was also ousted shortly after. The surviving Nguyen, Nguyen Anh was
supported by the French. He ruled as Emperor Gia Long. The French were rewarded with a privileged place at
court. Gia Long ruled an enlarged Vietnam, including the Mekong and Red river regions. His highly traditional
rule was continued by Minh Mang, who also embraced Confucianism. He persecuted the French catholic
community. French adventurers undertook the conquest of Vietnam and Cambodia. All of Vietnam was under
French control by the 1890s. French attempts to maximize their profits exacerbated overcrowding and the
migration of peasants to urban areas.
In the late 19th into the 20th centuries, the Vietnamese rallied around their ruler. The lack of support form Nguyen
and Confucian bureaucrats led to loss of faith in both. A Western-educated middle class emerged in the early
1900s, often adopting French ways. Rising nationalism coalesced around the secret Vietnamese Nationalist
Party (the VNZDD) in the 1920s. Failed uprising ending with a 1929 revolt weakened the party. The
Communist Party of Vietnam was left to rally resistance. The future Ho Chi Minh dominated the party.
Communist support helped the Communists oppose the French. Weakened by the Japanese invasion of Indochina
in 1941, French rule was left vulnerable to nationalist rebellion.
The nationalist movement, the Viet Minh, liberated portions of the country during World War II. When Japanese
rule ended, the Viet Minh were able to take power. Under Vo Nguyen Giap, the Vietnamese used guerilla
fighting against the French and Japanese. In 1945, Ho Chi Minh declared the independence of Vietnam. The
next year, the Frecnh, with British support, moved to retake the country. Guerilla warfare began anew. The
French were defeated at Dien Bien Phu in 1954.
Although the United States and the Viet Minh cooperated during World War II, anticommunist sentiment in the
United States following the war drove the two apart. The United States supported the presidency of Ngo Dinh
Diem. Diem’s attack on communists in the south, called the Viet Cong, led to further communist support of the
new National Liberation front. The United States sent increasing support, including nearly 500,000 troops in
1968, but was finally forced to retreat in the 1970s. Communist rule united the north and south.
Vietnam remains isolated, in part because of U.S. international pressure. Attempts to impose hard-line Marxism
failed to ease poverty. From the 1980s, Vietnam has been more open to outside investors, boosting economic
growth. The costs of entering the global economy include harsh conditions for workers and diminishing social
services.
Key Terms:
Hong Kong
Korean War
Ho Chi Minh
Kim Il-Sung
People’s Liberation Army
Mass Line
Great Leap Forward
Cultural Revolution
Deng Xiaoping
Red Guard
Chapter 34, Quiz Questions
1) In what year was the civil war in China decided by a Communist victory?
A) 1912.
B) 1940.
C) 1947.
D) 1958.
E) 1970.
2) Americans introduced all of the following reforms to Japan during their occupation EXCEPT
A) giving women the vote.
B) abolishing Shintoism as a state religion.
C) outlawing labor unions.
D) making the emperor a symbolic figurehead.
E) breaking up large land estates.
3) In what way was the restoration of an independent Korea complicated?
A) Korea had become a colony of China, which refused to restore independence.
B) Korea was divided into separate zones controlled by the U.S. and the Soviet Union.
183
C) Korea's government was claimed by surviving members of the old monarchy.
D) Korea had no prior experience as an independent government.
E) The former royal rulers of Korea wanted to be restored to power.
4) Which of the following statements most accurately reflects the situation in Korea following the Korean War?
A) Northern and southern Korea were rapidly reunited under a single, authoritarian government controlled by the Soviet
Union.
B) Northern Korea threw off its ties with China and the Soviet Union and sought a closer relationship with the U.S.
C) Korea remained divided with relatively authoritarian governments in both halves of the divided nation.
D) Southern Korea became fully democratic, but moved closer to political neutrality during the cold war.
E) Southern Korea was defeated after a surprise sea invasion and the defeat of its navy.
5) The Japanese political system after 1955
A) was marked by radical shifts between parties of the left and right.
B) was typified by the dominance of socialism.
C) revived many of the oligarchic features of earlier political tradition.
D) was intent on the destruction of the big business combines.
E) was characterized by the dominance of two major parties.
6) What was the weakness of the leadership of the Liberal Democratic Party in Japan in the 1980s?
A) Inability to provide economic growth
B) Corruption
C) Association with military policies of World War II
D) Adoption of a policy of nuclear armament
E) Its connection to the left
7) Which of the following was NOT a feature of government involvement in Japanese industry?
A) Setting production goals
B) Establishing investment goals
C) Limiting imports
D) Selecting the heads of the major corporations
E) Helping the education sector
8) Which of the following was NOT a factor in the amazing economic growth of Japan following the 1950s?
A) Cheap loans for technological innovation
B) Educational expansion
C) A growing population and a reduction in the agricultural labor force
D) A rapidly growing military-industrial complex
E) Japan's foreign policy
9) Which of the following represents a significant difference between Japanese and Western women in the later 20th
century?
A) Women in Japan participated actively in leisure activities with their husbands.
B) Japanese women had higher rates of divorce than their Western counterparts.
C) The Japanese feminist movement was confined to a small number of intellectuals.
D) Japanese women concentrated less on domestic duties than women in the West.
E) Women in Japan tended to marry very young but still did manage to make careers.
10) In what way was the Chinese Communist takeover of China different from the Bolshevik seizure of power in Russia?
A) China was already more industrialized than Russia at the time of the communist success.
B) The Russian seizure of power was accomplished without ridding the country of the ruling dynasty.
C) The Chinese Communists claimed a unified country and did not experience years of civil war after they came to power.
D) The Chinese Communist leadership was unable to move directly to the tasks of social reform and economic
development.
E) Russia was heavily industrialized whereas China was still mostly a rural-based society.
184
11) Which of the following statements is most accurate?
A) After the civil war that brought the Communists to power, the country was rapidly demilitarized.
B) The People's Liberation Army, the source of the Communist rise to power, gained dominance over the Communist
Party.
C) Most of China was administered by civilian bureaucrats recruited from the old Confucian scholar-gentry.
D) The army remained clearly subordinate to the Communist Party, with cadre advisors attached to military contingents.
E) The Chinese Communists allowed the old landed-gentry class to continue to exist and manage the state's farms.
12) Which of the following statements concerning Chinese domestic policies during the 1950s and 1960s is most
accurate?
A) Despite pledges made during the civil war, the Communist Party failed to undertake substantial land redistribution
programs.
B) With the introduction of the first five-year plan in 1953, the Communist leadership turned away from the peasantry.
C) Increasingly Mao came to embrace the old Confucian concept of a bureaucratic elite as the means of government.
D) Mao's primary trust came to rest in a group of intellectuals associated with the University of Beijing.
E) The Chinese Communist leadership began a massive attempt to reeducate the peasantry and create an industrialized
class.
13) Which of the following principles was NOT part of the Chinese Communist Party attitudes towards women?
A) Women should achieve legal equality
B) Career opportunities should be open to women, who should work outside of the household.
C) "Virtue was more important for women than learning."
D) "Women hold up half of the heavens."
E) "Women should be involved in their children's lives."
14) During the 1980s, all of the following were typical of the Chinese government EXCEPT
A) encouragement of private market production for the peasantry.
B) private enterprise within the industrial sector.
C) domination by the "pragmatists."
D) democratic reform.
E) repression of certain student groups.
15) In what way was the early nationalist organization in Vietnam similar to other third world nations?
A) It was composed of a Western-educated middle class.
B) It was Marxist on the model of the Communist Party of Russia.
C) It was drawn almost entirely from the peasantry.
D) Its goals were entirely peaceful.
E) It tried to appease all sides in the revolutionary conflict.
16) In part, the defeat of the United States in Vietnam in the 1970s resulted from
A) inadequate air power resulting in the use of chemical weapons.
B) an insufficient commitment by the U.S. government to win the war.
C) Vietnamese belief that the communists were fighting for independence.
D) the greater loss of American than Vietnamese lives.
E) the general superiority of the Viet Cong as soldiers.
Essay Questions:
Rebirth and Revolution: Nation-building in East Asia and the Pacific Rim
1. Discuss similarities in the preconditions of revolution in China and Vietnam.
2. In what ways did Mao’s concept of a peasant revolution lead to specific shifts in domestic policy during his rule?
185
3. In what ways have the Communist regimes of Vietnam and China retained ties to the traditional cultures of the
region?
4. How did the experience of decolonization in Vietnam differ form that of most third world nations?
186
Chapter 35, The End of the Cold War and the Shape of a New Era:
World History 1990-2006
Summary:
The collapse of the Soviet Union and its subject regimes ended the cold war. Global history took a sharp turn.
Colonialism’s end opened new possibilities for either human improvement or international and social conflicts, and for the
emergence of a truly globalized economy.
Key Concepts:
The End of the Cold War:





Decades of the Cold War were ended in the 1980s. What factors brought an end to Russian expansion?
Following Khrushchev, Soviet leadership lost its dynamism, at the same time that neighbors of the Soviet Union
broke away from Soviet dominance. Iran’s revolution frightened the Soviets, who invaded Afghanistan to create
a buffer. The Polish Solidarity movement threatened control in eastern Europe. Even China, which remained
communist, took a different course, distancing itself form the Soviet Union. The United States, under Jimmy
Carter, criticized Soviet human rights violations, while negotiating arms limitations. The Soviet Union’s invasion
of Afghanistan was denounced by the United States. Under Ronald Reagan, military spending soared, while the
“Reagan doctrine” promised help to any group fighting communism.
From 1985, the Soviet Union undertook reforms intended to boost its economy. While industrial production
slumped, social programs suffered.
In 1985, Mikhail Gorbachev took charge of the Soviet Union. He demonstrated his willingness to turn toward
the West. He announced his policy of glasnost, initiating a more open attitude toward political critique. Yet he
did not turn from communism. The Soviet Union was opened to foreign companies. Gorbachev next began a
new program, centered on perestroika, or economic restructuring. He called for lowered military spending, more
foreign investment and agricultural reform. He supported the creation, in 1988, of the Congress of People’s
Deputies. Gorbachev was elected president of the Soviet Union in 1990. Unrest among minority nationalities
rose, partially due o economic difficulties.
Outside the Soviet Union, Gorbachev’s programs had clear results. Bulgaria, Hungary, East Germany,
Czechoslovakia, and Poland moved away from communism or abandoned it altogether. The Berlin Wall was torn
down, and Germany was reunified in 1991. Some countries retained communism, but uder new, more moderate
regimes. Ethnic clashes occurred in many areas. Yugoslavia in particular was torn by violence among its
peoples. The decade of the 1990s was a period of economic uncertainty. Gorbachev gave soviet bloc nations the
right to self-determination.
Gorbachev’s regime and his programs were threatened by an unsuccessful coup in 1991. His leadership of the
Soviet Union was contested by leaders of the republics, especially the Russian Republic. The Russian leader,
Boris Yeltsin, declared that the Soviet Union was gone, calling for a commonwealth. The Commonwealth of
Independent States emerged, but was threatened form the outset by challenges form the Ukraine and Kazakhstan.
Economic difficulties in the late 1990s challenged Yeltsin in 199, keeping a firm hold on the media and opposing
Chechen calls for independence.
The Spread of Democracy:


Divergent forms of government communism, fascism, democracy were rivals for most of the 20th century. The
century ended with democracy in the ascendant. Spain, Portugal, and Greece, along with all Latin American
countries except Cuba, adopted democratic forms of government. South Korea, Taiwan, and the Philippines have
also chosen democracy. In Africa, democratic regimes included those of South Africa and Nigeria. Georgia and
Ukraine have also held democratic elections, as have several countries of the Middle East, including Saudi
Arabia.
China, North Korea, other Asian countries, and some in the Middle East have made other choices. In 1989,
Beijing student demonstrations in support of democracy were brutally repressed, many people were killed. After
200, questions arose concerning the promises of democracy. The U.S. supported non-democratic countries, while
democracies in Latin America experienced problems that brought the system into question.
187
The Great Powers and New Disputes:




Regional rivalries became clearer with the end of the Cold War.
When the Soviet Union was dismantled, repressed internal problems became obvious. Armenia and Azerbaijan,
formerly under Soviet rule, faced ethnic hostility. Yugoslavia dissolved as rival ethnic and religious groups faced
off. Slovenia, Croatia, Serbia, and Bosnia-Herzegovina became independent nations. Another conflict arose in
Kosovo, between Albanian nationalists and Serbians.
In Some areas of the world, old rivalries continued. In the Middle East, hostility between Iran and Iraq had led to
an Iraqi victory. Then Iraq invaded Kuwait, leading to the Persian Gulf War in 1991, which ended with the
defeat of Iraq. Israeli-Palestinian struggles continue. India and Pakistan have engaged in disputes over their
common border.
New conflicts arose elsewhere. European countries faced separatist or nationalist movements. Immigration,
especially from former colonies, caused intense problems in European countries, leading to new political
movements. Violent conflicts in Africa have plagued the continent. High death tolls have added to the problems
faced by African nations. The international response has often been slow.
The United States as Sole Superpower:


While Russia devoted less money to its military budget, U.S. military spending has been high. Fear of U.S.
aggression has led to new partnerships among other world powers. The U.N. has the ability to counter U.S.
power, but its role is not secure. In the United States itself, there is no consensus about how to use its monopoly
on power. The country has been reluctant to sign international agreements that may interfere with its sovereignty.
Terrorism aimed at the United States reached a high point with the bombing of the World Trade Center and the
Pentagon in 2001. The “War on Terror” has become an important facet in U.S. international relations. The
international community was supportive of the successful U.S. attack on Afghanistan, but the subsequent invasion
of Iraq has been subject to widespread criticism.
Key Terms:
Putin
Euro
War on Terror
Ronald Reagan
Persian Gulf War
Mikhail Gorbachev
Perestroika
Glasnost
Boris Yeltsin
Multinational Corporation
Chapter 35, Quiz Questions
1) Which of the following factors did NOT play a role in the demise of communism?
A) Soviet leadership
B) Reassertion of eastern European initiative
C) Western Europe's thriving Economy
D) The steadfastness of Chinese policy
E) Economic costs of the cold war
2) The term perestroika refers to
A) a new freedom to comment and criticize the Soviet government.
B) economic restructuring and more leeway for private ownership.
C) the Soviet space program.
D) the establishment of a liberal democracy.
E) an attempt by the Soviet leadership to allow non-communists to manage the economy.
3) The end of the cold war was associated with what large trend in the world at the end of the 20th century?
A) The rapid industrialization of the third world
B) The spread of multiparty democracy
C) The ending of world hunger
D) The dismantling of Western armed forces
188
E) A slow economic decline leading to a depression in 2005
4) A growing tactic of opponents of the ideals of globalization has been
A) terrorism.
B) communism.
C) institutional racism
D) political fundamentalism
E) conservative rallies
Essay Questions:
The End of the Cold War and the Shape of a New Era: World History 1990-2006
1. What trends in political organization and economic development can be identified in 20th century world
civilizations?
189
Chapter 36, Globalization and Resistance
Summary:
By the early 21st century, the unfolding of globalization the increasing interconnectedness of all world parts reflected the
close of the cold war and the lessening of international conflict, a movement to free markets, new technical developments
(especially the computer), and a general acceptance of global connections. Complicating factors to globalization were
lingering nationalism, an important religious surge, and terrorism.
Key Concepts:
Globalization: Causes and Process:





Globalization increased toward the end of the 20th century, partly due to the entry into world markets of such large
nations as China and members of the former Soviet Union. Only a few countries declined to join the global
economy. Internationalism replaced nationalism as a dominant cultural theme.
Technological improvements increased, making it easier to maintain contact over long distances. E-mail and the
development of the World Wide Web have revolutionized communications. Satellite television has made global
audiences possible for any one broadcast.
International investment has increased remarkably, comprising up to 40 percent of U.S. total investments.
Multinational corporations became a new force. Corporations sought cheap labor and undemanding
environmental policies. Raw materials were important, as they had been in the first phases of industrialization. In
some cases, multinationals were more powerful than the countries in which they operated. They were also able to
transfer their activities from one area to another. Outsourcing, the tactic of hiring outside workers, was used to
hire cheap labor wherever it could be found. While industrial nations made use of cheap labor in developing
countries, they generally offered higher wages than the local prevailing wages. The impact of globalization is
difficult to discern. Unemployment rates are high in areas that have not been successful in competing globally.
International demands have led to human rights issues, from the sale of body organs to increasing child labor in
some areas.
International patterns of migration established in the 1950s and 1960s continued into the new millennium.
Declining birth rates in some countries necessitated immigration. Large numbers of immigrants have led to
tension. Modern travel has made it easier to migrate to and from distant countries, transforming earlier
patterns of immigration.
Cultural exchange has reached new levels in the age of globalization. Greater scientific collaboration exists,
generally with English as the common language at the same time; fast-food restaurants such as McDonald’s have
covered the globe. American television shows and movies have also found world audiences. In the same way,
cultures around the world either celebrate American holidays, or celebrate their own holidays with greeting cards
and presents. Other nations, especially Japan, also exported their popular culture. One of the results of these
developments is the great increase in obesity, especially among children. Cultural globalization has been
mitigated in those areas where access to high technology is unavailable, and by adaptation of global culture to
local tastes and traditions.
Resistance and Alternatives:


Criticism of globalization has increased. Large rallies began in 199, pointing to the environmental dangers,
exploitation of cheap labor, and growing gaps between rich and poor, which critics attribute to globalization.
Critics have also claimed that gaps are emerging between rich and poor nations.
Nationalism has remained a vital force in some areas, in spite of growing internationalism. Resistance to foreign
cultures has included the French refusal to officially acknowledge the use of some English words. Religious
movements have provided some of the most powerful resistance to globalization. Russian Orthodoxy is only one
example of a religion that has gained new prominence. Fundamentalism, whether Protestant, Muslim, Hindu, or
Catholic, is a powerful force, that is often in opposition to globalization. Fundamentalism generally increases
intolerance and exclusivity. It has exacerbated, though it has not generally caused, hostility between or within
peoples. Terrorism is increasingly motivated by religious causes.
190
The Global Environment:



Environmental impacts changed in degree more than in kind. Efforts at industrialization increased the scale of
environmental hazards, for instance in China and the former Soviet Union. Southeast Asia is an area of
increasing extraction of natural resources, causing alarm in the world community. Two other areas of concern are
that the wealthiest countries consume products out of proportion to their populations, and that the same countries
also contribute more pollution compared to developing nations.
The greenhouse effect, an increase of gases that cause overheating of the earth, is now acknowledged by most
scientists. Sources of the gases that cause the problem include fossil fuel combustion, rice paddies, and
refrigeration. Predicted impacts include rising sea levels and dramatic changes to vegetation patterns.
Greenhouse gases are not new, but are produced in much greater quantities. Rainforest destruction is also not
new, but is occurring at greater rates, and causes concern because the forests are slow to grow back. International
conferences, held to address environmental concerns, have lacked support from the United States and other key
nations.
Epidemics, including AIDS and SARS, have become global issues.
Toward the Future:




Forecasting the future, while universally desired, has been shown to be problematic at best.
Trends can be studied to predict their course in the future. Yet, discerning trends can itself be difficult. The
existence of contradictory trends makes it hard to predict the ascendancy of one or the other.
One method for predicting the future exists in comparing past and present, in terms of “revolutions,” to develop
scenarios that describe that future.
Predicting the future is made difficult by the profound changes of the last hundred years. For instance, women’s
roles have been transformed, but it is not clear what current trends will predominate.
Key Terms:
Global Warming
AIDS
Internationalism
World Wide Web
Outsourcing
Cultural Exchange
Nationalism
Fundamentalism
McDonalds
Environmental Impacts
Chapter 36, Quiz Questions
1) What development led to the explosion of electronic communication in the 1990s?
A) Governments in many nations relaxed their censorship of ideas and communication.
B) Technology advances made communications devices, smaller, cheaper, and better
C) Many nations implemented huge programs to advance Internet, satellite, and cellular communication.
D) Religious and cultural resistance to globalization virtually disappeared.
E) Education systems around the world taught people how to use the devices.
2) On the whole, political institutions globalized at what rate compared to technology or business?
A) At about the same pace
B) Less rapidly
C) More rapidly
D) Political institutions never globalize.
E) There is an inverse relationship between the two.
3) Which of the following is NOT a trend running counter to globalization?
A) Nationalism
B) Religious differences
C) Ethnic competition
D) The Internet
E) Competition over economic resources
191
4) By the end of the last century, what percentage of humanity consumed a whopping four-fifths of all marketed goods
and services?
A) Four-fifths
B) One-half
C) Two-thirds
D) One-fifth
E) Three-fifths
Essay Questions:
Globalization and Resistance
1. The authors postulate several potential causal factors impacting civilization in the future, including population
growth, the exhaustion of frontiers, and technological advances associated with the “postindustrial world.”
Evaluate the impact such factors are likely to have on the future.
2. Consider the role of cultural identity versus the pace of internationalization in 20th-century cultures.
192
WRITING FOR THE AP EXAM
There are three types of essays all students must complete on the AP World History exam:
Document Based Question (DBQ) that asks students to read and analyze a set of documents and then write an essay
about them. Students will practice this skill throughout the course using the “Document Analysis” worksheet.
Documents may not always be written texts. A map, graph, chart, table, or visual image may also provide useful
evidence. Students need to practice analyzing individual texts or other data. There are worksheets on both map and graph
analysis to help students develop a system for analyzing these types of evidence. The Document Analysis Worksheet can
be used to analyze a photo by answering the questions with the photo in mind and broadly interpreting the word
“document.” If students need more specific practice analyzing documents, the teacher can help them by creating a
scaffolding assignment with questions that apply to a specific document.
In addition to analyzing individual evidence, students also need to consider a set of documents as a whole unit,
combining knowledge gleaned from individual documents to create an overall picture. They need to look at the whole
group and see how the documents can relate to each other. They need to be able to think of logical groups for the
documents, and they need to be able to determine viewpoints or important data that are not represented in the set, thus
possibly depriving them of a complete picture of the topic addressed in the Document Based Essay.
Change Over Time (COT) essay asks students to deal with broad changes in one or more regions of the world over at
least one of the course’s periods. To prepare students for the Change Over Time essay, have them practice by completing
Change Analysis worksheets regularly. Ask them to examine and analyze changes in one or more societies over a period
of time. Remind them of the world history themes and ask them to think broadly and in terms of how people’s approaches
to these topics evolved. What caused the evolution, and what is the impact of the change?
Comparative essay asks students to compare two or more societies on a set of issues. The Societal Comparison Sheet
assignments will get students used to looking at parallel characteristics in several different societies. They will assess the
level of similarity and difference among the societies, looking for breaks and continuities. They will also evaluate causes
and effects of the differences and similarities, thinking in terms of world history themes.
In addition to the specialized tasks for each specific kind of essay question, students also need to be able to use a
set of skills that generally fall under the category of “good history writing.” Historical essays require students to:
Develop a thesis that answers all parts of the question. If the essay does not complete all tasks required by the prompt,
it will not receive as good a score as those that do. Therefore, students need to practice identifying the tasks of the
question and making sure their thesis addresses all tasks. The thesis should define a student’s position on the question’s
topic and point to the details the student plans to address. Providing a “road map” in the thesis can help students stay on
track with their essays as they hurry to present all their points in a short period of time. Furthermore, students should keep
referring back to their thesis throughout the essay. In other words, the essay’s evidence should be directly tied to the
thesis and should offer further proof of the thesis.
Support the thesis with historical evidence. The historical evidence should be specific information that demonstrates
the accuracy of the thesis. It is not enough to simply write down some specific historical information. The student must
also take the next step and explain how the evidence supports the thesis, rather than leaving the reader to make the
connection.
Meet standards of good writing practices and use an appropriate style. In timed writing, students have to
demonstrate analytical writing skills in a short period of time with little opportunity to plan or revise. They should devote
their writing time to constructing an original and thoughtful thesis and then supporting it. Because of these “guerilla”
conditions, students should minimize the time they spend developing an introduction. They should also make sure that
their essay can stand without an elaborate conclusion because they simply may not have time to construct one.
Additionally, they should write about the past in past tense, use active voice, not use personal pronouns, avoid rhetorical
questions, and strive for correct spelling and grammar. While none of these “shoulds” are absolute rules, they are good
guidelines, and students should have clear reasons in mind if they disregard them.
The “Developing a Thesis and Planning an Essay” worksheet will help students write theses that state a position
and answer the prompt. Also, it will ask them to identify and rank their specific historical evidence according to its
strength.
193
Managing the Grading
If students are assigned the number of papers they need to practice their writing skills, teachers run the risk of continually
floundering in a sea of essays to grade. There are ways to keep students writing while maintaining your sanity.
First, students don’t always have to write a complete essay. Having them complete the “Developing a Thesis and
Planning an Essay” worksheets requires them to go through the most difficult part of essay writing, the thinking and
planning, without producing a complete essay for teachers to grade. Additionally, the worksheet makes it easy for
teachers to quickly find and evaluate the thesis and then see how the evidence supports the thesis. The entire class, a small
group, or an individual can complete this activity. It can be class work designed to put students through the process with
nothing being turned in. It can be used as a quiz to be completed by individuals or small groups. It can be homework to
be collected and graded. After students have completed several of these exercises with “low stakes” grades attached, they
can then choose worksheets that they would like to develop into a complete essay. In this way, students have practiced
planning many essays, but have not produced an unmanageable amount of complete works for their teacher to grade.
Once students are comfortable with the process, the teacher can then give students in-class essays to practice the timedwriting elements using topics that are new to the students. While the teacher works on grading those essays, students can
continue to practice the process.
Students can practice generating theses and proof in a “history journal” environment. The first few minutes of
class can be given to the students writing a quick response to a simple: “To what extent do you agree or disagree with the
following statement” prompt that the teacher has written on the board. (i.e. To what extent do you agree or disagree with
the following statement: “The western European Renaissance was not a distinct period in history. It was, instead, a
logical continuation of trends from the Late Middle Ages.”) After the students have had a few minutes to quickly write
their responses, they can read their writing to a peer partner and receive immediate feedback. Again, these short, timed
writings can be kept in a journal or they can be turned in as a quiz over writing skills or the content of assigned texts.
Keep in mind that practice essay topics do not need to come exclusively from the world region or time period
being currently studied. The AP exam is cumulative, and students need to remember that they are responsible for
retaining information throughout the entire course.
194
Developing a Thesis and Planning an Essay
What are the distinct tasks the prompt requires?
What general topics are you going to address in this essay? Write down the geographic regions, historical
periods, and themes you will discuss. Note any particular terms you will need to define in your essay.
Write a simple statement of your answer to the prompt:
What specific historical evidence leads you to your conclusion?
Item one:
How it supports your answer:
Item two:
How it supports your answer:
Item three:
How it supports your answer:
Item four:
How it supports your answer:
 Rank your evidence by strength. Put your strongest evidence first and finish with your weakest. That
way, if you run out of time, you will be missing only the weakest parts of your essay. Now that you’ve
analyzed your evidence, revise your position, if necessary.
195
 Rewrite your thesis into a formal statement that addresses all issues raised in the prompt, states your
position, and provides an indication of where you will go with your essay, including some reference to
your planned historical evidence. It is okay for your thesis to be several sentences, or a short, themed
paragraph, if you cannot fit all of that into one sentence.
 List the order in which you will present your historical proof.
Now you have created the skeleton for your essay. You have a strong thesis and a plan for supporting it. All
you have to do is add the transitions, connect the bones of your skeleton, and flesh it out with elaboration to
make a great essay.
196
Analyzing a Map
There are certain steps you should routinely follow when attempting to gather information from a map.
1.
What is the title of the map?
2.
What area does the map cover?
3. What time period does the map describe?
4.
What specific places are marked on the map? Why do you think they are marked?
5. Are there any insets (smaller maps that show a specific portion of the map as a whole)? What do they
show? Is the map in a series? What is the series trying to show?
Look at the key:
6.
What areas are shaded? What does the shading signify? What are the different colors? What
does each color signify?
7.
What symbols are used? What do they mean? Where are they located?
8. Summarize in 1-3 sentences the information that the map conveys.
197
Analyzing a Graph
1.
What is the title of the graph?
2.
What information is depicted on each axis?
3.
What are the increments of measurement on each axis? Are there any breaks designed to
demonstrate a break in the scale or a compression of data?
4.
What do the lines or bars on the graph represent?
5.
How do the lines or bars relate to each other?
Summarize in 1-3 sentences the information the graph conveys.
198
World History Generic DBQ Scoring Rubric
BASIC CORE (competence)
0-7 Points
1. Has acceptable thesis
1 Point
2. Addresses all documents. Understands the basic meaning of documents.
(May misinterpret one document.)
1 Point
3. Supports thesis with appropriate evidence from all documents.
2 Points
(Supports thesis with appropriate evidence from all but one document)
(1 Point)
4. Analyzes point of view in at least two documents
1 Point
5. Analyzes documents by grouping them in two or three ways, depending on the
question
1 Point
6. Identifies and explains the need for one type of appropriate additional
document or source
1 Point
Subtotal
/7 Points
EXPANDED CORE (excellence)
(Historical skills and knowledge required to show excellence)
Expands beyond basic core of 1-7 points. A student must earn 7 points in the basic
core area before earning points in the expanded core area.
Examples:







Has a clear, analytical, and comprehensive thesis
Shows careful and insightful analysis of the documents
Uses documents persuasively as evidence
Analyzes point of view in most or all documents
Analyzes the documents in additional ways - groupings, comparisons,
syntheses
Brings in relevant "outside" historical content
Explains why additional types of document(s) or sources are needed
0-2 Points
Subtotal
/2 Points
TOTAL
/9 Points
Likely AP Score
8-9
6-7
4-5
2-3
0-1
________ / 9
Class Points
48-50
42-47
35-41
30-34
27-29
________ / 50
199
World History Change and Continuity Over Time (CCOT) Essay Rubric
BASIC CORE (competence)
(Historical skills and knowledge required to show competence)
0-7 Points
1. Has acceptable thesis
(Addresses global issues and the time period(s) specified)
1 Point
2. Addresses all parts of the question, though not necessarily evenly or thoroughly
2 Points
(Addresses most parts of the question: for example, addresses change but not
continuity)
(1 Point)
3. Substantiates thesis with appropriate historical evidence
2 Points
(Partially substantiates thesis with appropriate historical evidence)
(1 Point)
4. Uses relevant world historical context effectively to explain change over time
and/or continuity
1 Point
5. Analyzes the process of continuity and change over time
1 Point
Subtotal
/7 Points
EXPANDED CORE (excellence)
(Historical skills and knowledge required to show excellence)
Expands beyond basic core of 1-7 points. A student must earn 7 points in the basic core
area before earning points in the expanded core area.
Examples:





Has a clear, analytical, and comprehensive thesis
Analyzes all issues of the question (as relevant): global context, chronology,
causation, change, continuity, effects, content
Addresses all parts of the question evenly
Provides ample historical evidence to substantiate thesis
Provides links with relevant ideas, events, trends in an innovative way
0-2 Points
Subtotal
/2 Points
TOTAL
/9 Points
Likely AP Score
8-9
6-7
4-5
2-3
0-1
________ / 9
Class Points
48-50
42-47
35-41
30-34
27-29
________ / 50
200
World History Compare and Contrast Essay Rubric.
BASIC CORE (competence)
0-7 Points
1. Has acceptable thesis
(Addresses comparison of the issues or themes specified.)
1 Point
2. Addresses all parts of the question, though not necessarily evenly or thoroughly
2 Points
(Addresses most parts of the question: for example, deals with differences but not
similarities.)
(1 Point)
3. Substantiates thesis with appropriate historical evidence
2 Points
(Partially substantiates thesis with appropriate historical evidence)
(1 Point)
4. Makes at least one relevant direct comparison between / among societies.
1 Point
5. Analyzes at least one reason for a similarity or difference identified in a direct
comparison.
1 Point
Subtotal
/7 Points
EXPANDED CORE (excellence)
Expands beyond basic core of 1-7 points. The basic core score of 7 must be achieved
before a student can earn expanded core points.
Examples:






Has a clear, analytical, and comprehensive thesis
Addresses all parts of the question thoroughly (as relevant): comparisons,
chronology, causation, connections, themes, interactions, content.
Provides ample historical evidence to substantiate thesis
Relates comparisons to larger global context.
Makes several direct comparisons consistently between or among societies.
Consistently analyzes the causes and effects of relevant similarities and
differences.
0-2 Points
Subtotal
/2 Points
TOTAL
/9 Points
Likely AP Score
8-9
6-7
4-5
2-3
0-1
________ / 9
Class Points
48-50
42-47
35-41
30-34
27-29
________ / 50
201
Leader Analysis Sheet
Name of leader:
Lifespan:
Title:
Country/region:
Years in power:
Political, social, and economic conditions prior to leader gaining power:
Ideology, motivation, goals:
Significant actions and events during term of power:
Short-term effects:
Long-term effects:
202
Peoples Analysis Sheet
Name of group:
Time period:
Location:
Important neighbors:
Strengths:
Weaknesses:
Impact on neighbors:
Legacy:
203
Conflict Analysis Sheet
Name of conflict:
Time period:
Type of conflict:
Underlying causes:
Immediate cause(s):
Turning points/important events:
Ending event(s):
End result:
Short-term effects:
Long-term effects:
204
Change Analysis Sheet
Society:
Time period:
Significant events during time period:
Characteristics at the beginning
of the time period:
Characteristics at the end
of the time period:
Political
Social
Economic
Artistic
Religious
Intellectual
Technological
Military
Geographic
Demographic
Women’s status
Causes and impact of changes:
205
Societal Comparison Sheet
Time period:
Significant events during time period:
Society One:
Society Two:
Characteristics of Society One:
Characteristics of Society Two:
Political
Social
Economic
Artistic
Religious
Intellectual
Technological
Military
Geographic
Demographic
Women’s status
Explanation of similarities and differences:
206
Document Analysis Sheet
Source (name and type):
Author:
Time period:
Society:
Political, social, economic characteristics at time written:
Purpose:
Tone:
Audience:
Point of view:
Important content:
Evidence of bias:
Assessment of validity:
207
The Dialectical Journal
Complete this double-entry journal while reading. In the left column, paraphrase an idea that is important or
interesting. Include the page number so others can locate the passage. In the right column write your response
to the concept or fact in the left column, by analyzing its importance.
paraphrase and page number
response
|
|
|
|
|
|
|
|
|
|
|
|
|
|
|
|
|
|
|
|
|
|
|
|
|
|
|
|
|
|
|
|
|
|
|
|
|
208
Inner/Outer Circle
Soon we will have an inner/outer circle discussion in class. You will be graded on your discussion, and you
will need to prepare for it. Here are the expectations for the discussion.
Discussion Expectations:
1. Be prepared. Read the texts for depth of understanding. Think about your reading. Take care to write
questions that are worth discussing and can be answered by the text. Write down answers to your questions.
2. Raise your hand and wait to be called upon. Do not raise your hand until the student currently speaking is
finished.
3. Look at the other students--not your desk--when talking.
4. Do not engage in side conversations in either circle.
5. Take notes with your head up when you are in the outer circle.
6. Positive points: contribute relevant facts, analysis, interpretation, evaluation; add new information--don't
just restate someone else's comment.
7. Negative points: not paying attention, interrupting, irrelevant comments, attacking other speakers,
monopolizing the conversations
To prepare for the discussion, you will write questions based on the assigned readings. You need to write
questions that are at high levels of thinking. This chart of thinking levels will help you make sure your
questions require thought and discussion.
Bloom's Taxonomy
1. Knowledge
arrange, define, duplicate, label, list, memorize, name, order, recognize, relate, recall,
repeat, reproduce, state
Name the . . . ., identify facts
2. Comprehension classify, describe, discuss, explain, express, identify, indicate, locate, recognize, report,
restate, review, select, translate
Explain what happened, tell what is meant, give reasons
209
3. Application
apply, choose, demonstrate, dramatize, employ, illustrate, interpret, operate, practice,
schedule, sketch, solve, use, write
Use the author's thesis, classify new information
4. Analysis
analyze, appraise, calculate, categorize, compare, contrast, criticize, differentiate,
discriminate, distinguish, examine, experiment, question, test
Answer why, make conclusions, differentiate facts from opinions, find supporting
evidence
5. Synthesis
arrange, assemble, collect, compose, construct, create, design, develop, formulate,
manage, organize, plan, prepare, propose, set up, write
Combine elements to make a new product/pattern, create solutions
6. Evaluation
appraise, argue, assess, attach, choose, compare, defend, estimate, judge, predict, rate,
core, select, support, value, evaluate
Compare theses, evaluate ideas, assess actions
Reading selection:
Author:
Questions:
210
Additional Questions Divided by Eras
1.
Foundations, ca. 8000 B.C.E–600 C.E.
To what extent did the characteristics of human populations change from ca. 8000 B.C.E to ca. 600 B.C.E?
(Consider size, location, and manner of accumulating food and shelter.)
2.
What changes (planned or unplanned) did humans make to the natural environment resulting from the advent of
agriculture and urban civilizations in this era?
3.
How did the natural environments of two of the following river civilizations influence developments in
technology, cultural achievements, and religious beliefs?
i. Mesopotamian societies, Indus River valley civilizations, Chinese, Meso-American/Andean
societies
4.
Analyze the reasons for the increasingly-wide trade networks in either the Eastern or Western hemispheres in the
period from ca. 8000 B.C.E.–600 C.E.
5.
Assess and account for the changes and continuities in how humans organized their societies across the period
from ca. 8000 B.C.E to ca. 600 B.C.E.
a. Use at least two of the following analytic categories: class systems, gender
b. systems, governmental systems, labor systems, nomadic vs. settled societies.
Postclassical Era, 600-1450
1.
2.
3.
4.
5.
6.
1.
2.
3.
4.
5.
6.
7.
Explain the spread of science and technology across Eurasia due to the existence of Dar al-Islam.
Compare the political institutions of two of the following empires: Tang-Song, Dar al-Islam, Western Europe,
Byzantine Empire.
In what ways did the Mongols contribute to the continued trends of cross-cultural interactions in Eurasia, and in
what ways did they cause discontinuities in those trends?
In what ways did the societies of sub-Saharan Africa and Europe interact with Islamic societies in this period?
Compare the causes of the spread of three of the following religions: Christianity, Judaism, Buddhism, Islam,
Confucianism, Hinduism.
Compare the technological and scientific achievements in two of the following societies: Classical China,
Classical Rome, Classical India, Classical Africa. Include information about their level of technological
development, the use of technology, and the societal attitude toward innovation.
Compare the effects of three of the nomadic migrations of the following on the settled societies into which they
migrated: Aztecs, Mongols, Turks, Vikings, Bantu.
Evaluate the role that the conversion to the Islamic faith had on the politics, economy, and society of the West
African kingdoms.
To what extent was the Indian Ocean region a coherent, connected “whole” in the post-classical era? Consider
political, economic, social, and cultural examples.
How did the consolidation of political empires in the post-classical empires affect the status of women? Choose
two of the following to analyze: Tang – Song; Aztec; Mali and Songhay, Dar al-Islam, Mongol Khanates.
Analyze the advantages and disadvantages of being a part of a tributary empire for (1) Japan, Korea, and Vietnam
under the Chinese; (2) Russia under the Tartars; and (3) peoples of Central America under the Aztecs.
Assess the accomplishments of the institution of the Roman Catholic Church in Western Europe in reintegrating
Western Europe into the Eastern hemisphere region in this period.
For one of the following civilizations, assess the impact of important political, economic, and social changes and
continuities as it moved from the river-civilization period through the classical and post classical period: China,
Indian, Mediterranean, Persian.
211
The Interaction of World Cultures, 1450–1750
1.
2.
3.
4.
5.
6.
7.
8.
9.
10.
11.
12.
13.
14.
15.
16.
17.
18.
19.
20.
21.
22.
23.
24.
Compare the labor systems in two of the following areas: Latin America, Russia, Ottoman Empire, Western
Europe.
Discuss the significance of the so-called Gunpowder Empires for international politics of this period.
Analyze the relationship of governmental/political structures to the acquisition of colonies.
What factors led to the creation of the first global economic network in the late 15th century?
Analyze the influence or importance of Islamic culture on the European Renaissance. (Consider economic,
cultural, and technological topics.)
What was the relationship of the Reformation with European political, economic, and social developments in the
16th-17th centuries?
Analyze the confluence of economic, social, and technological circumstances that led to the use of “unfree labor”
in North and South America by the Europeans in this era.
Why was sugar so profitable and why were sugar plantations so profitable for the Europeans in this period?
Compare the ways in which the Mongol khanates ruled and assimilated into Chinese, Persian, and Russian
societies.
Analyze the economic, cultural, and political relationships between settled/sedentary peoples and nomads in two
regions: Viking, Hungarian, Mongol, Aztec/Mexica, Bantu.
Compare the expansion of Russia with the expansion of the Ottoman Empire, the Western European empires, and
the Chinese empires in the 16-18th centuries. Pay particular attention to the politics of inclusion/exclusion of
various ethnic groups.
Compare European and Japanese feudal systems as social and political organizations.
Compare the economic, political, and cultural roles of cities in three of the following empires/regions: Inca,
Byzantine, Chinese, Mali.
Describe the role of women within the social and political structures in two of the following regions, and assess
the degree of change (or continuity) in women’s status in the post-classical period.
“Although there were many similarities between Sub-Saharan Africa and Western Europe around the year 1200
C.E., by 1450 C.E.they were less similar due to differences in trade problems, exposure to technological diffusion,
and seafaring conditions.” Assess the validity of this statement by describing the changes each region
experienced during this time period.
Analyze the impact of Islam on both China and Western Europe during this period.
Compare the two dominant civilizations of the Americas: the Aztecs and the Incas. Consider political, social,
economic, and cultural features.
Compare Islam and Confucianism in the following areas: acceptance of social and political protest; status and
condition of women; and tolerance for other beliefs within their society
Assess the impact of the Columbian Exchange by describing two of the following regions before and after 1492:
the Americas, Asia, Europe, Africa.
Assess the degree of change that occurred in Africa after the first wave of European contact in the 15th and early
16th centuries.
To what degree did Chinese society change during the Ming dynasty? (Be sure to include pieces of analysis from
the beginning/middle/end of the dynasty.)
Describe the key similarities and difference between China’s Zheng He expeditions (1405-1423) with those of
Western Europe in the late 15th and early 16th centuries.
Compare the European Renaissance works with Mughal artistic achievements.
Explain the role of the Chinese, Indian, and Islamic cultures in laying the foundations for European maritime
explorations in the 15th century.
Western Global Hegemony, 1750-1914
1.
2.
Compare two of the following colonial independence movements in terms of inspirations, goals, and counterrevolutionaries: Haitian, North America, South America, French.
Analyze the reasons why England and the Western Europeans were able to create industrial economies and the
Chinese were not.
212
3.
4.
5.
6.
7.
8.
9.
10.
11.
12.
13.
14.
Discuss the economic and social effects of an early industrial economy on the lower classes of society: rural
workers and urban workers. Be sure to include gender distinctions in your response.
Explain the economic relationship between the British colonial empire and the financing of the Industrial
Revolution.
Compare the early attempts to create an industrial economy in three of the following: England, U.S., Japan,
Russia, China, Egypt. (Consider political, economic, social, and religious categories in your analysis.)
Compare Russia’s interactions with the West with the interaction of one of the following other empires with the
West: Ottoman Empire, China, Tokagawa, Mughal India.
Compare the reasons (economic, political, social, cultural) for European countries acquiring colonies in the midto late 19th century.
How did the 19th century European/Western idea of nationalism influence both ideas about imperialism and ideas
about gender relations?
How did whether one was a settler or a non-settler colony affect the political, social, and cultural relations
between colonized and colonizers?
Describe the political, social, and economic changes brought to sub-Saharan Africa during the period of the new
imperialism (19th century).
Assess and explain the amount of change in women’s roles in two of these societies during the period of 17501914: Western European, Ottoman Empire, China, India, Sub-Saharan Africa, Latin America.
To what extent did the definition of “democracy” change from 1750 to 1914? Compare the colonization and
development of the US with colonization and development in one of the following areas: Australia, Canada, New
Zealand.
Compare the development of modernizing and nationalist movements in the Ottoman Empire and China in this
period.
Analyze the interplay or interconnection of industrialization, imperialism, and global conflict in Western Europe
and Africa in this period.
The 20th Century in World History, 1914 to the Present
1.
2.
3.
4.
5.
6.
7.
8.
9.
10.
11.
12.
To what extent did the two world wars end European global dominance?
To what extent did the creation of the League of Nations, United Nations, the Pan-Arab League, and the NonAligned Nations affect the patterns of global interactions in the 20th century?
Assess the political, economic, and social consequences of the world wars on two of the following: Russia, East
Asia, Sub-Saharan Africa.
Assess the effects of rapidly changing demographic and environmental trends in the 20th century on two of the
following regions: Latin America, Sub-Saharan Africa, India, USSR/Russia.
Discuss the extent to which the impacts of the globalization of science, technology, and culture have led to a
unification of the global community in the 20th century.
To what extent is genocide a phenomenon of the 20th /21st century?
Discuss the extent to which global economic developments of the 20th century have benefited two of the following
regions: the Americas, the Middle East, India.
Discuss the extent to which local opposition to the forces of globalization have succeeded in the second half of the
20th century.
To what extent did the definition of “democracy” change from 1914 to the present? (or from 1750 to the
present?)
Discuss the extent to which two of the following movements succeeded in their quests to change the status quo in
the 20th century: feminism, peasant protests, international Marxism, religious conservatives.
Identify and explain the important changes and continuities in the Russian empire from 1914 to the present.
Compare the social, economic, and political changes and continuities of Western Europe and Japan in the 20th
century.
213
13.
14.
15.
16.
17.
18.
19.
Compare the political, economic, and social causes and effects of two of the following revolutions: Russian,
Cuban, Mexican, Chinese, Iranian.
Compare the effects of two the following revolutions on women’s roles, status, and rights: Russian, Cuban,
Mexican, Chinese, Iranian.
To what extent are the categories of “First, Second, and Third World” useful terms for analyzing economic,
political, and social developments in the 20th century?
Compare the patterns of the post-war decolonizations in Africa and Asia.
Compare the legacies of the post-war end of colonization or neocolonialism in Africa, Asia, and Latin America.
Assess the advantages and disadvantages of high-tech warfare and guerrilla warfare in the 20th century. Use three
specific conflicts as examples.
To what extent did post-war art and culture diverge into new and different directions (different from the pre-war
era) in two of the following regions: Europe, Soviet Union, U.S., Middle East.
214
Timeline
Prehistoric Landmarks
2-2.5 million years ago----------- More humanlike species with larger brain size in eastern Africa.
120,000 years ago----------------- Arrival of Homo sapiens sapiens; later displacement of Neanderthals and other species
across Asia and Europe from initial center in Africa.
25,000 BCE------------------------ Passage of people to the Americas via land link from Asia.
8000-5000 BCE------------------- Further improvements in tool making; first development of agriculture; great expansion
in human population.
4000-3500 BCE------------------- Early use of bronze and copper tools.
1500 BCE-------------------------- Early use of iron tools and weapons.
River Valley Civilization
3500-2600 BCE------------------- Sumerian kingdom, development of cuneiform writing; river valley civilizations also in
Egypt and India.
1150-130 BCE--------------------- Books of the Jewish Old Testament written.
Classical Civilization
China
1029-258 BCE--------------------- China’s Zhou dynasty.
551-478 BCE---------------------- Confucius.
202-BCE-220 CE-----------------Han dynasty.
India
563-483 BCE---------------------- Gautama Buddha.
322-184 BCE---------------------- Mauryan dynasty.
Greece & Rome
800 BCE---------------------------- Rise of Greek city-states and economy; Homeric epics, Iliad and Odyssey; beginnings of
Rome.
330 BCE---------------------------- Macedonian Empire; Alexander the Great.
27 BCE----------------------------- Augustus Caesar; rise of Roman Empire; 476, fall of Rome.
313 CE------------------------------ Constantine adopts Christianity.
The Extension of Civilization
Middle East
570-632----------------------------- Muhammad and the foundation of Islam.
750-1258--------------------------- Abbasid caliphate.
Western Europe
800-814----------------------------- Charlemagne’s empire.
Americas
1350--------------------------------- Formation of Aztec Empire, height of Incan Empire.
Sub-Saharan Africa
1210-1400-------------------------- Empire of Mali.
East Asia
215
618-907----------------------------- Tang dynasty.
1236-1258-------------------------- Mongol conquests in Middle East, Russia.
1279--------------------------------- Toppling of Sung dynasty by Kublai Khan and Mongols.
Byzantium & Eastern Europe:
1054--------------------------------- Schism between Eastern and Western Christianity.
1453--------------------------------- Turkish capture of Constantinople; end of Byzantine Empire.
The Early Modern World
East Asia
1405-1433--------------------------Great Chinese fleets.
1600-1868--------------------------Tokugawa shogunate.
1644-1912--------------------------Qing dynasty.
India and Southeast Asia
1498---------------------------------Vasco da Gama (Portugal) to India.
1526-1761--------------------------Mughal Empire.
1857---------------------------------Mughal decline; rise of Sikh state 1708 and states of southern India.
1744, 1756-1763------------------French –British wars in India.
Western Civilization
1517---------------------------------Luther’s 95 theses; beginning of Protestant Reformation.
1519-1521--------------------------Magellan’s expedition around the world.
17th Century------------------------Scientific Revolution; Galileo (1564-1642), Newton (1642-1727).
1688-1690------------------------- Glorious Revolution in Britain, parliamentary regime; some religious toleration; political
writing of John Locke.
18th Century------------------------Enlightenment; Voltaire (1694-1778).
Russia & Eastern Europe
1552-1556--------------------------Russian expansion in Central Asia, western Siberia.
1689-1725--------------------------Peter the Great and partial Westernization.
Middle East (Ottoman Empire)
1683---------------------------------Failure of Ottoman assault of Vienna.
Latin America
1794---------------------------------Haitian uprising against France led by Toussaint L’ Ouverture; independence and end of
slavery there.
The World in the First Industrial Century
Western Civilization
1770--------------------------------Invention of the steam engine by Watt; Industrial Revolution begins.
1775-1783-------------------------American Revolution.
1789-1799-------------------------French Revolution.
1848--------------------------------Writings of Karl Marx.
1864-1871-------------------------German Unification.
1871-1914-------------------------Highpoint of Western imperialism.
1914-1918-------------------------World War I.
East Asia
1839-1842-------------------------Opium Wars in China.
1853--------------------------------Perry Expedition.
1868-1912-------------------------Meiji period in Japan.
216
1894-1895------------------------- Sino-Japanese War.
1904-1905------------------------- Russo-Japanese War.
1911-------------------------------- Chinese Revolution.
Middle East
1798-------------------------------- Napoleon’s Egyptian expedition.
1830-------------------------------- First Ottoman reform efforts.
Latin America
1808-------------------------------- Growing commercialization of Latin American economy.
1910----------------------------- ---Beginning of Mexican Revolution.
Sub-Saharan Africa
1807-1834------------------------ -Abolition of Atlantic slave trade.
1814----------------------------- ---Acquistion by British of Dutch South Africa.
1870------------------------------- -Rapid European Imperialist Gains.
India and Southeast Asia
1885-------------------------------- Formation of Indian National Congress.
Russia and Eastern Europe
1854-1856------------------------- Crimean War.
1861-------------------------------- Russian emancipation of serfs.
1884-------------------------------- Beginnings of Russian industrialization; Sergei Witte leading minister; completion of the
trans-Siberian railway.
1917--------------------------------Russian Revolution; abolition of the tsarist regime; Bolshevik victory.
217
The Exam
Test your knowledge of world history -- and potentially earn college credit while you're at it -- with the AP
World History Exam.
About the Exam
The three-hour-and-five-minute exam includes a 55-minute multiple-choice section and a 130-minute
free-response section.
Section I: Multiple-Choice
The multiple-choice section of the exam accounts for half of your exam score, and the free-response
section for the other half.
Section I consists of 70 multiple-choice questions designed to measure your knowledge of world history
from Period 1 to the present. This section follows the percentages listed below; questions will draw from
individual or multiple periods:
1
Technological and Environmental
Transformations
to c. 600 BCE
2
Organization and Reorganization of Human
Societies
c. 600 BCE to c. 600 CE
3
Regional and Transregional Interactions
c. 600 CE to c. 1450
4
Global Interactions
c. 1450 to c. 1750
5
Industrialization and Global Integration
c. 1750 to c. 1900
6
Accelerating Global Change and Realignments
c. 1900 to Present
Section II: Free-Response
In Section II, the free-response section of the exam, Part A begins with a mandatory 10-minute reading
period for the document-based question. You should answer the document-based question in
approximately 40 minutes. In Part B, you are asked to answer a question that deals with continuity and
change over time (covering at least one of the periods in the concept outline). You will have 40 minutes to
answer this question, 5 minutes of which should be spent planning and/or outlining your answer. In Part
C, you are asked to answer a comparative question that will focus on broad issues or themes in world
history and deal with at least two societies. You'll have 40 minutes to answer this question, 5 minutes of
which should be spent planning and/or outlining your answer.
Document-Based Essay Question
Put on your "historian" hat to demonstrate your ability to analyze source materials and develop an essay
based on those materials. Your goal: a unified essay that integrates your analysis of four to ten given
documents with your treatment of the topic. Comparative topics on the major themes will provide one of
the focuses of the DBQs, including comparative questions about different societies in situations of mutual
contact. The DBQ begins with a mandatory 10-minute reading period. Then you'll have 40 minutes to
write the essay.
The source materials are chosen for two reasons: the information they convey about the topic and the
perspective they offer on other documents used in the section. There is no one perfect DBQ answer; a
variety of approaches and responses are possible depending on your ability to understand the documents
and, ultimately, judge their significance. Remember: You'll most fully understand some of the documents
when you view them within the wider context of the entire series.
When writing the document-based essay, it's important to:

Refer to individual documents within the framework of the overall topic.

Uses all of the documents.

Discuss the materials in reference to the question -- don't just summarize them.

Cite documents by naming the author and/or by naming the document number.
218

There are no irrelevant or deliberately misleading documents.

It's important that you put your analytic skills to work and demonstrate that you understand
context, bias, and frame of reference regarding the documents' sources and the authors' points of view.
Group or juxtapose documents in a variety of ways (e.g. according to their ideas or points of view);
suggest reasons for similarities or differences in perspective among the documents; and identify possible
biases or inconsistencies within documents.

You'll be asked to explain the need for additional documents that would help you answer the
question more completely. You may also have to discuss which points of view are missing from the given
documents. Since the DBQ focuses on historical skills within a world history framework, remember to
place documents chronologically, culturally, and thematically.

You're not expected to know the author or topic of all the DBQ documents, or to include
information outside of the documents.
Continuity and Change-Over-Time Essay
The Continuity and Change-Over-Time-Essay focuses on large global issues such as technology, trade,
culture, migrations, or biological developments. It covers at least one of the periods in the course outline
and one or more cultural areas. You'll have 40 minutes to write the essay. It's recommended that you
spend five minutes planning and/or outlining your answer before you begin writing. You may be able to
choose different cases to illustrate your point.
Comparative Essay
In the final free-response essay you'll answer a comparative question that focuses on developments in two
or more societies, and their interactions with each other or with major themes or events (e.g. culture,
trade, religion, technology, migrations). You'll have 40 minutes to write the essay. It's recommended that
you spend five minutes planning and/or outlining your answer before you begin writing.
Scoring the Exam
The multiple-choice and free-response sections are each worth half of the final exam grade.
Themes and AP World History:
AP World History requires students to learn World History thematically. The Course is taught using five themes that form
threads that unify time periods and societies into a broad picture of history. The themes also help to compare and
analyze change and continuity over time.
Social--Development and transformation of social structures

Gender roles and relations

Family and kinship

Racial and ethnic constructions

Social and economic classes
Political--State-building, expansion, and conflict

Political structures and forms of governance

Empires

Nations and nationalism

Revolts and revolutions

Regional, trans-regional, and global structures and organizations
Interaction between humans and the environment

Demography and disease

Migration

Patterns of settlement

Technology
Cultural--Development and interaction of cultures

Religions

Belief systems, philosophies, and ideologies

Science and technology

The arts and architecture
Economic--Creation, expansion, and interaction of economic systems

Agricultural and pastoral production

Trade and commerce

Labor systems

Industrialization

Capitalism and socialism
219
Historical Periods, Key Concepts, and Theme’s
Historical Periods
1. Technological and Environmental Transformations to c. 600 B.C.E. 5%



Key Concept 1.1, Big Geography and the peopling of the earth.
Key Concept 1.2, The Neolithic Revolution and Early Agricultural Societies.
Key Concept 1.3, The Development and Interactions of Early Agricultural, Pastoral and Urban
Societies.
2. Organization and Reorganization of Human Societies c. 600 B.C.E. to c. 600 C.E.
15%



Key Concept 2.1, The Development and Codification of Religious and Cultural Traditions.
Key Concept 2.2, The Development of States and Empires
Key Concept 2.3, Emergence of Transregional Networks of Communication and Exchange.
3. Regional and Transregional Interactions c. 600 C.E. to c. 1450 20%



Key Concept 3.1, Expansion and Intensification of Communication and Exchange Networks.
Key Concept 3.2, Continuity and Innovation of State Forms and their Interactions.
Key Concept 3.3, Increased Economic Productive Capacity and Its consequences.
4. Global Interactions c. 1450 to c. 1750 20%



Key Concept 4.1, Globalizing Networks of Communication and Exchange.
Key Concept 4.2, new Forms of Social Organization and Modes of Production.
Key Concept 4.3, State Consolidation and Imperial Expansion.
5. Industrialization and Global Integration c. 1750 to c. 1900 20%




Key Concept 5.1, Industrialization and Global Capitalism.
Key Concept 5.2, Imperialism and Nation State Formation.
Key Concept 5.3, Nationalism, Revolution and Reform.
Key Concept 5.4, Global Migration
6. Accelerating Global Change and Realignments c. 1900 to the Present 20%



Key Concept 6.1, Science and the Environment.
Key Concept 6.2, Global Conflicts and Their Consequences.
Key Concept 6.3, new Conceptualizations of Global Economy, Society and Culture.
Theme 1: Interaction Between Humans and the Environment
• Demography and disease
• Migration
• Patterns of settlement
• Technology
The interaction between humans and the environment is a fundamental theme for world history. The
environment shaped human societies, but, increasingly, human societies also affected the environment. During
prehistory, humans interacted with the environment as hunters, fishers and foragers, and human migrations led
to the peopling of the earth. As the Neolithic revolution began, humans exploited their environments more
intensively, either as farmers or pastoralists. Environmental factors such as rainfall patterns, climate, and
available flora and fauna shaped the methods of exploitation used in different regions. Human exploitation of
220
the environment intensified as populations grew and as people migrated into new regions. As people flocked
into cities or established trade networks, new diseases emerged and spread, sometimes devastating an entire
region. During the Industrial Revolution, environmental exploitation increased exponentially. In recent
centuries, human effects on the environment —and the ability to master and exploit it — increased with the
development of more sophisticated technologies, the exploitation of new energy sources and a rapid increase in
human populations. By the twentieth century, large numbers of humans had begun to recognize their effect on
the environment and took steps toward a “green” movement to protect and work with the natural world instead
of exploiting it.
Theme 2: Development and Interaction of Cultures
• Religions
• Belief systems, philosophies, and ideologies
• Science and technology
• The arts and architecture
This theme explores the origins, uses, dissemination, and adaptation of ideas, beliefs, and knowledge within and
between societies. Studying the dominant belief system(s) or religions, philosophical interests, and technical
and artistic approaches can reveal how major groups in society view themselves and others, and how they
respond to multiple challenges. When people of different societies interact, they often share components of
their cultures, deliberately or not. The processes of adopting or adapting new belief and knowledge systems are
complex and often lead to historically novel cultural blends. A society’s culture may be investigated and
compared with other societies’ cultures as a way to reveal both what is unique to a culture and what it shares
with other cultures. It is also possible to analyze and trace particular cultural trends or ideas across human
societies.
Theme 3: State-Building, Expansion, and Conflict
• Political structures and forms of governance
• Empires
• Nations and nationalism
• Revolts and revolutions
• Regional, transregional, and global structures and organizations
This theme refers to the processes by which hierarchical systems of rule have been constructed and maintained
and to the conflicts generated through those processes. In particular, this theme encourages the comparative
study of different state forms (for example, kingdoms, empires, nation-states) across time and space, and the
interactions among them. Continuity and change are also embedded in this theme through attention to the
organizational and cultural foundations of long-term stability on one hand, and to internal and external causes of
conflict on the other. Students should examine and compare various forms of state development and expansion
in the context of various productive strategies (for example, agrarian, pastoral, mercantile), various cultural and
ideological foundations (for example, religions, philosophies, ideas of nationalism), various social and gender
structures, and in different environmental contexts. This theme also discusses different types of states, such as
autocracies and constitutional democracies. Finally, this theme encourages students to explore interstate
relations, including warfare, diplomacy, commercial and cultural exchange, and the formation of international
organizations.
Theme 4: Creation, Expansion, and Interaction of Economic Systems
• Agricultural and pastoral production
• Trade and commerce
• Labor systems
• Industrialization
• Capitalism and socialism
221
This theme surveys the diverse patterns and systems that human societies have developed as they exploit their
environments to produce, distribute, and consume desired goods and services across time and space. It stresses
major transitions in human economic activity, such as the growth and spread of agricultural, pastoral, and
industrial production; the development of various labor systems associated with these economic systems
(including different forms of household management and the use of coerced or free labor); and the ideologies,
values, and institutions (such as capitalism and socialism) that sustained them. This theme also calls Return to
the Table of Contents c The College Board AP World History Curriculum Framework 20 attention to patterns
of trade and commerce between various societies, with particular attention to the relationship between regional
and global networks of communication and exchange, and their effects on economic growth and decline. These
webs of interaction strongly influence cultural and technological diffusion, migration, state formation, social
classes, and human interaction with the environment.
Theme 5: Development and Transformation of Social Structures
• Gender roles and relations
• Family and kinship
• Racial and ethnic constructions
• Social and economic classes
This theme is about relations among human beings. All human societies develop ways of grouping their
members, as well as norms that govern interactions between individuals and social groups. Social stratification
comprises distinctions based on kinship systems, ethnic associations, and hierarchies of gender, race, wealth,
and class. The study of world history requires analysis of the processes through which social categories, roles,
and practices were created, maintained, and transformed. It also involves analysis of the connections between
changes in social structures and other historical shifts, especially trends in political economy, cultural
expression, and human ecology.
222
VOCABULARY CARD FORMAT
For each vocabulary term in AP World History you are required to complete a note card on 3 x 5 index cards. It is
important that you use 3 x 5 index cards and not a different size since other sizes make managing class sets of thirty or
more difficult. You will use these note-cards in composing your answers to study questions (forming “concept maps” and
“association maps”) and to study for quizzes, exams, and the AP exam. Ultimately, you will have a tremendous personal
resource that you can use to cement what you learn in this class for the long-term. In the short-run, you are required to
turn in your note-cards when asked.
GUIDELINES:
1. Use only white 3 x 5 index cards. Do Not use different sizes or colored cards. They can be ruled or not ruled.
YOU WILL NOT BE GIVEN CREDIT FOR CARDS THAT DO NOT MEET THESE SPECIFICATIONS.
2. On the front of each card write,
a. The vocabulary term, and
b. Whatever AP World history theme you think applies
You should have NO OTHER INFORMATION on the front of the card.
3. The back of the card should include the following:
a.
b.
c.
d.
A definition of the term, probably a single sentence.
The significance of the term (why it is important); a second sentence.
Which Big Concept it is associated with (just give the number, e.g. 1.2.)
A pen mark of the color that corresponds to the Era folder.
FRONT
SAMARKAND
Theme 1 – Humans & Environment
BACK
Concept: 3.1
Def:
Capital of Tamerlane’s empire & trading center for
Central Asia.
Sig:
City on Silk Road, example of trade entrepot. Cultural
mixing/syncretism.
(PEN MARK IN YELLOW)
223
METHODS FOR USING NOTECARDS
Note cards are an excellent resource for many different types of intelligence. They are portable, transferable
(someone else can use them), and can be manipulated in many different ways. They can be used to study in
ways that access several forms of intelligence. Below are some studying techniques using note cards:
1. Standard method – sequential.
This technique is probably what most students have been exposed to before entering high-school. One
takes a stack of note cards and simply works their way through it one at a time. This works well for people with
high logical/sequential intelligence.
2. Sorting method – eliminative
The sorting method eliminates cards that are learned as the cards are worked. The student creates three
piles of cards:
Pile 1 – Cards that are known and fully learned
Pile 2 – Cards that are somewhat known but not fully committed to memory.
Pile 3 – Cards that are unknown.
The student removes pile 1 and works their way through the remaining stacks (piles 2 & 3) until they have
learned and committed some to memory. They then sort their cards into the same three piles. This results in the
number of cards continually decreasing, and the amount of information committed to memory continually
increasing. At the end of the exercise all cards are “pile 1” cards. This technique also works well for people
with high logical/sequential
224